You are on page 1of 508
‘Mark this question => Question Id: 19595 Question 1 of 30 A.56-year-old is recently commenced on lefimnomide for sero-negative cheumatoid arthritis. Prior te the drug, her AST was 33 U/L and her ALT was 40 UiL which are now increased to 59 U/L and 72 WL respectively. Her FBC is normal, Alkaline phosphaiace=100 U/L and Bilrubin=12 mol/L. What isthe most appropriate management option? a) Continue leflunomide and monitor LFTs in one month +b) Stop the leflunomide and repeat tests in two weeks. ¢) Continme leflunomide and monitor LFTs in two weeks 4) Stop leflunomide and commence washout procedure, €) Stop leflunomide and seek urgent rheumatological advice Avewar (RERNGIRENY other UcersExptanation Report An Eos Question Explanation: Leflunomide is associated with serious hepatotoxicity Increased aminotransferases are commonly seen in association with therapy occurring in 15-20% of cases (less than a twofold rise) However, more serious elevation (greater than threefold) is seen in less then 5%. Generally, most hepatic events occur within the first sie months of use. Guidelines suggest that where there is a less than two fold elevation of transaminases, the drug should be stepped and the LFT repeated in two weeks, Ifthe results have retumed to normal then the drug can be recommenced, ‘As the active crug has such a long half @pproximately 15 days), in patients with severe elevations of LFTs, wash out veatment may be required to assist in exctetion/teduce absortion of the drug ‘This includes cholestyramine and actwated charcoal. ‘Mark this question => Question Id: 19595 Question 1 of 30 A.56-year-oldis recently commenced on lefhinomide for sero-negative rheumatoid arthritis. Prior to the drug, her AST was 33 U/L and her ALT was 40 U/L which are now increased to 59 U/L and 72 ULrespectively. Her FBC is normal. Alkaline phosphatase=100 U/L and Bilrubin=12 mol/L. What is the most appropriate management option? 8) Continue leflunomide and moniter LFTs in one month, Y © b) Stop the leflunomide and repeat tests in two weeks, ©) Continue leflunomide and monitor LFTs in two weeks 4) Stop leflunomide and commence washout procedwe. €) Stop leflunomide and seek urgent rheumatological advice Avewar (RERNGIRENY other UcersExptanation Report An Eos Question Explanation: Leflunomide is associated with serious hepatotoxicity Increased aminotransferases are commonly seen in association with therapy occurring in 15-20% of cases (less than a twofold rise) However, more serious elevation (greater than threefold) is seen in less then 5%. Generally, most hepatic events occur within the first sie months of use. Guidelines suggest that where there is a less than two fold elevation of transaminases, the drug should be stepped and the LFT repeated in two weeks, Ifthe results have retumed to normal then the drug can be recommenced, ‘As the active crug has such a long half @pproximately 15 days), in patients with severe elevations of LFTs, wash out veatment may be required to assist in exctetion/teduce absortion of the drug ‘This includes cholestyramine and actwated charcoal. 2726/2014 1 PM ‘Mark this question & => Question 2 of 30 Tenis elbow is associated with? a) Pain on pressure over the medical epicondyle +b) Pain on supination c) Pain on adduction of the thumb against resistance 4) Pain on wrist extension against resistance ¢) Pain on extension of the elbow Question Explanation: ‘Temnis elbow is due to lateral epicondylitis and is due to overuse/strain of the extensor muscles ofthe forearm, Golfer’s elbow is pain at the medial epicondyle. Consequently there is pain over the lateral epicondyle and the pam is exacerbated by wrist extension, 2726/2014 1 PM ‘Mark this question & => Question 2 of 30 ‘Temas elbow is associated with? a) Pain on pressure over the medical epicondyle +b) Pain on supination c) Pain on adduction ofthe thamb against resistance Y © 4) Pain on wrist extension against resistance ¢) Pain on extension of the elbow Question Explanation: ‘Temnis elbow is due to lateral epicondylitis and is due to overuse/strain of the extensor muscles ofthe forearm, Golfer’s elbow is pain at the medial epicondyle. Consequently there is pain over the lateral epicondyle and the pam is exacerbated by wrist extension, ‘Mark this question & => Question Td : 28322 Question 3 of 30 A31-year-old woman presents with Raynaud's phenomenon, Which one of the following clnical features suggests an underlying connective tissue disease? a) Involvement of toes ) One previous miscarriage in early pregnancy «) Symmetrical involvement of fingers @) Pernio «) Symptoms developed as a teenager Answer (UEISINRIAY) other User's Explanation Report An Evan Question Explanatioy Ahistory of Pemio or chilblains is suggestive of an underiying connective tissue disease. Other features suggestive of the potential presence of later development of an underlying connective disease include the onset of digital vasospasm after the age of 30, male sex, undateral ivelvement, and abnormal nail fold capillary changes in microscopy, scleradactyly, rashes and serological presence of autoantibodies. ‘Mark this question & => Question Td : 28322 Question 3 of 30 A31-year-old woman presents with Raynaud's phenomenon, Which one of the following clnical features suggests an underlying connective tissue disease? a) Involvement of toes ) One previous miscarriage in early pregnancy «) Symmetrical involvement of fingers Y¥ © A Pernio «) Symptoms developed as a teenager Answer (UEISINRIAY) other User's Explanation Report An Evan Question Explanatioy Ahistory of Pemio or chilblains is suggestive of an underiying connective tissue disease. Other features suggestive of the potential presence of later development of an underlying connective disease include the onset of digital vasospasm after the age of 30, male sex, undateral ivelvement, and abnormal nail fold capillary changes in microscopy, scleradactyly, rashes and serological presence of autoantibodies. ‘Mare this question a => Question Td : 46790 Question 4 of 30 A55 year old presents to you with pain and silfiess in both hands and knees of 6 months duration. You suspect rheumatoid arthritis, Tue physical examination finding that is most distinctive with a diggnosis is a) Unilateral joint tendemness/effisions b) Maculopapular rash ©) Toint stiffness worse in the morring, 4d) lridocyelitis €) Hepatosplenomegely Question Explanation: Rheumetoid arthritis usually affects the joints on both sides ofthe body. So, ifthe right hip is affected, so will the left. Some people will have only one atack of rheumatoid arthritis, but most will have one, then a period where they are arthrtis-free, and then another amack. These flare-ups usually continue until the time berween flare-ups is briefer and briefer, and the symptoms are-worse each time, Some flare-ups last only a few days; some last for weeks. The joints affected by rheumatoid arthritis have a membrane, called the synoviel membrane that produces synovial uid to keep the joints lubricated and to help the bones move smoothly around the joint When the synovial membrane becomes stiff and thick, it becomes difficult to move the joint Joint pain and stiffness is usually ‘worse frst thing in the morning, Other symptoms include redness and warmth around the affected joint, low fever, appetite and weight loss, and fatigue. Eventually, the joint becomes deformed, the cartilage is destroyed, and the unprotected bone begins to wear away. The joints can become ‘stuck’ in one position, called contractures. Rheumatoid arthritis can cause other problems in the body, such as vascultis (inflammation of the blood vessels), osteoporosis (loss of bone density), lung disease, rheumatoid nodules (stall bumps) under the skin, and blood disorders. ‘Mare this question a => Question Td : 46790 Question 4 of 30 A55 year old presents to you with pain and silfiess in both hands and knees of 6 months duration. You suspect rheumatoid arthritis, Tue physical examination finding that is most distinctive with a diggnosis is a) Unilateral joint tendemness/effisions b) Maculopapular rash Y © 6) Joint stiffiess worse in the morning 4d) lridocyelitis €) Hepatosplenomegely Question Explanation: Rheumetoid arthritis usually affects the joints on both sides ofthe body. So, ifthe right hip is affected, so will the left. Some people will have only one atack of rheumatoid arthritis, but most will have one, then a period where they are arthrtis-free, and then another amack. These flare-ups usually continue until the time berween flare-ups is briefer and briefer, and the symptoms are-worse each time, Some flare-ups last only a few days; some last for weeks. The joints affected by rheumatoid arthritis have a membrane, called the synoviel membrane that produces synovial uid to keep the joints lubricated and to help the bones move smoothly around the joint When the synovial membrane becomes stiff and thick, it becomes difficult to move the joint Joint pain and stiffness is usually ‘worse frst thing in the morning, Other symptoms include redness and warmth around the affected joint, low fever, appetite and weight loss, and fatigue. Eventually, the joint becomes deformed, the cartilage is destroyed, and the unprotected bone begins to wear away. The joints can become ‘stuck’ in one position, called contractures. Rheumatoid arthritis can cause other problems in the body, such as vascultis (inflammation of the blood vessels), osteoporosis (loss of bone density), lung disease, rheumatoid nodules (stall bumps) under the skin, and blood disorders. 2/26/2014 12:52:11 PM ‘Marke this question <=> Question Tad : 48742 Question 5 of 30 ‘A. 50) year old man has a hstory of chronic musculoskeletal pain, He hes been using frequent doses of Zxtra-Strength Tylenol with good results, He is concemed about the overdose and asks what his maximum daily dosage of acetamnophen should be. He weight 70 kg (154 1b). You tell him that the most he should take in 24 hours is a) 2000 mg b) 3000 mg ©) 4000 mg 4) 5000 mg 2) 6000 mg Answer | Banation | Other User's Explanation Report An Error Question Explanation ‘The maxinum daily dosage for all acetaminophen preparations is 4000 mg. Acetaminophen is used in more combination products than any other drug, for a number of different indications. An FDA panel has recommended that stronger warnings about hepatotoxicity be added to the label information for acetaminophen, Because it is used so frequently and is present in so many different preparations, care must be taken DOC to exceed the masimum 24-hour dosage in order to avoid hepatotoxicity 2/26/2014 12:52:11 PM ‘Marke this question <=> Question Tad : 48742 Question 5 of 30 ‘A. 50) year old man has a hstory of chronic musculoskeletal pain, He hes been using frequent doses of Zxtra-Strength Tylenol with good results, He is concemed about the overdose and asks what his maximum daily dosage of acetamnophen should be. He weight 70 kg (154 1b). You tell him that the most he should take in 24 hours is a) 2000 mg b) 3000 mg Y © ¢)4000 mg 4) 5000 mg 2) 6000 mg Answer | Banation | Other User's Explanation Report An Error Question Explanation ‘The maxinum daily dosage for all acetaminophen preparations is 4000 mg. Acetaminophen is used in more combination products than any other drug, for a number of different indications. An FDA panel has recommended that stronger warnings about hepatotoxicity be added to the label information for acetaminophen, Because it is used so frequently and is present in so many different preparations, care must be taken DOC to exceed the masimum 24-hour dosage in order to avoid hepatotoxicity 2262014 12:52:29 PM ‘Mark this question = => (Question Id : 48813 Question 6 of 30 AAS year old male presents 2 weeks after you see him for infectious diarthea caused by C. Iejuni. He has now developed unilateral lower limb weakness and bilateral distal parasthesia and decreased ankle tendon reflex. Te most likely dagnosis is a) Guillain Barre syndrome b) Multiple Sclerosis c) Myastheria Gravis ) Systemic Lupus Erythematosus Question Explanation: Guillain-Berré syncrome is the most common acquired inflammatory neuropathy. Ttis an acute, usually rapidly progressive inflammatory polyneuropathy charactenzed by muscular weakness and mid distal sensory loss. In most patents, the syndrome begins 1-3 weeks after an infectious disorder, surgery, or vaccination. Infection is the trigger in > 50% of patients; common pathogers inchide Campylobacter jejuni, enteric viruses, herpesviruses, and Mycoplasma. Flactid weakness predominates in most patients, its always more prominent than sensory abnormalities and may be most prominent proximally. Relatively symmetric weakness with paresthesias usually begins in the legs anc progresses to the arms, butit occasionally ‘begins in the arms of head Diagnose is clinical, If Guillain-Barré eyndrome is cuspected, patients chould be admitted to a hospital for electromyography (EMG), CSF analysis, and measurement of forced vital capacity. Guillain-Barré syndrome is a medical emergency, requiring constant monitoring and support of vital functions, typically in an ICU. Plasmapheresis helps when done early in the syndrome and is the treatment of choice in acutely il patients. Immune globulin is also effective when given ey. 2262014 12:52:29 PM ‘Mark this question = => (Question Id : 48813 Question 6 of 30 AAS year old male presents 2 weeks after you see him for infectious diarthea caused by C. Iejuni. He has now developed unilateral lower limb weakness and bilateral distal parasthesia and decreased ankle tendon reflex. The most lixely diagnosis is ¥ © a) Guilain Barre syndrome b) Multiple Sclerosis c) Myasthenia Gravis 4) Systemic Lupus Erythematoous Answer | Explanation Other User's Explanation Report An Error Question Explanation: Guillain-Berré syncrome is the most common acquired inflammatory neuropathy. Ttis an acute, usually rapidly progressive inflammatory polyneuropathy charactenzed by muscular weakness and mid distal sensory loss. In most patents, the syndrome begins 1-3 weeks after an infectious disorder, surgery, or vaccination. Infection is the trigger in > 50% of patients; common pathogers inchide Campylobacter jejuni, enteric viruses, herpesviruses, and Mycoplasma. Flactid weakness predominates in most patients, its always more prominent than sensory abnormalities and may be most prominent proximally. Relatively symmetric weakness with paresthesias usually begins in the legs anc progresses to the arms, butit occasionally ‘begins in the arms of head Diagnose is clinical, If Guillain-Barré eyndrome is cuspected, patients chould be admitted to a hospital for electromyography (EMG), CSF analysis, and measurement of forced vital capacity. Guillain-Barré syndrome is a medical emergency, requiring constant monitoring and support of vital functions, typically in an ICU. Plasmapheresis helps when done early in the syndrome and is the treatment of choice in acutely il patients. Immune globulin is also effective when given ey. ‘Mark this question >> Question Td : 50619 Question 7 of 30 A.19 year old girl presents with pain and swelling ofher right knee and no other symptoms, Joint efflsion is revealed on investigation. She has been sexually active since she was 15 and is seeing a gynecologist for follow-up of cervical dysplasia. Rheumatoid factor was negative. The most licely dhagnosis is a) Osteoarthritis ) Gouty arthritis ©) Psendogout 4) Systemic lupus erythematosus €) Gonococcal arthritis Question Explanation: Gonococcal arthtits is a more focal form of disseminated gonococcal infection (GI) that results in a frank septic arthritis with effusion Some patients have previous ot coincident symptoms of DGI. Usualy only 1 or 2 joints are involved, primarily the knees, ankles, wrists, and elbows. Onset is often acute, with fever, severe pain, and limtation of movement but may occur without constintional symptoms, Infected joints are swollen, and the overlying skin may be warmn andi red, Th 30 to 40% of patients with DGL, blood cultures are positive in the Ist week of liness. With septic arthritis, blood cultures are less often positive, but joint fuids are more often positive. Isolated, frenle, acute arthritis in a sexually active patient requires joint aspiration to diagnose gonococcel infection, Fluid is usually purulent (WECs > 20,000)uiL). Cultures of jomt uid are positive in 40 to 50%, but organisms are rarely wisible on Gram stain. PCE testing may be more sensitive buthas not been evaluated. ‘Mark this question >> Question Td : 50619 Question 7 of 30 A 19 year old girl presents with pain and swelling ofher right knee andno other symptoms, Joint effusion is revealed oa investigation. She has been sexually active since she was 15 and is seeing a gynecologist for follow-up of cervical dysplasia. Rheumatoid factor was negative. The most likely dagnosis is a) Osteoarthritis ') Gouty arthritis ©) Psendogout 4) Systemic inpus erythematosus VY © €) Gonococcal arthritis Question Explanation: Gonococcal arthtits is a more focal form of disseminated gonococcal infection (GI) that results in a frank septic arthritis with effusion Some patients have previous ot coincident symptoms of DGI. Usualy only 1 or 2 joints are involved, primarily the knees, ankles, wrists, and elbows. Onset is often acute, with fever, severe pain, and limtation of movement but may occur without constintional symptoms, Infected joints are swollen, and the overlying skin may be warmn andi red, Th 30 to 40% of patients with DGL, blood cultures are positive in the Ist week of liness. With septic arthritis, blood cultures are less often positive, but joint fuids are more often positive. Isolated, frenle, acute arthritis in a sexually active patient requires joint aspiration to diagnose gonococcel infection, Fluid is usually purulent (WECs > 20,000)uiL). Cultures of jomt uid are positive in 40 to 50%, but organisms are rarely wisible on Gram stain. PCE testing may be more sensitive buthas not been evaluated. 2/26/2014 1 Pu ‘Mark this question & => Question 8 of 30 AAl-year-old has acute right knee monoarthritis confirmed to be Gout, He also has a duodenal ulcer. Which of the following would is the best intial treatment for him? a) Allopurinal b) intra-articular corticosteroid injection ©) Indomethacin alone 4) Sulfinpyrazone @) Celecoxib Question Explanation: ‘All non-steroidals including Cox-I selective non-steroidals are contraindicated in the presence of active ulceration -Allopurinol should never be started in the presence of acute gout as the symptoms will be exacerbated. In large joint such as the knee the safest option would be to inject corticosteroid into the joint, Colchicine would also be an option but is associated with gastrointestinal (Gl) toxicity. Unicosunic drugs are used orally to treat chronic gout. 2/26/2014 1 Pu ‘Mark this question & => Question 8 of 30 AAl-ycar-sld has acute right knee monoarthritis confirmed to be Gout. He also has a duodenal ulcer. Which of the following would is the best initial treatment for him? a) Allopurinel Y © 5) Intra-articular corticosteroid injection. ) Indomethacin alone 4d) Sulfinpyrazone 2) Celecoxib Question Explanation: ‘All non-steroidals including Cox-I selective non-steroidals are contraindicated in the presence of active ulceration -Allopurinol should never be started in the presence of acute gout as the symptoms will be exacerbated. In large joint such as the knee the safest option would be to inject corticosteroid into the joint, Colchicine would also be an option but is associated with gastrointestinal (Gl) toxicity. Unicosunic drugs are used orally to treat chronic gout. 22672014 12:53:14 PM. 95 www. interface.ecdu.ok/". x \ Rl@emadkornran = Yahoo Ni \\ € > CB www interface edupk/medical-axams/test-analysis phprutid=14759 & apps FJ coogle A settings [) signin Cl imported Fromie 2 ge dle © oe yo Gaull Saye » © other bookmar ‘Maske this question <& => Question 14: 54940] Total Questions Question 9 of 30 A 55-year-old femele undergoes a DEXA scan which reveals a bone mincral density (BMD) T score of -2.55 at the hip and lumbar ee of the following may contribute to such a result? a) Cushing's Disease b) Acromegaly c) Delayed menopause d) Hypothyroidisra €) Obesity foe: BIBI 06 csccptenaton | reponan Err L Question Explanatios 2 3 ‘This patient has osteoporesis as defined by her abnormally low T score Diseases associsted with osteoporosis are - Vitamin D deficiency - Thyrotozicosis - Myeloma and - Hypegonadises “Myeloma and lymphoma are also associated with reduced BMD. Other associates include. ~ Rheumatoid arthritis ~ Renal failure ~ Corticosteroids - Early menopause - Slender habitus - Smoking ~ Lack of exercise - Family history ~ AgelSex and - Excess deohol RERPERERE SRE DER EEE SP ee eee XxXxXXXKXXKXKXXKKXKXKXKKXKXXKXKXKXXKKXKXKXKXKXXK 22672014 12:53:14 PM. 95 www. interface.ecdu.ok/". x \ Rl@emadkornran = Yahoo Ni \\ € > CB www interface edupk/medical-axams/test-analysis phprutid=14759 & apps FJ coogle A settings [) signin Cl imported Fromie 2 ge dle © oe yo Gaull Saye » © other bookmar ‘Maske this question <& => Question 14: 54940] Total Questions Question 9 of 30 A 55-year-old female undergoes a DEXA scan which reveals abone mineral density (BMD) T score of -2.55 at the hip and lumbar ee of the following may contribute to such a result? Y © a) Cushing's Disease b) Acromegaly c) Delayed menopause 4) Hypothyroidism €) Obesity Question Explanatior ‘This patient has osteoporesis as defined by her abnormally low T score Diseases associsted with osteoporosis are Report An Error - Vitamin D deficiency - Thyrotozicosis - Myeloma and - Hypegonadises “Myeloma and lymphoma are also associated with reduced BMD. Other associates include. ~ Rheumatoid arthritis ~ Renal failure ~ Corticosteroids - Early menopause - Slender habitus - Smoking ~ Lack of exercise - Family history ~ AgelSex and - Excess deohol RERPERERE SRE DER EEE SP ee eee XxXxXXXKXXKXKXXKKXKXKXKKXKXXKXKXKXXKKXKXKXKXKXXK ‘Mark this queston & => Question Td : 54971 Question 10 of 30 A. 53-year-old man with chronic alcokol abuse has gouty tophi, He is commenced on allopurinol but develops severe joint pains two days later. On examination he has a temperature of 39°C, and erythematous sweling ofhis hands, kaees and ankles. Investigations reves: irate 0.55 mmovf.||@.23-0.45)| [C-reactive protein[150 mg/l. |[€10) “Which of the following is the meat Hkely cause for his presentation? a) Acute pyrophosphate arthropathy ') Acute rheumatoid arthritis ©) Alopurinol allergy 4) Alcohol ingestion ¢) Treatment with allopurinol Question Explanation ‘This man is having an acute attack of gout following the initiation of antihyperuricemic (allopurinol) therapy. This can be minimized by not starting allopurinel immediately during or shortly after an acute aitacke of gout, abstaining from alcohol bingeing and gradually increasing the dose of allopurinol ‘Alcohol ingestion may also trigger an acute altack of gout, but in this case given the recent commencement of allopurinol, this is more likely to be the trigger. The most common feanures of allopurinol hypersensitviy are rash and fever. Joint sepsis affecting muliple Joints is unlikely. Acute pseudogout presents wth inflammaton of the large joints, the knee being most commonly affected. ‘Mark this queston & => Question Td : 54971 Question 10 of 30 A. 53-year-old man with chronic alcokol abuse has gouty tophi, He is commenced on allopurinol but develops severe joint pains two days later. On examination he has a temperature of 39°C, and erythematous sweling ofhis hands, kaees and ankles. Investigations reves: irate 0.55 mmovf.||@.23-0.45)| [C-reactive protein[150 mg/l. _|[(<10) ‘Which of the following is the most lcely cause for his presentation? a) Acute pyrophosphate arthropathy ') Acute rheumatoid arthritis c) Allopurinol allergy d) Alcohol ingestion Y © e) Treatment with allopurinol Question Explanation ‘This man is having an acute attack of gout following the initiation of antihyperuricemic (allopurinol) therapy. This can be minimized by not starting allopurinel immediately during or shortly after an acute aitacke of gout, abstaining from alcohol bingeing and gradually increasing the dose of allopurinol ‘Alcohol ingestion may also trigger an acute altack of gout, but in this case given the recent commencement of allopurinol, this is more likely to be the trigger. The most common feanures of allopurinol hypersensitviy are rash and fever. Joint sepsis affecting muliple Joints is unlikely. Acute pseudogout presents wth inflammaton of the large joints, the knee being most commonly affected. ‘Mack this question ez (Question Td : $9370 Question 11 of 30 ‘What would be radiographic feature of osteoarthritis of the knee joint? a) Marginal erosions 'b) Loss of articular cartilage with narrowing of the radiologic joint space c) Justa-articular osteopenia (demineralization) 4) Osteonecrosis (avascular necrosis) of the medial femoral condyle ) High riding patella (patella alta) Answer | beplanation Other User's Explanation Report An Error Question Explanation: Osteoarthstis (C.A) is a chronic arthropathy of an entire joint characterized by disruption and potential lose of joint cartilage along ‘with other joint changes, including bone hypertrophy (osteophyte formation). Symptoms include gradually developing pain aggravated, or triggered by activity, stiffness relieved < 30 min after activity, and occasional joint swelling. Diagnosis is confirmed by x-rays. (OA should be suspected in patients with gradual onset of symptoms and signs, particularly in older adults. IF OA is suspected, plain acreys should be obtained ofthe most syraptomnatic joints. X-rays generally reveal marginal osteophiytes, narrowing of the joint space, increased density of the subchondral bone, subchondral cyst formation, bony remodeling, and joint effusions. Standing x-rays of knees are more sensitive to joint space nacrowing, ‘Mack this question ez (Question Td : $9370 Question 11 of 30 ‘What would be radiographic feature of osteoarthritis of the knee joint? a) Marginal erosions Y © b) Less of articular cartilage with aarrowing of the radiologic joint space c) Justa-articular osteopenia (demineralization) 4) Osteonecrosis (avascular necrosis) of the medial femoral condyle ) High riding patella (patella alta) Answer | beplanation Other User's Explanation Report An Error Question Explanation: Osteoarthstis (C.A) is a chronic arthropathy of an entire joint characterized by disruption and potential lose of joint cartilage along ‘with other joint changes, including bone hypertrophy (osteophyte formation). Symptoms include gradually developing pain aggravated, or triggered by activity, stiffness relieved < 30 min after activity, and occasional joint swelling. Diagnosis is confirmed by x-rays. (OA should be suspected in patients with gradual onset of symptoms and signs, particularly in older adults. IF OA is suspected, plain acreys should be obtained ofthe most syraptomnatic joints. X-rays generally reveal marginal osteophiytes, narrowing of the joint space, increased density of the subchondral bone, subchondral cyst formation, bony remodeling, and joint effusions. Standing x-rays of knees are more sensitive to joint space nacrowing, 2/26/2014 1 PM. ‘Mark this question = => Question Id : 62264 Question 12 of 30 The appearance of pseudofractures on radiographs is associated with a) Osteomalacia, b) Osteoporosis, c) Paget's disease. 4) Osteopetrosis ©) Osteits fibrosa cystica. Arve UETNEEIR ote Users Explanation Report AnEvor Question Explanation: Pseudofractures, or Milkman’s lines, are areas of demineralzation seen in the bones in osteomalacia (softeniag of bone). They are not found in osteoporosis (reduction of bone mass), Paget's disease (increase in bone density), osteopetrosis, or osteitis fbrosa cystica The lastis a more severe form of bore resorption in which fbrosis occurs. 2/26/2014 1 PM. ‘Mark this question = => Question Id : 62264 Question 12 of 30 The appearance of pseudofractures on radiographs is associated with oY © a) Osteomalacia b) Osteoporosis, c) Paget's disease. 4) Osteopetrosis ©) Osteits fibrosa cystica. Arve UETNEEIR ote Users Explanation Report AnEvor Question Explanation: Pseudofractures, or Milkman’s lines, are areas of demineralzation seen in the bones in osteomalacia (softeniag of bone). They are not found in osteoporosis (reduction of bone mass), Paget's disease (increase in bone density), osteopetrosis, or osteitis fbrosa cystica The lastis a more severe form of bore resorption in which fbrosis occurs. 2/26/2014 12:54:09 PM ‘Mark this question & => Question Td ; 64438 Question 13 of 30 A.69 year old presents with an acutely red and swollen sight great toc without history of trauma The most usefil finding for making a diagnosis is this patient is which of the following? a) Persistent elevation of serum uric acid b) Good response to colchicines trial c) Radiograph showing joint erosion in the first metatarsophalangeal (MTE) joint 4d) An associated right ankle effision €) Painless elbow nodule Question Explanation: Gout is a disorder that results from deposits of sodiuma urate crystals, which accumula in the joints because ofhigh blood levels of uric acid (hyperuricemia), leading to attacks of painfil joint inflammation, Atacks of gout (acute gouty arthritis) can occur without warring They mey be triggered by an injury, surgery, consumption of large quantities of alcohol or purine rich food, fatigue, emotional stress, or ilmess. Typically, severe pain occurs suddenly in one or more joitts, often at night (probably because of the metabolic changes that occur when a person lies down, The pain becomes progressively worse and is offen excruciating, particularly when the jointis moved or touched. The joint becomes inflamed, Tt swells and feels warm, and the skin over the joint appears red or purplish, tight, and shiny A high uric acid level in the blood supports the diagnosis; however, this levelis often normal, especially during an acute attack, A blood test may show increased mumbers of white blood cells due to the inflammation caused by the urate crystals. The diagnosis is confirmed when needle shaped urate crystals are identified in a sample of a tophus or in joint ud removed (Joint aspiration) with a needle and viewed under a mecroscope with polanzed ight. 2/26/2014 12:54:09 PM ‘Mark this question & => Question Td ; 64438 Question 13 of 30 A.69 year old presests with an acutely red and swollen right great toc without history of trauma The most ucefil finding for making a diagnosis is this paticat is which of the following? Y © a) Persistent elevation of serum uric acid b) Good response to colchicines trial c) Radiograph showing joint erosion in the first metatarsophalangeal (MTE) joint 4) An ascociated right ankle effision ©) Painless elbow nodule Question Explanation: Gout is a disorder that results from deposits of sodiuma urate crystals, which accumula in the joints because ofhigh blood levels of uric acid (hyperuricemia), leading to attacks of painfil joint inflammation, Atacks of gout (acute gouty arthritis) can occur without warring They mey be triggered by an injury, surgery, consumption of large quantities of alcohol or purine rich food, fatigue, emotional stress, or ilmess. Typically, severe pain occurs suddenly in one or more joitts, often at night (probably because of the metabolic changes that occur when a person lies down, The pain becomes progressively worse and is offen excruciating, particularly when the jointis moved or touched. The joint becomes inflamed, Tt swells and feels warm, and the skin over the joint appears red or purplish, tight, and shiny A high uric acid level in the blood supports the diagnosis; however, this levelis often normal, especially during an acute attack, A blood test may show increased mumbers of white blood cells due to the inflammation caused by the urate crystals. The diagnosis is confirmed when needle shaped urate crystals are identified in a sample of a tophus or in joint ud removed (Joint aspiration) with a needle and viewed under a mecroscope with polanzed ight. ‘Marc this question & => Question Td : 67020 Question 14 of 30 A 36-year-old woman falls from a ladder and sustains a severe comminuted fracture of her distal humerus with intra-articular extension, Several days after surgery, she develops acute pain, swelling, and redness of the involved elbow, as well as fever and chills. The tceiest cause is a) A new fracture b) Osteomyelitis ©) Staphylococcal arthritis ) Meningococcal arthitis ©) Sporotrichosis Anewor ERNE) ote sere Explanation Report AnEror (Question Explanation: Staphylococcal arthntis would be the likeliest common cause of this patient's symptoms. A new fracture is unlikely and would not cause these symptoms, aor would Osteomyelitis which is a bone disorder and not a joint invader. Meningococcal arthritis or Sporotrichosis would be unlikely mn this setting, ‘Marc this question & => Question Td : 67020 Question 14 of 30 A 36-year-old woman falls from a ladder and sustains a severe comminuted fracture of her distal humerus with intra-articular extension, Several days after surgery, she develops acute pain, swelling, and redness of the involved elbow, as well as fever and chills. The tceiest cause is a) A new fracture b) Osteomyelitis VY © ©) Staphylococeal arthritis ) Meningococcal arthitis ©) Sporotrichosis Anewor ERNE) ote sere Explanation Report AnEror (Question Explanation: Staphylococcal arthntis would be the likeliest common cause of this patient's symptoms. A new fracture is unlikely and would not cause these symptoms, aor would Osteomyelitis which is a bone disorder and not a joint invader. Meningococcal arthritis or Sporotrichosis would be unlikely mn this setting, 2/26/2014 12:54:32 PM ‘Marke this question & => Question Id : 76226 Question 15 of 30 ‘Which of the following statements is NOT tue? a) Poeudogout can be accociated with diabetes. b) Deposition of calcium pyrophosphate dehydrogenase crystals is responsible for pseudogout c) Treatment of choice is Xanthine Ozadase inhibitors. 4) The crystals seen in the synovial uid in pseudogout are weakly positively bireftingent. ¢) Radiographic findings of pseudogout are commonly seen in the fibrocarilage. Question Explanation: “Allopurinel (Xanthine Oxidase inkibitor) does not benefit patients with pseudogout, which is due to deposition of ealeiam pyrophosphate dehydrogenase. Allopurmol prevents formation ofuric acid, which is not involved in this condition. The other four answers are correct. 2/26/2014 12:54:32 PM ‘Marke this question & => Question Id : 76226 Question 15 of 30 “Which of the following statements is NOT true? a) Poeudogout can be accociated with diabetes. b) Deposition of calcium pyrophosphate dehydrogenase crystals is responsible for pseudogout Y © c) Treatment of choice is Xanthine Oxdase inhibitors. 4) The crystals seen in the synovial uid in pseudogout are weakly positively bireftingent. ¢) Radiographic findings of pseudogout are commonly seen in the fibrocarilage. Question Explanation: “Allopurinel (Xanthine Oxidase inkibitor) does not benefit patients with pseudogout, which is due to deposition of ealeiam pyrophosphate dehydrogenase. Allopurmol prevents formation ofuric acid, which is not involved in this condition. The other four answers are correct. 2/26/2014 12:54:43 PM ‘Mark this question e& => Question Td : 89649 Question 16 of 30 A.54 year old woman develops symptoms consistent with left knee arthritis, Physical examination demonstrates a large effusion over the joint, which is aspirated for analysis. Finid enalysis reveals needle shaped negatively birefringent crystals, Which ofthe folowing is the most likely diagnosis? a) Gout. b) Pscudegout. c) Apatite disease. 4) Rheumatoid arthritis ©) None of the above. Answer [ Barisnaton | Other User's Explanation Report An Error Question Explanation: Gout can be diagnosed if intracellular monosodium urate crystals are seen in the aspirate of joint uid. These crystals are negatively birefiingent and needle shaped. Pseudogout or calcium pyrophosphate dihydrate (CPPD) crystals, are positively birefiingent and of varying shape, possibly thomboid, Apatice disease refers to the deposition of hydroxyapatite complexes, which can be detected only by electron microscopy. Rheumatoid arthrits does net produce a crystal induced synovits 2/26/2014 12:54:43 PM ‘Mark this question e& => Question Td : 89649 Question 16 of 30 A.54 year old woman develops symptoms consistent with left knee arthritis, Physical examination demonstrates a large effusion over the joint, which is aspirated for analysis. Finid enalysis reveals needle shaped negatively birefringent crystals, Which ofthe folowing is the most likely diagnosis? Y © a) Gout 'b) Pscudogout, c) Apatite disease. 4) Rheumatoid arthritis e) None of the above, Answer [ Barisnaton | Other User's Explanation Report An Error Question Explanation: Gout can be diagnosed if intracellular monosodium urate crystals are seen in the aspirate of joint uid. These crystals are negatively birefiingent and needle shaped. Pseudogout or calcium pyrophosphate dihydrate (CPPD) crystals, are positively birefiingent and of varying shape, possibly thomboid, Apatice disease refers to the deposition of hydroxyapatite complexes, which can be detected only by electron microscopy. Rheumatoid arthrits does net produce a crystal induced synovits ‘Mark this question <= => Question Id : 90917 Question 17 of 30 “What is the most common cause of septic arthritis in adults? a) Haemophilus influenzae b) Haemophilus ducreyi c) Streptococcus viridans 4) Escherickia. €) Staphylococcus aureus Answer (Bipianation | Other User's Explanation Report An Error Question Explanation: Staphylococcus aureus is the most common cause of septic arthritis in adults ‘Mark this question <= => Question Id : 90917 Question 17 of 30 “What is the most common cause of septic arthritis in adults? a) Haemophiius influenzae b) Hasmophilus ducreyi c) Streptococcus viridans ) Escherichia. © ) Staphylococcus aureus. Answer (Bipianation | Other User's Explanation Report An Error Question Explanation: Staphylococcus aureus is the most common cause of septic arthritis in adults ‘Maric this question = => Question Td: 91861 Question 18 of 30 A55 year old female wakes up one morning with a painful warm pinna and episclerits. She has a remote history of inflammation of her nose that was attributed to trauma, She is at increased risk for which one of the following? a) Tracheobronchial nflammation, b) Lung cancer, ©) Ottis media d) Simsiis €) Temporal arteritis. (Question Explanation: This patient has symptcms characteristic of relapsing polychondritis, an idiopathic inflammation of elastic cartilage. Tracheobronchial inflammation with the dreaded prospect of tracheomalacia and respiratory compromise is a potential outcome of relapsing polychondritis. Lang cencer (B), ottis media (C), sinusitis (D), and temporal artentis (E) are not associated with this condition. ‘Maric this question = => Question Td: 91861 Question 18 of 30 A.55 year old female wakes up one morning with a painful warm pinna and episcleritis. She has a remote history of inflammation of her nose that was attributed to trauma. She is at increased risk for which one of the following? Y © a)Tracheobronchial nflammation. 'b) Lung cancer. ©) Otitis media d) Sinusitis. e) Temporal arteritis. (Question Explanation: This patient has symptcms characteristic of relapsing polychondritis, an idiopathic inflammation of elastic cartilage. Tracheobronchial inflammation with the dreaded prospect of tracheomalacia and respiratory compromise is a potential outcome of relapsing polychondritis. Lang cencer (B), ottis media (C), sinusitis (D), and temporal artentis (E) are not associated with this condition. “Mark this question exo Question 1d: 92721 Question 19 of 30 ‘A.33 year old woman presents with tender fingertips upen exposure to the cold, arthralgias, dificity swallowing, and a blanching, erythematous rash on the skin, On examination she has hypertension and fibrosis of the skin, Investigations would most likely show vwihich one of the following? a) Anti-Ro antibody b) Anti-smooth muscle antibody ¢) Anti-ribomcleoprotein antibody 4) Anti-centromere antibody ©) Rheumatoid factor Answer | Explanation Other User's Explanation Report An Error Question Explanation: Anti-centromere antibody is associated with CREST syndrome, which is part of the scleroderma syndrome. This is a connective tissue disorder associated with calcinosis, Raynaud's discase, esophageal dysmotiity, scleroderma, and ielangicctasis. Anti-Ro antibody is associated with Sjogren's syndrome, Anti-smooth muscle antibody is associated with autoimmune chronic active hepatitis, This is common in young women, Anti-rbonucleoprotein antibody is associated with mixed cornective tissue disease, Rheumatoid factor is an IgM protein which is present in cheumatoid arthuitis “Mark this question exo Question 1d: 92721 Question 19 of 30 ‘A.33 year old woman presents with tender fingertips upen exposure to the cold, arthralgias, dificity swallowing, and a blanching, erythematous rash on the skin, On examination she has hypertension and fibrosis of the skin, Investigations would most likely show vwihich one of the following? a) Anti-Ro antibody 'b) Anti-smooth muscle antibody ¢) Anti-ribomcleoprotein antibody Y¥ © 4) Anti-centromere antibody e) Rheumatoid factor Answer | Explanation Other User's Explanation Report An Error Question Explanation: Anti-centromere antibody is associated with CREST syndrome, which is part of the scleroderma syndrome. This is a connective tissue disorder associated with calcinosis, Raynaud's discase, esophageal dysmotiity, scleroderma, and ielangicctasis. Anti-Ro antibody is associated with Sjogren's syndrome, Anti-smooth muscle antibody is associated with autoimmune chronic active hepatitis, This is common in young women, Anti-rbonucleoprotein antibody is associated with mixed cornective tissue disease, Rheumatoid factor is an IgM protein which is present in cheumatoid arthuitis ‘Mark this question =z Question 1d: 96155 Question 20 of 30 A woman aged 42 years presents with a history of white cyanotic appearing digis when she is exposed to cold which is promptly relieved by heat with a resultant hyperemia Her pulses are normnal and she has chronic nail and skin changes. This clinical condition is known as a) Caepal tunnel syndrome b) Peripheral vascular disease. ¢) Thromboangitis obliterans ) Raynaud's phenomenon, €) Peripheral embolic syndrome. Question Explanation: This is a classic description of Raynaud's phenomenon, which is a result of digital vasospasm upon exposure to cold. I occurs from a variety of causes. Carpal tunnel syndrome is in the cifferential diagnosis, but the symptoms of nerve compression in this syndrome are usually not related to temperanure, Peripheral vascular disease that is symptomatic in the digits is rare. Thromboangitis obiterans is a result of intimal prolferation in small to medium vessels that is associated with Raynaud's phenomenon, but is not responsible for the symptoms described in this patieat. It usually occurs in mea less than 30, ‘Mark this question =z Question 1d: 96155 Question 20 of 30 A.woman aged 42 years presents with a history of white cyanotic appearing digits when she is exposed to cold which is promptly relieved by heat with a resultant hyperemia Her pulses are normal and she has chronic mail and skin changes. This linival condition is known as a) Carpal tunnel syndrome ) Peripheral vascular disease ©) Throrehoangitis obliterans Y © d) Raynaud's phenomenon, ©) Peripheral embolic syndrome Question Explanation: This is a classic description of Raynaud's phenomenon, which is a result of digital vasospasm upon exposure to cold. I occurs from a variety of causes. Carpal tunnel syndrome is in the cifferential diagnosis, but the symptoms of nerve compression in this syndrome are usually not related to temperanure, Peripheral vascular disease that is symptomatic in the digits is rare. Thromboangitis obiterans is a result of intimal prolferation in small to medium vessels that is associated with Raynaud's phenomenon, but is not responsible for the symptoms described in this patieat. It usually occurs in mea less than 30, 2/26/2014 12:55:37 PM ‘Mark this question eqrz Question Td : 96580 Question 21 of 30 A.35 year old woman presents with an actte exanthem and a "strawberry tongue." The most likely diagnosis is a) Measles >) Erythema infectiosum. c) Rocky Mountain spotted fever @) Kawasaki's disease 2) Meningococcemia Question Explanation: The finding of a strawberry tongue is secn in such acute cxanthems as Kawasaki's disease and scaulet fever. 2/26/2014 12:55:37 PM ‘Mark this question eqrz Question Td : 96580 Question 21 of 30 A.35 year old woman presents with an acute exanthem and a "strawberry tongue." The most likely diagnosis is a) Measles >) Erythema infectiosum c) Rocky Mountain spotted fever Y © 4) Kawasalc's disease 2) Meningococcemia Question Explanation: The finding of a strawberry tongue is secn in such acute cxanthems as Kawasaki's disease and scaulet fever. 2/26/2014 12: 1 PH ‘Mark this question & => Question Td : 96873 Question 22 of 30 All of the following have a role in the management of acute gout, EXCEPT a) Allepurinal b) Corticosteroids ©) Colchicine ) Indomethacin €) Ibuprofen Avewor (UEQNBSHAN) other sors Explanation Repert An Error (Question Explanation: LAllopurinolis not effective in the management of acute gouty arthritis. Colchicine is the most effective agent and corticosteroids and nonsteroidal anti-inflammatory drugs such as indomethacin and ibuprofen are usefil adjuvants to colchicine 2/26/2014 12: 1 PH ‘Mark this question & => Question Td : 96873 Question 22 of 30 All of the following have a role in the management of acute gout, EXCEPT Y © a) Allopusind b) Corticosteroids ©) Colchicine ) Indomethacin €) Ibuprofen Avewor (UEQNBSHAN) other sors Explanation Repert An Error (Question Explanation: LAllopurinolis not effective in the management of acute gouty arthritis. Colchicine is the most effective agent and corticosteroids and nonsteroidal anti-inflammatory drugs such as indomethacin and ibuprofen are usefil adjuvants to colchicine 2/26/2014 12:56:03 PM ‘Mark this question = => Question Id: 97445 Question 23 of 30 ‘Drug that can be administered to pregnant women with active rheumatoid arthritis with relative safety if necessary is which of the following? 2) Pericillamine b) Gold c) Methotrexate 4) Prednisone @) Azathioprine Aveo: EEIEEIRIY ote: users explanation Report An Ever Question Explanation: Prednisone is relatively safe when necessary for the management of a flare up of rheumatoid arthritis during pregnancy. The cther agents can be toxic or teratogenic to the fetus. 2/26/2014 12:56:03 PM ‘Mark this question = => Question Id: 97445 Question 23 of 30 ‘Drug that can be administered to pregnant women with active rheumatoid arthritis with relative safety if necessary is which of the following? 2) Pericillamine b) Gold c) Methotrexate Y © 4) Prednisone @) Azathioprine Aveo: EEIEEIRIY ote: users explanation Report An Ever Question Explanation: Prednisone is relatively safe when necessary for the management of a flare up of rheumatoid arthritis during pregnancy. The cther agents can be toxic or teratogenic to the fetus. ‘Mark this question = => 103488 Question 24 of 30 Agent that has NOT been found to be usefil in the treatment of osteoporosis is a) Calcium ») Vitarnia D «) Viterain E a) Calcitonin 2) Estrogen Question Explanation: ‘Vitamin E has no known role in the management of esteoporesis. Calcium, vitamin D, calcitonin, and estrogen are all modalites of therapy mn osteoporosis. ‘Mark this question = => 103488 Question 24 of 30 Agent that has NOT been found to be usefil in the treatment of osteoporosis is a) Calcium ») Vitarnia D JY © c) Vitamin E a) Calcitonin 2) Estrogen Question Explanation: ‘Vitamin E has no known role in the management of esteoporesis. Calcium, vitamin D, calcitonin, and estrogen are all modalites of therapy mn osteoporosis. ‘Mark this question ==> Question 25 of 30 A.37 year cld woman has symptoms of dry eyes, dry mouth, and difficulty swallowing. Symptoms of vasculiis, nephritis, and polyneuropathy are also present. The serologic test of choice to make the diagnosis is a) Ant-Rofanti-La antbody b) Ant-nuclear antibody c) Anti-double-stranded DNA antibody 4) Ant-neutrophilic cytoplasmic antibody ¢) Ant-mitochondrial antibody Avowor (ENGIN tne veers txptanaion Report An Ever Question Explanation: This patient has Sjogren's syndrome, which is an autoimmune disease characterized by progressive destruction of exocrine glands Jeading to mucosal and conjunctival dynes. Congenital cardiac conduction defects are seen in fetuses of women with anti-Ro antibody (SSA). Xeroxtomia causes dress of the mouth and, this, difficulty swallowing Anti-miclear antibody (ANA) is very nonspecific and can occur mainly in connective tiseue disorders Anti-double-stranded DNA antibody is specific for lupus patients with renal disease. Asti-acutrophillc cytoplasmic antibody is seen in Wegener's granulomatosis, which is a necrotizing vasculitis, but this antibody is not specific for the disorder. Anti-mnitochonctial antibody is seen mainly in middle aged women with primery biliary cirrhosis. ‘Mark this question ==> Question 25 of 30 A.37 year cld woman has symptoms of dry eyes, dry mouth, and difficulty swallowing. Symptoms of vasculiis, nephritis, and polyneuropathy are also present. The serologic test of choice to make the diagnosis is Y © a) Ant-Rofanti-La antbody 6) Ant-nuclear antibody c) Anti-double-stranded DNA antibody 4) Antineutrophilic cytoplasmic antibody ¢) Ant-mtochondrial antibody Avowor (ENGIN tne veers txptanaion Report An Ever Question Explanation: This patient has Sjogren's syndrome, which is an autoimmune disease characterized by progressive destruction of exocrine glands Jeading to mucosal and conjunctival dynes. Congenital cardiac conduction defects are seen in fetuses of women with anti-Ro antibody (SSA). Xeroxtomia causes dress of the mouth and, this, difficulty swallowing Anti-miclear antibody (ANA) is very nonspecific and can occur mainly in connective tiseue disorders Anti-double-stranded DNA antibody is specific for lupus patients with renal disease. Asti-acutrophillc cytoplasmic antibody is seen in Wegener's granulomatosis, which is a necrotizing vasculitis, but this antibody is not specific for the disorder. Anti-mnitochonctial antibody is seen mainly in middle aged women with primery biliary cirrhosis. ‘Mark this question Ps] => Question Id: 119818 Question 26 of 30 A.35 year cld woman has recent onset of systemic Inpus erythematosus. She hes polyarthralgias, episodic mild skin rashes, and occasional pleurisy. What is the best inital treatment? a) High dose oral stercids b) Intravenous steroids. c) Hyéroxychloroquine. 4) Methowrexate 2) Cyclosporine. Answer | Boanation Other User's Explanation Report An Error Question Explanation: The antimalarial, bydroxychloroqune, is particularly usefil in patients with SLE who have joint pain, some skin involvement, and mid pleurisy. High dose oral steroids, intravenous steroids, and cyclosporine should be reserved for patients with very serious, potentially life threctening disease. Methotrexate is usually given to patients with more severe manifestations than the ones described in tis example, ‘Mark this question Ps] => Question Id: 119818 Question 26 of 30 A.35 year cld woman has recent onset of systemic Inpus erythematosus. She hes polyarthralgias, episodic mild skin rashes, and occasional pleurisy. What is the best inital treatment? a) High dose oral stercids b) Intravenous steroids. Y © 2) Hydrozychloroquine. 4) Methowrexate 2) Cyclosporine Answer | Boanation Other User's Explanation Report An Error Question Explanation: The antimalarial, bydroxychloroqune, is particularly usefil in patients with SLE who have joint pain, some skin involvement, and mid pleurisy. High dose oral steroids, intravenous steroids, and cyclosporine should be reserved for patients with very serious, potentially life threctening disease. Methotrexate is usually given to patients with more severe manifestations than the ones described in tis example, 2/26/2018 12:57:07 PM ‘Mark this question = => f Question 27 of 30 All of the following conditions typically alfect the lumbar spine, EXCEPT a) Ankylosing spondylitis b) Osteoarthuitis ©) Psoriatic arthritis 4) Forestier’s disease @) Rheumatoid arthuitis Question Explanation: Rheumatoid arhriis rarely affects the lumbar spine. Ankylosing spondylitis typically causes vertebral body squaring and ligamentous calcifications. Osteoarthritis commonly involves the lumbosacral spine Psoriatic arthritis can produce an ankylosing spondylitis lice syndrome in the spine. Forestier’s disease, or diffuse idiopathy spinal hyperostosis (DISH), characteristically involves the lumbar spine 2/26/2018 12:57:07 PM ‘Mark this question = => f Question 27 of 30 ‘All of the following conditions typically alec the lumbar spine, EXCEPT a) Ankylosing spondyitis ) Osteoarthtis ©) Psoriatic arthritis 4) Forzstiee's disease Y © 2) Rheumatoid arthritis Question Explanation: Rheumatoid arhriis rarely affects the lumbar spine. Ankylosing spondylitis typically causes vertebral body squaring and ligamentous calcifications. Osteoarthritis commonly involves the lumbosacral spine Psoriatic arthritis can produce an ankylosing spondylitis lice syndrome in the spine. Forestier’s disease, or diffuse idiopathy spinal hyperostosis (DISH), characteristically involves the lumbar spine 2/26/2014 12:57:19 PM ‘Matte this question & => Question 28 of 30 Symptom thatis more common in individuals with inflammatory arthritis of the back than in people with mechamucal low back pain is which one of the following? a) Moming stiffiess 'b) Radiation of pain te the leg c) Numbness of the toes d) History of trauma ¢) Absence of family history Answer | Explanation Other User's Explanation Report An Error Question Explanation: Thifernmatory back arthvitides, such as ankylosing spondylitis and the other seronegative spondsloarthvitides, are manifested by typical features of inflammatory arthritis, such as morning stiffness, gelling, elevated sedimentation rate, etc. Radiation of pain to the leg or numbness of the toes would be associated with a radiculopathy due to nerve root compression and are more common ina mechatical back syndrome, such as degenerative disc disease, lumbosacral strain, or a herniated disc. History of trauma is more common in a mechanical back syndrome. Individuals with inflammatory arthrtides are more likely to have positive family histories of inflammatory arthritis of the back or other joints, since some of these conditons do appear to have a hereditary component. 2/26/2014 12:57:19 PM ‘Matte this question & => Question 28 of 30 ‘Symptom that is more common in individuals with inflammatory arthritis of the back than in people with mechancal low back pain is which one of the following? Y © a) Moming stiffness b) Radiation of pain to the leg c) Numbness of the toes 4) History of trauma e) Absence of family history Answer | Explanation Other User's Explanation Report An Error Question Explanation: Thifernmatory back arthvitides, such as ankylosing spondylitis and the other seronegative spondsloarthvitides, are manifested by typical features of inflammatory arthritis, such as morning stiffness, gelling, elevated sedimentation rate, etc. Radiation of pain to the leg or numbness of the toes would be associated with a radiculopathy due to nerve root compression and are more common ina mechatical back syndrome, such as degenerative disc disease, lumbosacral strain, or a herniated disc. History of trauma is more common in a mechanical back syndrome. Individuals with inflammatory arthrtides are more likely to have positive family histories of inflammatory arthritis of the back or other joints, since some of these conditons do appear to have a hereditary component. ‘Mark this question <=> Question I Question 29 of 30 A condition that is usually NOT associated with an elevated creatine phosphokinase level is which one of the following? a) Acute myocardial infarction bv) Dermatomyositis, ) Renal failure 4) Acute tranma ¢) Polymyalgia rheumatica Question Explanation: Polymyalgia rheumatica typically produces high sedimentation rates, but CPK remains normal, Acute myocardial infarction, dermatomyositis, renal falure, and acute trauma can all cause elevated CPK levels ‘Mark this question <=> Question I Question 29 of 30 A condition that is usually NOT associated with an elevated creatine phosphokinase level is which one of the following? 2) Acute myocardial infarction ») Dermatomyositis ) Renal failure & Actte trauma Y © ©) Polymyalgia rheumatica Question Explanation: Polymyalgia rheumatica typically produces high sedimentation rates, but CPK remains normal, Acute myocardial infarction, dermatomyositis, renal falure, and acute trauma can all cause elevated CPK levels 2/26/2014 12:57:42 PM ‘Mark this question & Question Id : 132337 Question 30 of 30 A.45 year old dialysis patient presents with arthritis of the distal and proximal interphalangeal joints of the hands that is secondary to the deposition of pyramidal shaped postively birefringent crystals. After NSAID therapy he recovers without complication. The most likely diggnosis is a) Gouty arthritis b) Pseudogout c) Hydroxyapatite arthropatay 4) Calcium cxzalate induced arthritis 2) Calcium pyrophosphate deposition disease Question Explanation: Calcium oxalate induced arthritis typically presents with an arthritis of the DIP and PIP joints, Itis most commonly seen in dialysis patients who are ingesting large amounts of ascorbic acid. Ascorbic acid is metabolized to calcium oxalate, which then deposits in the stnall joints of the hand, Gouty arthritis typically presents with an inital arthritis of the metacarsophalangeal or ankle joints Pseudogout, ot calcium pyrophosphate deposition disease, is manifested initially with arthntis of the knees and wrists. Hydroxyapatite arthropzthy presents initially wath arthrits of the knees, shoulders, or hips. Report An Error 2/26/2014 12:57:42 PM ‘Mark this question & Question Id : 132337 Question 30 of 30 A.45 year old dialysis patient presents with arthritis of the distal and proximal interphalangeal joints of the hands that is secondary to the deposition of pyramidal shaped postively birefringent crystals. After NSAID therapy he recovers without complication. The most likely diggnosis is a) Gouty arthritis b) Pseudogout c) Hydroxyapatite arthropatay Y © @ Caleium oxalate induced arthritis 2) Caleium pyrophosphate deposition disease Question Explanation: Calcium oxalate induced arthritis typically presents with an arthritis of the DIP and PIP joints, Itis most commonly seen in dialysis patients who are ingesting large amounts of ascorbic acid. Ascorbic acid is metabolized to calcium oxalate, which then deposits in the stnall joints of the hand, Gouty arthritis typically presents with an inital arthritis of the metacarsophalangeal or ankle joints Pseudogout, ot calcium pyrophosphate deposition disease, is manifested initially with arthntis of the knees and wrists. Hydroxyapatite arthropzthy presents initially wath arthrits of the knees, shoulders, or hips. Report An Error ‘Mark this question => (Question Id : 10773 Question 1 of 30 A.69-year-old stroker and drinker has a two months history of dificulty raising his arms, ascending stairs, and a dry mouth, He has proximal weakness of all four linbs with absent tendon reflexes. CCR shows a right pleural effusion, What is the most icely diagnosis? a) Alcohel induced myopathy b) Myasthenia gravis ©) Polymnyalgia theumnatice 4) Eaton- Lambert syndrome ©) Polymyostis Answer | Exrianaton | Other User's Explanation Report An Error Question Explanation: Eaton - Lambert Syndrome is characterized by proximal muscle weakness (the craniel nerves and respiratory muscles are usually spared), depressed or absent tendon reflexes and autonomic features (¢.g. dry mouth, impotence etc), 70% of cases arc due to small cell lang cancer. Unlike myasthenia gravis exercise is associcted with increasing muscle strength and there is a negative response to Tensilon. Electromyography is useful in confirring the diagnosis where repeated nerve stinulations cause progressive increases in the size of the muscle action potental ‘Mark this question => (Question Id : 10773 Question 1 of 30 A.69-year-old stroker and drinker has a two months history of dificulty raising his arms, ascending stairs, and a dry mouth, He has proximal weakness of all four linbs with absent tendon reflexes. CCR shows a right pleural effusion, What is the most icely diagnosis? a) Alcohel induced myopathy b) Myasthenia gravis ©) Polymnyalgia theumnatice Y © 4) Baton- Lambert syndrome ©) Polymyostis Answer | Exrianaton | Other User's Explanation Report An Error Question Explanation: Eaton - Lambert Syndrome is characterized by proximal muscle weakness (the craniel nerves and respiratory muscles are usually spared), depressed or absent tendon reflexes and autonomic features (¢.g. dry mouth, impotence etc), 70% of cases arc due to small cell lang cancer. Unlike myasthenia gravis exercise is associcted with increasing muscle strength and there is a negative response to Tensilon. Electromyography is useful in confirring the diagnosis where repeated nerve stinulations cause progressive increases in the size of the muscle action potental 2/26/2014 1 PM. ‘Mar this question e => Question Td : 28262 Question 2 of 30 ‘Which of the following most accurately describes the mechanism of action of the bisphosphonates? a) Inhbition of osteoclast activity ) Acts through PTH receptors ©) Acts through calcitonin receptors ) Activates the calcium-sensing receptor ©) Upregulation of osteoblast activity (Question Explanation: The mechanistn of action of alendronate and risedronate, as well as newer agents, invoives the inhibition of the pathway that leads to the production of certain essential ipid compcunds inside osteoclasts, resulting ina series of events leading to decreased osteoclast activty and to the induction of cell death, 2/26/2014 1 PM. ‘Mar this question e => Question Td : 28262 Question 2 of 30 ‘Which of the following most accurately describes the mechanism of action of the bisphosphonates? Y¥ © a) Inhbition of osteoclast activity ') Acts through PTH receptors ©) Acts through calcitonin receptors d) Activates the calcium-sensing receptor €) Upregulation of osteoblast activity (Question Explanation: The mechanistn of action of alendronate and risedronate, as well as newer agents, invoives the inhibition of the pathway that leads to the production of certain essential ipid compcunds inside osteoclasts, resulting ina series of events leading to decreased osteoclast activty and to the induction of cell death, 2/26/2014 12:58:59 PM ‘Mark this question ez Question Td Question 3 of 30 49-year-old undergoes bone densitometty which shows bone mass decreased more than 2 standard deviations below the mean for her age in her left femoral head, wrist, and lumbar veriebral region. Six months later, the amount of bone lossiis seen to be increased by repeat densitometry examination, Which serum laboratory test abnormally is associated? a) Intact parathormone of 5 pmolll (1.2 -5.8) +b) Cortisol of 2060 mmol (110 -607) ©) Total serum globulin of 35 g ) Uric acid of 930 pmolt (149-446) €) Total cholesterol of 10 mmolf (<5.17) Question Explanation: She has osteoporosis with decreased bone mass. Most cases do not have a specific etiology, but Cushing’s syndrome with hypercortisolism can promote osteoporosis. Her age should make you suspicious. “Hypoparathyroidism is not going to accelerate bone loss. The bone resorption that accompanies hyperparathyroidism can cause osteoporosis. Over 25% of cases of osteoporosis are ‘primary’ with unknown cause Elevated serum globulin should make you suspect a monoclonal gammopathy, but myeloma leads to focal bone lytic lesions, ‘Hyperuricemia can be associated with gout that can cause focal bone destruction near affected joints, the bone mass overall is not decreased. 2/26/2014 12:58:59 PM ‘Mark this question ez Question Td Question 3 of 30 49-year-old undergoes bone densitometty which shows bone mass decreased more than 2 standard deviations below the mean for her age in her left femoral head, wrist, and lumbar veriebral region. Six months later, the amount of bone lossiis seen to be increased by repeat densitometry examination, Which serum laboratory test abnormally is associated? a) Intact parathormone of 5 pmolll (1.2 -5.8) Y © b) Cortisol of 2060 mmoll (110 -607) ©) Total serum globulin of 35 g ) Uric acid of 930 pmolt (149-446) €) Total cholesterol of 10 mmolf (<5.17) Question Explanation: She has osteoporosis with decreased bone mass. Most cases do not have a specific etiology, but Cushing’s syndrome with hypercortisolism can promote osteoporosis. Her age should make you suspicious. “Hypoparathyroidism is not going to accelerate bone loss. The bone resorption that accompanies hyperparathyroidism can cause osteoporosis. Over 25% of cases of osteoporosis are ‘primary’ with unknown cause Elevated serum globulin should make you suspect a monoclonal gammopathy, but myeloma leads to focal bone lytic lesions, ‘Hyperuricemia can be associated with gout that can cause focal bone destruction near affected joints, the bone mass overall is not decreased. ‘Mark this question = => (Question Id : 30418 Question 4 of 30 A44 year old femnale notices that she develops tingling and numbness over the palmar surface of her thumb, index, and middie fingers afer working several hours at her computer workstation making a document, She also reports that pain in the same area often occurs at night as well Her symptoms account for which pathological finding? a) Gout ) Hypertrophic osteoarthropathy c) Rheumatoid cethritis 4) Toxic peripheral neuropathy ¢) Localized tenosynovitis Anewor ERPNGEENI tne ueore Explanation Report An Ever Question Explanation: She has carpal tunnel syndrome an entrapment neuropathy of medan nerve. In this lady, tenosynouitis is worsened by repetitive motion ie. repetitive strain injury. ‘Mark this question = => (Question Id : 30418 Question 4 of 30 4.44 year old female notices that she develops tingling and numbness over the palmar surface of her thumb, index, and middle fingers after working several hours at her computer workstation making a document. She also reports that pain in the same area often occurs at night as well Her symptoms account for which pathological finding? a) Gout ) Hypertrophic osteoarthropathy ) Pheumatoid arthritis d) Toxic peripheral neuropathy Y © 2) Localized tenosynoritis Anewor ERPNGEENI tne ueore Explanation Report An Ever Question Explanation: She has carpal tunnel syndrome an entrapment neuropathy of medan nerve. In this lady, tenosynouitis is worsened by repetitive motion ie. repetitive strain injury. ‘Mark this question => Question Id : 30440 Question 5 of 30 A man in his 20's begins to note lower back oain and stiffness thatis persistent and diminishes with activity. During his 30's he also develops shoulder and hip arthritis, and in his 40's he is troubled by decreased mebilty of lumbar spine, No other medical problems are present These findings typically point to which of the following? a) Calcium pyrophosphate detydrate deposition disease b) Lyme disease ©) Ankylosing spondylitis 4d) Osteoarthritis €) Rheumatoid arthrits Question Explanation: He probably is also HLA B27 postive. The earlier in life the disease begins, the worse the prognosis There is a progressive bony ankylosis, especially of spine. RA typically involves small joints. Osteoarthritis typically involves a single large joint. Osteoarthtis typically involves a single large joint. Calcium pyrophosphate ditydrate deposition disease (Pseudogout) ismoretypical of the elderly and occurs in acute attacks ‘Mark this question => Question Id : 30440 Question 5 of 30 A man in his 20's begins to note lower back oain and stiffness thatis persistent and diminishes with activity. During his 30's he also develops shoulder and hip artarits, and in his 40's he is troubled by decreased mobility of lumbar spine, No other medical problems are present These findings typically point to which of the following? a) Calcium pyrophosphate detydrate deposition disease ) Lyme disease Y © ©) Ankylosing spondylitis 4) Osteoarthritis ¢) Rheumatoid arthritis Question Explanation: He probably is also HLA B27 postive. The earlier in life the disease begins, the worse the prognosis There is a progressive bony ankylosis, especially of spine. RA typically involves small joints. Osteoarthritis typically involves a single large joint. Osteoarthtis typically involves a single large joint. Calcium pyrophosphate ditydrate deposition disease (Pseudogout) ismoretypical of the elderly and occurs in acute attacks ‘Mark this question <=> Question 1d 30569 Question 6 of 30 Amen aged 71 years underwent an elective surgery for inguinal hernia, He developed acute monoarthitis of his right ankle on the second postoperative day. He was using a diuretic for hypertension, His temperature was 38°C on exemnination, Which one of the following is the most lely diagnosis? a) Acute rheumatoid arthritis b) Pseudogout ©) Septic arthritis ) Gout ¢) Traumatic synovitis (Question Explanation: The most likely diagnosis is gout, given the history of recent surgery and dinretic use. Pyrophoshate arthropatiy is less common, assoriated with deposition of pyrophosphate chiefly in the knees, second and third metacerpophalangeal joints and there may be a history of haemochrometosis. Rheumatoid arthritis most commonly matifests as a chronic polyarthritis and synovitis Septicaemia following an elective hemia repair would be uncommon as would traumatic smovitis. Why the fever you may ask? Gout is an inilammatory process and this is whet causes the fever. Fever may even be the most prominent feature of an attack of gout (Le. gout may be a cause of fever of unknown origin- it would suggest an inadequate history and examination of a patient with fever had been taken as well of coursel). Recent Prog Med. 1998 jan:89 (1):30-6. Acute gout is a cause of Systemic inflammatory response syndrome (SIRS) which is 2 or more changes of body temperature, heart rate, respratory fimction and peripheral leukocyte count (emedicine "Shock, sptic") ‘Mark this question <=> Question 1d 30569 Question 6 of 30 Aman aged 71 years underwent an elective surgery for inguinal hernia. He developed acute monoanthuitis of his right ankle oa the second postoperative day. He wes using a diuretic for hypertension, His temperature was 38°C on examination, Which one of the following is the most likely diagnosis? a) Acute cheamatoid arthritis ) Pseudogout ©) Septic arthritis Y © d Gout ¢) Traumatic synovitis (Question Explanation: The most likely diagnosis is gout, given the history of recent surgery and dinretic use. Pyrophoshate arthropatiy is less common, assoriated with deposition of pyrophosphate chiefly in the knees, second and third metacerpophalangeal joints and there may be a history of haemochrometosis. Rheumatoid arthritis most commonly matifests as a chronic polyarthritis and synovitis Septicaemia following an elective hemia repair would be uncommon as would traumatic smovitis. Why the fever you may ask? Gout is an inilammatory process and this is whet causes the fever. Fever may even be the most prominent feature of an attack of gout (Le. gout may be a cause of fever of unknown origin- it would suggest an inadequate history and examination of a patient with fever had been taken as well of coursel). Recent Prog Med. 1998 jan:89 (1):30-6. Acute gout is a cause of Systemic inflammatory response syndrome (SIRS) which is 2 or more changes of body temperature, heart rate, respratory fimction and peripheral leukocyte count (emedicine "Shock, sptic") ‘Mark this question => (Question Id : 30634 Question 7 of 30 A76 year old male presents with an acute onset of severe paia and swelling ofthe left wrist following a chest infection which he had two weeks previously. On examination, he had a temperature of 38°C and the left wrist was red, swollen and painfil, Which investigation is most appropriate in this patient? a) Erythrocytes sedimentation rate +b) Full blood count ¢) Serum urate concentration 4) X-ray-of the joint ¢) Joint aspiration Question Explanation: ‘The most relevant investigation with any one with a red, swollen and painful joint would be joint aspiration sending off for cultures and, analysis for crystals. Differential diagnoses clade gout (where serum urate may fall during acute attack) pseudogout and nnfection. “All diagnoses would be adequately addressed by joint aspiration. jon Report An Error ‘Mark this question => (Question Id : 30634 Question 7 of 30 A76 year cld male presents with an acute onset of severe pain and swelling of the left wrist following a chest infection which he had two weeks previously. On examination, he had a temperature of 38°C and the left wrist was red, swollen and painful. Which investigation is most appropriate in this patient? 4) Erythrocytes sedimentation rate ') Full blood count ©) Serum urate concentration & Xray-of the joint Y © 6) Joint aspiration Question Explanation: ‘The most relevant investigation with any one with a red, swollen and painful joint would be joint aspiration sending off for cultures and, analysis for crystals. Differential diagnoses clade gout (where serum urate may fall during acute attack) pseudogout and nnfection. “All diagnoses would be adequately addressed by joint aspiration. jon Report An Error ‘Mark this question e& => (Question Td : 48953 Question 8 of 30 Aa elderly man complains of lower back pain. X-ray reveals decreased joint space and osteophytes. The most appropriate diagnosis is a) Gout ) Degenerative joint disease (osteoarthritis) ©) Prendegont 4) Racumatoid arthritis Answer | Bplanation Other User's Explanation Report An Error Question Explanation: Osteoarthritis is a chronic arthropathy of an entire joint characterized by disruption and potential loss of joint cartilage along, with other joint changes, including bone hypertrophy (osteophyte formetion). Symptoms include gradually developing pain aggravated or triggered by activiy, stifuess relieved < 30 min after activity, and occasional joint swelling Diagnosis is confirmed by x-rays. Plain x-rays should be obiained of the most symptomatic joints. X-rays generally reveal marginal osteophytes, narrowing of the joint space, increased density of the subchondral bone, subchondral cyst formation, bony remodeling, and joint effusions. Standing x-rays of knees are more sensitive to joint space narrowing. Treatment involves physical measures (including rehabilitation), drugs, and surgery ‘Mark this question e& => (Question Td : 48953 Question 8 of 30 An elderly man complains of lower back pain. X-ray reveals decreased joint space and osteophytes. The most appropriate diagnosis is a) Gout ¥ © b) Degenerative joint disease (ostecarthritis) c) Psendogout d) Rheumatoid arthritis Answer | Bplanation Other User's Explanation Report An Error Question Explanation: Osteoarthritis is a chronic arthropathy of an entire joint characterized by disruption and potential loss of joint cartilage along, with other joint changes, including bone hypertrophy (osteophyte formetion). Symptoms include gradually developing pain aggravated or triggered by activiy, stifuess relieved < 30 min after activity, and occasional joint swelling Diagnosis is confirmed by x-rays. Plain x-rays should be obiained of the most symptomatic joints. X-rays generally reveal marginal osteophytes, narrowing of the joint space, increased density of the subchondral bone, subchondral cyst formation, bony remodeling, and joint effusions. Standing x-rays of knees are more sensitive to joint space narrowing. Treatment involves physical measures (including rehabilitation), drugs, and surgery ‘Mark this question = Question Id : 50790 Question 9 of 30 A.33 year old woman comes to you with a history ofthe gradual onset of pain and tendemess in the wrists and hands for 3 months She also complains of 1 hour of morning stiffness, she denies rash, fever, or skin changes. Oa physical examination she has spmimetiic swelling of the proximal interphalangeal joints aad metacarpophalangeal joints. Movement at these joirts is painfil. She hes no rash or mouth ulbers. Radiographs of the hands and wrists are negative, and chest fims are unremarkable, Her CBC is normal, but the erythrocyte sedimentation rate is elevated at 40 mun/hr. Latex fixation for rheumatoid factor is negative, and an antinuclear antibody: (AMA) testis negative. The most lileely diagnosis is a) Rheumatoid arthritis ) Systemic Inpus erythematosus 2) Sarcoidosis d) Lyme disease 2) Calcium pyrophosphate desposition disease Question Explanation: ‘The patient has rheumatoid arthritis (RA) by symptoms and physical findings. A positive latex fixation test for rheumatoid factor is not necessary for the diagnosis. A negative rheumatoid factor does not exciude RA. and a positive sheumatoid factor is not spectic. Rheumatoid factor is found in the serum of approximately 85% of adult patients with RA; in subjects without RA, the incidence of positive rheumatoid factor is 1%-5% and, increases with age. The ANA testis poritive in at least 95% of patients with systemic lupus erythematosus (SLE), but in only about 35% of paticnts with RA. Elevation of the erythrocyte scimentation rate is seen in many patients with RA, and the degree of elevation roughly parallels disease activiy. A mean of 6 months after the onset of Lyme disease, 60% of patients in the U.S. have brief anacks of asymmetric, oligoarticular arthritis, primarily in the large joints and especially in the knee ‘Mark this question = Question Id : 50790 Question 9 of 30 A.33 year old woman comes to you with a history ofthe gradual onset of pain and tendemess in the wrists and hands for 3 months She also complains of 1 hour of morning stiffness, she denies rash, fever, or skin changes. Oa physical examination she has spmimetiic swelling of the proximal interphalangeal joints and metacerpophalangeal joints. Movement at these joitts is painful, She hes no rash or mouth ulcers. Radbographs of the hands and wrists are negative, and chest fimns are unremarkable, Her CBC is normal, but the erythcocyte sedimentation rate ic elevated at 40 mabe. Latex fization for cheumatoid factor is negative, and an antinuclear antibody (AMA) tectic negative. The mest lcely diagnosis is Y © a) Rheumatoid arthritis ) Systemic Inpus erythematosus ©) Sarcoidosis 4) Lyme disease 2) Calcium pyrophosphate desposition disease Question Explanation: ‘The patient has rheumatoid arthritis (RA) by symptoms and physical findings. A positive latex fixation test for rheumatoid factor is not necessary for the diagnosis. A negative rheumatoid factor does not exciude RA. and a positive sheumatoid factor is not spectic. Rheumatoid factor is found in the serum of approximately 85% of adult patients with RA; in subjects without RA, the incidence of positive rheumatoid factor is 1%-5% and, increases with age. The ANA testis poritive in at least 95% of patients with systemic lupus erythematosus (SLE), but in only about 35% of paticnts with RA. Elevation of the erythrocyte scimentation rate is seen in many patients with RA, and the degree of elevation roughly parallels disease activiy. A mean of 6 months after the onset of Lyme disease, 60% of patients in the U.S. have brief anacks of asymmetric, oligoarticular arthritis, primarily in the large joints and especially in the knee ‘Mark this question =z Question 1d: 54951 Question 10 of 30 A.24-year-old non-smoker lady has left knee joint pain along 10 week old weight loss. With good appetite, she had occasional loose motions twice daily over this tine She neither smokes nor takes any medicines but takes modest quantities of alcohol. Family history of Hypothyroidism is +ve, Examination reveals a swollen, tender left knee joint with a small effusion, Which of the following ficely diagnosis? a) Behcet's disease b) Reiter‘s syndrome ©) Thyrotaxicosis 4) Inflammatory bowel disease ¢) Tuberculosis (Question Explanation: ‘The description of weight loss, diarrhea and a monojoligo-arthropathy suggest a diagnosis of inflammatory bowel disease. Reiter's is unlikely to present with olgoarthropathy and the diarrhea is usually acute. ‘Mark this question e => Question 10 of 30 Question Id : 54951 A.24-year-old non-smoker lady has left knee joint pain along 10 week old weight loss. With good appetite, she had occasional loose motions twice daily over this tine She neither smokes nor takes any medicines but takes modest quantities of alcohol. Family history of Hypothyroidism is +ve, Examination reveals a swollen, tender left knee joint with a small effusion, Which of the following ficely diagnosis? a) Behcet's disease b) Reiter‘s syndrome ©) Thyrotaxicosis Y © &) Inflammatory bowel disease ¢) Tuberculosis (Question Explanation: ‘The description of weight loss, diarrhea and a monojoligo-arthropathy suggest a diagnosis of inflammatory bowel disease. Reiter's is unlikely to present with olgoarthropathy and the diarrhea is usually acute. ‘Mark this question —& => ‘Question Id: 54992 Question 11 of 30 A, 29-year-old man presented with acute stifess and swelling of his knee and ankles, and a painful rash on his legs ESR was 87 mm in the first hour (0-15), Chest X-ray showed hilar lymphadenopathy. What is the likely outcome? a) Chronic arthritis ») Lung fibrosis c) Renal failure @) Skin ulceration ©) Spontaneous improvement Question Explanation: Report An Error The description is typical of acute Sarcoidosis with erythema nodosum, polyarthropathy and hilar Ismphadenopathy. This has good prognosis and usually resolves spontaneously over six to eight weeks ‘Mark this question —& => ‘Question Id: 54992 Question 11 of 30 A, 29-year-old man presented with acute stifess and swelling of his knee and ankles, and a painful rash on his legs ESR was 87 mm in the first hour (0-15), Chest X-ray showed hilar lymphadenopathy. What is the likely outcome? a) Chronic arthritis ») Lung fibrosis c) Renal failure @) Skin ulceration Y © ©) Spontaneous improvement Question Explanation: Report An Error The description is typical of acute Sarcoidosis with erythema nodosum, polyarthropathy and hilar Ismphadenopathy. This has good prognosis and usually resolves spontaneously over six to eight weeks 2/26/2014 1:00:51 PM Mat this questi qc Question 12 of 30 “What is the most effective treatment for osteoporosis? a) Exercise 'b) Analgesics 6) Vitamins 4) Bisphosphonates Question Explanation: Osteoporosis is @ progressive metabolic bone disease that decreases bone density (bone mass per unit volume), with deterioration of bone structure, Skeletal weakness leads to fractures with minor or inapparent trauna, particularly in the thoracic and lumbar spine, wrist, and hip. Acute ot chronic back pain is common. Diagnosis is by dual-energy X-ray absosptiometry. Bisphosphonates are first fine drug therapy. By inhibiting bone resorption, bisphosphonates preserve bone mass and can decrease vertebral and hip fractures, by 50%, Estrogen can preserve bone density and prevent fractures. Prevention and treatment involve Ca and vitamin D supplements, exercises to maximize bone and muscle strength and minimize the risk of falls, and drug therapy to preserve bone mass or stimulate new bone formation. 2/26/2014 1:00:51 PM Mat this questi qc Question 12 of 30 “What is the most effective treatment for osteoporosis? a) Exercise 'b) Analgesics 6) Vitamins ¥ © 4) Bisphosphonates Question Explanation: Osteoporosis is @ progressive metabolic bone disease that decreases bone density (bone mass per unit volume), with deterioration of bone structure, Skeletal weakness leads to fractures with minor or inapparent trauna, particularly in the thoracic and lumbar spine, wrist, and hip. Acute ot chronic back pain is common. Diagnosis is by dual-energy X-ray absosptiometry. Bisphosphonates are first fine drug therapy. By inhibiting bone resorption, bisphosphonates preserve bone mass and can decrease vertebral and hip fractures, by 50%, Estrogen can preserve bone density and prevent fractures. Prevention and treatment involve Ca and vitamin D supplements, exercises to maximize bone and muscle strength and minimize the risk of falls, and drug therapy to preserve bone mass or stimulate new bone formation. 2/26/2014 1:01:03 PM “Maric this question qc Question Id: 59711 Question 13 of 30 ‘A 24 year old wemen presents with fever, malaise, generalized arthralgas, and a skin rash, over the nose and malar eminences, The possible finding that has the greatest relative signiicance in the overall prognosis for the patiert is a) Immune complexes at the dermal epidermal junction in skin b) Pleurits c) Atypical verrucous vegetations of the mitral valve 4) Glomerular Subendothelial immune complex: deposition ©) Perivascular fbrosis in the spleen Answer | Biplanation Other User's Explanation Report An Error Question Explanation: Systemic lupus erythematosus is a chronic, multisystem, inflammatory disorder of probable autoimune etiology, occurring predominantly in young women, Many of the clinical manifestations of SLE are caused by the effects of circulating Immune complexes on vatious tissues of to the direct effects of antibodies to cell surface components. Common manifestations include acthralgias and arthritis; malar and other skin rashes; pleurtis or pericarditis; renal or CN'S involvement, and hematclogic eytopenia, ‘Diagnosis requires clinical and serologic cntena, Treatment of severe ongoing active disease requires corticosteroids, often hydroxychloroqune, and sometimes Immunosuppressants, 2/26/2014 1:01:03 PM “Maric this question qc Question Id: 59711 Question 13 of 30 ‘A 24 year old wemen presents with fever, malaise, generalized arthralgas, and a skin rash, over the nose and malar eminences, The possible finding that has the greatest relative signiicance in the overall prognosis for the patiert is a) Immune complexes at the dermal epidermal junction in skin b) Pleurits c) Atypical verrucous vegetations of the mitral valve Y © d) Glomerular Subendothelial immane complex deposition ©) Perivascular fbrosis in the spleen Answer | Biplanation Other User's Explanation Report An Error Question Explanation: Systemic lupus erythematosus is a chronic, multisystem, inflammatory disorder of probable autoimune etiology, occurring predominantly in young women, Many of the clinical manifestations of SLE are caused by the effects of circulating Immune complexes on vatious tissues of to the direct effects of antibodies to cell surface components. Common manifestations include acthralgias and arthritis; malar and other skin rashes; pleurtis or pericarditis; renal or CN'S involvement, and hematclogic eytopenia, ‘Diagnosis requires clinical and serologic cntena, Treatment of severe ongoing active disease requires corticosteroids, often hydroxychloroqune, and sometimes Immunosuppressants, ‘Marie this question eq => Question Td : 62720 Question 14 of 30 A.48 year woman has had rheumatoid arthritis for 8 years. Her hands now show moderate ulnar deviation of the fingers and she says her wrists and knees also hurt. She says, "Inad several drop attacks during the past 3 months." She characterizes these attacks as episodes of weakness and loss of feeling in her legs for several minutes. During one of these episodes, she became incontinert. On piysical examination she has facial plethora and swollen and painfil metacarpophalangeal and knee joints, bilaterally. The rest of the examination is normel. The most ikely cause of her 'drop attacks" is a) Adrenal insufficiency b) Atlanto-occipital instability c) Anaiety 4) Cardiac arrhythmia ©) Cerebral ischemia Answer | Biplanation Other User's Explanation Report An Error Question Explanation: Careful attention to her history however allows the diagnosis to ascertain with near certainty. The drop attacks are characterized by incontinence with weakness and loss of feeling in her legs. These symptoms point to a neurologic defect that affects both the parasympathetic and motor functions of the nervous systema, Atlento- occipital joint subluxation is a common problem for patients with, advenced theumatoid artis, as this patient clearly suffers from. Ifthe cervical spine were to suffer impingement from such subluxation, it would be expected that motor loss below the level of the impingemert aswell as loss of bladder control from interruption of the parasympathetic outflow (cranial component) ‘Marie this question eq => Question Td : 62720 Question 14 of 30 A.48 year woman has had rheumatoid arthntis for 8 years. Her hands now show moderate ulnar deviation ofthe fingers and she says her wrists and knees also hurt. She says, "Inad several drop attacks during the past 3 months." She characterizes these attacks as episodes of weakness and loss of feeling in her legs for several minutes. During one of these episodes, she became incontinert. On piysical examination she has facial plethora and swollen and painfil metacarpophalangeal and knee joints, bilaterally. The rest of the examination is normel. The most ikely cause of her 'drop attacks" is 8) Adeenal insufficiency o © b) Atlanto-cccipital instability c) Anaiety 4) Cardiac arrhythmia ©) Cerebral ischemia Answer | Biplanation Other User's Explanation Report An Error Question Explanation: Careful attention to her history however allows the diagnosis to ascertain with near certainty. The drop attacks are characterized by incontinence with weakness and loss of feeling in her legs. These symptoms point to a neurologic defect that affects both the parasympathetic and motor functions of the nervous systema, Atlento- occipital joint subluxation is a common problem for patients with, advenced theumatoid artis, as this patient clearly suffers from. Ifthe cervical spine were to suffer impingement from such subluxation, it would be expected that motor loss below the level of the impingemert aswell as loss of bladder control from interruption of the parasympathetic outflow (cranial component) (Maré tis question =r: Question 1d: 80251 Question 15 of 30 A.46-year-old construction worker lias to work alone at a heavy job that usually requires a two-men crew because his partner i ill ‘Next day, he wakes up with diffuse pain and stiffiess. Which one of the following tests is most likely to be abuontnal? a) Hemoglobin ) Antinuclear antibody ©) Flectrocardiogram. 9) Chest radiograph €) Creatine phosphokinase Question Explanation: ‘A creatine phosphokinase test would probably be abnormal, This individual is most likely to have rhabdomyolysis as a result of his unusually sitemuous physical activity and this would produce an elevation in his CPK enzymes. His hemoglobin should be normal. There is no evidence of connective tissue disease that would be associated with a positive antinuclear antibody. His electrocardiogram is not likely to be abnormal, since he does nothave any symptoms of coronary artery disease of pericarditis. His chest radiograph should be normal in the absence of any pulmonary symptoms (Maré tis question =r: Question 1d: 80251 Question 15 of 30 A 46-year-old construction worker has to work alone at aheavy job that usually requires a two-man crew because his partner is ill ‘Next day, he wakes up with diffuse pain and stiffuess. Which one of the following tests is most likely to be abnonnal? a) Hemoglobin ) Antinuclear antibody ©) Elestrocardiogram ) Chest radiograph YW © €) Creatine phosphokinase Question Explanation: ‘A creatine phosphokinase test would probably be abnormal, This individual is most likely to have rhabdomyolysis as a result of his unusually sitemuous physical activity and this would produce an elevation in his CPK enzymes. His hemoglobin should be normal. There is no evidence of connective tissue disease that would be associated with a positive antinuclear antibody. His electrocardiogram is not likely to be abnormal, since he does nothave any symptoms of coronary artery disease of pericarditis. His chest radiograph should be normal in the absence of any pulmonary symptoms 2/26/2014 1:01:41 PM Maik tis question rm Question 16 of 30 A 60 year old woman presents with complains of “feeling tired." She also complains of frequent one sided throbbing headaches Physical examination reveals tenderness over both temples. Investigations show elevated erytarocyte sedimentation rate. This patient ifnot treated aggressively can develop which one of the following? a) Arhritis b) Blindness ©) Deafhess ) Loss of the abilty to speak: €) Paralysis Question Explanation: This womnan is afficted with temporal (giant cell) arteritis. This is a panarteritis that can involve any of the branches of the aortic arch but classically affects the branches of the carotid system, Temporal arteritis commonly (40-50%) produces visual disturbances inchiding blindness, due to involvement of the ophthalmic artery. Biopsy of affected segments of arteries may be diagnostic, showing granulomatous lesions with giant cells, although nonspecific inflammatory infiltrates andor intimal fibrosis (generally with no disruption of the intemal elastic lamina) are also seen with some frequency. Stroke may occur in temporal arteritis, but this would not likely produce loss of all tactile sensation. Arthritis is not en associated feature of temporal artentis. Isolated deafness is not commonly seen with temporal arteritis, Loss of the ability to speck might conceivably result from a stroke affecting the inferior frontal gyrus or from a lesion of the brain stem centers controling phonetion, but visual disturbances are more common than stroke in temporal artentis. Paralysis is not a common complication of temporal arteritis. 2/26/2014 1:01:41 PM Maik tis question rm Question 16 of 30 A 60 year old woman presents with complains of "feeling tired." She also complains of frequent one sided throbbing headaches. Physical examination reveals tenderness over both temples. Investigations show elevated erythrocyte sedimentation rate. This patient if not treated aggressively can develop which one of the following? a) Arihriis Y © b) Blindness c) Deafhess d) Loss of the ability to speale €) Paralysis Question Explanation: This womnan is afficted with temporal (giant cell) arteritis. This is a panarteritis that can involve any of the branches of the aortic arch but classically affects the branches of the carotid system, Temporal arteritis commonly (40-50%) produces visual disturbances inchiding blindness, due to involvement of the ophthalmic artery. Biopsy of affected segments of arteries may be diagnostic, showing granulomatous lesions with giant cells, although nonspecific inflammatory infiltrates andor intimal fibrosis (generally with no disruption of the intemal elastic lamina) are also seen with some frequency. Stroke may occur in temporal arteritis, but this would not likely produce loss of all tactile sensation. Arthritis is not en associated feature of temporal artentis. Isolated deafness is not commonly seen with temporal arteritis, Loss of the ability to speck might conceivably result from a stroke affecting the inferior frontal gyrus or from a lesion of the brain stem centers controling phonetion, but visual disturbances are more common than stroke in temporal artentis. Paralysis is not a common complication of temporal arteritis. 2/26/2014 1:01:59 PM 7 veo erertacs chioky” * (Gaesdonran vam IST € > CB www interface edupk/medical-axams/test-analysis phprutid=14759 apps EJcoogie A settings [) signin Ci imported FromiE 2. webs dukes © a go call days Ww Gi other bookmar ‘Maske this question <= => (Question Td : 88674 Question 17 of 30 The x-ray below shows the hip joint of a 66 year old female. If aspiration of the joint id had been performed, the most accurate description of the aspirated fluid would be a) Clear fluid with high protein content and no significant numbers of inflammatory cells ') Clear fuid with high protein content; neutrophils and mononuclear cells, and culture positive for mycobacteria ©) Clear fiuid with high protein content; neutrophils and mononuclear cells; no bacteria and no erystals 4) Purulent Buidl with large numbers of nsutrophils and culture positive for bacteria ©) Purulent duid with large numbers of neutrophils and needle-shaped crystals newer (PEPER) ote: user's Explanation Repost An Err Question Explanation ‘The joint shows radiographic changes most suggestive of degenerative joint disease (osteoarthritis). These changes can include narrowing ofthe joint space increased density of suachondeal bone formation of osteophytes at the periphery ofthe joints and formation of pseadocysts in the subchondral marrow, Degenerative joint disease is not an inflammatory atthriis, and consequently the joint uid resembles normal joint uid andi without at marked inflammatory infitrate crystals, or microorganists. Total Questions KREEREEM eRe Meee Bok ZR RESRBRRBRRBE XxXxXXXKXXKXKXXKKXKXKXKKXKXXKXKXKXXKKXKXKXKXKXXK 2/26/2014 1:01:59 PM 7 veo erertacs chioky” * (Gaesdonran vam IST € > CB www interface edupk/medical-axams/test-analysis phprutid=14759 apps EJcoogie A settings [) signin Ci imported FromiE 2. webs dukes © a go call days Ww Gi other bookmar ‘Maske this question <= => (Question Td : 88674 Question 17 of 30 The x-ray below shows the hip joint of a 66 year old female. If aspiration of the joint id had been performed, the most accurate description of the aspirated fluid would be a) Clear fluid with high protein content and no significant numbers of inflammatory cells ') Clear fuid with high protein content; neutrophils and mononuclear cells, and culture positive for mycobacteria ©) Clear fiuid with high protein content; neutrophils and mononuclear cells; no bacteria and no erystals 4) Purulent Buidl with large numbers of nsutrophils and culture positive for bacteria ©) Purulent duid with large numbers of neutrophils and needle-shaped crystals newer (PEPER) ote: user's Explanation Repost An Err Question Explanation ‘The joint shows radiographic changes most suggestive of degenerative joint disease (osteoarthritis). These changes can include narrowing ofthe joint space increased density of suachondeal bone formation of osteophytes at the periphery ofthe joints and formation of pseadocysts in the subchondral marrow, Degenerative joint disease is not an inflammatory atthriis, and consequently the joint uid resembles normal joint uid andi without at marked inflammatory infitrate crystals, or microorganists. Total Questions KREEREEM eRe Meee Bok ZR RESRBRRBRRBE XxXxXXXKXXKXKXXKKXKXKXKKXKXXKXKXKXXKKXKXKXKXKXXK ‘Mark this question e=> Question 18 of 30 A.27 year old woman presents with a history ofrecent digital ulcerations. small nodules on her digits and around her knees that occasionally drain white material, and 2 sensation that she cannot filly open her mouth. "Whenever she gets cold her hands tum bine and get painful She also likely has noted which one offthe following symptoms? a) Migraines ) Rhinitis c) Esophageal refla 4) Podagra €) Helotrope rash Question Explanation: This patient has progressive systemic sclerosis, which often includes esophagedl reflux as one ofits manifestations. Migraines and rhinitis are not speciically associated with scleroderma, Podagra refers to acutz arthitis ofthe great toe resulting from gout. This patient's nodules are not consistent with tophi, which tend to be integular, large, and can occasionally drain. A heliowope rash is a purplish discoloration of the eyelids seen in dermatomrositis. Report An Error ‘Mark this question e=> Question 18 of 30 A.27 year old woman presents with a history ofrecent digital ulcerations. small nodules on her digits and around her knees that occasionally drain white material, and a sensation that she cannot filly open her mouth. Whenever she gets cold ker hands tum blue and get painful, She alco likely hac noted which one of the following eymptome? a) Migraines ) Rhinitis Y © c) Esophageal reflax 4) Podagra €) Helotrope rash Question Explanation: This patient has progressive systemic sclerosis, which often includes esophagedl reflux as one ofits manifestations. Migraines and rhinitis are not speciically associated with scleroderma, Podagra refers to acutz arthitis ofthe great toe resulting from gout. This patient's nodules are not consistent with tophi, which tend to be integular, large, and can occasionally drain. A heliowope rash is a purplish discoloration of the eyelids seen in dermatomrositis. Report An Error ‘Mark this question = => (Question Id : 92731 Question 19 of 30 “Which of the following observation does MOT support an immunological basis for the development of theumatic fever? a) The short latency between streptococcal infection and the development of sheumatic fever. ) Antistreptococcal antibodies, which cross-react with cardiac tissue, are preseat in sera, ©) High titers of streptococeal extracellilar and somatic antigens in sera ) Exaggerated cellilar reactivity to streptococcal cell membrane antigens ©) Increased frequency of class I histocompatibiity antigens in sera Anewor (REIS) other User's Ex Question Explanation: An immunologe theory for the development of rheumatic fever is supporied by the long latency between the onset of pharyngitis and the development of acute rheumatc fever. Mary persons with acute rheumatic fever have antistreptococcal antibodies, which cross- react with cardiac tissue. Higher titers of antibodies to streptococcal extracellular and somatic antigens are present in persons who develop theumatic fever when compered to pessons with uncomplicated streptococcal infections. Persons who develop acute theumatic fever also have an exaggerated cellular reactivity to streptococcal cell membrane antigens, as measured by ia vitro techriques. Some class II histocompatibiity antigens occur more frequently in persons who have developed rheumatic fever when compared to controls. ation Report An Error ‘Mark this question = => (Question Id : 92731 Question 19 of 30 “Which of the following observation does MOT support an immunological basis for the development of sheumatic fever? Y © a) The short latency between streptococcal infection and the development of sheumatic fever. ) Antistreptococcal antibodies, which cross-react with cardiac tissue, are preseat in sera, ©) High titers of streptococeal extracellilar and somatic antigens in sera ) Exaggerated cellilar reactivity to streptococcal cell membrane antigens ©) Increased frequency of class I histocompatibiity antigens in sera Anewor (REIS) other User's Ex Question Explanation: An immunologe theory for the development of rheumatic fever is supporied by the long latency between the onset of pharyngitis and the development of acute rheumatc fever. Mary persons with acute rheumatic fever have antistreptococcal antibodies, which cross- react with cardiac tissue. Higher titers of antibodies to streptococcal extracellular and somatic antigens are present in persons who develop theumatic fever when compered to pessons with uncomplicated streptococcal infections. Persons who develop acute theumatic fever also have an exaggerated cellular reactivity to streptococcal cell membrane antigens, as measured by ia vitro techriques. Some class II histocompatibiity antigens occur more frequently in persons who have developed rheumatic fever when compared to controls. ation Report An Error 2/26/2014 1:02:41 PM. ‘Mare this question e=> Question 20 of 30 Question Id: 93774 A perimenopausal woman has a bony swelling inthe distal in the distal inteepharyngeal joirt on physical examination, No inflammation is apparent The proximal intesphalangeal joint, hands, and wrists are not involved, and the patient is asymptomatic. What process is related to these findings? 2) Autoantibody formation ) Bacterial infection c) Crystal deposition 4) Joint trauma ©) Viral infection Answer | Explanation Other User's Explanation Report An Error Question Explanation: The disease is osicoartirtis, and the bony swellings are called "Heberden’s nodes", which may or may not be symptomatic. (Osteoarthritis is thought to be related to repentive joint trauma, Autoantibodies are mportant in theumatoid arthritis. Bacterial Infections cause septic arthritis. Crystal deposition is important in gout and pseudogout Viral infections can cause transient arthralgias and arthritis, 2/26/2014 1:02:41 PM. ‘Mare this question e=> Question 20 of 30 Question Id: 93774 A perimenopausal woman has a bony swelling inthe distal in the distal inteepharyngeal joirt on physical examination, No inflammation is apparent The proximal intesphalangeal joint, hands, and wrists are not involved, and the patient is asymptomatic. What process is related to these findings? a) Autoantibody formation. b) Bacterial infection ¢) Crystal depestion Y © @ Toint trauma ©) Viral infection Answer | Explanation Other User's Explanation Report An Error Question Explanation: The disease is osicoartirtis, and the bony swellings are called "Heberden’s nodes", which may or may not be symptomatic. (Osteoarthritis is thought to be related to repentive joint trauma, Autoantibodies are mportant in theumatoid arthritis. Bacterial Infections cause septic arthritis. Crystal deposition is important in gout and pseudogout Viral infections can cause transient arthralgias and arthritis, 2/26/2014 1:04:15 PM. Mack this question = >> Question Id 94524 Question 21 of 30 ‘A. 22 year old bisesmial male has a 4 week history of intermittent diarrhea, urethral cischarge, and pain in the right knee and left second toe. On examination he kas several oral ulcers, a clear urethral discharge, a scaly popular resh on palms and scles, onychelyss, sausage like swelling of the left second toe, and heat and swelling of the right knee. The results of Gram stains and cultures of urethral discharge are negative. No rheumatoid factor is present. Which of the following is the most likely diagnosis? a) Gonococeal arthritis 'b) Reiter’s eyndreme c) Behcet disease 4) Psoriatic arthritis ©) Acquired immmne deficiency syndrome Answer | Exvionation Other User's Explanation Report An Error Question Explanation: Reiter’s syndrome (reactive arthritis) is infarrmation of the joints and tendon attachments at the joints, often accomparied by inflammation of the eye's conjunctiva and the mucous membranes, such as those oftthe mouth and gentourinary tract, and by a distinctive rach, Reiter's syndrome is also called reactive arthritic because the joint inflammation appears to be areaction to an infection criginating in the intestine or genital tract. This syndrome is most common in men aged 20 to 40. Typically, symptoms begin 7 to 14 days after the infection Inflammation of the wethra result cither directly from infection of the wethra or even from areaction to the intestinal infection. Th men, inflammation of the urethra causes moderate pain and a discharge from the penis or arash on the glans ofthe penis (balanitis circinata). The prostate gland may be inflarned and painfil. The genital and urinary symptoms in women, if any occur, are usually mild, consisting of a slight vaginal discharge or uncomfortable urination. The conjunctiva becomes red and inflamed, causing itching or burning and excessive tearing. Joint pain and inflammation may be mild or severe, Several joints are usually affected at once, especially the knees, toe joints, and areas where tendons are attached to bones, such as atthe heels. Small, painless or tender sores can develop in the mouth. 2/26/2014 1:04:15 PM. Mack this question = >> Question Id 94524 Question 21 of 30 ‘A. 22 year old bisesmial male has a 4 week history of intermittent diarrhea, urethral cischarge, and pain in the right knee and left second toe. On examination he kas several oral ulcers, a clear urethral discharge, a scaly popular resh on palms and scles, onychelyss, sausage like swelling of the left second toe, and heat and swelling of the right knee. The results of Gram stains and cultures of urethral discharge are negative. No rheumatoid factor is present. Which of the following is the most likely diagnosis? a) Gonococeal arthritis JY © b) Reiter's syndrome c) Behcet disease 4) Psoriatic arthritis ©) Acquired immmne deficiency syndrome Answer | Exvionation Other User's Explanation Report An Error Question Explanation: Reiter’s syndrome (reactive arthritis) is infarrmation of the joints and tendon attachments at the joints, often accomparied by inflammation of the eye's conjunctiva and the mucous membranes, such as those oftthe mouth and gentourinary tract, and by a distinctive rach, Reiter's syndrome is also called reactive arthritic because the joint inflammation appears to be areaction to an infection criginating in the intestine or genital tract. This syndrome is most common in men aged 20 to 40. Typically, symptoms begin 7 to 14 days after the infection Inflammation of the wethra result cither directly from infection of the wethra or even from areaction to the intestinal infection. Th men, inflammation of the urethra causes moderate pain and a discharge from the penis or arash on the glans ofthe penis (balanitis circinata). The prostate gland may be inflarned and painfil. The genital and urinary symptoms in women, if any occur, are usually mild, consisting of a slight vaginal discharge or uncomfortable urination. The conjunctiva becomes red and inflamed, causing itching or burning and excessive tearing. Joint pain and inflammation may be mild or severe, Several joints are usually affected at once, especially the knees, toe joints, and areas where tendons are attached to bones, such as atthe heels. Small, painless or tender sores can develop in the mouth. 2/26/2014 1:04:30 PM ‘Mark this question = => Question Td : 95280 Question 22 of 30 A.4 year old woman complains of fatigue and chronic joint pain es well as moming stifftess, which lasts approximately an hour. Physical examination reveals nontender, rubbery nodules on her elbows. Biopsy of synowium from her knee joint is shown in a photomicrograph below. The most abundant cells in the inflammatory infitrate are which of the following? a) Eosinophils 'b) Langhans type gant cells c) Lymphocytes and plasma cells 4) Neutrophils ©) Type A and B synovial cells Answer | Explanation Other User's Explanation Report An Error Question Explanation: Lymphocytes and plasma cells aggregate near and around blood vessels (perivascular accumulation) in this example of chroric inflammation of the synovium. Lymphocytes have dari nuclei with litle visible cytoplasm. Plasma cells are larger with a distinst cytoplasm and an eccentric nucleus. The synovial lining is thickened from its normal 1- 2 layers. Note that you cid nothave to be able to recognize lymphocytes and plasma cells in the photomicrograph to answer this question, Because RA is associated with a chronic inflaramnatory infiltrate lymphocytes and plasina cells must be the correct answer. Eosinophils are not evident here and do not seer to play arole in RA. Langhans type giant cells, or multinucleated histiocytes, are not evident here but may appear during the later siages ofRA. Neutrophils are not evident here; they are mstead associated with acute inflammation Type A and B synovial cells are the two cell types of the synovial linng. They increase in number but are not the cells ofthe perivascular infiltrate 2/26/2014 1:04:30 PM ‘Mark this question = => Question Td : 95280 Question 22 of 30 A.43 year old woman complains of fatigue and chronic joint pain as well as moming sifihess, which lasts approstimately an hour Physical examination reveals nontender, rubbery nodules on her elbows. Biopsy of synowium from her knee joint is shown in a photomicrograph below. The most abundant cells in the inflammatory infitrate are which of the following? a) Eosinophils 'b) Langhans type gant cells Y © c) Lymphocytes and plasma cells 4) Neutrophils ©) Type A and B synovial cells Answer | Explanation Other User's Explanation Report An Error Question Explanation: Lymphocytes and plasma cells aggregate near and around blood vessels (perivascular accumulation) in this example of chroric inflammation of the synovium. Lymphocytes have dari nuclei with litle visible cytoplasm. Plasma cells are larger with a distinst cytoplasm and an eccentric nucleus. The synovial lining is thickened from its normal 1- 2 layers. Note that you cid nothave to be able to recognize lymphocytes and plasma cells in the photomicrograph to answer this question, Because RA is associated with a chronic inflaramnatory infiltrate lymphocytes and plasina cells must be the correct answer. Eosinophils are not evident here and do not seer to play arole in RA. Langhans type giant cells, or multinucleated histiocytes, are not evident here but may appear during the later siages ofRA. Neutrophils are not evident here; they are mstead associated with acute inflammation Type A and B synovial cells are the two cell types of the synovial linng. They increase in number but are not the cells ofthe perivascular infiltrate 2/26/2014 1:04:44 PM ‘Mat this question e& => Question Id : 95301 Question 23 of 30 A51 year old women presents with enlarged knuckles and large subcutaneous nodules hear her elbow. Her proximal interphalangeal joints (PIP) are hyperestended and her distal interphalangeal joints (DIP) are flexed. The nodules when biopsied would show which of the following histological appearance? a) Amorphous crystalline mass surrounded by macrophages 'b) Cystic space caused by myxoid degeneration of connective tissue ©) Darkly pigmented synovium with an exaberant, villous growth 9) Fibrinoid necrosis surrounded by palisading epithelioid cells €) Well-encapsulated nodule of polygonal cells within a tendon sheath Ancwor [UEQINURIY other Users Explanation Report AnEvot Question Explanation: Swen neck deformity (PIP joints hyperestended and DIP joints flexed), enlarged knuckles, and subcutaneous nodules are classic clues for rheumatoid arthrits. The subcutaneous rheumatoid nodules are composed histologically of areas of fibrincid necrosis surrounded by palisading epithelioid cells. Gout tophi are amorphous crystalline masses surrounded by macrophages. Ganglion cysts are small cystic spaces caused by myoid degeneration of connective tissue, Pigmented vilonodular synovitis causes darkly pigmented syneviumn with exuberant vilous growth, Nodular tenosynovitis causes a well encapsulated nodule of polygonal cells within a tendon sheeth. 2/26/2014 1:04:44 PM ‘Mat this question e& => Question Id : 95301 Question 23 of 30 A51 year old women presents with enlarged knuckles and large subcutaneous nodules hear her elbow. Her proximal interphalangeal joints (PIP) are hyperestended and her distal interphalangeal joints (DIP) are flexed. The nodules when biopsied would show which of the following histological appearance? a) Amorphous crystalline mass surrounded by macrophages 'b) Cystic space caused by myxoid degeneration of connective tissue ©) Darkly pigmented synovium with an exaberant, villous growth Y © A) Fibrinoid necrosis surrounded by palisading epithelioid cells €) Well-encapsulated nodule of polygonal cells within a tendon sheath Ancwor [UEQINURIY other Users Explanation Report AnEvot Question Explanation: Swen neck deformity (PIP joints hyperestended and DIP joints flexed), enlarged knuckles, and subcutaneous nodules are classic clues for rheumatoid arthrits. The subcutaneous rheumatoid nodules are composed histologically of areas of fibrincid necrosis surrounded by palisading epithelioid cells. Gout tophi are amorphous crystalline masses surrounded by macrophages. Ganglion cysts are small cystic spaces caused by myoid degeneration of connective tissue, Pigmented vilonodular synovitis causes darkly pigmented syneviumn with exuberant vilous growth, Nodular tenosynovitis causes a well encapsulated nodule of polygonal cells within a tendon sheeth. ‘Mark this question e>= Question Id : 98322 Question 24 of 30 ‘Modality of therapy that can be very helpfil in restoring some ofthe abiities necessary for the routine activities of daily living in patients with rheumatoid arthritis is a) Weightlifting b) Occupational therapy: c) Vocational rehabilitation 4) Jogging ©) Electrical bone graft stimulation Answer (Baotanation | Other User's Explanation Report An Error Question Explanation: (Occupational therapy, including splinting and the use of various adaptive tools for the activities of daily life, is one of the mainstays of management of patients with rheumatoid arthritis that is, unfortunately often overlooked. Weight lifting is contraindicated in peticats with active arthritis, Vocational rehabilitation is job retraining, which, while very important, does nothelp restore everyday activities to a patient's repertory of activities. Jogging is contraindicated in patients with active lower extremity disease and would not aid in ADIs. Electrical bone graft stimulation is often used for treatment of fracture aon ution, but has not been shown to be beneficial in the management of rheumatoid arthrts. ‘Mark this question e>= Question Id : 98322 Question 24 of 30 ‘Modality of therapy that can be very helpfil in restoring some ofthe abiities necessary for the routine activities of daily living in patients with rheumatoid arthritis is a) Weightlifting Y © b) Occupational therapy c) Vocational rehabilitation 4) Jogging €) Electrical bone graft stimulation Answer (Baotanation | Other User's Explanation Report An Error Question Explanation: (Occupational therapy, including splinting and the use of various adaptive tools for the activities of daily life, is one of the mainstays of management of patients with rheumatoid arthritis that is, unfortunately often overlooked. Weight lifting is contraindicated in peticats with active arthritis, Vocational rehabilitation is job retraining, which, while very important, does nothelp restore everyday activities to a patient's repertory of activities. Jogging is contraindicated in patients with active lower extremity disease and would not aid in ADIs. Electrical bone graft stimulation is often used for treatment of fracture aon ution, but has not been shown to be beneficial in the management of rheumatoid arthrts. 22672014 1:05:08 PM ‘Mark this question & => Question 25 of 30 Along with gout, colchicine is also beneficial in a) Rheumatoid arthritis b) Osteoarthritis c) Amyloid. ) Gaucher's disease ¢) Familial Mediterranean fever Ancwor [UESBWRIY other Users explanation Repor An Eos, Question Explanation: ‘One of the few conditions besides gout that responds to colchicine is famikal Mediterranean fever. Colchicine has no known effects in patients with rheumatoid arthrtis, osteoarthritis, amyloid, or Gaucher's disease 22672014 1:05:08 PM ‘Mark this question & => Question 25 of 30 Along with gout, colchicine is also beneficial in a) Rheumateid artis. b) Osteoarthritis. ©) Amyloid ) Gaucher's disease. Y © 2) Familial Mediterranean fever Ancwor [NESBA other Users Explanation Repor An Evo, Question Explanation: ‘One of the few conditions besides gout that responds to colchicine is famikal Mediterranean fever. Colchicine has no known effects in patients with rheumatoid arthrtis, osteoarthritis, amyloid, or Gaucher's disease ‘Mark this question =z Question Td: 113314 Question 26 of 30 “Which of the following is NOT an ireamunologic abnormality in systemic lupus erythematosis? a) Production of pathogenic subsets of autoantibodies and immune complexes is sustained. ) The mononuclear phagocyte system is unable to process immune complexes normally. ©) Idictype entiitiotype networks fall to suppress kyperactivated T- and B-cells 4) T-lymphocytes are skewed to suppressor finctions. ¢) Tolerance mechanisms that normally eliminate or inactwate strongly autoreactive T- and B- lymphocytes are impaired. Avewor (NEQNGNRANY) osherucersExplanation Report An Eos Question Explanation: ‘Tlymphocytes are heavily skewed toward helper cells. A sustained production of pathogenic subsets of autoantibodies and smmune complexes is characteristic of systemic lupus erythematosis. The monomuclear phagocytic system is unable to process these immune complexes nomnally, resulting 1n an incomplete clearing. In addition, idiotype antiiotype networks fail to suppress hyperactivated T- and B- cells. Tolerance mechanisms which normally eliminate autoreactive T- and B-lymphocytes are impaired. ‘Mark this question =z Question Td: 113314 Question 26 of 30 “Which of the following is NOT an ireamunologic abnormality in systemic lupus erythematosis? a) Production of pathogenic subsets of autoantibodies and immune complexes is sustained. ) The mononuclear phagocyte system is unable to process immune complexes normally. ©) Idictype entiitiotype networks fall to suppress kyperactivated T- and B-cells Y © d) T-lymphocytes are skewed to suppressor functions, ¢) Tolerance mechanisms that normally eliminate or inactwate strongly autoreactive T- and B- lymphocytes are impaired. Avewor (NEQNGNRANY) osherucersExplanation Report An Eos Question Explanation: ‘Tlymphocytes are heavily skewed toward helper cells. A sustained production of pathogenic subsets of autoantibodies and smmune complexes is characteristic of systemic lupus erythematosis. The monomuclear phagocytic system is unable to process these immune complexes nomnally, resulting 1n an incomplete clearing. In addition, idiotype antiiotype networks fail to suppress hyperactivated T- and B- cells. Tolerance mechanisms which normally eliminate autoreactive T- and B-lymphocytes are impaired. ‘Mark this question e& => Question Id : 118390 Question 27 of 30 A.52 year old man complains of bone pain, Lab data show a normal calcium and phosphate level but alkaline phosphatase levels are elevated Urinalysis reveals an elevated level of hydroxyprotine and roentengram stadies demonstrate dense, enlarged bones. Osteocalcin level is increased. The most likely diagnosis is a) Pott’s disease b) Osteogenic sarcoma c) Hbrogenesis imperfect ossium 9) Hyperparathyroidism ¢) Paget's disease Anewor (ERTREEIRY oor oor Ext Question Explanation: Paget's disease is a bone disorder that may be associated with paramyxowinis infections. Bone destruction is severe in Paget's disease, yet calcium levels remain normal. A diagnostic feature of Paget's disease is an increase in serum osteocalcin levels with increases in alkaline phosphatase levels, as well ation Report An Error ‘Mark this question e& => Question Id : 118390 Question 27 of 30 A.52 year old man complains of bone pain, Lab data show a normal calcium and phosphate level but alkaline phosphatase levels are elevated Urinalysis reveals an elevated level of hydroxyprotine and roentengram stadies demonstrate dense, enlarged bones. Osteocalcin level is increased. The most likely diagnosis is a) Pott’s disease b) Osteogenic sarcoma c) Hbrogenesis imperfect ossium 9) Hyperparathyroidism Y © e) Paget's disease Anewor (ERTREEIRY oor oor Ext Question Explanation: Paget's disease is a bone disorder that may be associated with paramyxowinis infections. Bone destruction is severe in Paget's disease, yet calcium levels remain normal. A diagnostic feature of Paget's disease is an increase in serum osteocalcin levels with increases in alkaline phosphatase levels, as well ation Report An Error ‘Mark this question =z Question Td: 119941 Question 28 of 30 4.55 year old woman who is taking procainamide for ber cardiac concition develops arthralgias, rash, myalgias, and a pleural effusion eight months later. She was found to have a fever of 101°. The anti-body titers that would be positive in this particular condition include a) Anti-double-stranded DNA antibody 'b) Anti-histone antibody ©) Anti-To-1 antibody 4d) Anti-SS-A antibody €) Ami-centromere antibody Question Explanation: “Anti-histone antibody is associated with drug induced inpus. This smidrome develops in approximately 15-20% of patients recesving hnydtalasine or procainamide. Mephitis or CINS involvement is not typical in this syndrome. Anti-double stranded DNVA antibodies ace prototype autoantibodies prominent in the sera of patients who have systemic lupus erythematosus. The tters are especially high ‘when lupus nephritis and hypocomplementemnia are present. Anti-Jo-1 antibody is found in patients who have polymyositis. Anti-SS- ‘A antibody is found in 75% of patients with Sjogren's syndrome. The passage of anti-SS-A antibody across the placenta has been. implicated in the development of complete heart block and cutaneous lupus in newborns. Ant-centromere antibody is directed against the constiction site of the chromosome in HEp-2 cells, the function of which is celular division. This pattern is found mn 50- 90% of patients with the CREST syndrome. ‘Mark this question =z Question Td: 119941 Question 28 of 30 A.55 ycar old woman who is taking procainamide for ber cardiac concition develops arthralgias, rash, myalgias, and a pleural effusion eight months later. She was found to have a fewer of 101°. The anti-body titers that would be positive in this paricular condition. include a) Anti-double-stranded DNA antibody Y © b) Anti-tistone antibody ©) Anti-To-1 artibody d) Anti-SS-A antibody €) Anti-centromere antibody Question Explanation: “Anti-histone antibody is associated with drug induced inpus. This smidrome develops in approximately 15-20% of patients recesving hnydtalasine or procainamide. Mephitis or CINS involvement is not typical in this syndrome. Anti-double stranded DNVA antibodies ace prototype autoantibodies prominent in the sera of patients who have systemic lupus erythematosus. The tters are especially high ‘when lupus nephritis and hypocomplementemnia are present. Anti-Jo-1 antibody is found in patients who have polymyositis. Anti-SS- ‘A antibody is found in 75% of patients with Sjogren's syndrome. The passage of anti-SS-A antibody across the placenta has been. implicated in the development of complete heart block and cutaneous lupus in newborns. Ant-centromere antibody is directed against the constiction site of the chromosome in HEp-2 cells, the function of which is celular division. This pattern is found mn 50- 90% of patients with the CREST syndrome. 2/26/2014 1:05:59 PM ‘Mark this question = => Question 29 of 30 “Which one of the following is commonly seen in osteoarthritis? a) Bamboo spine b) Chondrocaleinosis ©) DIP sclerosis 4) Peneil-in-cup erosions €) Ovethanging erosions Avewor (EQNS) other ucors Explanation Report An Evror Question Explanation: Question Id : 126053 (Osteoarthritis-DIP sclerosis-in contrast to rheumatoid arthritis, justa-articular bony sclerosis is commonly seen in osteoarthritis, especialy in the finger joints 2/26/2014 1:05:59 PM ‘Mark this question = => Question 29 of 30 “Which one of the following is commonly seen in osteoarthritis? a) Bamboo spine b) Chondrocaleinosis Y © 0) DD sclerosis 4) Peneil-in-cup erosions €) Ovethanging erosions Avewor (EQNS) other ucors Explanation Report An Evror Question Explanation: Question Id : 126053 (Osteoarthritis-DIP sclerosis-in contrast to rheumatoid arthritis, justa-articular bony sclerosis is commonly seen in osteoarthritis, especialy in the finger joints ‘Mark this question & Question Id : 126777 Question 30 of 30 74 year old female is dagnosed with pelymyalgia rheumatica, She also complains of a chronic headache. Low dose course of steroids improves her condition dramatically. The most lcely diagnosis is 2) Rheumstoid arthritis ) Dermatomyositis ©) Giant cell arteritis & Osteoarthritis ¢) None of the above Answer | Bolanation Other User's Explanation Report An Error Question Explanation: Giant cell arts, or temporal arteritis, is a vasculitis most commonly seen in elderly women, Ir involves the temporal arteries and is commonly seen in association with polymyalgia theumatica, Ibis highly responsive to steroid treatment, While rheurnatoid arthritis, dermatomyositis, and osteoarthntis may present with défuse muscle pain, they are not typically associated with chronic headaches ‘Mark this question & Question Id : 126777 Question 30 of 30 74 year old female is dagnosed with pelymyalgia rheumatica, She also complains of a chronic headache. Low dose course of steroids improves her condition dramatically. The most lcely diagnosis is a) Rheumatoid arthrtis, b) Dermatomyositis Y © 6) Giant cell arteritis & Osteoarthritis ¢) None of the above Answer | Bolanation Other User's Explanation Report An Error Question Explanation: Giant cell arts, or temporal arteritis, is a vasculitis most commonly seen in elderly women, Ir involves the temporal arteries and is commonly seen in association with polymyalgia theumatica, Ibis highly responsive to steroid treatment, While rheurnatoid arthritis, dermatomyositis, and osteoarthntis may present with défuse muscle pain, they are not typically associated with chronic headaches 2/26/2014 1:07:08 PM ‘Mark this question => (Question Td : 27039 Question 1 of 30 A.56-year-old female with a several month old MI complains of muscle aches and pains and is found to have an elevated creatine phosphokinase (CPE). She is on some medications to reduce recurrence of MI. Which of the following is the likely culprit? a) Carrot juice 6) Grapefiuit juice 6) Cranberry juice 4) Garic cloves €) Omega-3 fish oils Question Explanatio Grapefnt juice significantly increases serum concentrations of some statins. This is achieved by reducing the CYP3.A4-me-iated first-pass metabolism in the small intestine. On the other hand, grapefnut juice hhas no effect on the pharmacokinetics of pravastatm, sosuvastatin and fluvastatin Concomitant use of atorvastatin and large amounts of grapefruit juice should be avoided, or the dose of atorvastatin should be reduced accordingly CYP3A4 isa member of the cytochrome p450 inhibitor, thus may potentiate the elfects of warfarin. 2/26/2014 1:07:08 PM ‘Mark this question => (Question Td : 27039 Question 1 of 30 A.56-year-old female with a several month old MI complains of muscle aches and pains and is found to have an elevated creatine phosphokinase (CPE). She is on some medications to reduce recurrence of MI. Which of the following is the likely culprit? a) Carrot juice Y © b) Grapefuit juice 6) Cranberry juice 4) Garic cloves €) Omega-3 fish oils Question Explanatio Grapefnt juice significantly increases serum concentrations of some statins. This is achieved by reducing the CYP3.A4-me-iated first-pass metabolism in the small intestine. On the other hand, grapefnut juice hhas no effect on the pharmacokinetics of pravastatm, sosuvastatin and fluvastatin Concomitant use of atorvastatin and large amounts of grapefruit juice should be avoided, or the dose of atorvastatin should be reduced accordingly CYP3A4 isa member of the cytochrome p450 inhibitor, thus may potentiate the elfects of warfarin. ‘Mark this question = => Question Ta : 28292 Question 2 of 30 A, 25-year-old male has been receiving sulphasalazine for six months as treatment for Reiter's disease. His most recent series of blood tests were normal “When shouid he next be screened? a) Three weeks b) Two month c) Three months 4d) Seven months ©) One year Answer | Explanation Other User's Explanation Report An Error Question Explanation: Guidance suggests that during the first year of treatment with sulphasalazine, full blood count and liver function tests should be monitored every one to two weeks for the first three months then three monthly for the first year and then six monthly thereafter. Side effects of sulphasalazine include myelosuppression, macrocytosis, hypersensitivity and azoospermia in males. ‘Mark this question = => Question Ta : 28292 Question 2 of 30 A 25-year-old male has been receiving sulphasalazine for six months as treatment for Reiter‘s disease. His most recent series of blood tests were normal ‘When should ke next be screened? a) Three weeks b) Two month Y © 0) Three moaths 4) Seven months ) One year Answer | Explanation Other User's Explanation Report An Error Question Explanation: Guidance suggests that during the first year of treatment with sulphasalazine, full blood count and liver function tests should be monitored every one to two weeks for the first three months then three monthly for the first year and then six monthly thereafter. Side effects of sulphasalazine include myelosuppression, macrocytosis, hypersensitivity and azoospermia in males. 22672014 1:07:41 PM ‘Mark this question << => Question Td : 30514 Question 3 of 30 A17 years old gil presents with a history of polyarthralgia and marked early morting stiffiess for 3 months. Her symptoms are relieved by Diclofenac but she is becoming increasingly concerned about her symptoms which appear to be progressing. She is otherwise heathy apart from a history of acne which is well controlled on Minocycline, Her mother has severe rheumatoid arthritis, Investigations ESE 50 munfar (0-20) CRP 100 mat (<10) Bheamatoid fictor Negative ANA, Strongly positive (1: 1600) Anti-dsDIVA antibodies Negative kG 25 all (<15) “Wet is the most likely cause? a) Systemic Lupus Erythematosus ) Fibromyalgia c) Rheumatoid arthritis 4) Drug-induced SLE ¢) Sero-negative spondyloarthropathy Answer {Expisnation | Other User's Explanation Report An Error Question Explanation: The history strongly suggests an inflammatory problem and the elevated ESR and CRP confirm this. Rheumatoid arthritis and connective tissue disorder such as SLE would be on the dfferential diagnosis. The serology is atypical for theumatoid arthritis and the marked elevation ofthe CRP would be very unusual for SLE where characteristcally, CRP elevation indicates underlying bacterial infection or widespread serostis. The mos likely diagnosis is drug-induced SLE, Minocycline has been well documented as a cause of drug-induced SLE, Cheracteristically, the ESE. and CRP are both markedly elevated, the ANA is strongly positive and there is @ hypergammaglobulinaemia Asti-dsDNA antibodies are usualy negative. Symptoms usually improve following withdrawal ofthe drag but can take several months to resolve. 22672014 1:07:41 PM ‘Mark this question << => Question Td : 30514 Question 3 of 30 A17 years old gil presents with a history of polyarthralgia and marked early morting stiffiess for 3 months. Her symptoms are relieved by Diclofenac but she is becoming increasingly concerned about her symptoms which appear to be progressing. She is otherwise heathy apart from a history of acne which is well controlled on Minocycline, Her mother has severe rheumatoid arthritis, Investigations ESE 50 munfar (0-20) CRP 100 mat (<10) Bheamatoid fictor Negative ANA, Strongly positive (1: 1600) Anti-dsDIVA antibodies Negative kG 25 all (<15) “Wet is the most likely cause? a) Systemic Lupus Erythematosus ) Fibromyalgia c) Rheumatoid arthritis Y © d)Dniginduced SLE ¢) Sero-negative spondyloarthropathy Answer {Expisnation | Other User's Explanation Report An Error Question Explanation: The history strongly suggests an inflammatory problem and the elevated ESR and CRP confirm this. Rheumatoid arthritis and connective tissue disorder such as SLE would be on the dfferential diagnosis. The serology is atypical for theumatoid arthritis and the marked elevation ofthe CRP would be very unusual for SLE where characteristcally, CRP elevation indicates underlying bacterial infection or widespread serostis. The mos likely diagnosis is drug-induced SLE, Minocycline has been well documented as a cause of drug-induced SLE, Cheracteristically, the ESE. and CRP are both markedly elevated, the ANA is strongly positive and there is @ hypergammaglobulinaemia Asti-dsDNA antibodies are usualy negative. Symptoms usually improve following withdrawal ofthe drag but can take several months to resolve. 2/26/2014 10754 PM ‘Mare this question & => Question Td : 30613 Question 4 of 30 A.52 year old male has a history of lethargy, polydipsia, polyuna end pain and stifftess of the hands. Evidence of an arthropathy affecting the second and third metacarpophalangeal (MICP) joints is present on his both hands and X-ray of these sites reveal the evidence of degenerative disease. Hepatomegaly of Scmis also found in this patient. What is your lkely diagnosis? a) Gout 'b) Osteoarthritis c) Pyrophosphate arthropathy 4) Haemochromatosis ¢) Rheumatoid arthritis with amyloidosis Answer | Explanation Other User's Explanation Report An Error Question Explanation: These arc several rheumatic manifestations of hacmachrometosis. Classically there is a non-inflemmatory degenerative arthropathy affecting the second and third MCP joints with hook-like osteophytes on x ray. These joints are rarely involvedin primary osteoarthritis. Other joirts can also become involved in especially the hips, knee and shoulders anc occasionally the joint disease can resemble rheumatoid arthritis. Other cheumatic manifestations include acute pseadogout (pyrophosphete arthropathy) which present as an acute monoarthritis, asymptomatic choncrocalemosis and osteoporosis. 2/26/2014 10754 PM ‘Mare this question & => Question Td : 30613 Question 4 of 30 4.52 year old male has a history of lethargy, polydipsia, polyuna and pain and stifhiess of the hands. Evidence of an arthropathy affecting the second and third metacarpephalangeal (MCP) joints s present on his both hands and X-ray of these sites reveal the evidence of degenerative disease, Hepatomegely of Scmis also found inthis patient. What is your lcely diagnosis? a) Gout b) Osteoarthritis c) Pyrophosphate arthropathy Y © 4) Haemochromatosis 2) Rheumatoid arthritis with amyloidosis Answer | Explanation Other User's Explanation Report An Error Question Explanation: These arc several rheumatic manifestations of hacmachrometosis. Classically there is a non-inflemmatory degenerative arthropathy affecting the second and third MCP joints with hook-like osteophytes on x ray. These joints are rarely involvedin primary osteoarthritis. Other joirts can also become involved in especially the hips, knee and shoulders anc occasionally the joint disease can resemble rheumatoid arthritis. Other cheumatic manifestations include acute pseadogout (pyrophosphete arthropathy) which present as an acute monoarthritis, asymptomatic choncrocalemosis and osteoporosis. 2/26/2014 1:08:06 PM ‘Mark this question e&-=> Question 5 of 30 A. 66 year old male complains of bone pain especially in his spine. Lytic lesions in the vertebrae and skull were revealed on X-ray. The patienthad been anemic and hypercalcemic in the past. Which of the following is least likely to be present in this patient? a) Macroglobulinemia b) Bence Jones proteins c) Decreased resistance to infection ) Infitration of flat hones by plasma cells 2) Monoclonal gammopathy Question Explanation: ‘This is multiple myeloma. Macroglobulinemia is not typical of amutiple myeloma. 2/26/2014 1:08:06 PM ‘Mark this question e&-=> Question 5 of 30 A 66 year old male complains of bone pain especially in his spine. Lytic lesions in the vertebrae and skull were revealed on X-ray. The patient had been anemic and hypercalcemic in the past, Which of the following is least likely to be present in this patient? Y¥ © a) Macroglobulinemia b) Bence Jones proteins c) Decreased resistance to infection ) Infitration of flat hones by plasma cells 2) Monoclonal gammopathy Question Explanation: ‘This is multiple myeloma. Macroglobulinemia is not typical of amutiple myeloma. ‘Marke this question << => Question Td : 50509 Question 6 of 30 A.man develops swelling in his knee Sweling is red warm on examination An arthrocentesis is performed and comes back showing thomboid postively bifingent crystals. What is the most appropriate treatment? a) Allopurinol 'b) Probenacid ©) Colehucine 4) Indomethacin Question Explanation: Peeudogout (CPPD - caleiim pyrophosphate dihydrate crystal deposition disease) is a disorder caused by deposits of calcium pyrophosphate crystals in the cartilage and then in the uid of the joints, leading to intermittent attacks of painfil joint irflammation, Symptoms vary widely. Some people have attacks of painful joint inflammation, usually in the knees, wrists, or other relatively large joints. Diagnosis is by taking Quid from an inflamed joint through a needle (joint aspiration). Positively biftingent calcium pyrophosphate crystals (rather than urate crystals) are found in the joint fluid, Treatment is with nonsteroidal antiinflammatory drugs (NSAIDs) such, as indomethacin, ‘Marke this question << => Question Td : 50509 Question 6 of 30 A.man develops swelling in his knee Sweling is red warm on examination An arthrocentesis is performed and comes back showing thomboid postively bifingent crystals. What is the most appropriate treatment? a) Allopurinol b) Probenacid ©) Colehucine Y © 4) Indomethacin Question Explanation: Peeudogout (CPPD - caleiim pyrophosphate dihydrate crystal deposition disease) is a disorder caused by deposits of calcium pyrophosphate crystals in the cartilage and then in the uid of the joints, leading to intermittent attacks of painfil joint irflammation, Symptoms vary widely. Some people have attacks of painful joint inflammation, usually in the knees, wrists, or other relatively large joints. Diagnosis is by taking Quid from an inflamed joint through a needle (joint aspiration). Positively biftingent calcium pyrophosphate crystals (rather than urate crystals) are found in the joint fluid, Treatment is with nonsteroidal antiinflammatory drugs (NSAIDs) such, as indomethacin, ‘Mark this question & => Question Td : 50529 Question 7 of 30 A42 year old woman presents with painful swelling of the joints ofher fingers, excessive fatigue and a malar rash. She is most likely suffering from a) Psoriasis b) Pseudogout ©) Rheumatoid artheitis 4) Systemic lupus erythematosus (SLE) €) Serum sickness Avcwor (UEIAIRIRN) other Users Explanation Report An Evat Question Explanation: Systemic lupus erythematosus ss a chronic, mubisystem, inflammatory disorder of probable autoimraune etiology, occurring predominantly in young women, Common manifestations include arthralgas and arthritis malar end other skin rashes, pleuritis or pericarditis, renal or CNS involvement and hematologic cytopenia. Diagnosis requires clinical and serologic criteria Treatment of severe ongoing active disease requires corticosteroids, often hydroxychloroquine, and sometimes immuncsuppressants ‘Mark this question & => Question Td : 50529 Question 7 of 30 A.42 year old woman presents wih painful swelling of the joints of her fingers, excessive fatigue and a malar rash, She is most likely suffering from a) Psoriasis ) Pseudogout ©) Rheumatoid arthritis Y © d) Systemic lupus erythematosus (SLE) ¢) Serum sickness Avcwor (UEIAIRIRN) other Users Explanation Report An Evat Question Explanation: Systemic lupus erythematosus ss a chronic, mubisystem, inflammatory disorder of probable autoimraune etiology, occurring predominantly in young women, Common manifestations include arthralgas and arthritis malar end other skin rashes, pleuritis or pericarditis, renal or CNS involvement and hematologic cytopenia. Diagnosis requires clinical and serologic criteria Treatment of severe ongoing active disease requires corticosteroids, often hydroxychloroquine, and sometimes immuncsuppressants 4561. ‘Mark this question ez Question Td Question 8 of 30 ATA year old female presents with a pain in her hips and shoulders bilaterally for 1 month, accompanied by marked stiffness in the morting In addition, she reports a 4lb weight loss and fatigue. She denies fever, chills, sweats, nausea, vornting, swallowing disturbances, or charges in bowel habits. She specifically denies any visual symptoms of headache. Her physical examination is unremarkable, except for changes suggestive of osteoarthritis in the knees and hands, Laboratory testing reveals a hemoglobin level of 1LB g/4L. QV 13.0-16.0), a hematocrit of 36% (40-45), and an erythrocyte sedimentation rate of 84 mavhr (QT=20). The most appropriate management at this point is, a) Begin prednisone. 60 mg daily, and refer for a temporal artery biopsy as soon as possible ) Begin prednisone, 60 mg daily, and plan to refer for a temporal artery biopsy her symtoms do not respond c) Begin prednisone, 15 mg daily, with no plans for a biopsy 4) Begin a COX-2 inhibitor such as Rofecoxib (Views), with no plans for a biopsy ¢) Refer for a temporal artery biopsy and wat for the results befbre starting drug therapy Anowor UEIFNEEERIY ot: ere Explanation Report An Ever Question Explanation: Itis generally not considered necessary to refer patients with classic polymyalgia rheumatica for a temporal artery biopsy in the abseace of syraptoms or signs of giant cell atentis (e.g, headache, visual complaints, jaw claudication, fever, scalp tendemess, abnormal findascopic exam). Corticosteroids should be started at relatively low doses (10-20 mg daly) and the patient followed for what should be rapid clinical response. 4561. ‘Mark this question ez Question Td Question 8 of 30 A74 year old female preseats wit a pain in her hips and shoulders bilaterally for 1 month, accompanied by marked stiffness in the morning In addition, she reports 2 4lb weight loss and fetigue. She denies fever, chills, sweats, nausea, vomiting, swallowing disturbances, or changes in bowel habits. She specifically denies any visual symptoms of headache. Her physical examination is unremarkable, except for changes suggestive of ostecarthrtis in the knees and hands. Laboratory testing reveals a hemoglobin level of LL.B g/dL. QV 13.0-16.0), a hematocrit of 36% (IT 40-45), and an erythrocyte sedimentation rate of 84 mm/hr Q¥<20), The most appropriate management at this pointis a) Begin prednisone, 60 mg daily, and refer for a temporal artery biopsy as soon as possible ) Begin srednisone, €0 mg daily, and plan to refer for a temporal artery biopsy ifher symotoms do not respond JY © c) Begin prednisone, 15 mg daily, with no plans for a biopsy 4) Begin a COX-2 inhibitor such as Rofevoxib (Vioxx), with no plans for a biopsy ) Refer for a temporal artery biopsy and wat for the results before starting crug therapy Anowor UEIFNEEERIY ot: ere Explanation Report An Ever Question Explanation: Itis generally not considered necessary to refer patients with classic polymyalgia rheumatica for a temporal artery biopsy in the abseace of syraptoms or signs of giant cell atentis (e.g, headache, visual complaints, jaw claudication, fever, scalp tendemess, abnormal findascopic exam). Corticosteroids should be started at relatively low doses (10-20 mg daly) and the patient followed for what should be rapid clinical response. 2/26/2014 1:08:50 PM ‘Mark this question e& => (Question Id : 54701 Question 9 of 30 “Which is not associated with HLA-B27? a) Reiter syndrome ') Psoriatic Arthritis ©) Ankylosing Spondyitis 4) Reactive arthitis ©) Bebcet syndrome (Question Explanation: Spondyloarthropathies share certain clinical characteristics (eg, back pain, uveitis, GI symptoms, and rashes). Some are strongly associated with the HLA-B27 allele, Clinical and genetic similasities suggest that they also share similar causes or pathophysiologies. ‘Rheumatoid factor is negative in the spondyloarthropathies (hence, they are also called the seronegative spondyloarthropathies). They include ankylosing spondyltis, reactive arthritis, psoriatic arthrits and Reiter's synerome. 2/26/2014 1:08:50 PM ‘Mark this question e& => (Question Id : 54701 Question 9 of 30 “Which is not associated with HLA-B27? a) Reiter syndrome ') Psoriatic Arthritis ©) Ankylosing Spondyitis ) Reactive artis Y¥ © €) Behcet syndrome (Question Explanation: Spondyloarthropathies share certain clinical characteristics (eg, back pain, uveitis, GI symptoms, and rashes). Some are strongly associated with the HLA-B27 allele, Clinical and genetic similasities suggest that they also share similar causes or pathophysiologies. ‘Rheumatoid factor is negative in the spondyloarthropathies (hence, they are also called the seronegative spondyloarthropathies). They include ankylosing spondyltis, reactive arthritis, psoriatic arthrits and Reiter's synerome. ‘Mark this question e => Question Td : 59481 Question 10 of 30 The first line therapy for mild t> moderately severe psotiasis confined to the elbows and knees is a) Phototherapy using ultraviolet B light b) Methotrexate ©) Betamethasone dipropionate (Diprolene) 4) Btretinate (Tegison) (Question Explanation: The majority of psoriasis patients can be managed with topical agents such as betamethasone diproplonate. Systemic treatment is reserved for patients with disabling psotiass that does not respond the topical treatment. This would include phototherapy, methorexate, and etretinate. ‘Mark this question e => Question Td : 59481 Question 10 of 30 The first-line therapy for mild to moderately severe psoriasis confined to the elbows and knees is a) Phototherapy using ultraviolet B light b) Methotrexate Y © c) Betamethasone dipropionate (Diprolene) 4) Biretinate (Tegison) (Question Explanation: The majority of psoriasis patients can be managed with topical agents such as betamethasone diproplonate. Systemic treatment is reserved for patients with disabling psotiass that does not respond the topical treatment. This would include phototherapy, methorexate, and etretinate. 2/26/2014 1:09:17 PM ‘Mark this question => Question Id : 60309 Question 11 of 30 A.43-year-old female has dermatomycsitis. Which one of the following autoantibodies is seen in the highest frequency in dermatomyositis? a) Anticneutrophil cytoplasmic antibodies b) Anti-jo-1 antibodies c) Rheumatoid Factor 4) Anti-dsDNTA antibodies ¢) Anti-Ro antibodies Answer | Explanation Other User's Explanation Report An Error Question Explanation: ‘Asti-jo-1 antibodies are found in polymyositis and dermatomyositis. These are the only rheumatic diseases in which these types of antibodies. 2/26/2014 1:09:17 PM ‘Mark this question => Question Id : 60309 Question 11 of 30 A.43-year-old female has dermatomycsitis. Which one of the following autoantibodies is seen in the highest frequency in dermatomyositis? a) Anti-neutrophil cytoplasmic antibodies Y © b) Ami-jo-1 antibodies c) Rheumatoid Factor 4) Anti-dsDNTA antibodies ¢) Anti-Ro antibodies Answer | Explanation Other User's Explanation Report An Error Question Explanation: ‘Asti-jo-1 antibodies are found in polymyositis and dermatomyositis. These are the only rheumatic diseases in which these types of antibodies. ‘Mark this question ez Question Id: 62700 Question 12 of 30 A72 year old women presents with acute knee arthritis Radiographs reveal meriscal calcification (Chonckocalcinosis). Synovial fid analysis reveals weally positive bireffingent rhomboid shaped crystals. The crystals are most likely composed of a) Monosodum urate b) Calcium hydroxyapatite c) Cholesterol 4) Calcium pyrophosphate dihydrate Question Explanation: Calcium pyrophosphate dihydrate (CPED) crystal deposition discase involves intra articular endlor extra articular deposition of CPPD crystals. Manifestations are protean and may be minimal or include intermittent attacks of acute arthritis anda degenerative arthropathy that is often severe. Diagnosis requires identification of CPPD crystals in synovial fluid, Diagnosis is established by identfying rhomboid or rod-shaped, weakly positively birettingent crystals on polarized light microscopy of synovial fluid, ‘Mark this question ez Question Id: 62700 Question 12 of 30 A72 year old women presents with acute knee arthritis Radiographs reveal meriscal calcification (Chonckocalcinosis). Synovial fid analysis reveals weakly positive bireftingent rhomboid shaped crystals. The crystals are most likely composed of a) Monosodiuen urate 6) Calcium Lydroxyapatite c) Cholesterol Y © 4) Calcium pyrophosphate dihydrate Question Explanation: Calcium pyrophosphate dihydrate (CPED) crystal deposition discase involves intra articular endlor extra articular deposition of CPPD crystals. Manifestations are protean and may be minimal or include intermittent attacks of acute arthritis anda degenerative arthropathy that is often severe. Diagnosis requires identification of CPPD crystals in synovial fluid, Diagnosis is established by identfying rhomboid or rod-shaped, weakly positively birettingent crystals on polarized light microscopy of synovial fluid, 2/26/2014 1:09:40 PM ‘Mark this question = => Question 13 of 30 ‘All of the following features are commonly present in ostzoarthutis EXCEPT a) The DIP involvement ) The PIP involvement c) Metacarpophalangeal involvement ) Knee involvement @) Ankle involvement Question Explanation: Metacarpophalangeal involvement is a feature of rheumatoid arthritis. All of the other choices are common sites of osteoarthritis, 2/26/2014 1:09:40 PM ‘Mark this question = => Question 13 of 30 All of the following features are commonly present in osteoarthuitis EXCEPT a) The DIP involvement ) The PIP involvement Y © 0) Metacarpophalangeal involvement od) Knee iavelvemeat @) Ankle involvement Question Explanation: Metacarpophalangeal involvement is a feature of rheumatoid arthritis. All of the other choices are common sites of osteoarthritis, ‘Mark this question Question Td : 67356 Question 14 of 30 A.30 year old women has migratory arthrits of her ankles and wrists and a vesiculopustular skin eruption. She is afebrile, Physical examination reveals tenosynovitis of her waists, The most lixely diagnosis is a) Gonococcal arthritis b) Reiter's syndrome c) Rheumatoid arthritis 4) Meningococcemia ©) Systemic lupus erythematosus Answer | Explanation) Other User's Explanation Report An Error Question Explanation: Systemic lupus erythematosus is a chronic, multisystem, inflammetory disorder of probable autoimmune etiology, occurring predominantly in young women, Common manifestations include arthralgias and artitis, malar and other skin rashes, pleuritis or pericarditis, renal or CNS involvement; and hematologic cytopenia, Diagnosis requires clinical and serologic criteria Treament of severe ongoing active disease requires corticosteroids, often tydroxychloroquine, and sometimes inununosuppressents ‘Mark this question Question Td : 67356 Question 14 of 30 A.30 year old women has migratory arthrits of her ankles and wrists and a vesiculopustular skin eruption. She is afebrile, Physical examination reveals tenosynovitis of her waists, The most lixely diagnosis is a) Gonococeal arthritis ) Reiter's syndrome c) Rheumatoid arthritis 4) Meningococcemia JV © e) Systemic lupus erythematosus Answer | Explanation) Other User's Explanation Report An Error Question Explanation: Systemic lupus erythematosus is a chronic, multisystem, inflammetory disorder of probable autoimmune etiology, occurring predominantly in young women, Common manifestations include arthralgias and artitis, malar and other skin rashes, pleuritis or pericarditis, renal or CNS involvement; and hematologic cytopenia, Diagnosis requires clinical and serologic criteria Treament of severe ongoing active disease requires corticosteroids, often tydroxychloroquine, and sometimes inununosuppressents 2/26/2014 1:10:00 PM ‘Mark this question Question Td: 70110 Question 15 of 30 A73-year-old male with adenocarcinoma of the lung presents with prositnal muscle weakness and an elevated creatine phosphckinase level in bis serum, The likeliest diagnosis is a) Myasthenia gravis b) Inclusion body myositis, ©) Guillain-Barre syndrome 4) A parencoplastic syndrome @) Pressure effects of the cancer on the sympathetic chain Question Explanation: This patient most ely has a polymyostis-ilke syndrome which is paraneoplastic in origin, Le. itis a side-effect of the tumor itself Polyinyositis-dermatomyositis is one ofthe more common paraneoplastic syndromes in older adults. Myasthenia gravis can also be a paraneoplastic syndrome, butt does not produce elevated CPK levels Guillain-Barre syndrome, an ascending paralysis, can also be paraneoplastic, but it does not fit this picture. Inclusion body mycsitis is a viral ilness that is not associeted with tumors, Although lmng cancer can compress sympathetic chain, itis unlikely to produce proximal weakness. 2/26/2014 1:10:00 PM ‘Mark this question Question Td: 70110 Question 15 of 30 A.73-year-old male with adenocarcinoma of the ung presents with proximal muscle weakness and an elevated creatine phosphokinase level in his serum, The likeliest diagnosis is a) Myasthenia gravis ») Inclusion body myositis, ©) Guillain-Barre syndrome ¥ © d)Aparancoplastic syndrome. 2) Pressure effects of the cancer on the sympathetic chain. Question Explanation: This patient most ely has a polymyostis-ilke syndrome which is paraneoplastic in origin, Le. itis a side-effect of the tumor itself Polyinyositis-dermatomyositis is one ofthe more common paraneoplastic syndromes in older adults. Myasthenia gravis can also be a paraneoplastic syndrome, butt does not produce elevated CPK levels Guillain-Barre syndrome, an ascending paralysis, can also be paraneoplastic, but it does not fit this picture. Inclusion body mycsitis is a viral ilness that is not associeted with tumors, Although lmng cancer can compress sympathetic chain, itis unlikely to produce proximal weakness. ‘Mark. this question => Question Td = 70209 Question 16 of 30 A44-year-old men presents with fatigue, low-grade fever, and weakness, and complains that he can no longer clitub up into the cab ofhis truck. He most likely has a) An elevated creatine phosphokinase level ) Balanits ©) Headaches 4) A normal electromyogram. ©) A positive rheumatoid factor. Avewor [UEQNISNAN) other cor Explanation Report An Ever Question Explanation: ‘This patent's symptoms are most consistent with polymyositis, which is associated with an elevated CPK, but not with balatitis, head-aches, or positive sheumatoid factor. The electromyogram is usually abnormal in polymyositis. ‘Mark. this question => Question Td = 70209 Question 16 of 30 A44-year-old men presents with fatigue, low-grade fever, and weakness, and complains that he can no longer clitub up into the cab ofhis truck. He most iely has Y © a) Anelevated creatine phosphokinase level, ) Balan. c) Headaches d) A normal clectromyogram. €) A positive rheumateid factor. Avewor [UEQNISNAN) other cor Explanation Report An Ever Question Explanation: ‘This patent's symptoms are most consistent with polymyositis, which is associated with an elevated CPK, but not with balatitis, head-aches, or positive sheumatoid factor. The electromyogram is usually abnormal in polymyositis. ‘Mark this question =z Question Td: 72542 Question 17 of 30 A 53-year-old who had an upper respiratory tract infection presents with progressive weakness in the lower extremities. Deep- tendon reflexes ae decreased A few days later, he develops paralysis of both proximal and distal muscles of both extremities. CSF protrin is 108 mg/dl. What is the correct diagnosis? a) Myastheria gravis b) Multiple sclerosis ©) Shy-Drager syndrome 4) Guillain-Barre syndrome ©) Amyowrophic lateral sclerosis £) Dermatomyositis Question Explanation: Guillain-Barre syndrome is an acute immune polyneuropathy which mainly affects the motor nerves. Ttis associated with a prior upper respiratory infection or Campylobacter jejuni. Death can occur because this neurologic disease can cause paralysis of the respiratory system, Myasthenia gravis is an autoimmune disease caused by circulating antibodies against the acetylcholine receptors in the postsynaptic muscle membranes. It presents initially with extraocular muscle weekness causing ptoss and diplopia, Multiple sclerosis can present intialy with optic neuritis. Cerebral spinal-luid analysis reveals oigoclonal bands, ard the CT of the brain reveals white matter lesions Steroids are the treatment of choice. Initially Shy-Drager syndrome is caused by degeneration of the central autonomic neurons. Amyotrophic lateral sclerosis presents initially as upper extremity fasiculations ‘The cerebral spinal-fiuid analysis is normal. It is a motor neuron disease with degeneration of the anterior hom cells of the spinal cord Early on, deep tendon reflexes are preserved. Dermatomyositis usually involves skin lesions on the eyelids with puffiness, ‘Mark this question =z Question Td: 72542 Question 17 of 30 A 53-year-old who had an upper respiratory tract infection presents with progressive weakness in the lower extremities. Deep- tendon zeflexes are decreased. A few days later, he develops paralysis ofboth prozimal and distal muscles of both extremities. CSF protein is 108 mg/dl, What is the cozrect diagnosis? a) Myasthenia gravis 'b) Multiple sclerosis ©) Shy-Drager syndrome YM © 4) Guillain-Barre syndrome ©) Amyotrophic lateral sclerosis £) Detmatomyositis Question Explanation: Guillain-Barre syndrome is an acute immune polyneuropathy which mainly affects the motor nerves. Ttis associated with a prior upper respiratory infection or Campylobacter jejuni. Death can occur because this neurologic disease can cause paralysis of the respiratory system, Myasthenia gravis is an autoimmune disease caused by circulating antibodies against the acetylcholine receptors in the postsynaptic muscle membranes. It presents initially with extraocular muscle weekness causing ptoss and diplopia, Multiple sclerosis can present intialy with optic neuritis. Cerebral spinal-luid analysis reveals oigoclonal bands, ard the CT of the brain reveals white matter lesions Steroids are the treatment of choice. Initially Shy-Drager syndrome is caused by degeneration of the central autonomic neurons. Amyotrophic lateral sclerosis presents initially as upper extremity fasiculations ‘The cerebral spinal-fiuid analysis is normal. It is a motor neuron disease with degeneration of the anterior hom cells of the spinal cord Early on, deep tendon reflexes are preserved. Dermatomyositis usually involves skin lesions on the eyelids with puffiness, ‘Maric this question =r: Question 1d: 97665 Question 18 of 30 A.16 year old Mexican boy presents with pain and defortity of his sight hand and forearm. Radiograph of the hand and forearms demonsirates erosion of the terminal phalanges and mnatkced metaphyseal resorption of the ulna and sadus. The boy also has chronic renal failure but has not been teated with dialysis These problems are caused by excessive secretion of which of the following hormones? a) Aldosterone ) Calcitonin ©) Parathyroid hormone od) Renin e) Vitamin D (Question Explanation: Tn renal failure, the ability of the kidney to secrete phosphate is impaled The resultant byperphosphatemia causes hypocalcemia and triggers excretion of large amounts of parathyroid hormone. The released parathyroid hormone is a major contributor to bony changes (€.g., osteitis fibrosa, a form of localized bone resorption) seen in some patents with chronic renal failure, Aldosterone is a regulator of serum sodium, Calcitonin levels are usually low in chronic renal falure, unless the parathyroids have so hypertrophied as to cause tertiary hyperparathyroidism with hypercalcemia, Reain is normally secreted by the kidney and may be either decreased or increased in varying stages and forms of kidney disease, Regerdless, renin regulates blood pressure and aldosterone secretion, rather than bone metabolisin, The active form of vitamin D cholecaliferol is formed in the kidney from vitemin D absorbed from the gut and then processed by the liver. Uremia interrupts this pathway and consequertly causes a functional vitamin D deficiency. ‘Maric this question =r: Question 1d: 97665 Question 18 of 30 A.16 year old Mexican boy presents with pain and deformity of his sight hand and forearm. Radiograph of the hand and forearrn demonsirates erosion of the terminal phalanges and marked metaphyseal resorption of ihe ulna and radius. The boy also has chronic renal falure but has not been treated with dialysis, These problems are caused by excessive secretion of which of the following hormones? a) Aldosterone ) Calcitonin Y © ©) Parathyroid hormone ) Renin ) Vitamin D (Question Explanation: Tn renal failure, the ability of the kidney to secrete phosphate is impaled The resultant byperphosphatemia causes hypocalcemia and triggers excretion of large amounts of parathyroid hormone. The released parathyroid hormone is a major contributor to bony changes (€.g., osteitis fibrosa, a form of localized bone resorption) seen in some patents with chronic renal failure, Aldosterone is a regulator of serum sodium, Calcitonin levels are usually low in chronic renal falure, unless the parathyroids have so hypertrophied as to cause tertiary hyperparathyroidism with hypercalcemia, Reain is normally secreted by the kidney and may be either decreased or increased in varying stages and forms of kidney disease, Regerdless, renin regulates blood pressure and aldosterone secretion, rather than bone metabolisin, The active form of vitamin D cholecaliferol is formed in the kidney from vitemin D absorbed from the gut and then processed by the liver. Uremia interrupts this pathway and consequertly causes a functional vitamin D deficiency. 2/26/2014 1:10:48 PM ‘Mark this question = => Question Id : 89739 Question 19 of 30 A.20 year old Japanese woman presents with sudden left upper extremity hemiparesis and blindness in the left eye. Exanination demonstrates absent pulses in both arms and bruits in the carotid arteries. Her ESR is 100 mmfhr. The most likely diagnosis is a) Thromboangitis obliterans ) Temporal arteritis 6) Takayasu's arteritis 4) Cholesteral microembol €) Raynaud's disease Question Explanation: ‘This disease is more cornmon in Asian females ands called “pulseless disease." It is caused by vasculitis of arteries from an aortic arch. There is increase in collateral flow in the shoulder, chest, and neck areas. Presentation is usually a neurological defecit. Steroids or bypass acterial grafts ae the only treatmert. Thromboangitis oblierens is more common in young men who smoke There is an inflammatory occlusion of more distal arteries with loss of circulation in toes and lingers. Temporal arterts is more common n the elderly, Cholesterol microemb olism from atherosclerotic plaques can canse neurologic defects, but this is more commonly seen in the elderly Raynaud's disease is symmetric, intermittent attacks of painful pallor or cyanosis in the fingers which is precipitated by cold 2/26/2014 1:10:48 PM ‘Mark this question = => Question Id : 89739 Question 19 of 30 A.20 year old Japanese woman presents with sudden left upper extremity hemiparesis and blindness in the left eye. Exanination demonstrates absent pulses in both arms and bruits in the carotid arteries. Her ESR is 100 mmfhr. The most likely diagnosis is a) Thromboangitis obliterans ) Temporal arteritis JV © 6) Takayasu's arteritis 4) Cholesteral microembol €) Raynaud's disease Question Explanation: ‘This disease is more cornmon in Asian females ands called “pulseless disease." It is caused by vasculitis of arteries from an aortic arch. There is increase in collateral flow in the shoulder, chest, and neck areas. Presentation is usually a neurological defecit. Steroids or bypass acterial grafts ae the only treatmert. Thromboangitis oblierens is more common in young men who smoke There is an inflammatory occlusion of more distal arteries with loss of circulation in toes and lingers. Temporal arterts is more common n the elderly, Cholesterol microemb olism from atherosclerotic plaques can canse neurologic defects, but this is more commonly seen in the elderly Raynaud's disease is symmetric, intermittent attacks of painful pallor or cyanosis in the fingers which is precipitated by cold 2/26/2014 1:10:59 PM ‘Mark this question e& => Question Td : 95352 Question 20 of 30 Aa old woman presents to her physician that eyes and mouth are always dry. She also complains of sore, aching joints, which she says have limited her abiity to perform her daily activities. Aspiration of joint uid would most likely reveal which of the following? a) Calcium pyrophosphate crystals ) Excessive cartilage loss and reactive bone formation c) Monosodium urate crystals 4) Non proliferative synovitis with many neutrophils and gram negative dplococci ©) Proliferative synovitis with many lymphocytes macrophages, and plasma cells Answer | Berianaton |) Other User's Explanation Report An Error Question Explanation: Dey eyes and mouth in an elderly woman are probably due to Sjégren syndrome, which is an autoimmune attack on the salivary and tear glands. Sjégren syndrome is associated, in some cases, with rheumatoid arthritis, Join: findings in sheumetoid arthritis are highlighted by a proliferative synovitis with many lymphocytes, macrophages, and plasma cells. Osteowthritis appears to be due to repetitive low level trauma to joincs and is mariced by excessive cartlage less and reactive bone formation Gout and pseudogout can cause arthritis secondary to crystal deposition in joints. Gonococcal arthritis is a systemic complication of gonorrhea, typically observed in young women who have had a recent sexual encounter. 2/26/2014 1:10:59 PM ‘Mark this question e& => Question Td : 95352 Question 20 of 30 Aa old woman presents to her physician that eyes and mouth are always dry. She also complains of sore, aching joints, which she says have limited her abiity to perform her daily activities. Aspiration of joint uid would most likely reveal which of the following? a) Calcium pyrophosphate crystals ) Excessive cartilage loss and reactive bone formation c) Monosodium urate crystals 4) Non proliferative synovitis with many neutrophils and gram negative dplococci JV © =) Proliferative synovitis with many lymphocytes macrophages, and plasma cells Answer | Berianaton |) Other User's Explanation Report An Error Question Explanation: Dey eyes and mouth in an elderly woman are probably due to Sjégren syndrome, which is an autoimmune attack on the salivary and tear glands. Sjégren syndrome is associated, in some cases, with rheumatoid arthritis, Join: findings in sheumetoid arthritis are highlighted by a proliferative synovitis with many lymphocytes, macrophages, and plasma cells. Osteowthritis appears to be due to repetitive low level trauma to joincs and is mariced by excessive cartlage less and reactive bone formation Gout and pseudogout can cause arthritis secondary to crystal deposition in joints. Gonococcal arthritis is a systemic complication of gonorrhea, typically observed in young women who have had a recent sexual encounter. ‘Mark this question e= Question 21 of 30 Cinical sign that is described by pain on dorsiflexion is which one of the following? a) Grey Tumer’s sign >) Homan's sign c) Murphy's sign @) Cullen's sign ®) Peoas sign Question Explanation: Pancreatitis can produce both a periumbilical hematoma (Cullen's sign) andl a flank ecchymoses (Grey-Tumer’s sign). Homan's sign refers to pain on dorsiflexion in the presence ofa calf deep venous thrombosis, but is not very sensitive or specific. Abdominal pain under the right costal margin daring simultaneous palpation and inspiration is Murphy's sign, which can be found in acure cholecystitis, The psoas sign is when right leg extension produces abdominal pain, and is found in acute appendictis (the appendix overlies the psoas muscle). Crohn's disease, a subset of inflammatory bowel disease, is not commonly associated with any of these clnical signs. n Report An Etror ‘Mark this question e= Question 21 of 30 Cinical sign that is described by pain on dorsiflexion is which one of the following? a) Grey Turner's sign Y © >) Homan’s sign c) Murphy's sign @) Cullen's sign 2) Peoae sign Question Explanation: Pancreatitis can produce both a periumbilical hematoma (Cullen's sign) andl a flank ecchymoses (Grey-Tumer’s sign). Homan's sign refers to pain on dorsiflexion in the presence ofa calf deep venous thrombosis, but is not very sensitive or specific. Abdominal pain under the right costal margin daring simultaneous palpation and inspiration is Murphy's sign, which can be found in acure cholecystitis, The psoas sign is when right leg extension produces abdominal pain, and is found in acute appendictis (the appendix overlies the psoas muscle). Crohn's disease, a subset of inflammatory bowel disease, is not commonly associated with any of these clnical signs. n Report An Etror 2/26/2014 1:11:23 PM ‘Mark this question = => Question Td: 96710 Question 22 of 30 4.33 year old man of Middle Eastern ongin presented with uveitis, genital ulcers, and recurrent oral ulcers. Joint pain in the knees and ankles was also present, Investigaiions were positive for HLA-BS1. The most lkely diagnosis is a) Sarcoidoss +b) Systemic lupus erythematosus c) Beheet’s disease 4) Polyarteriis nodosa ©) Ankylosing spondyttis Question Explanation: Behcet's disease is a multisystem syndrome associated with uveitis and genital and oral uicers, as well as phlebitis and arthritis of the kenees. I occurs most frequently in the Mediterranean, the Middle East, and Japan, affecting men significently more often than wornen, Sarcoidosis is also a multisystem disease capable of causing uveitis and arthralgia. However, it manifests most often in the Jungs and tends to affect northern Europeans and American Blacks more than other ethnic groups. Systemic lupus erythematosus is more common in women, Itis an autoimmune disease associated with anti miclear antbody and symptoms of serositis, cerebritis, and renal disease. Polyarteriis nodosais a disease of unknown orign associated with large vessel vasculitis, Isis more common in tales and is associated with renal disease and renal artery aneurysms. Ankylosing spondylitis is associated with HLA-B27. It is more common in males. Symptomatic sacroilitis is the hallmark of this disease. 2/26/2014 1:11:23 PM ‘Mark this question = => Question Td: 96710 Question 22 of 30 4.33 year old man of Middle Eastern ongin presented with uveitis, genital ulcers, and recurrent oral ulcers. Joint pain in the knees and ankles was also present, Investigaiions were positive for HLA-BS1. The most lkely diagnosis is a) Sarcoidoss +b) Systemic pus erythematosus Y © ¢) Behret's disease 4d) Polyarteritis nodosa €) Ankylosing spondyttis Question Explanation: Behcet's disease is a multisystem syndrome associated with uveitis and genital and oral uicers, as well as phlebitis and arthritis of the kenees. I occurs most frequently in the Mediterranean, the Middle East, and Japan, affecting men significently more often than wornen, Sarcoidosis is also a multisystem disease capable of causing uveitis and arthralgia. However, it manifests most often in the Jungs and tends to affect northern Europeans and American Blacks more than other ethnic groups. Systemic lupus erythematosus is more common in women, Itis an autoimmune disease associated with anti miclear antbody and symptoms of serositis, cerebritis, and renal disease. Polyarteriis nodosais a disease of unknown orign associated with large vessel vasculitis, Isis more common in tales and is associated with renal disease and renal artery aneurysms. Ankylosing spondylitis is associated with HLA-B27. It is more common in males. Symptomatic sacroilitis is the hallmark of this disease. 2/26/2014 1:11:34 PM ‘Mark this question e& => Question Id: 119231 Question 23 of 30 “What is a major distinguishing characteristic of the urinalysis in patients with acute thabdomyolysis? a) Elevated CPK levels b) Hemataria. ©) Myoglobinaria ) Oxalate crystals €) Hyaline casts Ancwor [UEISINMIAN) other User's Explanation Report An Exar (Question Explanation: MMyoglobinuria occurs in acute rhabdomyolysis as a consequence of acute reuscle damage. Elevated CPK levels cen occur in many other muscle diseases as well. Hematuria is not a common manifestation of rhabdomyolysis. Oxalate crystals are not typical of thabdomyolysis, nor are hyaline casts, 2/26/2014 1:11:34 PM ‘Mark this question e& => Question Id: 119231 Question 23 of 30 “Whar is a major distinguishing characteristic of the urinalysis in patients with acute thabdomyolysis? a) Elevated CPE levels ) Hemaburia Y © ©) Myoglobimuria. ) Oxalate crystals ©) Hyaline casts Ancwor [UEISINMIAN) other User's Explanation Report An Exar (Question Explanation: MMyoglobinuria occurs in acute rhabdomyolysis as a consequence of acute reuscle damage. Elevated CPK levels cen occur in many other muscle diseases as well. Hematuria is not a common manifestation of rhabdomyolysis. Oxalate crystals are not typical of thabdomyolysis, nor are hyaline casts, 2/26/2014 1:11:47 PM ‘Mark this question e& => Question Id: 119654 Question 24 of 30 A.42 year old woman with lupus erythematous presents with the signs and symptoms of active systemic lupus erythematosus. The physician should do which one of the following, in addition for treatment for acute lupus emergency? a) Cross match the patient for four units +) Stop all corticosteroids ) Rule out or treat the patient for infection. &) Start prophylactic anticonwulsive therapy *) Start IV heparin Question Explanation: Paticats with lupus are very susceptible to severe systemic infections that complicate the discase. Infection is often not distingnishable fiom the active systemic lupus and should be treated until infection is ruled out as it can be life threatening Blood transfusions are not part of the acute treatment of lupus, and conticosteroid treatment is essential and should not be stopped. Whie lupus may lead to seizures, prophylanis is generally not recommended. TV heparin should be avoided, as these patients can present wth pulmonary hemorrhage, 2726/2014 PM ‘Mark this question e& => Question 24 of 30 A.42 year old woman with lupus erythematous presents with the signs and symptoms of active systemic lupus erythematosus. The physician should do which one of the following, in addition for treatment for acute lupus emergency? a) Cross match the patient for four units ») Stop all corticosteroids Y © c) Rule out or treat the patient for infection. &) Start prophylactic anticonwulsive therapy 2) Start IV heparin Answer Bariananon Other User's Explanation Report An Error Question Explanation: Paticats with lupus are very susceptible to severe systemic infections that complicate the discase. Infection is often not distingnishable fiom the active systemic lupus and should be treated until infection is ruled out as it can be life threatening Blood transfusions are not part of the acute treatment of lupus, and conticosteroid treatment is essential and should not be stopped. Whie lupus may lead to seizures, prophylanis is generally not recommended. TV heparin should be avoided, as these patients can present wth pulmonary hemorrhage, 26/2014 1:12:00 PM ‘Matte this question & => Question 25 of 30 “Which one of the following is associated with psoriatic arthritis? a) Bamboo spine b) Chondrocaleinosis c) DIP sclerosis 4) Justa-articular demineralization ©) Pencil in cup erosions Answer | Bxpianation | Other User's Explanation Report An Error Question Explanation: Psoriatic arthitis-pencil-in-cup erosions-this is the classic radiographic finding in this condition. 26/2014 1:12:00 PM ‘Matte this question & => Question 25 of 30 “Which one of the following is associated with psoriatic arthritis? a) Bamboo spine b) Chondrocaleinosis c) DIP sclerosis 4) Justa-articular demineralization SY © e) Pencil in cup erosions Answer | Bxpianation | Other User's Explanation Report An Error Question Explanation: Psoriatic arthitis-pencil-in-cup erosions-this is the classic radiographic finding in this condition. 2/26/2014 1:12:24 PM ‘Mark this question & => Question 26 of 30 ‘Which of the following is commonly seen in gout? 2) Chondrocaleinosis ) DIP sclerosis ©) Tuxta-atticular demineralization 4) Pencil-in-cup erosions €) Overhanging erosions newer (EQISIRHNYY other Users Explanation Report An Enor Question Explanatio (Gout-overhanging erosions-this is the pattern often seen when. unc acid deposits damage cartilage. Question Id ; 126063 2/26/2014 1:12:24 PM ‘Mark this question & => Question 26 of 30 ‘Which of the following is commonly seen in gout? 2) Chondrocaleinosis ) DIP sclerosis ©) Tuxta-atticular demineralization 4) Pencil-in-cup erosions Y © €) Overhanging erosions anower [UEQISIRHNYY other Users Explanation Report An Exar Question Explanatio (Gout-overhanging erosions-this is the pattern often seen when. unc acid deposits damage cartilage. Question Id ; 126063 ‘Mark this question e => 133502 Question 27 of 30 “Which of the following condition is NOT frequently associated with the HLA-B27 antigen? a) Ankylosing spondyitis 6) Tleerative colitis. ©) Psoriatic arthritis ) kets ) Scleroderma Avewor (NEQNGNRANY) osherucersExplanation Report An Eos (Question Explanation: ‘As many as 90% of Cancasian Americans with ankylosing sponcylitis are HLA-B27 positive, with slightly lower percentages in other efinic groups. An increased incidence of HLA-B27 positivity is found in people with ulcerative colts, especially those with spondyltis ancVor sacrotits, individuals with psoriatc arthritis, and patients with iris, which is often associated with the other HLA- B27-positive conditions ‘Mark this question e => 133502 Question 27 of 30 “Which of the following condition is NOT frequently associated with the HLA-B27 antigen? a) Ankylosing spondyitis 6) Tleerative colitis. ©) Psoriatic arthritis ) kets Y © 2) Scleroderma Avewor (NEQNGNRA) other UcersExplanation Report An Exos (Question Explanation: ‘As many as 90% of Cancasian Americans with ankylosing sponcylitis are HLA-B27 positive, with slightly lower percentages in other efinic groups. An increased incidence of HLA-B27 positivity is found in people with ulcerative colts, especially those with spondyltis ancVor sacrotits, individuals with psoriatc arthritis, and patients with iris, which is often associated with the other HLA- B27-positive conditions 2726/2014 1:12:46 PM ‘Mark this question e& => Question Id : 136222 Question 28 of 30 ‘Which one of the following is NOT a common manifestation of progressive systemic sclerosis? a) Ramaud's b) Pulmonary hypertension. ©) Pericarditis, 4) Low complement levels e) Malabsorption, Answer | Berianation Other User's Explanation Report An Error Question Explanation: Low complement levels are not typical of progressive systemic sclerosis, bur Raynaud's, pulmonary hypertension, pericarditis, and malabsorption are all associated with this condition, 2726/2014 1:12:46 PM ‘Mark this question e& => Question Id : 136222 Question 28 of 30 “Which one of the folowing is NOT a commen manifestation of progressive systemic sclerosis? a) Ramaud's b) Pulmonary hypertension. ©) Pericarditis Y © &) Low complement levels e) Malabsorption. Answer | Berianation Other User's Explanation Report An Error Question Explanation: Low complement levels are not typical of progressive systemic sclerosis, bur Raynaud's, pulmonary hypertension, pericarditis, and malabsorption are all associated with this condition, ‘Mark this question == Question 29 of 30 A.35 year old patent presents because of a pani right wrist, Examination reveals thet the toe is exquisitely painful, indamed, and ‘warm, Similar condition has happened before, generally occurring the moming after a previous evening's ‘partying "Toint fuid aspiration would most likely show a) Giant cells and acid fast positive rods b) Needle like crystals thet are strongly negatively bireftingent c) Numerous neutrophils and gram negative rods 4) Numerous neutrophils and intracellular gram-negative cocci 2) Roughly Cuboidal csystals that are weakly positively bireffinzent Question Explanation: The disease is acute gout, which ie characterized by strongly negatively birefbingent needle like crystals of uric acid in the joint uid. ‘An acute gouty arthritis is the common endpoint of a group of disorders that produce hyperuricemia, resulting in the deposition of monosodium urate within the joint space. These include increased aucleic acid tamover leading to hyperuricemia found with leukemia, reduced excretion with chronic renal disease, overproduction with Lesch Nyhan syndrome (HGPRT deficiency), impaired excretion with alcohol use, as well as unknown enzyme defects (primary gout). Tubercular arthritis causes granuioma formation with giant cells and acid fast mycobacteria, Infectious arthritis due to Salmonella (sickle cell patients) or Haemophilus (children) would be associated. with neutrophils and grain negetive rods in joint uid. Infectious arthritis due to N. gonorrhoeae would be associated with neutrophils and intracelular gram-negative cocci. Calcium pyrophosphate (pseudogout) produces roughly cuboidal crystals that ale weakly positively bireffingent ‘Mark this question == Question 29 of 30 A.35 year old patent presents because of a pani right wrist, Examination reveals thet the toe is exquisitely painful, indamed, and ‘warm, Similar condition has happened before, generally occurring the moming after a previous evening's ‘partying "Toint fuid aspiration would most likely show a) Giant cells and acid fast positive rods Y © ») Needle like crystals thet are strongly negatively birefringent c) Numerous neutrophils and gram negative rods 4) Numerous neutrophils and intracellular gram-negative cocci @) Rovahly Cubsidal erystals that are weakly positively bireftingent Question Explanation: The disease is acute gout, which ie characterized by strongly negatively birefbingent needle like crystals of uric acid in the joint uid. ‘An acute gouty arthritis is the common endpoint of a group of disorders that produce hyperuricemia, resulting in the deposition of monosodium urate within the joint space. These include increased aucleic acid tamover leading to hyperuricemia found with leukemia, reduced excretion with chronic renal disease, overproduction with Lesch Nyhan syndrome (HGPRT deficiency), impaired excretion with alcohol use, as well as unknown enzyme defects (primary gout). Tubercular arthritis causes granuioma formation with giant cells and acid fast mycobacteria, Infectious arthritis due to Salmonella (sickle cell patients) or Haemophilus (children) would be associated. with neutrophils and grain negetive rods in joint uid. Infectious arthritis due to N. gonorrhoeae would be associated with neutrophils and intracelular gram-negative cocci. Calcium pyrophosphate (pseudogout) produces roughly cuboidal crystals that ale weakly positively bireffingent oo a EEE € 2 © DB www. interface.edupk/mecical-exams/test-analysis pprutid=14759 Apps [9 Google A Settings [) Signin CI Imported FromlE 2... abe dole Eo yo Geil Sap we Other bookmar ‘Maske this question <= (Question Id : 213745 Question 30 of 30 ‘A 21-year-old male has chronic low back pain for the past 8 months. It begins frequently at aight, radiates down the thighs clong, pronounced stiffiess and moderate limitation of back motion, Tendemess is +ve. No GITIGUT symptoms exist A diastolic murmur along the left steal border exists. ESR and RF are negative. His x-ray fln is shown below. ib ‘Which of the followng is the nest best step in management? a) Intra-articular corticosteroid injection b) Lew-dose oxycodone c) Methotrexate 4) Nonsteroidal anti-inflammatory drugs ©) Oral prednisone 8 Spinal bracing Answer | Baplanation | Other User's Explanation Report An Error Question Explanation: The paiient’s young age, occurrence of pain at nigh, negativity of rheumatoid factor and especially, characteristic bamboo spine seen. on his x-ray are corsistent with ankylosing spondylitis. This is one of the seronegative spondyloarttropathies, characterized by onset before 40 years of age, absence of circulating autoantibodies, frequent association with HL.A-B127 histocompatibiity antigen, and commen involvement of the spinal column Ankylosing spondylis should be suspected in any young person complaining of chronic low back pain and confirmed by radiographs or CT scans of sacroiliac jomts The disease usually progresses to involve the whole vertebral column (as in the image above), producing ankylosis and respiratory failure secondary to restrictive lung disease, Treatment initially consists of NSAIDS, which s the manstay of therapy. In those with breakthrough symptoms, a low-dose opicte such as oxycodone may be added. Physical therapy and exercise are also extremely important in the treatment of these patients Jn those with persistent peripheral joint disease and sacroiltis, intra-articular corticosteroid injections may be used as adjanctive therapy, Oral steroids are generally avoided in the treaiment of ankylosing spondylitis because their effectis thought to be oflittle value and other reasonable akernatives are available. Disease-modifving antiheumatic drugs, or DMARDS, including methotrexate, can be used in those who are unresponsive to the use of NSAIDS and other conservative measures Spinal bracing is indicated in the management of scoliosis and is not usefill in ankvlosing spondvitis. Total Questions Ig RO OK I IB bs to SRBNSBRBRRBEBRBE xxxXKXXXKKXKXKKKXKKKKXKXKKXKKKKKKKKKKM oo a EEE € 2 © DB www. interface.edupk/mecical-exams/test-analysis pprutid=14759 Apps [9 Google A Settings [) Signin CI Imported FromlE 2... abe dole Eo yo Geil Sap we Other bookmar ‘Maske this question <= (Question Id : 213745 Question 30 of 30 ‘A 21-year-old male has chronic low back pain for the past 8 months. It begins frequently at aight, radiates down the thighs clong, pronounced stiffiess and moderate limitation of back motion, Tendemess is +ve. No GITIGUT symptoms exist A diastolic murmur along the left steal border exists. ESR and RF are negative. His x-ray fln is shown below. ib “Which of the followng is the next best step in management? a) Intra-articular corticosteroid injection b) Low-dose oxysodone c) Methotrexate Y © d) Nonsteroidal anti-inflammatory drugs ©) Oral prednisone £) Spinal bracing Answer | Baplanation | Other User's Explanation Report An Error Question Explanation: The paiient’s young age, occurrence of pain at nigh, negativity of rheumatoid factor and especially, characteristic bamboo spine seen. on his x-ray are corsistent with ankylosing spondylitis. This is one of the seronegative spondyloarttropathies, characterized by onset before 40 years of age, absence of circulating autoantibodies, frequent association with HL.A-B127 histocompatibiity antigen, and commen involvement of the spinal column Ankylosing spondylis should be suspected in any young person complaining of chronic low back pain and confirmed by radiographs or CT scans of sacroiliac jomts The disease usually progresses to involve the whole vertebral column (as in the image above), producing ankylosis and respiratory failure secondary to restrictive lung disease, Treatment initially consists of NSAIDS, which s the manstay of therapy. In those with breakthrough symptoms, a low-dose opicte such as oxycodone may be added. Physical therapy and exercise are also extremely important in the treatment of these patients Jn those with persistent peripheral joint disease and sacroiltis, intra-articular corticosteroid injections may be used as adjanctive therapy, Oral steroids are generally avoided in the treaiment of ankylosing spondylitis because their effectis thought to be oflittle value and other reasonable akernatives are available. Disease-modifving antiheumatic drugs, or DMARDS, including methotrexate, can be used in those who are unresponsive to the use of NSAIDS and other conservative measures Spinal bracing is indicated in the management of scoliosis and is not usefill in ankvlosing spondvitis. Total Questions Ig RO OK I IB bs to SRBNSBRBRRBEBRBE xxxXKXXXKKXKXKKKXKKKKXKXKKXKKKKKKKKKKM 2/27/2014 1:57:48 PM ‘Maric this question => Question Id: 28272 Question 1 of 10. A. 29-year-old has a3 month history of arthralgia aloag swelling of the distal interphalangeal joints of the middle and ring fingers of the hhand and wrist on the right plus a swollea lef ankle. She had no medical history ofnote. Her ESR=41 mmr. Diagnose a) Acute exacerbation of osteoarthritis b) Rheumatoid arthritis c) Sjogren's syndrome 4) Psoriatic arthropatty ) Systemic iupus erythematous Question Explanation: ‘This woman has psoriatic arthritis. Synovits is indicative of an inflammatory arthritis Rheumatoid atthnis typically effects the metacarpophalengeal and proximal interphalangeal joints and tends to be asymmetrical, Joint involvement in symmetric lupus erythematous occurs in the form of a polyarticular arthralgia, frequently symmettical and episodic Ttense tendoritis is more common than synovitis and can lead to deforming reversible subluzation of joints without erosive disease (laccoud’s arthropathy). ‘A short, shking history of marked acute polyarticular symptoms occur with systemic (vital) infection. Sjogren's syndrome will usually involve sheumatoid acthritis along other clinical findings. 2/27/2014 1:57:48 PM ‘Maric this question => Question Id: 28272 Question 1 of 10. A. 29-year-old has a3 month history of arthralgia aloag swelling of the distal interphalangeal joints of the middle and ring fingers of the hhand and wrist on the right plus a swollea lef ankle. She had no medical history ofnote. Her ESR=41 mmr. Diagnose a) Acute exacerbation of osteoarthritis b) Rheumatoid arthritis c) Sjogren's syndrome V © @ Psoriatic arttropathy ) Systemic iupus erythematous Question Explanation: ‘This woman has psoriatic arthritis. Synovits is indicative of an inflammatory arthritis Rheumatoid atthnis typically effects the metacarpophalengeal and proximal interphalangeal joints and tends to be asymmetrical, Joint involvement in symmetric lupus erythematous occurs in the form of a polyarticular arthralgia, frequently symmettical and episodic Ttense tendoritis is more common than synovitis and can lead to deforming reversible subluzation of joints without erosive disease (laccoud’s arthropathy). ‘A short, shking history of marked acute polyarticular symptoms occur with systemic (vital) infection. Sjogren's syndrome will usually involve sheumatoid acthritis along other clinical findings. 2/27/2014 1:58:00 PM ‘Maris this question & => Question Id : 30698 Question 2 of 10 A.44 year old male attends you for an insurance medical and is in good health. Examination was normal but investigations reveal that he has a senum urate concentration of 0.55 mmol/L (0.25-0.45). Which management is most appropriate in this patient? a) Start Allopurinol b) Start Colchicine c) Lifestyle advice 4) Start Diclofenac ) Stat Prednisolone ower EPIRA) ote: users Explanation Repost AnEvor Question Explanation: The most appropriate treatment for this asymptomatic man with an isolated slightly elevated urate is lifestyle adrice wth an appropriately reduced purine diet, increased exercise and reduced alcohol consumption. 2/27/2014 1:58:00 PM ‘Maris this question & => Question Id : 30698 Question 2 of 10 A.44 year old male attends you for an insurance medical and is in good health. Examination was normal but investigations reveal that he has a senum urate concentration of 0.55 mmol/L (0.25-0.45). Which management is most appropriate in this patient? a) Start Allopurine! ) Stat Colchicine Y © c) Lifestyle advice 4) Start Diclofenac ) Stat Prednisolone ower EPIRA) ote: users Explanation Repost AnEvor Question Explanation: The most appropriate treatment for this asymptomatic man with an isolated slightly elevated urate is lifestyle adrice wth an appropriately reduced purine diet, increased exercise and reduced alcohol consumption. 2:27 2014 1:58:11 PM. ‘Marie this question & => Question Td : 51749 Question 3 of 10 ATS year old black female has moderetely severe pain due to osteoarthritis. She is also on medication for a seizure disorder. Whea choosing medications to management her chronic pain, which one of the following should be used with caution because of her history of seizures? a) Salsalate Disalvid) b) Rofecoxib (iow) c) Hydrocodone (Lortab) 4) Oxycodone (OxyContin) e) Tramadol (tram) Answer | Bplanation Other User's Explanation Report An Error Question Explanation: According to the clinical practice guidelines for management of persistent pain in older persons, tramadol has efficacy and safety sitnilar to those of equienalgesic doses of codzine and hydrocodone, However, because of the threat of seizures (rare but potential), tramadol should be used with caution in patients with a history of seizure disorder or those taking other medications that lower seizure thresholds, 2:27 2014 1:58:11 PM. ‘Marie this question & => Question Td : 51749 Question 3 of 10 ATS year old black female has moderetely severe pain due to osteoarthritis. She is also on medication for a seizure disorder. Whea choosing medications to management her chronic pain, which one of the following should be used with caution because of her history of seizures? a) Salsalate (Disaleid) +) Rofecouib (Vix) c) Hydrocodone (Lortab) 4) Oxycodone (OxyContin) WV © ¢) Tramadol (Ctra) Answer | Bplanation Other User's Explanation Report An Error Question Explanation: According to the clinical practice guidelines for management of persistent pain in older persons, tramadol has efficacy and safety sitnilar to those of equienalgesic doses of codzine and hydrocodone, However, because of the threat of seizures (rare but potential), tramadol should be used with caution in patients with a history of seizure disorder or those taking other medications that lower seizure thresholds, ‘Maric this question = => Question Td : 54641 Question 4 of 10 271 year old women suffered a lumbar vertebral fracture 3 years ago. At that time she had a dual energy K-ray absorptiometry DEXA) scan, wilh aT score of -2.6, and was placed on alendronate (Fosamax), calcium, and vitamin D. She recently quit smoking Her BMLis 21. & DEXA scan today shows her bone mineral density to be -2.1. Most appropziate in the management of this patient is a) Replace alendronate with raloxifene (Evista) ) Stop alendronate, but continue calcium and vitarnin D ©) Add ralosifene to her regimen ) Make no change to her regimen ¢) Add teriparatide Fortzo) to her regimen) Anewor (EIPMNEER) oer cre Explanation Report AnEvor Question Explanation: Even though the patient's DEA has improved and she is techaically osteopenc, she stil has risk factors for osteoporosis, including recent smoking, low BMI, and a prior fragilty fracture. She should continue her current regimen ‘Maric this question = => Question 4 of 10 Question Id : 54641 ATI year old woman suffered a lumbar vertebral fracture 3 years ago. At that time she had a dual energy X-ray absorptiometry (DEXA) scan, with a T score of -2.6, and was placed on alendronate Fosamax), calciun, and vitanin D. She recently quit smoking Her BMTis 21. A DEXA scan today shows her bone mineral density to be -2.1. Most appropriate in the management of this patient is a) Replace alendronate with raloxifene (Evista) ) Stop alendronate, but continue calcium and vitarnin D ©) Add ralosifene to her regimen Y © d) Make no change te her regimen, ¢) Add teriparatide Fortzo) to her regimen) Anewor (EIPMNEER) oer cre Explanation Report AnEvor Question Explanation: Even though the patient's DEA has improved and she is techaically osteopenc, she stil has risk factors for osteoporosis, including recent smoking, low BMI, and a prior fragilty fracture. She should continue her current regimen ‘Mark this question e => Question Td : 62570 Question 5 of 10 A.women complains of a severe heatlache and jaw pain when she chews. Complains of shoulder pain and bilateral wrist weakness are also present. Initial lab tests show an ESR of 75 (normal is less than 30). The most appropriate diagnosis is a) Temporal Artertis b) Rheumatoid Arthritis c) Polymyositis 4d) Sarcoidosis Question Explanation: Temporal arteritis is a chronic inflammatory disease of large blood vessels, particularly the carotid artery and its branches, occurring primarily in older adults. Simutaneously polymyalgia rheumatica is common, Focal symptoms and signs may include headaches, vistal disturbances, temporal artery tenderness, and pain in the jaw muscles while chewing Fever, weight loss, malaise, and fatigue are also common, ESR is typically very high. Diagnosis is most often by biopsy. Treatment with high dose corticosteroids is usvally highly effective and prevents visual loss. ‘Mark this question e => Question Td : 62570 Question 5 of 10 A.women complains of a severe heatlache and jaw pain when she chews. Complains of shoulder pain and bilateral wrist weakness are also present, Initial ab tests show an ESR of 75 (normal is less than 30). The most appropriate diagnosis is Y © a) Temporal Artertis b) Rheumatoid Arthritis c) Polymyositis 4d) Sarcoidosis Question Explanation: Temporal arteritis is a chronic inflammatory disease of large blood vessels, particularly the carotid artery and its branches, occurring primarily in older adults. Simutaneously polymyalgia rheumatica is common, Focal symptoms and signs may include headaches, vistal disturbances, temporal artery tenderness, and pain in the jaw muscles while chewing Fever, weight loss, malaise, and fatigue are also common, ESR is typically very high. Diagnosis is most often by biopsy. Treatment with high dose corticosteroids is usvally highly effective and prevents visual loss. 2/27/2014 1:58:46 PM ‘Mark this question << => Question 6 of 10 A 22-year-old woman suddenly develops total paralysis of the extremities and trunk after eating a large carbohydrate dinner. This has occurred in the past with recuirent attacks of flaccid weakness. Her grandmother also had sirailar symptoms. Her serum potassium is 2.5 meqL and her giucose is 120 mg/dl. What is the diagnosis? a) Embole stroke b) Myotcaia congenital ©) Thyrotoxicoss 4) Hyperaldosteroaisim ©) Hypokalemic periodic paralysis Answer | Explanation Other User's Explanation Report An Error Question Explanation: Hypokalemic periodic paralysis is a genetic disease, and sudden paralysis can occur when the potassium level drops. Hyperkalemic periodic paralysis is ancther entity when the potassium level is elevaied Muscle biopsy would reveal vacuoles within muscle Sbers. High carbohydrate intake leads to increased movement of glucose and potassium into skeletal mascle, leading to an acute state of severe bypokelemia, Embelic stroke would be uncommon in this age group. Tk would also cause focal neurological deficits. Myotonia congenital is arate disorder. The paiient becomes stiff when he exercises and the muscles loosen up as activity continues. Cold ex- acerbates the symptoms. Thyrotoxicosis would be caused by excess thyroid hormone-producing tachycardia, ciaphoresis, and hypetreflena Grave's disease is the most common etiology Hyperaldosteronism would cause hypertension, hypokalemia, and hypematremia Paralysis is uncommon. 2/27/2014 1:58:46 PM ‘Mark this question << => Question 6 of 10 A. 22-year-old woman suddenly develops total paralysis of the extremities and trunk after eating a large carbohydrate dinner. This has occurred in the past with recurrent attacks of flaccid weakness. Her grandmother also had similar symptoms. Her serum potassim is 25 meqL and her gucose is 120 mg/dl. What is the diagnosis? a) Embole stroke 'b) Myotonia congenital c) Thyrotoxicosis 4) Hyperaldosteroaism Y © &) Hypokalemic periodic paralysis Answer | Explanation Other User's Explanation Report An Error Question Explanation: Hypokalemic periodic paralysis is a genetic disease, and sudden paralysis can occur when the potassium level drops. Hyperkalemic periodic paralysis is ancther entity when the potassium level is elevaied Muscle biopsy would reveal vacuoles within muscle Sbers. High carbohydrate intake leads to increased movement of glucose and potassium into skeletal mascle, leading to an acute state of severe bypokelemia, Embelic stroke would be uncommon in this age group. Tk would also cause focal neurological deficits. Myotonia congenital is arate disorder. The paiient becomes stiff when he exercises and the muscles loosen up as activity continues. Cold ex- acerbates the symptoms. Thyrotoxicosis would be caused by excess thyroid hormone-producing tachycardia, ciaphoresis, and hypetreflena Grave's disease is the most common etiology Hyperaldosteronism would cause hypertension, hypokalemia, and hypematremia Paralysis is uncommon. ‘Mark this question => Question Id : 94242 Question 7 of 10 ‘Tre statement regarding the use of alendronate (Fosamax) for the treatment of osteoporosis in men is a) Is effectiveness is similar to that seen in women +) It is ineffective in patients with Paget’s diseases ©) Ikis comtraindicated in patients taking NSAIDs 4) It causes a decrease in height Avewor [JEXSIURNY) oter Users Explanation Report An Evor Question Explanation: Tn men with hypogonadism, testosterone therapy has limited efficacy, and the efficacy of other therapies for osteoporosis in men has not been evahated, Bisphosphonates, incudng alendronate, are indicated for treatment of Paget's disease. They decrease bone pain, decrease bone resorption, and increase new bone formation The proportion of men whose height decreased by at least 10 mm over 2-year period was 13% in the placebo group and 3% in the clendronate group. The frequency of adverse gastrointestinal effects in the two groups was similar despite the fact that 3696 of the men in the placebo group and 41% of those in the alendronate group reported taking NSAIDs dunng the study. ‘Mark this question => Question Id : 94242 Question 7 of 10 ‘True statement regarding the use of alendronate (Fosamax) for the treatment of osteoporosis in men is, Y © a) lis effectiveness is similar to that scen in women +b) Itis ineffective in patients with Paget's diseases 6) It is contraincicated in patients taking NSAIDs 4) Tb causes a decrease in height Avewor [JEXSIURNY) oter Users Explanation Report An Evor Question Explanation: Tn men with hypogonadism, testosterone therapy has limited efficacy, and the efficacy of other therapies for osteoporosis in men has not been evahated, Bisphosphonates, incudng alendronate, are indicated for treatment of Paget's disease. They decrease bone pain, decrease bone resorption, and increase new bone formation The proportion of men whose height decreased by at least 10 mm over 2-year period was 13% in the placebo group and 3% in the clendronate group. The frequency of adverse gastrointestinal effects in the two groups was similar despite the fact that 3696 of the men in the placebo group and 41% of those in the alendronate group reported taking NSAIDs dunng the study. ‘Mark this question & => Question Id: 96114 Question 8 of 10 Awomen aged 45 years presents with complains of dry mouth and dry eyes. The physician suspects 2 diagnosis of Sjogren's syndrome. Antibodies found atthe highest fiequency in Sjogren's syndrome are which of the folowing? a) Anti-neutrophil cytoplasmic antibodies ) Anti-RNA antibodies ©) AntiedsDIA antibodies 4) Anti-Ro antibodies @) Anti-Jo-I antibodies Answer [UEQSINSHRN) other User's Ee Question Explanation: Anti-Ro antibodies are found in 60- 70% of cases of Sjégren’s syndrome, Anti-RNA antibodies (B) are also found in cases of Sjogren's syncrome, ation Report An Error ‘Mark this question & => Question Id: 96114 Question 8 of 10 Awomen aged 45 years presents with complains of dry mouth and dry eyes. The physician suspects 2 diagnosis of Sjogren's syndrome. Antibodies found atthe highest fiequency in Sjogren's syndrome are which of the folowing? a) Anti-neutrophil cytoplasmic antibodies ) Anti-RNA antibodies ©) AntiedsDIA antibodies ¥ © A) Anti-Ro antibodies @) Anti-Jo-I antibodies Answer [UEQSINSHRN) other User's Ee Question Explanation: Anti-Ro antibodies are found in 60- 70% of cases of Sjégren’s syndrome, Anti-RNA antibodies (B) are also found in cases of Sjogren's syncrome, ation Report An Error 2/27/2014 1:59:20 PM ‘Mark this question & => Question Id : 121210 Question 9 of 10 ‘Medication thet is beneficial in the treatment of both rheumatoid arthritis and carcinomatous meningitis is which one of the following? a) Azethioprine +b) Cyclophosphamide ©) Solganol 4) Phenoprofen ©) Methotrexate Answer (Bxvianaton | Other User's Explanation Report An Error Question Explanation: Methotrexate is very usefulin the ireatment of sheumatoid arthritis and, especialy when administered intrethecally, is beneficial in carcinomatous meningitis as well. Neither azathioprine nor cyclophosphamide is useful for carcinomatous meningitis, but each is beneticial in rheumatoid arthritis and also in other forms of cancer. Solganol has no role in cancer treatment, Phenoprofen, a nonsteroidal anti inflammatory agent is not useful in cancer chemotherapy. 2/27/2014 1:59:20 PM ‘Mark this question & => Question Id : 121210 Question 9 of 10 ‘Medication thet is beneficial in the treatment of both rheumatoid arthritis and carcinomatous meningitis is which one of the following? a) Azethioprine +b) Cyclophosphamide ©) Solganol 4) Phenoprofen Y © ¢) Methotrexate Answer (Bxvianaton | Other User's Explanation Report An Error Question Explanation: Methotrexate is very usefulin the ireatment of sheumatoid arthritis and, especialy when administered intrethecally, is beneficial in carcinomatous meningitis as well. Neither azathioprine nor cyclophosphamide is useful for carcinomatous meningitis, but each is beneticial in rheumatoid arthritis and also in other forms of cancer. Solganol has no role in cancer treatment, Phenoprofen, a nonsteroidal anti inflammatory agent is not useful in cancer chemotherapy. ‘Mark this question & Question Td : 136523 Question 10 of 10 A.23 year old woman presents with low grade fever, abdominal pain, and malzise. Examination showed pallor with no signs of lymphadeopathy, murmur, or abdominal findings. Lab data revealed lymphopenia and a creatinine level greater than 4.0. Slight alopecia was also noted. The most ikely diagnosis is a) Hodgkin's disease 'b) Sarcoidosis c) Systemic lupus erythematosus 4) Chronic fatigue syndrome ©) Tuberculosis Answer | Explanation Other User's Explanation Report An Error Question Explanation: This young woman has systemic lupus erythematosis, an autoimmune disease of unlenown etiology. Mubiple organ systems are affected Her abdominal pain is secondary to serositis. Membranous proliferative disease is common in the kidneys, and leukopenia and lymphopenia are also scen, as well as pleuritis and a malar rash on the face. Infection is very common in these patients. Hodgkin ‘s disease would present as peripheral lymphadenopathy but no renal dysfunction, Sarcoidosis would present with pulmonary syinptoms most of the time, but it also affects all organ systems. Chronic fatigue syndrome would also present with adenopathy and maybe low-grade fever, but kidney dysfunction is rare. Tuberculosis can also affect all organ systems inchuding kidneys, joints, ovaries, and the peritoneal cavity However, the main organ system affected is the respiratory system, and this patient has no pulmonary symptoms of TB; such as hemoptysis or cough. ‘Mark this question & Question Td : 136523 Question 10 of 10 ‘A. 23 year old woman presenis with low grade fever, abdorsinal pain, and malaise. Examination showed pallor with no signs of Iymphadeopatay, murmur, or abdominal findings. Lab data revealed Iymphopenia and a creatinine level greater than 4.01. Slight alopecia was also noted. The mest ikkely diagnosis is a) Hodgkin's dsease +b) Sarcoidosis Y © ¢) Systemic lupus erythematosus 4) Chroric fatigue syndrome ©) Tuberculosis Answer | Explanation Other User's Explanation Report An Error Question Explanation: This young woman has systemic lupus erythematosis, an autoimmune disease of unlenown etiology. Mubiple organ systems are affected Her abdominal pain is secondary to serositis. Membranous proliferative disease is common in the kidneys, and leukopenia and lymphopenia are also scen, as well as pleuritis and a malar rash on the face. Infection is very common in these patients. Hodgkin ‘s disease would present as peripheral lymphadenopathy but no renal dysfunction, Sarcoidosis would present with pulmonary syinptoms most of the time, but it also affects all organ systems. Chronic fatigue syndrome would also present with adenopathy and maybe low-grade fever, but kidney dysfunction is rare. Tuberculosis can also affect all organ systems inchuding kidneys, joints, ovaries, and the peritoneal cavity However, the main organ system affected is the respiratory system, and this patient has no pulmonary symptoms of TB; such as hemoptysis or cough. ‘Maré this question => Question 1 of 30 A 76-year-old with hyperosmolar non-Ketotic hyperglycemia has ared, hot and swollen knee. Which of the following is most useful inthe diagnosis of the swollen Ienee joint? a) C-ANCA b) CRP c) Orthopedic referral for joint washout 4) Joint Aspiration ©) Rheumatoid factor Question Explanation: Joint aspiration is the best option in this context. Ibis a simple procedure with a high diqgnostio yield. Sencing the joint aspiration for IMICIS in a blood culture borte may increase yield, The risk of introducing infection into the knee joint during simple aspiration by non-experts is 1 in 10,000 procedures, so the procedure is safe ‘Maré this question => Question 1 of 30 A 76-year-old with hyperosmolar non-Ketotic hyperglycemia ha: ared, hot and swollen knee. Which of the following is most usefil inthe diagnosis of the ewollen knee joint? a) C-ANCA ) CRE c) Orthopedic referral for joint washout Y © 4) Joint Aspiration ©) Rheumatoid factor Question Explanation: Joint aspiration is the best option in this context. Ibis a simple procedure with a high diqgnostio yield. Sencing the joint aspiration for IMICIS in a blood culture borte may increase yield, The risk of introducing infection into the knee joint during simple aspiration by non-experts is 1 in 10,000 procedures, so the procedure is safe 2/27/2014 2:00:40 PM ‘Mark this question e& => (Question Id : 28312 Question 2 of 30 A43-year-old has six month old dyspepsia and a 3 year history of Raynaud's phenomenon, On exatrination she had telangiectasia Her investigations reveal an ESR of 40 mur and positive anticentromere antibodies. Which of the following is a typical lave complication of this disorder? a) Alopecia b) Pulmonary hypertension ©) Butterfly skin rach 9) Erosive polyarthropathy €) Myositis Avewor (EQNS) other Ucors Explanation Report An Evror Question Explanation: Linited scleroderma is characterized by Raynaud's phenomenon, peripheral skin involvement, skin calcification, telangiectasia, nail fold capillary dilatation and ant:-centromere antibodies in 70-80% of patients Pulmonary hypertension with or without interstitial ling disease is a characteristic late complication of this disorder. 2/27/2014 2:00:40 PM ‘Mark this question e& => (Question Id : 28312 Question 2 of 30 4.43-year-old has six month old dyspepsia and a3 year history of Raynaud's phenomenon. On examination she had telangiectasia Her investigations reveal an ESR of 40 mmv/ar and positive anticentromere antibodies. Which of the following is a typical late complication of this disorder? a) Alopecia ¥ © b) Pulmonary hypertension c) Buterfy skein rash d) Erosive polyarthropathy €) Myositis Avewor (EQNS) other Ucors Explanation Report An Evror Question Explanation: Linited scleroderma is characterized by Raynaud's phenomenon, peripheral skin involvement, skin calcification, telangiectasia, nail fold capillary dilatation and ant:-centromere antibodies in 70-80% of patients Pulmonary hypertension with or without interstitial ling disease is a characteristic late complication of this disorder. ‘Mark this question & => (Question Td : 28362 Question 3 of 30 A 66-year-old mele has inadequate pain rebef for his hip osteoarthritis, He takes Paracetamol and codeine 30mg four titnes daily but thas found litle benefit, What is the most likely explanation for the lack of efficacy here? a) Inadequate dose of Codeine b) Fast acetylator status ©) Ipratropium accelerates the metabolism of codeine 4) Impaired absorption of Codeine ¢) Interaction of Paracetamol with Codeine Answer [UEQSINSHRN) other User's Ee Question Explanation: The most likely explanation is that the codeine dose is inadequate, Studies have shown that Paracetamol Ig combined with codeine at dose of 60mg have the best analgesic outcomes Ipratropium does not increase the metabolism of codeine, ation Report An Error ‘Mark this question & => (Question Td : 28362 Question 3 of 30 4. 66-year-old male has inadequate pain retef for his hip osteoarthritis. He takes Paracetamol and codeine 30mg four times daily but ‘has found little benefit. What is the most likely explanation for the lack of efficacy here? Y © a) Inadequate dose of Codeine ) Fast acetylater status ©) Ipratropium accelerates the metabolism of codeine d) Impaired absorption of Codeine 2) Interaction of Paracetamol with Codeine Answer [UEQSINSHRN) other User's Ee Question Explanation: The most likely explanation is that the codeine dose is inadequate, Studies have shown that Paracetamol Ig combined with codeine at dose of 60mg have the best analgesic outcomes Ipratropium does not increase the metabolism of codeine, ation Report An Error 2/27/2014 2:01:02 PM ‘Mark this question & => Question Td : 28372 Question 4 of 30 A.21-year-old lady presents has typical erythema nodosum, She has alow grade fever and bilateral ankle arthritis. No history of travel abroad present and no medication taken. Which of the following would be the most appropriate investigation for this patient? a) Banum enema b) Exythrocyte sedimentation rate (ESR) c) Upper gastrointestinal (GD endoscopy ) Chest X-ray ¢) C-Reactive protein Question Explanation: Erythema nodosun is commonly idiopathic Trean also be related to streptococcal infections, acute Sarcoidosis or related to drugs such as the oral contraceptive ill, sulphonamides and penicillin, Rarer causes include infarnmacory bowel disease, tuberculosis, Bechet's disease and other connective tissue disorders. In this case, a chest x-ray would be the most helpful investigation as this may identify bilateral hilar lmphadenopathy which together with a bilateral anile arthropathy would strongly support a diagnosis of acute Sarcoidosis. ‘Investigation of the bowel is unlikely to help in the absence of any bowel symptoms. ESR is non-specific. Chest X ray is a much better option than CRP which is agan nonspecific 2/27/2014 2:01:02 PM ‘Mark this question & => Question Td : 28372 Question 4 of 30 A 21-year-old lady presents has typical erythema nodosum. She has alow grade fever and bilateral anlcle arthritis. No history of travel abroad present and xo medication taken, Which of the following would be the most appropriate investigation for this paticat? a) Banum enema ) Erythrocyte sedimentation rate (ESR) c) Upper gastrcintestinal (GD) endoscopy Y © &) Chest X-ray ©) C-Reactive protein Question Explanation: Erythema nodosun is commonly idiopathic Trean also be related to streptococcal infections, acute Sarcoidosis or related to drugs such as the oral contraceptive ill, sulphonamides and penicillin, Rarer causes include infarnmacory bowel disease, tuberculosis, Bechet's disease and other connective tissue disorders. In this case, a chest x-ray would be the most helpful investigation as this may identify bilateral hilar lmphadenopathy which together with a bilateral anile arthropathy would strongly support a diagnosis of acute Sarcoidosis. ‘Investigation of the bowel is unlikely to help in the absence of any bowel symptoms. ESR is non-specific. Chest X ray is a much better option than CRP which is agan nonspecific ‘Mark this question €e>=> Question 5 of 30 A51 year old lady presents with dry eyes, a dry mouth, an erythematous rash and polyerthralgia Investigations show: Ant-Ro/SSA antibodies strongly positive Rheumatoid factor Positive kG 45 aL (<15) IgM Nowmal kA Nocmal Kappallambda ratio Normal “Which of the followingic the most Lkely diagnosis? a) Hyperviscosity syndrome 'b) Myeloma associated Vesculitis c) Rheumatoid arthritis with cecoadary Sjogren's Syndrome 4) Systemic Lupus Erythematosus ©) Primary Sjogren's Syndrome Question Explanation: Question Td : 30538 The clinical features and the serology are typical of primary Sjogren’s syndrome (occur alone and more likely to have postive anti Ro SSA antibodies than secondary sjogren’s). Hypergammaglobulinasmia is present in 80% of indivisuals, ANA and Anti-Ro/SSA antibodies are present in approximately 90% of individuals as is a weakly positive rheumatoid factor. The normal keppallambda ratio confirms the hypergammaglobulina is polyclonal. Typically secondary sjogren’s has pre-existent Rheumatoid or SLE before the development of Sjogren's symptoms ‘Mark this question €e>=> Question 5 of 30 A51 year old lady presents with dry eyes, a dry mouth, an erythematous rash and polyerthralgia Investigations show: Ant-Ro/SSA antibodies strongly positive Rheumatoid factor Positive kG 45 aL (<15) IgM Nowmal kA Nocmal Kappallambda ratio Normal ‘Which of the followings the most Hcely diagnosis? a) Hyperviscosity syndrome 'b) Myeloma associated Vesculitis c) Rheumatoid arthritis with secondary Sjogren’s Syndrome d) Systemic Lupus Erythematosus © ©)Primary Sjogren’s Syndrome Question Explanation: Question Td : 30538 The clinical features and the serology are typical of primary Sjogren’s syndrome (occur alone and more likely to have postive anti Ro SSA antibodies than secondary sjogren’s). Hypergammaglobulinasmia is present in 80% of indivisuals, ANA and Anti-Ro/SSA antibodies are present in approximately 90% of individuals as is a weakly positive rheumatoid factor. The normal keppallambda ratio confirms the hypergammaglobulina is polyclonal. Typically secondary sjogren’s has pre-existent Rheumatoid or SLE before the development of Sjogren's symptoms ‘Marie this question << => Question Td : 46770 Question 6 of 30 A 25 year old female returns to the office a follow-up wisit. She was last seen 2 weeks ago because of a facial rash that worsens with sun exposure, She complains of joint pain today. She says, ‘It just hurts everywhere, sometimes my knees hurt, and sometimes t's ny elbows or my ankles, AndI have been feeling tired all the time. I just don't have any energy.’ She denies any joint swelling Urinalysis shows 3+ proteins. You consider the diagnosis systemic Inpus erythematosus and plan additional dagnostic testing IEshe hhas systemic Inpus erythematosus, studies are most likely to show a) Decreased numbers of helper T cells b) Increased manbers of plasma celle c) Decreased serum concentrations of C3 and C4 4) Serum Antimicrosomal antibodies ¢) Serum antiplatelet antibodies Answer | Explanation Other User's Explanation Report An Error Question Explanation: Lupus is a chronic inflammatory disease that can affect warions parts of the body, especially the skin, joints, blood, and kidneys -Arthralgia and skin rashes are very common findings in these patients. The ANA antibody test is often used a a first test in the diagnostic evaluation of lupus. Laboratory tesis which meacure complement levels in the blood are also of some value. Complement is a blood protein that, with antibodies, destroys bacteria, It is an "ampiifier' of immune function, Ifthe total blood complement level is, ow, or the C3 or C4 complement values are low, and the person also has a positive ANA, some weight is added to the diagnosis of lupus. Low C3 and C4 complement levels in individuals with positive ANIA test results may also be indicative of lupus kidney disease, Decreased helper (CD4) T cells ere characteristic of certain hereditary cell deficiencies as well as acquired ones such as ‘AIDS. Increased B cells are characteristic of cerain leukemias, lymphomas, or disorders such as multiple myeloma Serum antimicrosomal antibodies are characteristic of subacute lymphocytic thyroiditis. This occurs most often in the postparmim period but may also occur sporadically. Antimicrosomal antibodies are present in 50 to 80% of patients, while antithyroid peroxidase antibodies ace present in nearly all patients. Serum antiplatelet antibodies are found in disorders such as heparin- induced thrombocytopenia and certain idiopathic thrombocytopenias. ‘Marie this question << => Question Td : 46770 Question 6 of 30 A 25 year old female rehuras to the office a follow-up visit. She was last seen 2 weeks ago because of a facial rash that worsens with sun exposure. She complains of joint pain today. She says, "It just hurts everywhere, sometimes my knees hurt, and sometimes i's my elbows or my ankles. And have been feeling tired all the time. I just don’t have any energy." She denies any joint swelling Uninalysis shows 3+ proteins. You consider the diagnosis systemic lnpus erythematosus and plan additional diagnostic testing Ifshe hhas systemic Inpus erythematosus, studies are most likely to show a) Decreased numbers of helper T cells 'b) Increased numbers of plesma cells Y © c) Decreased serum concentrations of C3 and C4 4) Serum Antimicrosomal antibodies €) Serum antiplatelet antibodies Answer | Explanation Other User's Explanation Report An Error Question Explanation: Lupus is a chronic inflammatory disease that can affect warions parts of the body, especially the skin, joints, blood, and kidneys -Arthralgia and skin rashes are very common findings in these patients. The ANA antibody test is often used a a first test in the diagnostic evaluation of lupus. Laboratory tesis which meacure complement levels in the blood are also of some value. Complement is a blood protein that, with antibodies, destroys bacteria, It is an "ampiifier' of immune function, Ifthe total blood complement level is, ow, or the C3 or C4 complement values are low, and the person also has a positive ANA, some weight is added to the diagnosis of lupus. Low C3 and C4 complement levels in individuals with positive ANIA test results may also be indicative of lupus kidney disease, Decreased helper (CD4) T cells ere characteristic of certain hereditary cell deficiencies as well as acquired ones such as ‘AIDS. Increased B cells are characteristic of cerain leukemias, lymphomas, or disorders such as multiple myeloma Serum antimicrosomal antibodies are characteristic of subacute lymphocytic thyroiditis. This occurs most often in the postparmim period but may also occur sporadically. Antimicrosomal antibodies are present in 50 to 80% of patients, while antithyroid peroxidase antibodies ace present in nearly all patients. Serum antiplatelet antibodies are found in disorders such as heparin- induced thrombocytopenia and certain idiopathic thrombocytopenias. 2/27/2014 2:01:44 PM ‘Mark this question <= => Question Ta : 48983 Question 7 of 30 A.55 year old man presents with extreme pain and swelling of the lett middle finger. He suifered a small puncture wound to the volar aspect of the finger at the level of the distal flezor crease four days ago. Passive extension of the proxitnal interphalangeal (PIP) and distal interphalangeal (DIP) joints severely aggravates his symptoms. The most lily diagnosis is a) Pulp space infection b) Cellulitis c) Mid-palmar space infection 4) Septic arthritis distal interphalangeal (DIE) joint ¢) Septic tenossnovitis Answer | Explanation Other User's Explanation Report An Error Question Explanation: These conditions are degeneration (tendincpathy) and associated inflammation (tendinitis) of a tendon or inflammation of the tendon sheath lining (tenosynovitis). Symptoms usually incfude pain with motion and tenderness on palpation, Chronic tendon detericration or inflammation can cause scars that restrict motion, Volar flexor tenosynovitis is one of the most common musculoskeletal entiies and is ofen overlooked. Pain occurs in the paln on the volar aspect ofthe thumb or other digits and may ratliate distally. The diagnosis is made by eliciting tendemess on palpation of the tendon and sheath and by noting swelling and possibly a nodule, In later stages, triggering or snapping of the digit occurs on flexion (trigger finger). Diagnosis is clinical. Treatment includes rest, NSAIDs, and semetimes corticosteroid injections, 2/27/2014 2:01:44 PM ‘Mark this question <= => Question Ta : 48983 Question 7 of 30 A.55 year old man presents with extreme pain and swelling of the lett middle finger. He suifered a small puncture wound to the volar aspect of the finger at the level of the distal lezor crease four days ago. Passive extension of the proximal iterphalengeal (PIP) and distal interphalangeal (DIP) joints severely aggravates his symptoms The most likely diagnosis is a) Pulp space infection b) Cellulitis c) Mid-palmar space infection 4) Septic arthritis distal interphalangeal (DIE) joint Y © &) Septic tenosmnonitis Answer | Explanation Other User's Explanation Report An Error Question Explanation: These conditions are degeneration (tendincpathy) and associated inflammation (tendinitis) of a tendon or inflammation of the tendon sheath lining (tenosynovitis). Symptoms usually incfude pain with motion and tenderness on palpation, Chronic tendon detericration or inflammation can cause scars that restrict motion, Volar flexor tenosynovitis is one of the most common musculoskeletal entiies and is ofen overlooked. Pain occurs in the paln on the volar aspect ofthe thumb or other digits and may ratliate distally. The diagnosis is made by eliciting tendemess on palpation of the tendon and sheath and by noting swelling and possibly a nodule, In later stages, triggering or snapping of the digit occurs on flexion (trigger finger). Diagnosis is clinical. Treatment includes rest, NSAIDs, and semetimes corticosteroid injections, 2/27/2014 2:01:57 PM ‘Mark this question & => Question Id : 54389 Question 8 of 30 A.woman who is positive for ANA presents with ankle edema, arthralgia, protein in her urine and a butterfly rash on her face. What is the most likely diagnosis? a) Lupus b) Sclerodesma c) Multiple sclerosis 4d) Rheumatoid arthritis Question Explanatior Systemic lupus erythematosus is a chronic. multisystem, inflammatory disorder of probable autoimmune etiology. occurring predominantly in young women Common manifestations incinde arthralgias and arthritis; malar and other skin rahes, pleurtis or pericarditis, renal or CNS involvement, and hematologic cytopenia. Diagnosis requires clinical and serologic criteria The Fluorescent test for ANA is the best screen for SLE, Treatment of severe ongoing active disease requires corlicesteroids, often hydroxychloroquine, and sometiines inmunosuppressants, 2/27/2014 2:01:57 PM ‘Mark this question & => Question Id : 54389 Question 8 of 30 A.woman who is positive for ANA presents with ankle edema, arthralgia, protein in her urine and a butterfly rash on her face. What is the most likely diagnosis? Y © a) Lupus ) Sclerodesma ©) Muttiple sclerosie 4) Rheumatoid artaritis Question Explanatior Systemic lupus erythematosus is a chronic. multisystem, inflammatory disorder of probable autoimmune etiology. occurring predominantly in young women Common manifestations incinde arthralgias and arthritis; malar and other skin rahes, pleurtis or pericarditis, renal or CNS involvement, and hematologic cytopenia. Diagnosis requires clinical and serologic criteria The Fluorescent test for ANA is the best screen for SLE, Treatment of severe ongoing active disease requires corlicesteroids, often hydroxychloroquine, and sometiines inmunosuppressants, ‘Mark this question e& => (Question Td : 54849 Question 9 of 30 A 32-year-old pale and unwell lady has chronic pain that changes from day to day, often focusing in the lower back. She wakes up fiequently at night, and is unrefieshed in the moming. She also has intermittent constipation and diarrhea, Exemination and blood tests are normal but there is tenderness in multiple arsas. Diagnose a) Depressive disorder ») Hypothyroidism ©) Schizophrenia 4d) Fibromyalgia @) Somatoform disorder Question Explanation: Fibromyalgia is becoming a recognized medical diagnosis, and is based on the presence of pain in all four quadrants of the body, as wall as tenderness in 11 of 18 anatomically defined tigger areas, Approximately 50% of patients with fibromyalgia complain of diarshea and constipation, often associated with abnormal blosting ‘Momning fatigue is present in a large proportion of these patients, and patients often look unwell, and may appear depressed and anxious. Other features include tissue swelling, morning stiffness and sleep disorders, Somatoform disorders are a group of psychological discrders in which a patient experiences physical symptoms despite the absence of an underlying medical concition that can fully explain their presence. The clinical picture here is too close to that of fioromyalgia to be a somatoform disorder. ‘Mark this question e& => (Question Td : 54849 Question 9 of 30 A 32-year-old pale and unwelllady has chronic pain that changes from day to day, often focusing in the lower back. She wakes up frequently a night, and is uurefiesked in the morning. She also has intermittent constipation aad diahea. Exeinination and bleod tests are normal butthere is tenderness in multiple areas, Diagnose a) Depressive disorder ) Hypothyroidism ¢) Schizophrenia Y © & Fibromyalgia @) Somatoform disorder Question Explanation: Fibromyalgia is becoming a recognized medical diagnosis, and is based on the presence of pain in all four quadrants of the body, as wall as tenderness in 11 of 18 anatomically defined tigger areas, Approximately 50% of patients with fibromyalgia complain of diarshea and constipation, often associated with abnormal blosting ‘Momning fatigue is present in a large proportion of these patients, and patients often look unwell, and may appear depressed and anxious. Other features include tissue swelling, morning stiffness and sleep disorders, Somatoform disorders are a group of psychological discrders in which a patient experiences physical symptoms despite the absence of an underlying medical concition that can fully explain their presence. The clinical picture here is too close to that of fioromyalgia to be a somatoform disorder. 2/27 2014 2:02:21 PM ‘Mark this question qe => Question 10 of 30 A.69-year-old woman complained of pain at the base of her right thm. There was tendemess and swelling of the right frst carpo- metacarpal joint. “What is the most likely diagnosis? a) Avascular necrosis of the scaphoid. b) De Quervain's tenosynovitis c) Psoriatic arthritis 4) Osteoarthritis e) Rheumatoid Answer | Explanation Other User's Explanation Report An Error Question Explanation: Osteoarthritis of the frst carpometacarpal joirt is extremely common and in a 69-years old lady is the most licely diagnosis, ‘Swelling is usually bony hard and due to osteephyte formation which can lead to the appearance of squaring of the hand, De Quervain’s tenosynovitis is a common overuse condition which presents with pain at the base of the thumb but is not associated with joint sweling ‘This jomt can be affected in rheumatoid arthritis and psoriatic arthnits but rarely on its own. 2/27 2014 2:02:21 PM ‘Mark this question qe => Question 10 of 30 A, 69-year-old woman complained of pain at the base of her sight thumb. There was tenderness and swelling ofthe right rst carpo- metacarpal joint. “What is the most lcely diagnosis? a) Avascular necrosis of the scaphoid b) De Quervain's tenosynovitis c) Pscriatie arthritis Y © 4) Osteoarthritis 2) Rheumatoid Answer | Explanation Other User's Explanation Report An Error Question Explanation: Osteoarthritis of the frst carpometacarpal joirt is extremely common and in a 69-years old lady is the most licely diagnosis, ‘Swelling is usually bony hard and due to osteephyte formation which can lead to the appearance of squaring of the hand, De Quervain’s tenosynovitis is a common overuse condition which presents with pain at the base of the thumb but is not associated with joint sweling ‘This jomt can be affected in rheumatoid arthritis and psoriatic arthnits but rarely on its own. ‘Mar this question = => Question Td : 56043 Question 11 of 30 AT 1-year-old has bilateral shouler pain and especially carly moming, He has lost one stone in last eight weeks end complains of feeling lethargic with loss of appetite. Investigations revealed a very high ESR (100 mn‘hr), Normochromic normocytic anetnia end a positive rheurnatoid factor. The most likely diggnosis is: a) Polyartertis nodosa ) Polymyalgia heumatica ©) Polymyositis rheumatica d) Rheumatoid arthritis €) Systemic Lupus Erythematous Answer [UERBURINY orber Users Explanation Report An Evo Question Explanation: “This conditon is polymyalgia theumatic. It's associated with weight loss, anemia and malase. Itis associated with false positive rheumatoid factor especially in the elderly. Positive rheumatoid factor does not make a diagnosis of rheumatoid arthritis ‘Mar this question = => Question Td : 56043 Question 11 of 30 A71-ycar-old has bilateral shouler pain and especially carly moming, He has lost one stone in last eight weeks and complains of feeling lethargic with loss of appetite. Investigations revealed a very high ESR (100 mm‘hr), Normochromic normocytic anemia anda positive rheumatoid factor. The most likely diagnosis is a) Polyattertis nodosa ¥ © b) Polymyalgia cheumatica c) Polymyositis rheumatiea d) Rheumatoid arthritis €) Systemic Lupus Erythematous Answer [UERBURINY orber Users Explanation Report An Evo Question Explanation: “This conditon is polymyalgia theumatic. It's associated with weight loss, anemia and malase. Itis associated with false positive rheumatoid factor especially in the elderly. Positive rheumatoid factor does not make a diagnosis of rheumatoid arthritis 2/27/2014 2.0; PM ‘Mark this question ez Question Id: 61788 Question 12 of 30 A post menopausal worman comes to yon to discuss hormone replacement therapy (HRT). A history of migraine and a strong family history of osteoporosis are present. An appropriate choice would be 2) Offer combination oval estrogen-progesterone HRT ») Offer progesterone only ©) Offer oral estrogen only 4) Offer biphosphonate only Anower ERNEEIRI cine users bxptans Question Explanation: Oral forms of estrogen and progesterone can cause several side effects one of which is migraines. For this patient the only medicine appropriate is a bisphosphonate like Actonel or Fosamax to prevent her from developing osteoporosis n Report An Error 2/27/2014 2.0; PM ‘Mark this question ez Question Id: 61788 Question 12 of 30 A post menopausal worman comes to yon to discuss hormone replacement therapy (HRT). A history of migraine and a strong family history of osteoporosis are present. An appropriate choice would be a) Offer combination oral estrogen-progesterone HRT +b) Offer progesterone only ¢) Offer oral estrogen only ¥ © 4) Offer biphosphonate only Anower ERNEEIRI cine users bxptans Question Explanation: Oral forms of estrogen and progesterone can cause several side effects one of which is migraines. For this patient the only medicine appropriate is a bisphosphonate like Actonel or Fosamax to prevent her from developing osteoporosis n Report An Error ‘Mark this question e => Question Td : 62560 Question 13 of 30 The diagnostic test of choice for the diagnosis of myasthenia gravis is a) EEG ) Tensilon test c) MRI of brain 4) SCF analysis Question Explanation: Myasthenia gravis is an autoimmune disorder of episodic muscle weakness and easy faiigebily caused by antibody and cell mediated, destruction of acetylcholine receptors. IF is more common among young women and older men but may occur at any age. Symptoms worsen with muscle activity and lessen with rest Diagnosis is by IV edrophonium (tensilon) challenge, which briefly lessens the weakness. Treatment incluces anticholinesterase drugs, immunosuppressants, corticosteroids, thymectomy, and plasmapheresis. ‘Mark this question e => Question Td : 62560 Question 13 of 30 ‘The diagnostic test of choice for the diagnosis of myasthenia gravis is a) EEG Y © b) Tersilon test c) MRI of brain d) SCF analysis Question Explanation: Myasthenia gravis is an autoimmune disorder of episodic muscle weakness and easy faiigebily caused by antibody and cell mediated, destruction of acetylcholine receptors. IF is more common among young women and older men but may occur at any age. Symptoms worsen with muscle activity and lessen with rest Diagnosis is by IV edrophonium (tensilon) challenge, which briefly lessens the weakness. Treatment incluces anticholinesterase drugs, immunosuppressants, corticosteroids, thymectomy, and plasmapheresis. 2/27/2014 2:03:09 PM ‘Marte this question & => Question Id : 79939 Question 14 of 30 A 34-year-old woman presents with a history of progressive siffhess ofher fingers and cold intolerance of ber hands and feet. ‘Which one of the following is LEAST likely to be associeted with her condition? a) Subcutaneous nochiles b) Digital ulcers c) Esophagitis 4) Aortic dilatation c) Renal insufficiency Question Explanation: Aortic dilatation is unlikely in this patient This patient most likely has progressive systemic sclerosis (scleroderma), which is associated with calcinosis (subcutaneous nodules) and dgital ulcerations due to Raynaud's phenomenon and resulting vasoconstriction, Many patients with sclerodema have esophagitis as a result of reflux and esophageal strictures. Unfortunately many also develop renal insufficiency as a result of glomerulosclerosis and vasoconstriction. 2/27/2014 2:03:09 PM ‘Marte this question & => Question Id : 79939 Question 14 of 30 ‘A 34-year-old woman precents with a histery of progressive siiiess oflaer fingers and cold intolerance of her hands and feet. Which one of the following is LEAST likely to be associated wih her condition? a) Subcutaneous nodules b) Digital ulcers c) Esophagitis Y © & Aortic dilatation c) Renal insufficiency. Question Explanation: Aortic dilatation is unlikely in this patient This patient most likely has progressive systemic sclerosis (scleroderma), which is associated with calcinosis (subcutaneous nodules) and dgital ulcerations due to Raynaud's phenomenon and resulting vasoconstriction, Many patients with sclerodema have esophagitis as a result of reflux and esophageal strictures. Unfortunately many also develop renal insufficiency as a result of glomerulosclerosis and vasoconstriction. ‘Mark this question Question Id Question 15 of 30 A 33-year-old woman complains of severe joint pain in her elbow and knee joints. An arthocentesis test is performed WBC count is shown to be 4,000 cells imicrolter in the synovial uid, yet no bacteria can be cubured, Which one of the following findings would be diagnostic for the presence of rheumatoid artarits? a) T-cells specific for myelin basic protein ») Autoantibody directed at myelin basic protein, ¢) Autoantibedy directed at type T collagen 4) Antoantibody directed at serum immnnoglobutins 6) T-cells specific for serum immunoglobutins Question Explanation: Autoantbody directed at serum immunoglobulins, or rheumatoid factor, is seen only in rheumatoid arthritis ‘Mark this question Question Id Question 15 of 30 A 33-year-old woman complains of severe joint pain in her elbow and knee joints. An arthocentesis test is performed WBC count is shown to be 4,000 cells imicrolter in the synovial uid, yet no bacteria can be cubured, Which one of the following findings would be diagnostic for the presence of rheumatoid artarits? a) T-celle epecifio for myelin bacis protein ) Autoantibody directed at myelin basic protein ©) Autoantibedy directed at typeI collagen Y © d) Autoantibody directed at serum immunoglobulins 2) T-cells specific for serum ismrmmoglobutins Question Explanation: Autoantbody directed at serum immunoglobulins, or rheumatoid factor, is seen only in rheumatoid arthritis 2727/2014 2:03:40 PM ‘Mark this question e => Question Td : 81136 Question 16 of 30 “Which one of the following drugs is contraindicated in acute gouty arthritis? a) Indomethacin ) Colchicine ©) Gincocarticeids 4) Allopurinol ¢) All of the above drugs are indicated Question Explanation: ‘Any agent that causes a change in the uric acid level during an acute gouty attack can actually exacerbate the attack: Therefore, allopurinol, which reduces the serum uric acid level, is not mdicated for acute attacks. Allopurinal is used for suppression of future attacks, Indomethacin (A), cochicine (B) and steroids (C) can be used in the acute atteck. 2727/2014 2:03:40 PM ‘Mark this question e => Question Td : 81136 Question 16 of 30 “Which one of the following drugs is contraindicated in acute gouty arthritis? a) Indomethacin ) Colchicine ©) Gincocarticeids Y¥ © &) Allepurinol ¢) All of the above drugs are indicated Question Explanation: ‘Any agent that causes a change in the uric acid level during an acute gouty attack can actually exacerbate the attack: Therefore, allopurinol, which reduces the serum uric acid level, is not mdicated for acute attacks. Allopurinal is used for suppression of future attacks, Indomethacin (A), cochicine (B) and steroids (C) can be used in the acute atteck. ‘Mark this question rz Question Id: 82036 Question 17 of 30 A.69 year oldman developed a violaceous discoloration around the eves and a violet erythema over the knuckles that spared the skin over the phalanges. He complained of arthralgia several months later and noted difficulty rising from a chair. The most common and specific serologic marker found in this disease is which of the following? a) Anti-nuclear antibody b) Anti-Ro/Anti-la antibody ©) Anti-To-1 antibody 4) Ant-histone antibody ) Anti-centromeres antibody Answer [ Barisnaton | Other User's Explanation Report An Error Question Explanation: Dermatomyositis is esscciated with heliotrope (violateous color) of the eyclids and Gottron’s papules on the knuckles, It is associated with proximal muscle weakness ands autcimmune in nature. Anti-Jo-1 (C) is the serological marker most specific for dermatcmyostis. AITA (A) is associated with lupus. Anti-histone antibody (D) is associated with drug-related lupus. Anti- centromeres antibody (E) is associated with scleroderma and anti-Ro/Anti-la antibody (B) with Sjogren's syndrome. ‘Mark this question rz Question Id: 82036 Question 17 of 30 A.69 year oldman developed a violaceous discoloration around the eves and a violet erythema over the knuckles that spared the skin over the phalanges. He complained of arthralgia several months later and noted difficulty rising fror a chair. The most common and speclic serologic marlcer found in thie diceace ic which of the following! a) Anti-nuclear antibody b) Anti-Ro/Anti-la antibody Y © c) Anti-Jo-1 antibody 4) Anti-histone antibody ) Anti-centromeres antibody Answer [ Barisnaton | Other User's Explanation Report An Error Question Explanation: Dermatomyositis is esscciated with heliotrope (violateous color) of the eyclids and Gottron’s papules on the knuckles, It is associated with proximal muscle weakness ands autcimmune in nature. Anti-Jo-1 (C) is the serological marker most specific for dermatcmyostis. AITA (A) is associated with lupus. Anti-histone antibody (D) is associated with drug-related lupus. Anti- centromeres antibody (E) is associated with scleroderma and anti-Ro/Anti-la antibody (B) with Sjogren's syndrome. ‘Mark this question ==> Question Id: 91891 Question 18 of 30 The group thatis at greatest risk for development of inflammatory osteoacthatis of the hands is a) Teenagers b) Young children ¢) Elderly men 4) Blacks ¢) Women Answer | Bevionation | Other User's Explanation Report An Error Question Explanation: Tnflannmnatory osteoarthritis of the hands is most common in women, can develop early or late in fk, and can be hereditary in women within families. Teenagers (A), young children (B), elderly men (C), and blacks (D) are not at great risk for this condition. ‘Mark this question ==> Question Id: 91891 Question 18 of 30 The group thatis at greatest risk for development of inflammatory osteoacthatis of the hands is a) Teenagers b) Young children ¢) Elderly men 4) Blacks Y © ¢) Women Answer | Bevionation | Other User's Explanation Report An Error Question Explanation: Tnflannmnatory osteoarthritis of the hands is most common in women, can develop early or late in fk, and can be hereditary in women within families. Teenagers (A), young children (B), elderly men (C), and blacks (D) are not at great risk for this condition. 2/27/2014 2:04:20 PM ‘Mark this question = => Question Td : 92105 Question 19 of 30 A48-year-old woman with rheumatoid arthritis is found io have thrombocytosis (platelet count of 700,000) on a routine blood exarn? What statement regarding her is correct? a) Her arthritis is active. ) She is tkely to develop thrombi ©) She should be put on anticoagulation 4) She probably has a mysloproliferative disorder. ) The possibilty of an occult malignancy should be evalueted. Question Explanatios ‘Thrombocytosis is a common indicator of active inflammation and is often seen in patients with shenmatoid artis flares. Such individuals are aot at signicant risk for clotting accidents and do not need anticoagulation. A myeloproliferatwve disease or humor is unllely. Report An Error 2/27/2014 2:04:20 PM ‘Mark this question = => Question Td : 92105 Question 19 of 30 4.48-ycar-old woman with rheumatoid arthritis is found to have thrombecytesis (platelet count of 700,000) on a routine blood exam? What statement regarding her is correct? Y © a) Her arthrits is active. ») Shes lcely to develop thrombi. ¢) She should be put on anticoagulation. d) She probably has a mycloproliferative disorder. e) The possibility of an occult malignancy should be evaluated Question Explanatios ‘Thrombocytosis is a common indicator of active inflammation and is often seen in patients with shenmatoid artis flares. Such individuals are aot at signicant risk for clotting accidents and do not need anticoagulation. A myeloproliferatwve disease or humor is unllely. Report An Error ‘Mark this question ez Question Td : 94504 Question 20 of 30 A.22 year old registered nurse comes to the employee health clinic because she says, "I'm too tired to work." She has had increasing fatigue, malaise and anorexia curing the past several days. Investigations show. ALT 1160 UML, PT 13 sec, Biincbin 1.8 mg/dL, HBsAg positive She is instructed to rest at home and return in 3 days if no new symptoms develop. Two days later she calls to say that she has now developed an uticarial rash and swelling of the joints ofher fingers. What is the most correct statement about her condition at this time? 4) The arthritis and rash are unrelated to her liver disease b) The arthritis and rash are the result of an associated immune complex disorder c) Ibis unlikely that her blood is infectious 4) She has a 50% sisk for developing chronic liver disease 2) She should be given hepatitis B immune globulin Question Explanation: In approximately 10% of HBV infected patients, an immune complex-mediated "serum sickness lice" reaction is seen. Fever, ghmeruionephaitis, arthralgas, and urticarial maculopapular rash may also be present. Arthritis like picture involving the joints of the hands, wrists, and elbows may also be see. These syrnptoms resolve with the onset of jaundice, and hence ate related to the progress of iver disease. The presence of HBsAg in the blood implies infectivity via both semzal contact and exposure to blood In 5-10% of cases, HB infection progresses to chronic hepatitis from there there’s a 1% chance of progressing to fulminant hepatiis. Once the patient is already infected, administering hepatitis B snmunoglobulin is of no benefit Report An Error ‘Mark this question ez Question Td : 94504 Question 20 of 30 A 22 year old registered murse comes to the employee health clin because she says, "I'mtoo tited to work." She has had increasing fatigue, malaise and anorexia curing the past several days. Investigations show. ALT 1160 UML, PT 13 sec, Biincbin 1.8 mg/dL, HBsAg positive She is instructed to rest at home and return in 3 days if no new symptoms develop. Two days later she calls to say that she has now developed an uticarial rash and swelling of the joints ofher fingers. What is the most correct statement about her condition at this time? @) The arthritis and rash are unrelated to her liver disease Y © b) The arthritis and rash are the result of an associated immune complex disorder c) Ibis unlikely that her blood is infectious 4) She has a 50% sisk for developing chronic liver disease ¢) She should be given hepatitis B immune alobulin Question Explanation: In approximately 10% of HBV infected patients, an immune complex-mediated "serum sickness lice" reaction is seen. Fever, ghmeruionephaitis, arthralgas, and urticarial maculopapular rash may also be present. Arthritis like picture involving the joints of the hands, wrists, and elbows may also be see. These syrnptoms resolve with the onset of jaundice, and hence ate related to the progress of iver disease. The presence of HBsAg in the blood implies infectivity via both semzal contact and exposure to blood In 5-10% of cases, HB infection progresses to chronic hepatitis from there there’s a 1% chance of progressing to fulminant hepatiis. Once the patient is already infected, administering hepatitis B snmunoglobulin is of no benefit Report An Error 2/27/2014 2:04:52 PM ‘Marc this question e& => Question Td : 98170 Question 21 of 30 ‘Which of the following should NOT be a treatment of a patient with rheumatoid arthritis and myasthenia gravis? a) Penicillamine ) Intramnascular geld injections ©) Azathioprine od) Methotrexate €) Steroids. Anewor (UENINRIAN other User's Explanation Report An Enron Question Explanation: Penicillamine can induce a myasthenia like syndrome as one ofits potential adverse effects. None of the other agents can do this, so their use would be appropnate in such a patient. 2/27/2014 2:04:52 PM ‘Marc this question e& => Question Td : 98170 Question 21 of 30 ‘Which of the following should NOT be a treatment of a patient with rheumatoid arthritis and myasthenia gravis? Y © a) Penicillamine ') Intamaseular geld injections c) Azathioprine d) Methotrexate. €) Steroids. Anewor (UENINRIAN other User's Explanation Report An Enron Question Explanation: Penicillamine can induce a myasthenia like syndrome as one ofits potential adverse effects. None of the other agents can do this, so their use would be appropnate in such a patient. ‘Mark this question ez = 100107 Question 22 of 30 A.44 year old man presents with an acute attack of severe first right toe pain, No fever, trauma, or other medical problems are present. On aspiration ofthe joint duid waat would the physician see under a polarizing microscope? 4) Positively bireftingent crystals ) Negatively bireftingent crystals ©) Bhomboid-chaped crystale 4) Tophi «) Calcium pyrophosphate dihydrate crystals Question Explanation: ‘This man has an acute atack of gouty arthritis, and aspiration of the synovial flud of the toe would reveal sodium urate monchydrate crystals, which are negahvely byeffingent under a polarizing microscope. Pseudogout s characterized by inflammaton, mainly in the knees, with rhomboid shaped calomm pyrophosphate crystals which ere postively birefringent under a polarizing microscope, Tophi are chalky material made up ofurate crystals and granulemas of epithelial cells and are found in artcular cartlages and subcutaneous n Report An Error tissues. ‘Mark this question ez = 100107 Question 22 of 30 4.44 year old man presents with an acute attack of severe first right toe pain. No fever, trauma, or other medical problems are present. On aspiration of the joint duid what would the physician see under a polarizing microscope? a) Positively birefringent crystals Y © b) Negatively birefringent crystals c) Rhomboid-shaped crystals 4) Tophi ¢) Calcium pyrophosphate diayclate crystals Question Explanation: ‘This man has an acute atack of gouty arthritis, and aspiration of the synovial flud of the toe would reveal sodium urate monchydrate crystals, which are negahvely byeffingent under a polarizing microscope. Pseudogout s characterized by inflammaton, mainly in the knees, with rhomboid shaped calomm pyrophosphate crystals which ere postively birefringent under a polarizing microscope, Tophi are chalky material made up ofurate crystals and granulemas of epithelial cells and are found in artcular cartlages and subcutaneous n Report An Error tissues. ‘Mat this question & => Question Td : 103075 Question 23 of 30 A 56 year old man’s examination shows unusual bluish black pigmentation over his face and fingers. His urine sample tuned black on adding sodium hydroxide. Spinal films revealed intervertebral disk spaces showing greater raciodensity than the adjacent vertebrae. ‘The most ikely diagnosis is a) Ochronesis 'b)Peephyria cutanea tarda c) Systemic lupus erythematosus 4) Pellagra ¢) Sturge Weber syndrome Answer | Explanation Other User's Explanation Report An Error Question Explanation: Ochronosis (also known as alleaptomuria) is an autosomal recessive trait characterized by deficiency in homogentisic arid oxidase, an enzyme necessary in the production of the acid produced by the metabolism of phenylalanine and tyrosine. Ochronesis can produce ankylosis of joints and other carilaginous stractares. Darlcening of the skin is present which represents deposits of homogentisic acid in cartilage and the dermis, Porphyria cutanea tarda also causes skin discoloration. In contrast to ochronosis, either hyper or hhypopigmentation may develop. Patients also develop vesicular and bullous lesions, which were not scen in this patient. Lupus patients can present with a photosensitive rash on the trunk and face along with arthralgias, malaise, and an elevation of antinuclear antibody tters. A "inalar" rash is common. Pellegra is secondary to niacin deficiency. The characteristic rash found in this nutritional eficiency is a dark, scaling, blistering lesion which is sharply demarcated, This dermatitis is usually preceded by diarrhea. Pellagra is associated with dementia, diarrhea and dermatitis. Sturge Weber syndrome is associated with seizures and a hemangiometous lesion on the face. Visval loss can occur because of involvement of the ophthalnic division of the trigeminal nerve. ‘Mat this question & => Question Td : 103075 Question 23 of 30 .A.56 year old man’s examination shows unusual bluish black pigmentation over his face and fingers. His urine sample tumed black on adding sodium hydroxide. Spinal films revealed intervertebral disk spaces showing greater radiodensity than the adjacent vertebrae ‘The most icely diagnosis is Y © a) Ochronosis 'b) Porphyria cutanea tarda c) Systemic lupus erythematosus 4) Pellagra ¢) Sturge Weber syndrome Answer | Explanation Other User's Explanation Report An Error Question Explanation: Ochronosis (also known as alleaptomuria) is an autosomal recessive trait characterized by deficiency in homogentisic arid oxidase, an enzyme necessary in the production of the acid produced by the metabolism of phenylalanine and tyrosine. Ochronesis can produce ankylosis of joints and other carilaginous stractares. Darlcening of the skin is present which represents deposits of homogentisic acid in cartilage and the dermis, Porphyria cutanea tarda also causes skin discoloration. In contrast to ochronosis, either hyper or hhypopigmentation may develop. Patients also develop vesicular and bullous lesions, which were not scen in this patient. Lupus patients can present with a photosensitive rash on the trunk and face along with arthralgias, malaise, and an elevation of antinuclear antibody tters. A "inalar" rash is common. Pellegra is secondary to niacin deficiency. The characteristic rash found in this nutritional eficiency is a dark, scaling, blistering lesion which is sharply demarcated, This dermatitis is usually preceded by diarrhea. Pellagra is associated with dementia, diarrhea and dermatitis. Sturge Weber syndrome is associated with seizures and a hemangiometous lesion on the face. Visval loss can occur because of involvement of the ophthalnic division of the trigeminal nerve. ‘Mark this question = => Question Id: 109942 Question 24 of 30 “Which of the fellowing testis most appropriate for the diagnosis of gout? a) Gram stain and culture of synoviel fui. ) Examination of synovial uid for presence of urate crystals. «) Radiographic evalustion with clinical examination @) Scrum muscle enzyme measurement ¢) Arteriogram. Avewor (NEQNIIRHAN) other Ucors Explanation Report An Error Question Explanation: Goutis a disease of excess urate formation and subsequent depostion of unis acid crystals in tissue, Examinetion of the synovial fluid wih detection of urate crystals is diagnostic of gout. ‘Mark this question = => Question Id: 109942 Question 24 of 30 ‘Which of the following test is most appropriate for the diagnosis of gout? a) Gram stain and culture of synovial uid © ») Examination of synovial fluid for presence of urate crystals. 2) Radiographic evaluation with clinical examination. d) Scrum muscle enzyme measurement, 2) Arteriogram. Avewor (NEQNIIRHAN) other Ucors Explanation Report An Error Question Explanation: Goutis a disease of excess urate formation and subsequent depostion of unis acid crystals in tissue, Examinetion of the synovial fluid wih detection of urate crystals is diagnostic of gout. 2/27 72014 2:05:42 PM ‘Mark this question = => ‘Question Td : 114240 Question 25 of 30 A prisoner of age 40 years was exercising vigorously in his prison cell, lifing weights, when he suddenly felt weakness, malaise, and muscle pain, He has never experienced this before, athough he has been working out for many years. He was found to have ant elevated creatinine phosphokinase level and no red blood cells in the urine, The treatment of choice is which one ofthe following? a) Pryridostigmine bromide (Mestinon) ') Steroids ©) Intravenous hydration ) Azathioprine ©) Antibiotics Ancwor (UEIURINY other Users Explanation Report AnEroe (Question Explanation: ‘This patient hes rhabdomyclosis secondary to muscle breakdown and release of myoglobin. The urine would be dark brown from myoglobinuria, and vigorous intravenous hydration would prevent kidney faiure. Pyridostigmine is an antichelinesterase medication used in patents with myasthena grams. This disease presents with bulbar and ocular muscle weakness. Steroids are used in polymyalgia rheumatca, which is common in elderly patients and associated with proximal muscle weakness. Azathioprine is a cytotoxic drug used in autoimmune disease. Antibiotics would not be used in thabdomyolosis unless the underlying eticlogy was a bacterial infection 2/27 72014 2:05:42 PM ‘Mark this question = => ‘Question Td : 114240 Question 25 of 30 A prisoner of age 40 years was exercising vigorously in his prison cell, lifing weights, when he suddenly felt weakness, malaise, and muscle pain, He has never experienced this before, athough he has been working out for many years. He was found to have ant elevated creatinine phosphokinase level and no red blood cells in the urine, The treatment of choice is which one ofthe following? a) Pryridostigmine bromide (Mestinon) ') Steroids JV © ©) Intravenous hydration ) Azathioprine ©) Antibiotics Ancwor (UEIURINY other Users Explanation Report AnEroe (Question Explanation: ‘This patient hes rhabdomyclosis secondary to muscle breakdown and release of myoglobin. The urine would be dark brown from myoglobinuria, and vigorous intravenous hydration would prevent kidney faiure. Pyridostigmine is an antichelinesterase medication used in patents with myasthena grams. This disease presents with bulbar and ocular muscle weakness. Steroids are used in polymyalgia rheumatca, which is common in elderly patients and associated with proximal muscle weakness. Azathioprine is a cytotoxic drug used in autoimmune disease. Antibiotics would not be used in thabdomyolosis unless the underlying eticlogy was a bacterial infection 2/27/2014 2:06:02 PM ‘Mark this question & => Question 26 of 30 Common vertebral column Sinding in ankylosing spondylitis is a) Bamboo spine 'b) Chonéroralcinosis ©) Fua-articular demineralization 4) Penoil-in-sup erosions €) Ovethanging erosions Question Explanation: Barnboo spine is the common vertebral column finding on radiographs in anieylosing spondylitis 2/27/2014 2:06:02 PM ‘Mark this question & => Question 26 of 30 ‘Common vertebral column Sinding in ankylosing spondylitis is Y¥ © a) Bamboo spine 'b) Chonéroralcinosis ©) Fua-articular demineralization 4) Penoil-in-sup erosions €) Ovethanging erosions Question Explanation: Barnboo spine is the common vertebral column finding on radiographs in anieylosing spondylitis 2/27/2014 2:06:14 PM ‘Mark this question & => Question Td : 126630 Question 27 of 30 AST year old man presents to the ER with an acutely swollen, painful, and red right great toe for frst time, Serum uric acid level is elevated and has monosodium urate crystals in synovial uid aspircted from his toe. He responds wellto oral indomethacin. Which one of the following forms a part of an appropriate discharge plan? a) Daily oral colchicine and aretam apoointment for a 24 hour une collection in three weeks +) Daily oral allopurinal ¢) Daily oral allopurinol and a remem appointment for a 24- hour urine collection in three weeks 4) Low purine diet and follow up as needed. £) Oral penicillamine Answer | Exvianaion Other User's Explanation Report An Error Question Explanation: Since this is the patient's frct attack, an agent such as colchicine, which is relatively specific for gout and well tolerated in most individuals at low doses, is appropriate. A retum visit with a 24 hour urine collection for uric acid clearance is helpfil in deciding whether to treet with allopurinel (func acid excretion is high) or with probenecid (Euric acid excretion is low). This patieat will need a second agent because of his elevated serum uric acid, Starting daily allopurinol is not correct, since beginning an agent that changes the uric acid equilibrium, be it up or down, can precipitate anew attack in the setting of a very recent attack. Option C is similarly not correct, Option D is not correct since few commonly consumed foods are high in purines and adjustment of diet is usually not helpful Penicilamine is not of any value in gout. 2/27/2014 2:06:14 PM ‘Mark this question & => Question Td : 126630 Question 27 of 30 AST year old man presents to the ER with an acutely swollen, painful, and red right great toe for frst time, Serum uric acid level is elevated and has monosodium urate crystals in synovial uid aspircted from his toe. He responds wellto oral indomethacin. Which one of the following forms a part of an appropriate discharge plan? Y © a) Daily oral colchicine and a retum appointment for a 24 howr urine collection in three weeks +) Daily oral allopurinal ¢) Daily oral allopurinol and a remem appointment for a 24- hour urine collection in three weeks 4) Low purine diet and follow up as needed. £) Oral penicillamine Answer | Exvianaion Other User's Explanation Report An Error Question Explanation: Since this is the patient's frct attack, an agent such as colchicine, which is relatively specific for gout and well tolerated in most individuals at low doses, is appropriate. A retum visit with a 24 hour urine collection for uric acid clearance is helpfil in deciding whether to treet with allopurinel (func acid excretion is high) or with probenecid (Euric acid excretion is low). This patieat will need a second agent because of his elevated serum uric acid, Starting daily allopurinol is not correct, since beginning an agent that changes the uric acid equilibrium, be it up or down, can precipitate anew attack in the setting of a very recent attack. Option C is similarly not correct, Option D is not correct since few commonly consumed foods are high in purines and adjustment of diet is usually not helpful Penicilamine is not of any value in gout. 2/27/2014 2:06:26 PM ‘Mark this question ez Question Td: 136566 Question 28 of 30 A.26 year old women presents with severe arthritis thatis acute in onset, fever, and a tender rash on both her lower extremities. Her CXR was normal and nodular, tender, erytuematous lesions were noted on the aatesior suxface of both her legs. The most likely diagnosis is a) Rheumatoid arthritis ) Heertfortd’s syndrome ) Loeffgren’s syndrome ) Lyme disease ©) Erythema multiforme Anewor [UEBRINY other Users Explanation Report An Eos Question Explanation: ‘This patent has Loetfgren’s syndrome, which has a good prognosis in sarcoid patients. The leg nodules are erythema nodosum. ‘Treatment is nonsteroidal anti-inflammatory drugs. Kheumatoid arthritis would present as pain in the wrist joints and hands, unusual to present as a rash and fever. Heerifortd’s syndrome involves the parotid glands in sarcoid patients. Lyme disease is caused by a tick bite, and the rash is called erythema migrars, which is oval to round shaped and macular with central clearing The arthritis in Lyme disease presents few weeks to months after the bite. Erythema multiforme consists of "target" lesions of concenttic circles of erythema with a central vesicle or bulla. The causes of this skin reaction are multiple penicilin, sulfonamides, herpes, and mycoplasma 2/27/2014 2:06:26 PM ‘Mark this question ez Question Td: 136566 Question 28 of 30 A.26 year old women presents with severe arthritis thatis acute in onset, fever, and a tender rash on both her lower extremities. Her CXR was normal and nodular, tender, erytuematous lesions were noted on the aatesior suxface of both her legs. The most likely diagnosis is a) Rheumatoid arthritis ) Heertfortd’s syndrome Y © ©) Loeffgren’s syndrome ) Lyme disease ©) Erythema multiforme Anewor [UEBRINY other Users Explanation Report An Eos Question Explanation: ‘This patent has Loetfgren’s syndrome, which has a good prognosis in sarcoid patients. The leg nodules are erythema nodosum. ‘Treatment is nonsteroidal anti-inflammatory drugs. Kheumatoid arthritis would present as pain in the wrist joints and hands, unusual to present as a rash and fever. Heerifortd’s syndrome involves the parotid glands in sarcoid patients. Lyme disease is caused by a tick bite, and the rash is called erythema migrars, which is oval to round shaped and macular with central clearing The arthritis in Lyme disease presents few weeks to months after the bite. Erythema multiforme consists of "target" lesions of concenttic circles of erythema with a central vesicle or bulla. The causes of this skin reaction are multiple penicilin, sulfonamides, herpes, and mycoplasma ‘Mark this question ez Question Id: 150678 Question 29 of 30 Antibody studies given below suggest which one of the following disease? [Ant-ritonscleoprotein axti-RNP) [i titer Rheumatoid factor (RF) flow titer [Anti-single stranded DNA (anti-ssDNA) flow titer |Anti-double-stranded DNA (anti-dsDNA)| not detected) [Anti-Sinith antigen (anti-Sin) fnot detected |Antia Scl-70 not detected) a) CREST syndrome ) Diffuse scleroderma c) Drug-induced lupus 4d) Mixed connective tissue disease €) Systemic lupas erythematosus (SLE) Answer | Biplanation | Other User's Explanation Report An Error Question Explanation: ‘Mixeed connective tissue disease (MCTD) is clinically an overlap autoimmune disorder, including joint pain, myalgias, pleurisy, esophageal dysmotiliy, and skin disease. Interestingly, the condition is usually characterized by high anibody titers to ribomucleoprotein (RNB), which serves as a ciscase marker. The condtion is apparently immunologically distinct from other connective tissue disorders, Low titer RF and anti-ssDNA are common features of MCTD, but other autoantibodies are less commonly observed. Anti-centromere antibody is the cistinctive marker for the CREST varient of scleroderma, characterized by calcinosis, Raynaud phenomenon, esophageal dysmotiity, sclerodactyly, and telangiectasia, Anti Scl 70 is the distinctive marker for diffuse scleroderma, or systemic sclerosis, a multisystem disorder in which fibrosis of the skin, blood vessels, and viscera occurs. High titers of anti-histone antibody without other autoantibodies is the distinctive marker for drug-induced lupus. This disorder is patticularly associated with acministration of L hydralazine and procainamide, High titers of anti-dsDINA is the distinctive marker for systemic lupus erythematosus (SLE) ‘Mark this question ez Question Id: 150678 Question 29 of 30 Antibody studies given below suggest which one of the following disease? [Ant-ritonscleoprotein axti-RNP) [i titer Rheumatoid factor (RF) flow titer [Anti-single stranded DNA (anti-ssDNA) flow titer |Anti-double-stranded DNA (anti-dsDNA)| not detected) [Anti-Sinith antigen (anti-Sin) fnot detected |Antia Scl-70 not detected) a) CREST syndrome b) Diffuse scleroderma c) Drug-induced lupus Y © @ Mixed connective tissue disease ) Systemic lupus erythematosus (SLE) Answer | Biplanation | Other User's Explanation Report An Error Question Explanation: ‘Mixeed connective tissue disease (MCTD) is clinically an overlap autoimmune disorder, including joint pain, myalgias, pleurisy, esophageal dysmotiliy, and skin disease. Interestingly, the condition is usually characterized by high anibody titers to ribomucleoprotein (RNB), which serves as a ciscase marker. The condtion is apparently immunologically distinct from other connective tissue disorders, Low titer RF and anti-ssDNA are common features of MCTD, but other autoantibodies are less commonly observed. Anti-centromere antibody is the cistinctive marker for the CREST varient of scleroderma, characterized by calcinosis, Raynaud phenomenon, esophageal dysmotiity, sclerodactyly, and telangiectasia, Anti Scl 70 is the distinctive marker for diffuse scleroderma, or systemic sclerosis, a multisystem disorder in which fibrosis of the skin, blood vessels, and viscera occurs. High titers of anti-histone antibody without other autoantibodies is the distinctive marker for drug-induced lupus. This disorder is patticularly associated with acministration of L hydralazine and procainamide, High titers of anti-dsDINA is the distinctive marker for systemic lupus erythematosus (SLE) ‘Mark this question e Question Id : 150785 Question 30 of 30 A.47 year old woman has a feeling of sand in her eyes and reports difficulty swallowing such foods as crackers or toast. Biopsy of an enlarged salivary gland reveals lymphocytic infiltration. There is hyperplasia of the ductal linngs and signs of fibrosis and hyalinization of the acini. The pair of tests that would likely reveal positive results in this patient includes a) Anti-centromere antibody and rheumatoid factor 'b) Anti-Scl-70 antibody and anti-Smith antibody ©) Anti-Smith antibody and anti-double stranded DNA antibody 4) Rheumatoid factor and anti-double stranded DNA, ©) Rheumatoid factor and anti-SS-A antibody Question Explanation: This patient has Sjogren syndrome, which is an autoimmune dicease characterized by lymphocytic infiltration of exocrine glands resulting in dry mouth (xerostomia) and dry eyes (keratoconjunctivitis sicca). Patients have an increased risk: of malignant lymphome. “Autoantibodies produced include anti-Ro (SS-A), anti-La (S$-B), antinaclear antibodies and rheumatoid factor. Anti-centromere antibodies are a very spetific marker for CREST syndrome. Rheumatoid factor (IgM antibodies ageinst one’s own IgG) is usually postive in rheumatoid arthritis, but may be seen in low titers in patients with other autoimmune diseases and chronic inflammatory conditions. Anti-Sc!-70 antibodies are seen in patients with scleroderma Anti-Smith antibody is seen in systemic lupus erythematosus. Anti-Smith antibody and anti-double stranded DNA antibody are associated with systemic lupus erythematosus. ‘Rheumatoid factor is seen in approximately 80% of patients with rheumetoid arthritis and in low tters in other autoimmune disorders. Anti-double stranded DNA (anti-dsDNA) is seen in patients with SLE. ‘Mark this question e Question Id : 150785 Question 30 of 30 A.47 year old woman has a feeling of sand in her eyes and reports difficulty swallowing such foods as crackers or toast. Biopsy of an enlarged salivary gland reveals lymphocytic infiltration. There is hyperplasia of the ductal linngs and signs of fibrosis and hyalinization of the acini. The pair of tests that would likely reveal positive results in this patient includes 8) Asti-centromere antibody and rheumatoid factor 'b) Anti-Scl-70 antibody and anti-Smith antibody ©) Anti-Smith antibody and anti-double stranded DNA antibody 4) Rheumatoid factor and anti-double stranded DNA, J © ©) Rheumatoid factor and anti-SS-A antibody Question Explanation: This patient has Sjogren syndrome, which is an autoimmune dicease characterized by lymphocytic infiltration of exocrine glands resulting in dry mouth (xerostomia) and dry eyes (keratoconjunctivitis sicca). Patients have an increased risk: of malignant lymphome. “Autoantibodies produced include anti-Ro (SS-A), anti-La (S$-B), antinaclear antibodies and rheumatoid factor. Anti-centromere antibodies are a very spetific marker for CREST syndrome. Rheumatoid factor (IgM antibodies ageinst one’s own IgG) is usually postive in rheumatoid arthritis, but may be seen in low titers in patients with other autoimmune diseases and chronic inflammatory conditions. Anti-Sc!-70 antibodies are seen in patients with scleroderma Anti-Smith antibody is seen in systemic lupus erythematosus. Anti-Smith antibody and anti-double stranded DNA antibody are associated with systemic lupus erythematosus. ‘Rheumatoid factor is seen in approximately 80% of patients with rheumetoid arthritis and in low tters in other autoimmune disorders. Anti-double stranded DNA (anti-dsDNA) is seen in patients with SLE. 2/27/2018 2:24:23 PM ‘Mark this question => Question Td : 20041 Question 1 of 30 A.52-year-old female has rheumatoid arthritis. She states that she is allergic to Penicilin and C'o-Trimoxazole. Therefore, which of the following drags is contraindicated? 4) Azathioprine ) Sulphasalazine ©) Ciclosporin 4) Gold therapy ©) Metaotrexate Question Explanation: Both co-trimoxazole and sulphesalazine contain sulphonamide groups and hence an allergy to co-trimoxazole would be a contra- indication to the use of sulphasalazine. Co-trimoxazole is a combination of D-aminosalicyclic acid and suapyridine 2/27/2018 2:24:23 PM ‘Mark this question => Question Td : 20041 Question 1 of 30 A.52-year-old female has rheumatoid arthritis. She states that she is allergic to Penicilin and C'o-Trimoxazole. Therefore, which of the following drags is contraindicated? 4) Azathioprine Y © b) Sulphasatazine ©) Ciclosporin 4) Gold therapy ©) Metaotrexate Question Explanation: Both co-trimoxazole and sulphesalazine contain sulphonamide groups and hence an allergy to co-trimoxazole would be a contra- indication to the use of sulphasalazine. Co-trimoxazole is a combination of D-aminosalicyclic acid and suapyridine 2/27/2014 2:21:44 PM ‘Mark this question & => Question Id : 28212 Question 2 of 30 A 3l-year-old male presents with a week history of a painful right leg Past medical history reveals thet he had erythema nodosum and recurrent oral and scrotal ulceration, Examination reveals a diffusely swollen left leg. What is the most likely cause of his swollen leg? a) Cells b) Venous thrombosis c) Lymphedema 9) Pyomyositis ¢) Ruptured Popliteal Baker's) cyst Question Explanation: This man has clinical features of Behcet's syndrome, He has had erythema nodosum (EN) 50% ofpatients with Bechet’s have an episode of EN throughout the course ofthe disease. The condition is a systemic vasculitis typified by: + Recurrent aphihous ulcers. © Genital ulcers © Uveitis © Skin lesions. ‘Venous thrombosis is a characteristic manifestation of Behcet's, The most likely case of this man’s swollen leg is therefore venous thrombosis. 2072018 2:21:41 PM ‘Mark this question <~ Question 2 of 30 Question Id : 28212 A 3l-year-old male presents with a week history of a painful right leg Past medical history reveals thet he had erythema nodosum and recurrent oral and scrotal ulceration, Examination reveals a diffusely swollen left leg. What is the most likely cause of his swollen leg? a) Cefalts Y © b) Venous thrombosis c) Lymphedema 4) Pyemyositis ©) Ruptured Popliteal Baker's) cyst Answer (| Bxrianaton | Other User's Explanation Report An Error Question Explanation: This man has clinical features of Behcet's syndrome, He has had erythema nodosum (EN) 50% ofpatients with Bechet’s have an episode of EN throughout the course ofthe disease. The condition is a systemic vasculitis typified by: + Recurrent aphihous ulcers. © Genial ulcers © Uveitis © Skin lesions ‘Venous thrombosis is a characteristic manifestation of Behcet's, The most likely case of this man’s swollen leg is therefore venous thrombosis. 2/27/2014 2:21:54 PM ‘Mark this question e& => Question Td : 28382 Question 3 of 30 A. 23-year-old boy with known hereditary angioneurotic edema presents with a recurrent fever, arthralgia and a rash on the face and the upper chest, Despite treatmert, he has recurrent attacks and required adrenaline on several occasicns. His CA levels have been persistently reduced secondary to this. What is the most likely canse for his current symptoms? ) Systemic taps erythematosus (SLE) b) Detmetomyesitis ) Erythema Multiforme 4) Psoriasis with arthropathy 2) Viral iiness Question Explanation: Hereditary angioneurotic edema is characterized by deficiency of C1 esterase inhibitor. This leads to persistent activation of the classical complement pathway and C4 levels are frequently low secondary to activation and consumption, TFtreatment fails to normalize the C4 levels and they remain persistently low, these patients are at an increased risk of developing SLE, 2/27/2014 2:21:54 PM ‘Mark this question e& => Question Td : 28382 Question 3 of 30 A. 23-year-old boy with known hereditary angioneurotic edema presents with a recurrent fever, arthralgia and a rash on the face and the upper chest, Despite treatmert, he has recurrent attacks and required adrenaline on several occasicns. His CA levels have been persistently reduced secondary to this. What is the most likely canse for his current symptoms? Y © a) Systemic iupus erythematosus (SLE) b) Detmetomyesitis ) Erythema Multiforme 4) Psoriasis with arthropathy 2) Viral iiness Question Explanation: Hereditary angioneurotic edema is characterized by deficiency of C1 esterase inhibitor. This leads to persistent activation of the classical complement pathway and C4 levels are frequently low secondary to activation and consumption, TFtreatment fails to normalize the C4 levels and they remain persistently low, these patients are at an increased risk of developing SLE, 2/27/2014 2:22:08 PM. ‘Mark this question = => Question 4 of 30 Armiddle aged man has had increasing back pain and right hip pain for the past 10 years. The pain is worse at the end of the day. He has bony enlargement ofthe distal interphalangeal jomts He is otherwise healthy with no prior medical history. Presence of prominent osteophytes involving the vertebral bodies is seen on the spine radiograph. Radiograph of the pelvis shows sclerosis with narrowing of the joint space at the right acetabulum seen Which of the following pathologic processed is most likely to be taking place in this patient? a) Gont 'b) Lyme disease c) Osteoarthritis 4) Osteomyelitis €) Rheumatoid arthritis Answer | Explanation Other User's Explanation Report An Error Question Explanation: Degenerative osteoarthritis is a common and progressive condition that becomes more frequent and symptomatic with aging There is, erosion and loss of articular cartilage. Rheumatoid arthntis typically involves small joints of the hands and fect most severely, and there is a destructive pannus thatleads to marked joint deformity. A gouty arthritis is more likely to be accompanied by swelling and deformity with joint destruction, The pain is not related to usage. Osteomyelitis represents an ongoing infection that produces merked bone deformity not just joint narrowing Lyme disease produces a chronic arthritis but it is typically preceded by a deer tick bite with askia lesion. Iris much less common than osteoarthritis, 2/27/2014 2:22:08 PM. ‘Mark this question = => Question 4 of 30 Amite aged man has had increasing back pain and right hip pain for the past 10 years. The pain is worse at the end of the day. He has bony enlargement of the dstal interphalangeal jonts. He is otherwise healthy with no prior medical history. Presence of prominent osteophytes involving the vertebral bodies is seen on the spine radiograph. Radiograph of the pelvis shows sclerosis wih narrowing of the joint space atthe right acetabulum seen Which of the following pathologic processed is most likely to be taking place in this patient? a) Gout b) Lyme disease Y¥ © c) Osteoarthritis 4) Osteomyelitis ©) Rheutmateid atthrits Answer | Explanation Other User's Explanation Report An Error Question Explanation: Degenerative osteoarthritis is a common and progressive condition that becomes more frequent and symptomatic with aging There is, erosion and loss of articular cartilage. Rheumatoid arthntis typically involves small joints of the hands and fect most severely, and there is a destructive pannus thatleads to marked joint deformity. A gouty arthritis is more likely to be accompanied by swelling and deformity with joint destruction, The pain is not related to usage. Osteomyelitis represents an ongoing infection that produces merked bone deformity not just joint narrowing Lyme disease produces a chronic arthritis but it is typically preceded by a deer tick bite with askia lesion. Iris much less common than osteoarthritis, 2/27/2014 2:22:25 PM / feaemad.omran- Yahoo x )/ 9% ww.interface.edupkjn x tele] =: € GD www. interface edu pk /medical-exams/tast-analysis phpautid=14759 Apps FJ coogie A setungs [) signin Cl imported Fromie 2 .. abe awl En yo Geil Sap Ww Gi other bookmar ‘Maske this question <& => (Question Td : 30558 Question 5 of 30 A 52-years-cld woman presented with a two week history of malaise and lower limb joint pain, associated with a vasculiic rash over her shins, thighs and buttocks Investigations revealed: Haemoglobin 9.8 g/dL (11.5-16.5) Platelet count 275 109 /L (150-400 x109) Serum creatinine concentration 452 mol (60-110) Antinuclear antibodies Negative Antineutrophl cytoplasmic antibodies Negative Antiglemerutar basement membrane antibodies Negative Dipetis urinalysis Blood H+ Protein + ‘What is the most likely diagnosis? a) Amayloidosis b) Hemolytic uremic syndrome c) Membranous neptropathy 4) Myeloma ¢) Henoch-Schonlein nephritis Question Explanation ‘The distribution of the rash together with lower limb joint pains end renal involvement are most suggestive of Henoch-Schonlein purpura This usually occurs in children aged 2-10 years but can occur in any age group. The orly way of differentiating this condition from other small vessel vasculitides is by biopsy the hallmark being Ig deposition in vessel walls on drect immunofluorescence ‘Membranous nephropathy is a histclogical diagnosis and usually presents with proteinuria only as does amyloidosis. Myeloma can rarely cause vasculitis which is ANCA negative but this is rare and unlicely. HUS causes haemoglobinuria rather than active renal sediment Total Questions ZR BARRE ERE RE RRR ER EBB E EEE Re oR ee XxXxXXXKXXKXKXXKKXKXKXKKXKXXKXKXKXXKKXKXKXKXKXXK 2/27/2014 2:22:25 PM / feaemad.omran- Yahoo x )/ 9% ww.interface.edupkjn x tele] =: € GD www. interface edu pk /medical-exams/tast-analysis phpautid=14759 Apps FJ coogie A setungs [) signin Cl imported Fromie 2 .. abe awl En yo Geil Sap Ww Gi other bookmar ‘Maske this question <& => (Question Td : 30558 Question 5 of 30 A 52-years-cld woman presented with a two week history of malaise and lower limb joint pain, associated with a vasculiic rash over her shins, thighs and buttocks Investigations revealed: Haemoglobin 9.8 g/dL (11.5-16.5) Platelet count 275 109 /L (150-400 x109) Serum creatinine concentration 452 mol (60-110) Antinuclear antibodies Negative Antineutrophl cytoplasmic antibodies Negative Antiglemerutar basement membrane antibodies Negative Dipetis urinalysis Blood H+ Protein + ‘What is the most likely diagnosis? a) Amayloidosis b) Hemolytic uremic syndrome c) Membranous neptropathy 4) Myeloma Y © e)Henoch-Schonlein nephritis Question Explanation ‘The distribution of the rash together with lower limb joint pains end renal involvement are most suggestive of Henoch-Schonlein purpura This usually occurs in children aged 2-10 years but can occur in any age group. The orly way of differentiating this condition from other small vessel vasculitides is by biopsy the hallmark being Ig deposition in vessel walls on drect immunofluorescence ‘Membranous nephropathy is a histclogical diagnosis and usually presents with proteinuria only as does amyloidosis. Myeloma can rarely cause vasculitis which is ANCA negative but this is rare and unlicely. HUS causes haemoglobinuria rather than active renal sediment Total Questions ZR BARRE ERE RE RRR ER EBB E EEE Re oR ee XxXxXXXKXXKXKXXKKXKXKXKKXKXXKXKXKXXKKXKXKXKXKXXK 2/27 2014 2:22:39 PM ‘Mark this question = i Question 6 of 30 A.55 year old male presents with acute onset on paiaful great right toe. Itis swollen, red and tender. This is his first episode. You decide to order a serum uric acid and 24 hour urine for uric atid, You would then prescribe which of the following? a) Colchicine b) Indomethacin ¢) Allopurinol 4) Probenacid Answer | Bxrianation Other User's Explanation Report An Error Question Explanation: Goutis a disease that can occur when purine metabolism gives rise to uric acid, Uric acid is normally excreted in the urine, But in certain people is done, These patients are called undersecreters. While other patients can excrete the uric acid normally but their body produces larger than normal amounts, Taese patients are called overproducers. The treatment of acute gout involves NSAIDS such as indomsthacin, Colchicine is also prescribed butis not as well tolerated due to side effects, The treatment of chronic gout involves allopurinol for overproducers and probenacid for undersecretors, The tests ordered (serum uric acid and 24 une for uric acid) should help classifying which category this patients in. 2/27 2014 2:22:39 PM ‘Mark this question = i Question 6 of 30 A.55 year old male presents with acute onset on paiaful great right toe. Itis swollen, red and tender. This is his first episode. You decide to order a serum uric acid and 24 hour urine for uric atid, You would then prescribe which of the following? a) Colchicine Y¥ © b) Indomethacin ¢) Allopurinol 4d) Probenacid Answer | Bxrianation Other User's Explanation Report An Error Question Explanation: Goutis a disease that can occur when purine metabolism gives rise to uric acid, Uric acid is normally excreted in the urine, But in certain people is done, These patients are called undersecreters. While other patients can excrete the uric acid normally but their body produces larger than normal amounts, Taese patients are called overproducers. The treatment of acute gout involves NSAIDS such as indomsthacin, Colchicine is also prescribed butis not as well tolerated due to side effects, The treatment of chronic gout involves allopurinol for overproducers and probenacid for undersecretors, The tests ordered (serum uric acid and 24 une for uric acid) should help classifying which category this patients in. 2/27/2014 2:22:54 PM S@emad.omran- Yahoo i x ) 93 www.mterface.ecupkin x \ S 5 € CB www interface.edu.pk /mectical-axcams/tast-analysis, php ?ut ops J coogle S\ seungs [) sinin Cl imperted Homie 2. wubb Qala Eon go cou ayer Ww Gi other bookmar ‘Maske this question = => Question Id : 51719 Question 7 of 30 A 64 year old man comes to you with pain swelling of the left great toc at the metatarsophalangeal joint, Examination shows itis crythematosus, warm, swollen, and tender to touch, The patient has a history of diabetes mellitus controlled by dict, and hypertension. His medications inclade hydrochiorothiazide, 25 mg/day. A CBC and blood chemistry profiles are normal, except for a uic acid level of 92 mg/dL QF 3.6-8 5). True staternent regarding this situation is a) This attack should resolve spontaneously in 3-4 days b) Allopurinol (Zyloprim) therapy should be started c) The elevated uric level establishes the diagnosic of gout 4) Intra articular steroid injection should be avoided €) Stopping the hydrochlorothiazide may control the hyperuricemia Anower UEQIENEAR) oe: Users Explanation Repo An Err Question Explanation: ‘This isa typical presentation for gout. Blevated uric acid levels are not necessary for the diagnosis, as there are some patients with nonmal uric acid levels who sill have gout. Conversely, hyperuricemia does not establish the diagnosis of gout. Bisk factors for the development of gout include several enzyme deficiencies, renal insufficiency, hypertension, obesty, moonshine ingestion (causing lead exposure), and alcohol abuse. There are several medications thet elevate uric acid, including diuretics, low dose salicylates, niacin, cyclosporine, ethambutol, and pyratinamide. A typical gout attack such as the one described will resolve spontaneously within 2 ‘weeks without treatment. In patients who have an acute monoarticular arthritis in addition to gout, other diagnoses such as osteoarthritis, pseudogout, and infection rmust he consiclered. The diagnosis of gontis established by aspiration of synoviel fhid or tophi, with characteristic wc acid orystale detected by polarized light microscopy Trestment can consist of NSAIDs in healthy individuals, Indomethacin ic considered the drug of choice. Corticosteroids oan alse be uced, and are particularly helpfil when the pationt has renal insufficiency. Intra articular injections of a certicostoroid such as triamcinolone are usefil, and intrerauscular corticosteroids may be especially usefil in patients with polyarticular gout. Colchicine may be used, but may cause diathea, Life style changes such as weight loss, discontinuing alcohol use, and changing antihypertensive therapy is often all thatis needed to control the lyperuicemia and thus prevent further attacks. Colchicine can be used for prophylaxis as well, although it does not alter liyperucicemia or prevent tophi ftom forming, Ifa patient has more than two attacks per year, urate chlorine therapy is indicated A 24-hour urine collection to identify whether the patient is an under excretor or an overproducer of uric acid would indicate the correct medication, Overproducers are treated with alloourinol, while under excretor benefit fiom probenecid if renal fimction is normal and there is no history of kidney stones. Total Questions be EE et RO ee DI I Ris BE i ZR RERRRRBRI XxxXxXXXKXKXKKXKXKXKKKKXKKKKXKKKKXKKKKKXKXK 2/27/2014 2:22:54 PM S@emad.omran- Yahoo i x ) 93 www.mterface.ecupkin x \ S 5 € CB www interface.edu.pk /mectical-axcams/tast-analysis, php ?ut ops J coogle S\ seungs [) sinin Cl imperted Homie 2. wubb Qala Eon go cou ayer Ww Gi other bookmar ‘Maske this question = => Question Id : 51719 Question 7 of 30 A 64 year old man comes to you with pain swelling of the left great toc at the metatarsophalangeal joint, Examination shows itis crythematosus, warm, swollen, and tender to touch, The patient has a history of diabetes mellitus controlled by dict, and hypertension. His medications inclade hydrochiorothiazide, 25 mg/day. A CBC and blood chemistry profiles are normal, except for a uic acid level of 92 mg/dL QF 3.6-8 5). True staternent regarding this situation is a) This attack should resolve spontaneously in 3-4 days b) Allopurinol (Zyloprim) therapy should be started c) The elevated uric level establishes the diagnosic of gout 4) Intra articular steroid injection should be avoided Y © ¢) Stopping the hydrochlorothiazide may control the hyperuricemia Anower (UBQIRREAR) oe: Users Explanation Report An Err Question Explanation: ‘This isa typical presentation for gout. Blevated uric acid levels are not necessary for the diagnosis, as there are some patients with nonmal uric acid levels who sill have gout. Conversely, hyperuricemia does not establish the diagnosis of gout. Bisk factors for the development of gout include several enzyme deficiencies, renal insufficiency, hypertension, obesty, moonshine ingestion (causing lead exposure), and alcohol abuse. There are several medications thet elevate uric acid, including diuretics, low dose salicylates, niacin, cyclosporine, ethambutol, and pyratinamide. A typical gout attack such as the one described will resolve spontaneously within 2 ‘weeks without treatment. In patients who have an acute monoarticular arthritis in addition to gout, other diagnoses such as osteoarthritis, pseudogout, and infection rmust he consiclered. The diagnosis of gontis established by aspiration of synoviel fhid or tophi, with characteristic wc acid orystale detected by polarized light microscopy Trestment can consist of NSAIDs in healthy individuals, Indomethacin ic considered the drug of choice. Corticosteroids oan alse be uced, and are particularly helpfil when the pationt has renal insufficiency. Intra articular injections of a certicostoroid such as triamcinolone are usefil, and intrerauscular corticosteroids may be especially usefil in patients with polyarticular gout. Colchicine may be used, but may cause diathea, Life style changes such as weight loss, discontinuing alcohol use, and changing antihypertensive therapy is often all thatis needed to control the lyperuicemia and thus prevent further attacks. Colchicine can be used for prophylaxis as well, although it does not alter liyperucicemia or prevent tophi ftom forming, Ifa patient has more than two attacks per year, urate chlorine therapy is indicated A 24-hour urine collection to identify whether the patient is an under excretor or an overproducer of uric acid would indicate the correct medication, Overproducers are treated with alloourinol, while under excretor benefit fiom probenecid if renal fimction is normal and there is no history of kidney stones. Total Questions be EE et RO ee DI I Ris BE i ZR RERRRRBRI XxxXxXXXKXKXKKXKXKXKKKKXKKKKXKKKKXKKKKKXKXK ‘Mark this question <= Question Id : 54359 Question 8 of 30 A patient with rheumatoid arthritis who has been taking aspitin present with a normocytic normochromic anemia, All of the following are true about his anemia, except a) Could be because of blood loss b) It will respond to treatment with vitamin B12 c) MCV of 1 ) Blevated reticulocyte count Question Explanatior “When anormocytic normechromic anemia is encountered, the anemia can have 3 possible etiologies (1e. blood loss, hemolysis, decreased production). In most anemias, one ofthese causes is the dominant factor. ‘Hemolysis will show an increased reticulocyte count, Notmecytic anemnias have an MC value between 80 and 100, Vitamnin B12 is given for anemia due to vitamin B12 deficiency. This type of anemia would be macrocytc. ‘Mark this question <= Question Id : 54359 Question 8 of 30 A patient with rheumatoid arthritis who has been taking aspirin present with a normocytic normochromic anemia. All of the following are true about his anema, except a) Could be because of blood loss Y © b) Iwill respond to treatment wit vitamin B12 c) MCV of 91 d) Elevated reticulocyte count Question Explanatior “When anormocytic normechromic anemia is encountered, the anemia can have 3 possible etiologies (1e. blood loss, hemolysis, decreased production). In most anemias, one ofthese causes is the dominant factor. ‘Hemolysis will show an increased reticulocyte count, Notmecytic anemnias have an MC value between 80 and 100, Vitamnin B12 is given for anemia due to vitamin B12 deficiency. This type of anemia would be macrocytc. ‘Mari this question <=> Question Td: 54551 Question 9 of 30 A.66 year old women is concerned about osteoporosis. The study of choice for determining whether or not she as osteoporosis is a) A quantitative CT scan of the hip and spine ) Dual energy X-ray absorptiometry (DEXA) of the hip and spine ©) Plain radiographs of the hip and spine 4) Ultrasonography of the heal ©) A DEXA scan of the middle phalans of the non-dominant hand Question Explanation: Dual energy X-ray absorptiometry (DEXA) of the hip and pine is the method of choice for assessment of bone mineral densiy. ‘Quantitative CT is the most sensitive method, but results in substantially greater radiation exposure. Plain radiographs are no sensitive enough to diagnose osteoporosis until total bone density has decteased by 50%. The predictive value of DEXA scans and ubrasound examinations of peripheral bones in assessing fracture risk at the hip of verterbrae is not clear. ‘Mari this question <=> Question Td: 54551 Question 9 of 30 A.66 year old women is concerned about osteoporosis. The study of choice for determining whether or not she as osteoporosis is a) A quantitative CT scan of the hip and spine Y © b) Dual energy X-ray absorptiometry (DEXA) of the hip and spine ©) Plain radiographs of the hip and spine 4) Ultrasonography of the heal ©) A DEXA scan of the middle phalans of the non-dominant hand Question Explanation: Dual energy X-ray absorptiometry (DEXA) of the hip and pine is the method of choice for assessment of bone mineral densiy. ‘Quantitative CT is the most sensitive method, but results in substantially greater radiation exposure. Plain radiographs are no sensitive enough to diagnose osteoporosis until total bone density has decteased by 50%. The predictive value of DEXA scans and ubrasound examinations of peripheral bones in assessing fracture risk at the hip of verterbrae is not clear. ‘Mark this question ez Question Td : 54731 Question 10 of 30 Guillain-Barre syndrome is not associated with a) Proximal muscle weakness ) Areflexia ©) Treatment with antibiotics 4) Infection with C. jejuni Answer | Beplanation Other User's Explanation Report An Error Question Explanation: Guillain-Baeré syndrome is an acute, usually rapidly progressive inflammatory polyneuropethy characterized by muscular weakness and mild distal sensory loss, Cause is thought io be autoimmune. In about 2/3 of patients, the syndrome begins 5 days to 3 weeks after an infectious disorder, surgery, or vaccination, Infection is the trigger in » 50% of patients, common pathogens include Campylobacter jejuri, enteric viruses, herpesviruses (including cytomegalovirus and those causing infectious mononucleosis), and Mycoplasma Flaccid weakness predominates in most patients, itis always mote prominent than sensory abnommdlities and may be most prorrinent proximally. Relatively symmettic weakness with paresthesias usually begins in the legs and progresses to the armns, but it occasionally begins in the anns or head, Tn 90% of patients, weakness is manimal at 3 weeks. Deep tendon reflexes are lost. Sphincters are usualy spared, Facial and oropharyngeal muscles are weak in > 50% of patients with severe disease. Respiratory paralysis severe enough to require endotracheal intubation and mechanical ventilation occurs in 5 to 10%. ‘Diagnosis is clinical. Treatment includes plasmapheresis, y-globulin, and, for severe cases, mechanical ventilation, ‘Mark this question ez Question Td : 54731 Question 10 of 30 Guillain-Barre syndrome is not associated with a) Proximal muscle weakness ) Areflexia JY © ©) Treatment with antibiotics 4) Infection with C. jejuni Answer | Beplanation Other User's Explanation Report An Error Question Explanation: Guillain-Baeré syndrome is an acute, usually rapidly progressive inflammatory polyneuropethy characterized by muscular weakness and mild distal sensory loss, Cause is thought io be autoimmune. In about 2/3 of patients, the syndrome begins 5 days to 3 weeks after an infectious disorder, surgery, or vaccination, Infection is the trigger in » 50% of patients, common pathogens include Campylobacter jejuri, enteric viruses, herpesviruses (including cytomegalovirus and those causing infectious mononucleosis), and Mycoplasma Flaccid weakness predominates in most patients, itis always mote prominent than sensory abnommdlities and may be most prorrinent proximally. Relatively symmettic weakness with paresthesias usually begins in the legs and progresses to the armns, but it occasionally begins in the anns or head, Tn 90% of patients, weakness is manimal at 3 weeks. Deep tendon reflexes are lost. Sphincters are usualy spared, Facial and oropharyngeal muscles are weak in > 50% of patients with severe disease. Respiratory paralysis severe enough to require endotracheal intubation and mechanical ventilation occurs in 5 to 10%. ‘Diagnosis is clinical. Treatment includes plasmapheresis, y-globulin, and, for severe cases, mechanical ventilation, ‘Mark this question & => (Question Td : 54839 Question 11 of 30 A. 72-year-old with chronic renal impairment and atrial fibtillation for which he takes warfann, presents with an acutely tender and red left big toe Investigations reveal [Serum Creatinine] 200 micromolil|(50-100) [Serum Urate jo. Slmmoll — (0.12-0. 42)) ‘Which of the Following is the most appropriate treatment for this man’s presentation? 2) Allopurnol ) Prednicolone ©) Colchicine 4) Diclofenac ©) Acerarinophen 8) Probenecid. Answer | Bopanation Other User's Explanation Report An Error Question Explanation: This man presents with acute gout, has chronic renal impairment, AF and takes warfarin NSADDs would be the treatment of choice but may cause deterioration in renal function and would be associated with an increased risk of bleeding in the elderly. The adverse effects of colchicines (esp. GI symptoms) would be more dkely in the elderly and should probably be avoided in those with renal impairment of this degree. Thus, Steroids are probably the best option. Allopurinol may well precipitate/estacerbate acute gout and. are used once the acute attack has settled folowing adequate treatment Unicosune drugs (probenecid) are used orally to treat chronic gout and not acute. ‘Mark this question & => (Question Td : 54839 Question 11 of 30 A. 72-year-old with chronic renal impairment and atrial fibtillation for which he takes warfann, presents with an acutely tender and red left big toe Investigations reveal [Serum Creatinine] 200 micromolil|(50-100) [Serum Urate jo. Slmmoll — (0.12-0. 42)) ‘Which of the Following is the most appropriate treatment for this man’s presentation? 2) Allopurinol Y © b) Prednisolone ©) Colchicine 4) Dielofenae ¢) Acetaminophen 8) Probenecid. Answer | Bopanation Other User's Explanation Report An Error Question Explanation: This man presents with acute gout, has chronic renal impairment, AF and takes warfarin NSADDs would be the treatment of choice but may cause deterioration in renal function and would be associated with an increased risk of bleeding in the elderly. The adverse effects of colchicines (esp. GI symptoms) would be more dkely in the elderly and should probably be avoided in those with renal impairment of this degree. Thus, Steroids are probably the best option. Allopurinol may well precipitate/estacerbate acute gout and. are used once the acute attack has settled folowing adequate treatment Unicosune drugs (probenecid) are used orally to treat chronic gout and not acute. ‘Mate this question = => (Question Td : 55322 Question 12 of 30 A patient with long standing rheumatoid arthritis, who has been taking medications for it, develops sinninus. The most likely cause is a) Meneire disease b) Aspirin toxicity c) Labyrinthiis 4) Acoustic neuroma Question Explanation: Rheumetoid arthrtis (RA) is a chronic autoimmune disease, producing damege mediated by cytokines, chemokines, and metalloproteases. Peripheral joints (eg, wrists, metacarpophalangeal joints) are syrametrically inflamed, often resulting in progressive destruction of articular structures, usually accompanied by systemic symptoms. Diagnosis requires specific clinical, laboratory, and radiclogic criteria. If patient takes aspirin daily (for RA, for example), tt can cause or worsen tinnitus, For this reason, aspirin is no longer used for RA, as effective doses are after toxic. n—-Report An Error ‘Mate this question = => (Question Td : 55322 Question 12 of 30 4 patient with long standing rheumatoid arthritis, who has been taking medications for it, develops tinnitus. The most likely cause is a) Meneire disease Y © b) Aspirin toxicity c) Labyrinthitis d) Acoustic neuroma Question Explanation: Rheumetoid arthrtis (RA) is a chronic autoimmune disease, producing damege mediated by cytokines, chemokines, and metalloproteases. Peripheral joints (eg, wrists, metacarpophalangeal joints) are syrametrically inflamed, often resulting in progressive destruction of articular structures, usually accompanied by systemic symptoms. Diagnosis requires specific clinical, laboratory, and radiclogic criteria. If patient takes aspirin daily (for RA, for example), tt can cause or worsen tinnitus, For this reason, aspirin is no longer used for RA, as effective doses are after toxic. n—-Report An Error ‘Mark this question <= => (Question Id: 57008 Question 13 of 30 A 63-year-old lady is suffering from pain and stiftess of her shoulders and difficully getting out of a chair ‘Which of the following ‘would support a diagnosis of Polymyalgia theumatica? a) Hypothermia 'b) Musele tendemess ©) Proximal muscle weakness 4) Pelvic girdle sitiness e) Weight gein Question Explanation: Polymyalgia rheumatica presents with early morning stiffness of the shoulder and pelvic grdles, fever, ancresia, weightloss and malaise. There is no muscle tendemess or weakness and the feet are aever affected. Investigations may reveal - Normochromic/ normocytic anemia. - Raised erythrocyte sedimentation rate (ESR) often > 50 mmfhr - Raised alkaline phosphatase (ALP) and - Raised C-reactive protein (CRE). Featares of giant cell arteritis should be sought - Headache - Visual disturbance - Transisnt ischemic attacks (TLAs) - Jaw claudication and - Thickened, tender, pulseless temporal arteries. Diagnos is by temporal artery biopsy andlor characteristics response to steroids, ‘Mark this question <= => (Question Id: 57008 Question 13 of 30 A 63-year-old lady is suffering from pain and stiffness of her shoulders and difficulty getting out of a chair Which of the following ‘would support a diagnosis of Folymyalgia rheumatica? a) Hypothermia b) Muscle tendemess c) Proximal muscle weakness Y © 4) Pelvic girdle stifiness ©) Weight gein Question Explanation: Polymyalgia rheumatica presents with early morning stiffness of the shoulder and pelvic grdles, fever, ancresia, weightloss and malaise. There is no muscle tendemess or weakness and the feet are aever affected. Investigations may reveal - Normochromic/ normocytic anemia. - Raised erythrocyte sedimentation rate (ESR) often > 50 mmfhr - Raised alkaline phosphatase (ALP) and - Raised C-reactive protein (CRE). Featares of giant cell arteritis should be sought - Headache - Visual disturbance - Transisnt ischemic attacks (TLAs) - Jaw claudication and - Thickened, tender, pulseless temporal arteries. Diagnos is by temporal artery biopsy andlor characteristics response to steroids, 2027/2014 ‘Mark this question & => Question Id ; 60320 Question 14 of 30 PM. A4L-year-old with rheumatoid arthritis treated with a potentially remittive agent has a low oxygen saturation on routine oximetry prior to upper gastrointestinal endoscopy for severe dyspepsia. Further studies reveal elevated carboxymethemoglobin, The remittive agent she is receivingis probably a) Auranofin ) Azathioprine. ©) Cyclophospharaide. 4) Dapsone. «) Fenicillamine newer (UBINSTANY cine: users Explanation Repost An Ear Question Explanation: Dapsone can produce carbexymethemoglobmemia and low oxygen saturation in susceptible individuals. None of the other agents cause this problem. 2027/2014 ‘Mark this question & => Question Id ; 60320 Question 14 of 30 PM. A4L-year-old with rheumatoid arthritis treated with a potentially remittive agent has a low oxygen saturation on routine oximetry prior to upper gastrointestinal endoscopy for severe dyspepsia. Further studies reveal elevated carboxymethemoglobin, The remittive agent she is receivingis probably a) Auranofin ) Azathioprine. ©) Cyclophospharaide. Y © 4 Dapsone ¢) Penicillamine. newer (UBINSTANY cine: users Explanation Repost An Ear Question Explanation: Dapsone can produce carbexymethemoglobmemia and low oxygen saturation in susceptible individuals. None of the other agents cause this problem. 2272014 2:24:35 PM ‘Mark this question = => ‘Question Id : 62369 5 of 30 Question ‘Which one of the following groups is NOT at increased risk for osteoporosis? a) Elderly men b) Postmenopausal woman ) Adolescents 4) Patients receiving long-teem, high-dose prednisone ©) A 19-year-old male in a body cast after sustaining a spinal facture Question Explanation: Adolescents have a very low incidence of osteoporosis. All of the other groups are at increased risk. 2272014 2:24:35 PM ‘Mark this question = => ‘Question Id : 62369 5 of 30 Question ‘Which one of the following groups is NOT at increased risk for osteoporosis? a) Elderly men b) Postmenopausal woman Y © 6) Adolescents 4) Patients receiving long-teem, high-dose prednisone ©) A 19-year-old male in a body cast after sustaining a spinal facture Question Explanation: Adolescents have a very low incidence of osteoporosis. All of the other groups are at increased risk. 2/27/2014 2:24:53 PM ‘Mark this question = => Question Td : 63878 Question 16 of 30 A.women with Iupus suddenly develops a fever and notes that her urine is dati in color. Analysis of her wine reveals the presence of red cells, She has had some joint aching and been feeling unwell for afew days. Her malar rashis about the same as usual for her. “The feature thet suggests she is expenencing an exacerbetion of her disease 1s a) C3 and C4 are both normal b) C3is low and C4 is normal ¢) CHS, C3 and C4 is normal d) C3 and C4 are low Ancwor [UEIRINRIAN) other Users Explanation Report An Evan (Question Explanation: Serum complement levels (C3 and C4) indicate consumption of immune complexes. Elevated or normal serum complement levels indicate that immune complexes are being cleared. During alupus exacerbation, immune complexes are not cleared from the body. Therefore, decreesing serum complement levels are consistent with exacerbation of the disease (lupus flare) 2/27/2014 2:24:53 PM ‘Mark this question = => Question Td : 63878 Question 16 of 30 A.women with Iupus suddenly develops a fever and notes that her urine is dati in color. Analysis of her wine reveals the presence of red cells, She has had some joint aching and been feeling unwell for afew days. Her malar rashis about the same as usual for her. “The feature thet suggests she is expenencing an exacerbetion of her disease 1s a) C3 and C4 are both normal b) C3is low and CA is normal ¢) CHS, C3 and C4 is normal Y¥ © 4) C3 and C4 are bw Ancwor (UBISINRIAN) other User's Explanation Report An Exot (Question Explanation: Serum complement levels (C3 and C4) indicate consumption of immune complexes. Elevated or normal serum complement levels indicate that immune complexes are being cleared. During alupus exacerbation, immune complexes are not cleared from the body. Therefore, decreesing serum complement levels are consistent with exacerbation of the disease (lupus flare) ‘Mark this question <=> Question Id: 87093 Question 17 of 30 An 82 year old man has onset low back pain, X-ray of the lower back and pelvis shows sclerotic changes in the lower vertebrae and in focal areas throughout the pelvis. The radiologist’ report states that the sclerotic changes may represent osteoarthritis, however, a neoplastic process cannot be excluded. The most cost effective method in the initial work-up of this patient is a) Bone marrow aspirate and biopsy b) Digtal rectal exam c) Prostate-specific antigen 4) Radiomclide bone scan ¢) Serum alkaline phosphatase Question Explanation: (Osteoarthritis is the most common rheumatologic disease, the prevalence of which increases with age. It primarily involves weight bearing joints, hence its diswibution in the lower vertebrae, pelvic bones, and proximal femur. Sclerotic bone, representing reactive bone formation, develops as a reaction to injury and is responsible for the slightly elevated serum alkaline phosphatase levels thet normally occur in much of the elderly population. IF prostate cancer with osteoblastic (bone forming) metastases to the vertebral column and pelvis were present in this patient, a digital rectal exam would be the most cost effective initial step in the work-up. With advanced prostate cancer, the gland would very likely be erlarged and hard ("stony"), A bone marrow aspirate and biopsy is not usualy part of the normal work-up of possible metastatic prostate cancer and has no place in the evaluation of osteoarthntis. A prostate specific entigen (PSA) level should be ordered in this patient, but not as the initial step in the work-up, since it does not distinguish hyperplasia from cancer and is firly expensive. In known cases of prostate cancer the PSA is a measure of tumor burden. and is used to monitor recurrences when folowing patients who have been teated for prostate cancer. A radionuclide bone scan is commonly used to rule out metastasis in patents with prostate cancer. Itis expensive and is not used as a screening test for prostate cancer. The serum alkaline phosphatase is typically elevated in metastatic prostate cancer due to ostecblastic activity in the metastatic foci, However, an elevated serum alkaline phosphatase is non-specific, since it may be shghtly increased in osteoarthritis (reactive bone formation) as well as in liver disease, ‘Mark this question <=> Question Id: 87093 Question 17 of 30 An 82 year old man has onset low back pain. X-ray of the lower back and pelvis shows sclerotic changes in the lower vertebrae and in focal areas throughout the pelvis. The radiologisi's report states that the sclerotic changes may represent osteoarthriiis, however, a neoplastic process cannot be excluded. The most cost effective method in the initial work-up of this patient is a) Bore marrow aspirate and biopsy Y © ’) Digtal rectal exam c) Prostate-specific antigen d) Radiomelide boa scan 2) Serum alkaline phosphatase Question Explanation: (Osteoarthritis is the most common rheumatologic disease, the prevalence of which increases with age. It primarily involves weight bearing joints, hence its diswibution in the lower vertebrae, pelvic bones, and proximal femur. Sclerotic bone, representing reactive bone formation, develops as a reaction to injury and is responsible for the slightly elevated serum alkaline phosphatase levels thet normally occur in much of the elderly population. IF prostate cancer with osteoblastic (bone forming) metastases to the vertebral column and pelvis were present in this patient, a digital rectal exam would be the most cost effective initial step in the work-up. With advanced prostate cancer, the gland would very likely be erlarged and hard ("stony"), A bone marrow aspirate and biopsy is not usualy part of the normal work-up of possible metastatic prostate cancer and has no place in the evaluation of osteoarthntis. A prostate specific entigen (PSA) level should be ordered in this patient, but not as the initial step in the work-up, since it does not distinguish hyperplasia from cancer and is firly expensive. In known cases of prostate cancer the PSA is a measure of tumor burden. and is used to monitor recurrences when folowing patients who have been teated for prostate cancer. A radionuclide bone scan is commonly used to rule out metastasis in patents with prostate cancer. Itis expensive and is not used as a screening test for prostate cancer. The serum alkaline phosphatase is typically elevated in metastatic prostate cancer due to ostecblastic activity in the metastatic foci, However, an elevated serum alkaline phosphatase is non-specific, since it may be shghtly increased in osteoarthritis (reactive bone formation) as well as in liver disease, ‘Mark this question ez Question Td : 87686 Question 18 of 30 A.62 year old woman presents with progressive tightening and hardening of the skin on her arms and face that has recently caused disfigurement and caused her difficulty in performing manual activities. You suspect an autcirmune disorder and order an antinuclear antibody panel. The results show antitopoisomerase I and no anticentromere antibodies. Ifthe patient is left untreated, her disease will most likely do which do which of the folowing? a) Conwert to a dermatologic malignancy ) Progress to involve other skin surfaces c) Progress to potentially fatal systemic fibrosis d) Remit completely 2) Stabilize, with residual dermal fibrosis Question Explanation: This patient has scleroderma (or systemic sclerosis), an autoimmune connective tissue disorder. The skin is most frequently involved inthis disease and is characterized by excessive tissue fibrosis. There is evidence for bth an immunologic and vascular etiology to the disease, Almost all patients with scleroderma have antinuclear antibodies. Those with the antitopoisomerase antibody usually develop diffuse systemic sclerosis, and they usually die from consequences of systemic disease such as pulmonery fibrosis or malignart hypertension, Scleroderma does not predispose to dermatologic malignancies Progression to involving other skin surfaces without visceral volvement suggests limited systemic sclerosis, associated with an anticentromere antibody. 96% of patents with another limited form of systemic sclerosis, the CREST syndrome (calcinosis, Raynaud phenomenoa, esophageal dysmotiity, sclerodactyly, telangiectasias), also have an anticentromere antibody. Sclerodermais a slowly progressive disease that will not stop or spontaneously reverse on its own, n Report An Error ‘Mark this question ez Question Td : 87686 Question 18 of 30 A.62 year old women presents with progressive tightening and hardening ofthe skin on her ans and face that has recently caused disfigurement and caused her difficulty in performing manual activities. You suspect an autcimmune disorder and order an antinuclear antibody panel. The results show anttopoisomerase I and no anticertromere antibodies. Ifthe patient is left untreated, her disease will most likely do which do which of the folowing? a) Conwert to a dermatologic malignancy ) Progress to involve other skin surfaces V © 2) Progress to potentially fatal systemic fibrosis, @) Remit completely «) Stabilize, with residual denmal fibrosis Question Explanation: This patient has scleroderma (or systemic sclerosis), an autoimmune connective tissue disorder. The skin is most frequently involved inthis disease and is characterized by excessive tissue fibrosis. There is evidence for bth an immunologic and vascular etiology to the disease, Almost all patients with scleroderma have antinuclear antibodies. Those with the antitopoisomerase antibody usually develop diffuse systemic sclerosis, and they usually die from consequences of systemic disease such as pulmonery fibrosis or malignart hypertension, Scleroderma does not predispose to dermatologic malignancies Progression to involving other skin surfaces without visceral volvement suggests limited systemic sclerosis, associated with an anticentromere antibody. 96% of patents with another limited form of systemic sclerosis, the CREST syndrome (calcinosis, Raynaud phenomenoa, esophageal dysmotiity, sclerodactyly, telangiectasias), also have an anticentromere antibody. Sclerodermais a slowly progressive disease that will not stop or spontaneously reverse on its own, n Report An Error 2/27/2014 2:25:37 PM S@emad.omran- Yahoo i x ) 93 www.mterface.ecupkin x \ S 5 € CB www interface.edu.pk /mectical-axcams/tast-analysis, php ?ut ops J coogle S\ seungs [) sinin Cl imperted Homie 2. wubb Qala Eon go cou ayer Ww Gi other bookmar ‘Maske this question <= => Question Id : 88067 Question 19 of 30 A 10 year old boy has a history of degenerative changes and pain in both shoulders and his left knee. Decreased ronge of motion is present. A complete blood count with differential is normel, and rheumatoid factor is not present, A sample of wine collected for routine urinalysis turns black upon standing at room temperature. The child is most ikely suffering fiom a) Childhood polycystic disease b) Ochronosis c) Paroxysmal nocturnal hemoglobinuria 4) Phenylkcetonuria €) Rhabdomyolysis Anower (UERIENEAR oho: Users Exon Question Explanatio ‘This boy's presentation, including the negative rheumatoid factor, suggests ostecarthritis at a very early age. The urine turning black on standing is classically associated with allcaptonuria (ochronosis), an arthritic disease caused by a deficiency of homogentisic acid oxidase (homogentisate 1, 2-dioxygenase). Allkaptomuria is an autosomal recessive disorder in which the lack of homogentisic oxidase blocks the metabolism of phenylclanine tyrosine at the level of homogentisic acid Thus, homogentisic acid accurmulates in the body. A large amount is excreted, imparting a black color to the urine if alowed to stand and undergo oxidation. The retained homogentisic acid selectively binds to collagen in connective tissues, tendons, and cartlage, imparting to these a blue black pigmentation (ochrenosis) most evident in the ears, nose, and cheeks. The most serious consequences of the orhronosis, however, stem from deposits of the pigment in the articular cartilages ofthe jeints. Tn some obscure menner, the pigmentation causes the cartilage to lose its normal resiliency and become brittle and fibrilated. Wear and tear erosion of this abnormal cartlage leade te demdetion of the cubchondral bone, and often tiny fragments of the fibrilated cartilage are driven into the underiying bone, worsening the damage. The vertebral cohiran, particularly the intervertebral dico, is the prime site of attack but lator the knees, shoulder, and hips maybs affected. The small joints of the hand and fect are usually spared. The metabclic defsctis present from birth, but the degenerative arthropathy usually develops slowly and becomes evident later. Childhood polycystic kicney disease is arare autosomal recessive disorder characterized by cystic enlargement of the kidneys and, usually, the liver. Death often occurs in infancy. Paroxysmal nocnumnal hemoglobinuria is associated with hemolytic anemia and reddish-brown urine. Phenylketonuria is classically associated with a deficiency of phenylalanine hydroxylase and with mental retardation, Rhabdomyolysis may occur with traumatic injury, muscle ischemia, seizures, excessive exercise, heat stroke, malignant hyperthermia, alcoholism, and various infectious or metabolic disorders It would be associated with reddish urine Report An Error Total Questions be EE fet RO Wo I tt BseeRE ZR RERRRRBRI XxxXxXXXKXKXKKXKXKXKKKKXKKKKXKKKKXKKKKKXKXK 2/27/2014 2:25:37 PM S@emad.omran- Yahoo i x ) 93 www.mterface.ecupkin x \ S 5 € CB www interface.edu.pk /mectical-axcams/tast-analysis, php ?ut ops J coogle S\ seungs [) sinin Cl imperted Homie 2. wubb Qala Eon go cou ayer Ww Gi other bookmar ‘Maske this question <= => Question Id : 88067 Question 19 of 30 A.10 year old bey has a history of degenerative changes and pain in both shoulders and his left knee. Decreased range of motion is present A complete blood count with differcatial is normal, and rheumatoid factor is not present. A sample of urine collected for routine urinalysis turns black upon sianding at room temperature. The child is most likely suffering from a) Childhood polycystic disease Y © b) Ochronosis c) Paroxysmal noctumal hemoglobinuria d) Phenyllcetonuria ¢) Rhabdomydlysis Anower (UERIENEAR oho: Users Exon Question Explanatio ‘This boy's presentation, including the negative rheumatoid factor, suggests ostecarthritis at a very early age. The urine turning black on standing is classically associated with allcaptonuria (ochronosis), an arthritic disease caused by a deficiency of homogentisic acid oxidase (homogentisate 1, 2-dioxygenase). Allkaptomuria is an autosomal recessive disorder in which the lack of homogentisic oxidase blocks the metabolism of phenylclanine tyrosine at the level of homogentisic acid Thus, homogentisic acid accurmulates in the body. A large amount is excreted, imparting a black color to the urine if alowed to stand and undergo oxidation. The retained homogentisic acid selectively binds to collagen in connective tissues, tendons, and cartlage, imparting to these a blue black pigmentation (ochrenosis) most evident in the ears, nose, and cheeks. The most serious consequences of the orhronosis, however, stem from deposits of the pigment in the articular cartilages ofthe jeints. Tn some obscure menner, the pigmentation causes the cartilage to lose its normal resiliency and become brittle and fibrilated. Wear and tear erosion of this abnormal cartlage leade te demdetion of the cubchondral bone, and often tiny fragments of the fibrilated cartilage are driven into the underiying bone, worsening the damage. The vertebral cohiran, particularly the intervertebral dico, is the prime site of attack but lator the knees, shoulder, and hips maybs affected. The small joints of the hand and fect are usually spared. The metabclic defsctis present from birth, but the degenerative arthropathy usually develops slowly and becomes evident later. Childhood polycystic kicney disease is arare autosomal recessive disorder characterized by cystic enlargement of the kidneys and, usually, the liver. Death often occurs in infancy. Paroxysmal nocnumnal hemoglobinuria is associated with hemolytic anemia and reddish-brown urine. Phenylketonuria is classically associated with a deficiency of phenylalanine hydroxylase and with mental retardation, Rhabdomyolysis may occur with traumatic injury, muscle ischemia, seizures, excessive exercise, heat stroke, malignant hyperthermia, alcoholism, and various infectious or metabolic disorders It would be associated with reddish urine Report An Error Total Questions be EE fet RO Wo I tt BseeRE ZR RERRRRBRI XxxXxXXXKXKXKKXKXKXKKKKXKKKKXKKKKXKKKKKXKXK ‘Mark this question e& => (Question Id : 96610 Question 20 of 30 A.62 year old woman presents with fatigue, morning stifiess for over 30 minutes, inability to comb her hair because of arm weakness, and inability to get up fiom a chair without assistance. These symptoms have developed gradually over several months. Examination revealed proximal muscle weakness. Investigation show the ESR. to be 70 mavhr. Rheumatoid factor is negative and creatinine kinase level is negative. The correct diagnosis is a) Polymyostis ) Giant cell arteritis ¢) Polymyalgia rheumatica @) Rheumatoid axthuitis «) Fibromyalgia syndrome Ancwer UERERNGRERY tne sere bxplanation Report An Ever (Question Explanatio Polymyalgia rheumatica is associated with temporal artentis 40% of the time, Patients are usually over 60 years of age. A normal EMG, creatinine kinase level, end muscle biopsy are seen. Polymyositis is an inflammatory myopathy. There is symmetrical weakness oflimb girdles, neck muscles, and pharynx. One would see an elevation of the creatinine kinase level and an abnormal EMG. Giant cell atentis is associated with unilateral headache and possible blindness fom involvement of the ophthalmic artery Rheumatoid arthriis usually presents as joint stiffness in the moming and nivolvement initially of the west joints. Rheumatoid factor (IgM) would be positive. Fibromyalgia is common in young patients and is associated with certam trigger points in the body ‘Mark this question e& => (Question Id : 96610 Question 20 of 30 A.62 year ld woman presents wih fatigue, morning stiffuess for over 30 minutes, inability to comb her hair because of ann weakness, and inabiliy to get up fiom a chair without assistance. These symptoms have developed gradually over several months. Exarrination revealed proximal muscle weakness. Investigation show the ESR to be 70 mem/hr. Rheumatoid factor is negative and creatinine kinase level is negative. The correct diagnosis is a) Polymnyosiis ) Giant cell arteritis Y © 2) Polymyalga rheumatica ) Rheumatoid arthritis ©) Fibromyalgia syndrome Ancwer UERERNGRERY tne sere bxplanation Report An Ever (Question Explanatio Polymyalgia rheumatica is associated with temporal artentis 40% of the time, Patients are usually over 60 years of age. A normal EMG, creatinine kinase level, end muscle biopsy are seen. Polymyositis is an inflammatory myopathy. There is symmetrical weakness oflimb girdles, neck muscles, and pharynx. One would see an elevation of the creatinine kinase level and an abnormal EMG. Giant cell atentis is associated with unilateral headache and possible blindness fom involvement of the ophthalmic artery Rheumatoid arthriis usually presents as joint stiffness in the moming and nivolvement initially of the west joints. Rheumatoid factor (IgM) would be positive. Fibromyalgia is common in young patients and is associated with certam trigger points in the body ‘Mark this question & => Question Id: 111092 Question 21 of 30 Roentegram studies of the spine in a 38 year old woman reveal no abnormalities. A follow up radiographic evaluation is performed approximately three months later and sacroiiitis is detected. The most ely diagnosis is a) Reiter's syndrome b) Psoriatic arthritis c) Arthritis associated with gastrointestinal defects 4) Rheumatoid artiitis 2) Osteoarthritis Question Explanation: Acthritis associated with gastrointestinal defects can present either with normal radiologic studies, or else with sacroilis of the spine Reiter’s syndrome and psoriatic arthritis patients present with asymmetric and nonmarginal syndestophytes, while ankylosing spondyiitic patients present with alinear pater of the lumbar spine (versus the notmal curved pattern), as well as a diffuse syndesmophyte formation. ‘Mark this question & => Question Id: 111092 Question 21 of 30 Roentegram studies of the spine in a 38 year old woman reveal no abnormalities. A follow up radiographic evaluation is performed approximately three months later and sacroiiitis is detected. The most ely diagnosis is a) Reiter’s syndrome b) Psoriatic arthuitis Y © 0c) Arthnitis associated with gastrointestinal defects 4) Rheumatoid artiitis 2) Osteoarthsitis Question Explanation: Acthritis associated with gastrointestinal defects can present either with normal radiologic studies, or else with sacroilis of the spine Reiter’s syndrome and psoriatic arthritis patients present with asymmetric and nonmarginal syndestophytes, while ankylosing spondyiitic patients present with alinear pater of the lumbar spine (versus the notmal curved pattern), as well as a diffuse syndesmophyte formation. ‘Mark this question eq => Question Td : 113202 Question 22 of 30 A.36 year old female with a long history of steroid treated lupus presents with severe right hip pain, Whatis the most licely diagnosis? a) Osteoarthritis b) Septic arthritis ©) Osteonecrosis 9) Ankylosing spondyitis ¢) Rheumatoid arthritis Avewor (NEQNGNRANY) osherucersExplanation Report An Eos Question Explanation: “This patent likely has osteonecrosis of her hip. Both chronic steroid use ard lupus itself are associated with increased occurrence of osteonecrosis. Ostecarturits, septic arthritis, ankylosing spondylitis, and rheumatoid arthrits are all unlicely to present m this manner. ‘Mark this question eq => Question Td : 113202 Question 22 of 30 A.36 year old female with a long history of steroid treated lupus presents with severe right hip pain. Whatis the most likely diagnosis? a) Osteoarthritis. ) Septic artis JY © ©) Osteonecrosis 4) Ankylosing spondylitis, ¢) Rheumatoid arthritis Avewor (NEQNGNRANY) osherucersExplanation Report An Eos Question Explanation: “This patent likely has osteonecrosis of her hip. Both chronic steroid use ard lupus itself are associated with increased occurrence of osteonecrosis. Ostecarturits, septic arthritis, ankylosing spondylitis, and rheumatoid arthrits are all unlicely to present m this manner. 2/27/2014 2:26:24 PM ‘Mark this question & => Question Id : 120682 Question 23 of 30 AS year old male as a swollen, extremely painfal night knee. He has a history of a few similar episodes over the last five years, involving either knees or anicles. He also has low grade fever. Synovial hid aspiration reveals no crystals or bacteria but a white count of 35,000. Choncrocalcinosis is revealed on knee radiography. The treatment of choice in this patient is a) Oral predaisone ) Intravenous colchicine. ) Oral allopurinol 4) Oral indomethacin. ¢) Intraauticular methyjprednisolone Ancwor (UEIFNRER) tne srs Explanation Report AnEror (Question Explanation: This patient probably hes pseudogout, or calcium pyrophosphate dihydrate crystal deposition, which is manifested by the chondrocalcinosis evident by radiography Calcium pyrophosphate dihydrate crystals are stall, rhomboid shaped, and relatively weakly positive birefringent. They may be scanty ir synovial fluid and dificult to see even if present. The synovial fhid leukocyte court is usually elevated, as itis here Oral ndomethacin or another nonsteroidal anti mflammatory agent is the treatment of choice m this condition. Oral prednisone should help but would be avoided because of its many potential side effects, unless there were some absolute contraindication to nonsteroidals (¢.g., renal insufficiency, allergy). Intravenous colchicine would be appropriate for acute gout, but does not typically have much effect on pseudogout. Oral allopurinol would not be of any value in pseudogou: and should not be used in acute gout because ofits likelihood of acutely altering the uric acid equiibrium and prolonging an attack. Intraarticular methylprednisolone might be beneficial, but also should be reserved for instances in which a nonsteroidal is contraindicated or snsufficient, 2/27/2014 2:26:24 PM ‘Mark this question & => Question Id : 120682 Question 23 of 30 AS year old male as a swollen, extremely painfal night knee. He has a history of a few similar episodes over the last five years, involving either knees or anicles. He also has low grade fever. Synovial hid aspiration reveals no crystals or bacteria but a white count of 35,000. Choncrocalcinosis is revealed on knee radiography. The treatment of choice in this patient is a) Oral predaisone ) Intravenous colchicine ) Oral allopurinol Y © A) Oral indomethacin. ¢) Intraauticular methyjprednisolone Ancwor (UEIFNRER) tne srs Explanation Report AnEror (Question Explanation: This patient probably hes pseudogout, or calcium pyrophosphate dihydrate crystal deposition, which is manifested by the chondrocalcinosis evident by radiography Calcium pyrophosphate dihydrate crystals are stall, rhomboid shaped, and relatively weakly positive birefringent. They may be scanty ir synovial fluid and dificult to see even if present. The synovial fhid leukocyte court is usually elevated, as itis here Oral ndomethacin or another nonsteroidal anti mflammatory agent is the treatment of choice m this condition. Oral prednisone should help but would be avoided because of its many potential side effects, unless there were some absolute contraindication to nonsteroidals (¢.g., renal insufficiency, allergy). Intravenous colchicine would be appropriate for acute gout, but does not typically have much effect on pseudogout. Oral allopurinol would not be of any value in pseudogou: and should not be used in acute gout because ofits likelihood of acutely altering the uric acid equiibrium and prolonging an attack. Intraarticular methylprednisolone might be beneficial, but also should be reserved for instances in which a nonsteroidal is contraindicated or snsufficient, ‘Mark this question => Question 24 of 30 ‘Which one of the following is NOT commonly associated with rheumatoid arthritis? a) Sjégrens syndrome, b) Symmetrical polyarthnts. c) Anemia, ) Glomerulonephaitis €) Pencardia effusion Question Explanation: Glomerulonephritis is not associated with rheumatoid arthritis. In contrast to many other connective tissue diseases, such as lupus and scleroderma, the kidneys are rarely involved in patients with rheumatoid arthritis, Sjégren’s syndrome is the complex of kceratoconjunctiviis sicca (dry eyes), xerostomia (dry mouth), and a connective tissue disease, often sheumateid arthritis. Symmetrical polyarthritis, especialy of the small joints of the hands and feet, is the classical joint feature of adult onsct thoumatoil arthritis. Most patients with rheumatoid arthritis have an anemia consistent with chronic disease, ie, normocytic and normochromic. Asymptomatic pericarcial effusions are noted in as many as 50% of people with rheumatcid arthritis who undergo echocardiography. ‘Mark this question eo Question 24 of 30 ‘Which one of the followings NOT commonly associated with rheumat>id arthrits? a) Siégrea‘s syndrome b) Symmetrical polyarthritis c) Anernia Y © d) Glomerulonephuitis ¢) Pencardial effusion Question Explanation: Glomerulonephritis is not associated with rheumatoid arthritis. In contrast to many other connective tissue diseases, such as lupus and scleroderma, the kidneys are rarely involved in patients with rheumatoid arthritis, Sjégren’s syndrome is the complex of kceratoconjunctiviis sicca (dry eyes), xerostomia (dry mouth), and a connective tissue disease, often sheumateid arthritis. Symmetrical polyarthritis, especialy of the small joints of the hands and feet, is the classical joint feature of adult onsct thoumatoil arthritis. Most patients with rheumatoid arthritis have an anemia consistent with chronic disease, ie, normocytic and normochromic. Asymptomatic pericarcial effusions are noted in as many as 50% of people with rheumatcid arthritis who undergo echocardiography. 2:27 2014 2:26:45 PM ‘Mare this question e => Question Id: 129495 Question 25 of 30 FALSE statement regarding systemic lupus erythematosis is which one of the following? a) Ibis increased in monozygotic as compared to dizygotic twins +b) It ie increased in women during childbearing years c) The relative risk of developing its increases as the number of susceptbiliy genes increases 4) Ultcaviolet light produces disease fares ©) Dg induced and spontaneous diseases have the same clinical and attozntibody picture Answer | Explanation Other User's Explanation Report An Error Question Explanation: Drug induced lupus presents clinically and immunologically differently compared with the spontaneous form of the disease A genetic predisposition for systemic lupus erythematosis (SLE) is suggested by the increared concordance in monozygotic a3 compared with slzygotic twins, a 10% frequency of patients with more then one affected family member, and correlations of MHC class II end TIT genes with the disease, The increased prevalence of the disease in women of childbearing age, as well as abnormalities of estrogens and ancrogens in affected persons, suggests a hormonel influence in the development of the disease The more genes susceptibility a person has, the greater the relative risk for developing SLE. I hes been calculated that at least three ot four genes are necessary for the disease to develop. Ultraviolet light, especially UV-B, causes the disease to flare in more than half the cases. 2:27 2014 2:26:45 PM ‘Mare this question e => Question Id: 129495 Question 25 of 30 FALSE statement regarding systemic Inpus erythematosis is which one of the following? a) I's increased in monczygotis as compared to dizygotic twins b) Ib is increased in women during childbearing years, c) The relative risk of developing it s increases as the number of susceptibility genes increases. 4) Ultraviolet light produces diseace fares. ¥ © 2)Dnug induced and spontaneous diseases have the same clinical and axtoentibody picture Answer | Explanation Other User's Explanation Report An Error Question Explanation: Drug induced lupus presents clinically and immunologically differently compared with the spontaneous form of the disease A genetic predisposition for systemic lupus erythematosis (SLE) is suggested by the increared concordance in monozygotic a3 compared with slzygotic twins, a 10% frequency of patients with more then one affected family member, and correlations of MHC class II end TIT genes with the disease, The increased prevalence of the disease in women of childbearing age, as well as abnormalities of estrogens and ancrogens in affected persons, suggests a hormonel influence in the development of the disease The more genes susceptibility a person has, the greater the relative risk for developing SLE. I hes been calculated that at least three ot four genes are necessary for the disease to develop. Ultraviolet light, especially UV-B, causes the disease to flare in more than half the cases. ‘Mark this question e => Question Id: 131498 Question 26 of 30 A31 year old woman has dry eyes, drmess of the mouth, and increased epistaxis. Examination showed xerostomia, keretoconjunciivitissicea, and dryness of the nasal mucosa. She was found to have positive titers of anti-SSE antibody. Useful diagnostic test in this condtion is a) Schilling test 'b) Gastric motiity test ©) Schirmer test ) Biopsy of the liver to quantify copper content €)Methacholine challenge test Question Explanatio This patient has Sjégren's syndrome, which is an autoimnmnane disease involving the salivary and lacrimel glands. The Schitmer test involves testing for dry eyes. Less than 5 mm oF weiting on fiter paper in five mimutes is considered a positive test. The Schilling test involves testing for vitamin B12 deficiency by radio-labeled cobalamin. The gastric motility tect involves testing for gastroparesis, which can occurin scleroderma or diabetic autonomic dysfunction, The gold standard to diagnose Wilson's disease is to quantitate the amount of copper in the liver by doing a biopsy. The methacholine challenge testis used to elicit symmptoms of wheezing in a patient suspected of having asthma. ‘Mark this question e => Question Id: 131498 Question 26 of 30 A31 year old woman has dry eyes, drmess of the mouth, and increased epistaxis. Examination showed xerostomia, keretoconjunciivitissicea, and dryness of the nasal mucosa. She was found to have positive titers of anti-SSE antibody. Useful agnostic test in this condtion is a) Schilling test 'b) Gastric motiity test Y © 0) Schirmer test d) Biopsy of the liver to quantify copper content €)Methacholine challenge test Question Explanatio This patient has Sjégren's syndrome, which is an autoimnmnane disease involving the salivary and lacrimel glands. The Schitmer test involves testing for dry eyes. Less than 5 mm oF weiting on fiter paper in five mimutes is considered a positive test. The Schilling test involves testing for vitamin B12 deficiency by radio-labeled cobalamin. The gastric motility tect involves testing for gastroparesis, which can occurin scleroderma or diabetic autonomic dysfunction, The gold standard to diagnose Wilson's disease is to quantitate the amount of copper in the liver by doing a biopsy. The methacholine challenge testis used to elicit symmptoms of wheezing in a patient suspected of having asthma. 2/27/2014 2:27:08 PM ‘Mark this question e& => Question 27 of 30 A.19 year cld boy presents with acute onset of a hot, pani, swollen knee. Which ore of the following test should be performed first? a) Urinalysis b) Lumbar puncture c) Arthrocentesis 4) Chest radiograph 2) Tuberonlin skin test Question Explanation: ‘An acutely swollen, hot, painfil joint is a medical emergency until the possibility of septic arthritis has been elminated, This can only bbe done by culturing synovial fluid obtained at the time of arthroceatesis. Some of the other tests might ultimately be necessary ifthe patient did have an infecied knee, in order to assess the extent of dissemination of the infection. Thus, a winalysis, 2 lumbar puncture with cultre of cerebrospinal Quid, and a chest radiograph to assess the possibilty of pneumonia all might be considered as later imerventions. another bacterial etiology for the knee arthritis was not found, and tuberculosis was a consideration in the differential diagnosis, then a taberculin skin test (PPD) would be reasonable 2/27/2014 2:27:08 PM ‘Mark this question e& => Question 27 of 30 A.19 year cld boy presents with acute onset of a hot, pani, swollen knee. Which ore of the following test should be performed first? a) Urinalysis b) Lumbar puncture Y © o) Asthrocentesis 4) Chest radiograph 2) Tuberonlin skin test Question Explanation: ‘An acutely swollen, hot, painfil joint is a medical emergency until the possibility of septic arthritis has been elminated, This can only bbe done by culturing synovial fluid obtained at the time of arthroceatesis. Some of the other tests might ultimately be necessary ifthe patient did have an infecied knee, in order to assess the extent of dissemination of the infection. Thus, a winalysis, 2 lumbar puncture with cultre of cerebrospinal Quid, and a chest radiograph to assess the possibilty of pneumonia all might be considered as later imerventions. another bacterial etiology for the knee arthritis was not found, and tuberculosis was a consideration in the differential diagnosis, then a taberculin skin test (PPD) would be reasonable ‘Mark this question e& => Question Td : 139489 Question 28 of 30 HLA testing of a 36 year old man with severe left hip pain and whose past history is sgniicant is positve for a a grayish, scaly rash on forehead and hands for several years, but has not required treatment, shows HLA-B27 positvity. HLA-B27 haplotype is strongly associated with which one of the following disease? a) Acute anterior uveitis b) Budd-Chiari syndrome ©) Goodpastire syndrome 4) Reye syndrome €) Sjogren syndrome Question Explanation: ‘The patient has psoriatic arthritis which is very strongly associated with HLA-B27, Psoriatic arthritis affects young men (who more often get sacroiltis) and females (who more often get the inflammatory form). The arthritis tends to develop after the rash in most patients, as in the one described above, The etiology of psoriatic arthritis is unknown: however it certeinly has an autoinnmne and genetic component, HLA-R27 is also associated with Reiter symdrome, which can he remembered as the syndrome in which you "can't see (anterior uveitis conjunctivitis), can't pee (urethritis), and can’t climb a tree a (arthvtis)." HLA-B27 is also associated with acute anterior uveitis, postgonccoccal arthritis, and anleylosing spondylitis. Budd-Chiori syndrome is liver disease secondary to ‘occlusion of the inferior vena cava or hepatis veins, There is no arthritic involvement, The most commen cause in this country is malignancy; and in the Far East, itis congenital webs in the IVC. Goodpasture syndrome is pulmonary hemorrhage and glomerular discese secondary to antibodies to basement membrane (GBM). Ibis one of the combined pulmonary renal syndromes, has an autoimmune etiology, and is most common in middle aged, Jewish men, Diagnosis is made by kidney biopsy and immune Quorescent staining which shows the antibodies attached to the GBM. Reye syndrome is an often fatal liver disease that can follow vial infection. (Cf note, it can be precipitated by use of salicylates in children. Sjogren syndrome is an autoimmune disease that damages salivary and tear glands. Petients typically have dry eyes and mouths ‘Mark this question e& => Question Td : 139489 Question 28 of 30 ‘HLA testing of a 36 year old man with severe left hip pain and whose past history ‘s significant is positwe for a a grayish, scaly rash ‘on forehead and hands for several years, but has not required treatment, shows HLA-B27 positivity. HLA-B27 haplotype is strongly associated with which one of the following disease? Y © a) Acute anterior uveitis 'b) Budd-Chiari syndrome c) Goodpasture syndrome 4) Reye syndrome €) Sioarea syndrome Question Explanation: ‘The patient has psoriatic arthritis which is very strongly associated with HLA-B27, Psoriatic arthritis affects young men (who more often get sacroiltis) and females (who more often get the inflammatory form). The arthritis tends to develop after the rash in most patients, as in the one described above, The etiology of psoriatic arthritis is unknown: however it certeinly has an autoinnmne and genetic component, HLA-R27 is also associated with Reiter symdrome, which can he remembered as the syndrome in which you "can't see (anterior uveitis conjunctivitis), can't pee (urethritis), and can’t climb a tree a (arthvtis)." HLA-B27 is also associated with acute anterior uveitis, postgonccoccal arthritis, and anleylosing spondylitis. Budd-Chiori syndrome is liver disease secondary to ‘occlusion of the inferior vena cava or hepatis veins, There is no arthritic involvement, The most commen cause in this country is malignancy; and in the Far East, itis congenital webs in the IVC. Goodpasture syndrome is pulmonary hemorrhage and glomerular discese secondary to antibodies to basement membrane (GBM). Ibis one of the combined pulmonary renal syndromes, has an autoimmune etiology, and is most common in middle aged, Jewish men, Diagnosis is made by kidney biopsy and immune Quorescent staining which shows the antibodies attached to the GBM. Reye syndrome is an often fatal liver disease that can follow vial infection. (Cf note, it can be precipitated by use of salicylates in children. Sjogren syndrome is an autoimmune disease that damages salivary and tear glands. Petients typically have dry eyes and mouths ‘Mari tis question = => Question Td 159650 Question 29 of 30 ASA year old male has reddering and itching of his skin, Examination shows erythematous areas covered by a silvery scale on the extensor sufaces of his elbow and knees. Pain in his hands and knees is also present, Joint involvement in ths condition is pathologically similar to that in which condition? a) Gout b) Osteoarthritis ©) Psendogout 4) Rheumatoid arthritis €) Suppurative arthritis Question Explanation: Psoriasis is associated with rheumatoid arthritis in 5% of cases, Hand involvement is common. The inflammatory process affects the tendon sheaths as well, leading to the characteristic sausage fingers. Pathologically psoriasis related anti is similar to rheumatoid arthritis, bur the process tends to be less destructive, Large joitts may also be involved. Psoriatic arthritis is a member ofthe category of seronegative spondyloanhropathies, which are rheumatoid factor negative and commonly associated with HLA-B27 haplotype (ankylosing spondyliis end Reter syndrome are other conditions in this group). Gout and pseudogout are crystal related inflammatory athropathies. Gout is caused by abnormal deposition of monosodium urate crystals, and pseudogout by deposition of calciun pyrophosphate, Neither of these conditions is associated with psoriasis. Degenerative joint disease or osteoarthritis is primanly a disease of cartilage, which undergoes progressive destructive changes marked by excessive carilage loss and reactive bone formation, Rheumatoid arthritis and related disorders are instead cue to an inflammatory process that begins with synovial membrane inflammation. Suppurative arthritis is caused by purulent bacteria that localize to joints, either by hematogenous dissernination or by spread from contiguous infectious foci. The most frequent organisms include S. aureus S. pyogenes, H. influenzae and gram negative bacteria, such as Salmonella E. coil and Pseudomonas. Suppurative arthnitis is an acute disease, manifesting with septic fever, and local infammatory signs of nvolved joints. ‘Mari tis question = => Question Td 159650 Question 29 of 30 A 54 year old male has reddening and itching of his skin. Examination shows erythematous areas covered by a silvery scale on the extensor surfaces of his elbow and knees. Pain in his hands and knees is also present. Joint invelvement in this condition is pathologically siilar to that in which condition? a) Gout ) Osteoarthritis ) Psendogout Y © &) Rheumatoid arthritis ©) Suppurative arthritis Question Explanation: Psoriasis is associated with rheumatoid arthritis in 5% of cases, Hand involvement is common. The inflammatory process affects the tendon sheaths as well, leading to the characteristic sausage fingers. Pathologically psoriasis related anti is similar to rheumatoid arthritis, bur the process tends to be less destructive, Large joitts may also be involved. Psoriatic arthritis is a member ofthe category of seronegative spondyloanhropathies, which are rheumatoid factor negative and commonly associated with HLA-B27 haplotype (ankylosing spondyliis end Reter syndrome are other conditions in this group). Gout and pseudogout are crystal related inflammatory athropathies. Gout is caused by abnormal deposition of monosodium urate crystals, and pseudogout by deposition of calciun pyrophosphate, Neither of these conditions is associated with psoriasis. Degenerative joint disease or osteoarthritis is primanly a disease of cartilage, which undergoes progressive destructive changes marked by excessive carilage loss and reactive bone formation, Rheumatoid arthritis and related disorders are instead cue to an inflammatory process that begins with synovial membrane inflammation. Suppurative arthritis is caused by purulent bacteria that localize to joints, either by hematogenous dissernination or by spread from contiguous infectious foci. The most frequent organisms include S. aureus S. pyogenes, H. influenzae and gram negative bacteria, such as Salmonella E. coil and Pseudomonas. Suppurative arthnitis is an acute disease, manifesting with septic fever, and local infammatory signs of nvolved joints. ‘Mark this question = (Question Td : 150924 Question 30 of 30 AAS year old woman has cifficulty swallowing, calcifying subcutaneous lesions, and diminished circulation to the fingers. Her fingers have thickened skin and are extended in an inflexible position, What autoantbodies are likely to be present in this patient? a) Ant-centromere antibody ) Anti-histone antibody ©) Ant-SS-A ) Ant $8-B ©) Rheumatoid factor Answer | Bolanation Other User's Explanation Report An Error Question Explanation: The symptoms described in the question are classic for CREST syndrome. This condition is an exemple of localized scleroderma with slow progression to lfe threatening complications. CREST syndrome is the combination of calcinosis, Raynaud's phenomenon, esophageal dysmotiliy, sclerodactyly, and telangiectasia Arti centromere antibodies are reported in a high percentage of patients with CREST, butin 2 stall percentage of pure scleroderma parents. Centromeres are the portion of the chromosome where spindle fibers atach during cell division. Anti-histone antibody is a marker for drug-induced lupus erythematosus, Anti-SS-A is amerker for Sjogren's syndrome, characterized by dry eyes and cry mouth. Anti-SS-B is an autoantibody directed against sibonucleoproteins, and is a marker for Sjogren’s syndrome, Rheumatoid factor is generally an IgM autoantibody directed aganstthe Fe portion of IgG. RF is positive in 80% of patents with rheumatoid arthritis and may also be found in low titers in patients with chronic infections, other autormmune diseases such as SLE and Sjogren's syndrome, or chronic pulmonary’, hepatic, or renal diseases. ‘Mark this question = (Question Td : 150924 Question 30 of 30 AAS year old woman has cifficulty swallowing, calcifying subcutaneous lesions, and diminished circulation to the fingers. Her fingers have thickened skin and are extended in an inflexitle postion, What autoantbodies are likely to be present in this patient? Y © a) Ant-centromere antibody ) Ant-histone antibody ©) Ant-SS-A d) Anti-SS-B ) Rheumatoid factor Answer | Bolanation Other User's Explanation Report An Error Question Explanation: The symptoms described in the question are classic for CREST syndrome. This condition is an exemple of localized scleroderma with slow progression to lfe threatening complications. CREST syndrome is the combination of calcinosis, Raynaud's phenomenon, esophageal dysmotiliy, sclerodactyly, and telangiectasia Arti centromere antibodies are reported in a high percentage of patients with CREST, butin 2 stall percentage of pure scleroderma parents. Centromeres are the portion of the chromosome where spindle fibers atach during cell division. Anti-histone antibody is a marker for drug-induced lupus erythematosus, Anti-SS-A is amerker for Sjogren's syndrome, characterized by dry eyes and cry mouth. Anti-SS-B is an autoantibody directed against sibonucleoproteins, and is a marker for Sjogren’s syndrome, Rheumatoid factor is generally an IgM autoantibody directed aganstthe Fe portion of IgG. RF is positive in 80% of patents with rheumatoid arthritis and may also be found in low titers in patients with chronic infections, other autormmune diseases such as SLE and Sjogren's syndrome, or chronic pulmonary’, hepatic, or renal diseases. ‘Maré this question => Question Id : 24722 Question 1 of 30 A 73-year-old man presents with painful lumps in his feet and is diagnosed with gout. Following initial treatment with non-steroidal anti-inflammatory agents, he is started on Allopurinol. How does this work? a) Inhibition of microtubule assembly ) Inhibits Kanthine oxidase areversibly ©) Inhibits cyclooxygenase TI 4) Inhibition of renal reuptake of uric acid ©) Inhibits Xanthine oxidase reversibly Anewor (NENINRAN) other User's Explanation Report AnEvror Question Explanation: Allopurinol inhibits xanthine oxidase irreversibly, the enzyme involved in the conversion of purines into uric acid. ‘Maré this question => Question Id : 24722 Question 1 of 30 A 73-year-old man presents with painful lumps in his feet and is diagnosed with gout. Following initial treatment with non-steroidal anti-inflammatory agents, he is started on Allopurinol. How does this work? a) Inhibition of microtubule assembly Y © b) Inhibits Xanthine oxidase rreversibly ©) Inhbits cyclooxygenase TL d) Inhibition of renal reuptake of une acid. e) Inhibits Xanthine oxidase reversibly Anewor (NENINRAN) other User's Explanation Report AnEvror Question Explanation: Allopurinol inhibits xanthine oxidase irreversibly, the enzyme involved in the conversion of purines into uric acid. 2/27/2014 2:29:08 PM ‘Mare this question <= => Question Id : 28142 Question 2 of 30 ‘A teenager treated for sexvally transmitted disease three months age hes alarge effusion in the right knee. Syaovial Guid analysis revealed a white cell count of 16 x 1031 but culture was negative, Which one of the following organisms is the most likely cause? a) Chlamydia Trachomatis b) Herpes Simplex c) Treponema Pallidun 4) Neisseria Gonomhea ©) Trichomonas Vaginalis Answer | Explanation Other User's Explanation Report An Error Question Explanation: Bacteria are the most common cause of moncarthits Staphylococcus aureus and gonococei are the most common causes of septic arthritis Neisseria gonorrhoeae occurs in young adults and is often preceded by a migratory tendonitis or arthritis. Gram stain is positive in 25% and culture positive in 50% ‘This patient has been treated previously for a semually acquired infection and this may be why the culture is negative, 2/27/2014 2:29:08 PM ‘Mare this question <= => Question Id : 28142 Question 2 of 30 ‘A teenager treated for sexvally transmitted disease three months age hes alarge effusion in the right knee. Syaovial Guid analysis revealed a white cell count of 16 x 1031 but culture was negative, Which one of the following organisms is the most likely cause? a) Chlamydia Trachomatis b) Herpes Simplex c) Treponema Palidum V © 4) Neisseria Gonorrhea e) Trichomonas Vaginalis Answer | Explanation Other User's Explanation Report An Error Question Explanation: Bacteria are the most common cause of moncarthits Staphylococcus aureus and gonococei are the most common causes of septic arthritis Neisseria gonorrhoeae occurs in young adults and is often preceded by a migratory tendonitis or arthritis. Gram stain is positive in 25% and culture positive in 50% ‘This patient has been treated previously for a semually acquired infection and this may be why the culture is negative, ‘Mark this question e=> Question 3 of 30 Question Id : 28192 A.74-year-old lady presents with pain and swelling of the let wrist. Three weeks ago she received an intra-articular steroid injection into the wrist as treatment of chronic pain which was felt to be due to osteoarthritis. On examination, the join is erythematous, swollen and tender. Results reveal [White cell count 12.5 x 10% [LDH concentration hoo UL. [Rheumatoid Factor pave ‘Gray of wrist revealed a bony destruction of the joint and wrist aspiration revealed only a dey tap, Whatis the most dagnosis? a) Acute Gout bb) Septic arthiritis c) Acute inflammatory reaction related to Osteoarthritis 4) Acute cheumatoid asthitis ©) Byrophosphate arthropathy Question Explanation: This patients has had an invasive procedure performed relatively recently for suggested OA. Unforiunately the risks associated with intraarticular injection includes joitt infection which appears to be the case here. The positive rheumatoid factor is a red herring, is mildly positive here and is found in 2.5% of the population and may be reised in association with Ca, SLE and infection ‘Mark this question e=> Question 3 of 30 Question Id : 28192 A.74-year-old lady presents with pain and swelling of the let wrist. Three weeks ago she received an intra-articular steroid injection into the wrist as treatment of chronic pain which was felt to be due to osteoarthritis. On examination, the join is erythematous, swollen and tender. Results reveal [White cell count 12.5 x 10% [LDH concentration hoo UL. [Rheumatoid Factor pave ‘Gray of wrist revealed a bony destruction of the joint and wrist aspiration revealed only a dey tap, Whatis the most dagnosis? a) Acute Gout Y © ») Septic arthiritis c) Acute inflammatory reaction related to Osteoarthritis 4) Acute cheumatoid asthitis ©) Byrophosphate arthropathy Question Explanation: This patients has had an invasive procedure performed relatively recently for suggested OA. Unforiunately the risks associated with intraarticular injection includes joitt infection which appears to be the case here. The positive rheumatoid factor is a red herring, is mildly positive here and is found in 2.5% of the population and may be reised in association with Ca, SLE and infection 2/27 2014 2:29:34 PM ‘Mark this question & => Question Ta : 28282 Question 4 of 30 A 25-year-old promising zthlete is diagnosed with chronic fatigue syndrome. Which of the following treatments is indicated? a) Group therapy 'b) Prednisolone ©) Seroxat 4) Graded exercise therapy ¢) Thyroxine Question Explanatio The diagnosis and management of chronic fatigue syndrome. or myalgic encephalitis, has recently been reviewed by NICE (2007) ‘The main features which need to he present to confirm a diagnosis of chronic fatige syndrome are that it 1. Isnew in onset, persistent or recurrent and unexplained by other conditions. 2. Is characterized by post exertional malaise. 3. Results in a substantial reduction in activity level “Associated symptoms include hypersomnia (or insomnia muscle) or joint pain without inflammation and painful lymph nodes without lymphadenopathy, headaches and cognitive dysfunction. Red flag symptoms which suggest another diagnosis include significam weight loss, inammatory arthropathy or connective tissue disease, and localizing or focal neurological signs. 2/27 2014 2:29:34 PM ‘Mark this question & => Question Ta : 28282 Question 4 of 30 A. 25-year-old promising athlete is diagnosed with chronic fatigue syndrome. Which of the following treatments is indicated? a) Group therapy 'b) Prednisolone c) Seroxat Y © 4) Graded exercise therapy ¢) Thyroxine Question Explanatio The diagnosis and management of chronic fatigue syndrome. or myalgic encephalitis, has recently been reviewed by NICE (2007) ‘The main features which need to he present to confirm a diagnosis of chronic fatige syndrome are that it 1. Isnew in onset, persistent or recurrent and unexplained by other conditions. 2. Is characterized by post exertional malaise. 3. Results in a substantial reduction in activity level “Associated symptoms include hypersomnia (or insomnia muscle) or joint pain without inflammation and painful lymph nodes without lymphadenopathy, headaches and cognitive dysfunction. Red flag symptoms which suggest another diagnosis include significam weight loss, inammatory arthropathy or connective tissue disease, and localizing or focal neurological signs. 2:27 2014 2:29:46 PM ‘Mark this question = => (Question Id : 30059 Question 5 of 30 A.47 year old man has a history of pain in his left knee for several years. No joint swelingis present. The pain decreases as he moves about during the day. The disease process involved is probably which one of the folowing? a) Osteothondroma b) Osteomalacia c) Osteoarthritis 4) Osteopetrosis 2) Osteoporosis Answer | Berisnacon) Other User's Explanation Report An Error Question Explanation: (Osteoarthritis usually involves a large joint. The pain usually diminishes with movment, but recurs with reuse or prolonged use of the affecied joint, Osicoporosis woud be uncommon in a 45 years old male. Back pain is a more typical symptom for osteoporosis ‘Osteochondroma could be located ebout the knee, but the pain would probably be exacerbated by movement or local trauma, The findings with osteomalacia osteopetrosis an uncommon inherited metabolic disorder, leads to brittle bones that precispose to fractures, 2:27 2014 2:29:46 PM ‘Mark this question = => (Question Id : 30059 Question 5 of 30 AAT year old man has a history of pain in hs left knee for several years. No joint swellingis present. The pain decreases as he moves about during the day. The disease process involved is probably which one of the following? @) Osteochondroma ) Osteomalacia Y © 5) Osteoarthritis 4) Osteopetrosis 2) Osteoporosis Answer | Berisnacon) Other User's Explanation Report An Error Question Explanation: (Osteoarthritis usually involves a large joint. The pain usually diminishes with movment, but recurs with reuse or prolonged use of the affecied joint, Osicoporosis woud be uncommon in a 45 years old male. Back pain is a more typical symptom for osteoporosis ‘Osteochondroma could be located ebout the knee, but the pain would probably be exacerbated by movement or local trauma, The findings with osteomalacia osteopetrosis an uncommon inherited metabolic disorder, leads to brittle bones that precispose to fractures, ‘Mark this question = => Question Ta : 30428 Question 6 of 30 Alady aged 67 presented with complain of pain at the base of her right tumb, Tendemess and swelling of right first carpometacarpal joint was present on examination, Which one of the following is the most likely diagnosis? a) Avascular necrosis of the scaphoid b) Osteoarthritis c) de Quervain's tenosynovitis 4) Psoriatic arthritis ©) Rheumatoid arthritis Answer | Explanation Other User's Explanation Report An Error Question Explanation: Osteoarthritis of the 1st carpometacaarpal joint is extremely common and in a 67 year old lady is the most likely diegnosis. Swelling is usually bony hard and due to osteophyte formation which can Ikad to the appearance of squaring of the hand. De Quervains tensynovitis is a common overuse condition which present with pain at the base of the thumb butis not associated with joint swelling This joint can be affected in RA and psoriatic arthritis but rarely on its own. ‘Mark this question = => Question Ta : 30428 Question 6 of 30 Alady aged 67 presented with complain of pain at the base of her right tumb, Tendemess and swelling of right first carpometacarpal joint was present on examination, Which one of the following is the most likely diagnosis? a) Avascular necrosis of the scaphoid Y © b) Osteoarthritis c) de Quervain's tenosynovitis 4) Psoriatic arthritis ©) Rheumateid artis Answer | Explanation Other User's Explanation Report An Error Question Explanation: Osteoarthritis of the 1st carpometacaarpal joint is extremely common and in a 67 year old lady is the most likely diegnosis. Swelling is usually bony hard and due to osteophyte formation which can Ikad to the appearance of squaring of the hand. De Quervains tensynovitis is a common overuse condition which present with pain at the base of the thumb butis not associated with joint swelling This joint can be affected in RA and psoriatic arthritis but rarely on its own. 2/27/2014 2:30:10 PM ‘Mark this question e& => Question Id: 30548 Question 7 of 30 Affemale aged 36 years, who was two months postpartum, presented with complains of joint pain, skin rash and fever. On investigation ESR was found to be 40 mm/hour (0-20). What is the most likely diagnosis? a) Reactive arthritis ) Rheumatoid arthritis 1) Sarcoidosis ) Systemic lupus erythematosus 8) Viral arthrtis Question Explanatioy This is a poor question. The symptoms are non-specific and to answer one needs to know the rature and distribution of the rash and. the severity and pattem of the fever. SLE is the most likely to give a combination of joint pains, rash and fever. Documented persistent or recurrent fevers are not generally a feature of the other concitions. ‘The fect that the patient is 2 months postpartum is, atrelevant 2/27/2014 2:30:10 PM ‘Mark this question e& => Question Id: 30548 Question 7 of 30 A female aged 36 years, who was two months postpartum, presented with complains of joint pain, skin rash and fever. On investigation ESR was found to be 40 murvhour (0-20). ‘What is the mostlikely diagnosis? a) Reactive arthntis ) Rheumatoid arthritis 1) Sarcoidosis Y¥ © &) Systemic lapus erythematosus 2) Viral artheitis Question Explanatioy This is a poor question. The symptoms are non-specific and to answer one needs to know the rature and distribution of the rash and. the severity and pattem of the fever. SLE is the most likely to give a combination of joint pains, rash and fever. Documented persistent or recurrent fevers are not generally a feature of the other concitions. ‘The fect that the patient is 2 months postpartum is, atrelevant 2027/2014 PM ‘Mark this question = => Question Td : 30688 Question 8 of 30 Alady of 38 years of age presents to you with a fecling of being tired all the time. Out of the following, which clinical finding is consistent with chronic fetigue syndrome? a) Lymphadenopathy 6) Sleep apnoea ©) Swelling of the metacarpophalangeal joints 4) Weight loss €) Sore throat Question Explanatio ‘The diagnosis and menagement of chronic fatigue syndrome or myalgic encephalitis has recently been reviewed by NICE (2007). The main features which need to be present to corfirm a diagnosis are fatigue that i |. Is new in onset, persistent or recurrent and unexplained by other conditions, 2. Is characterized by post-exertional malaise 3, resultin a substantial reduction in activity level Report An Error “Associated symptoms incinde lypersomnia or insomnia, muscle or joint pain, without Iymphadenopatty, headaches and cognitive dysfunction, Red flag symptoms which suggest another dagnosis include significant weight loss, inflammatory arthropathy or connective tissue disease and Localizing or focal neurological signs. 2027/2014 PM ‘Mark this question = => Question Td : 30688 Question 8 of 30 A lady of 38 years of age presents to you with a feeling of being tied all the time. Out of the Following, which clinical finding is consistent with chronic fatigue syndrome? a) Lymphadenopatty ») Sleep apnoea ©) Swelling of the metacarpophalangeal joints 4) Weightloss Y © €) Sore throat Question Explanatio ‘The diagnosis and menagement of chronic fatigue syndrome or myalgic encephalitis has recently been reviewed by NICE (2007). The main features which need to be present to corfirm a diagnosis are fatigue that i |. Is new in onset, persistent or recurrent and unexplained by other conditions, 2. Is characterized by post-exertional malaise 3, resultin a substantial reduction in activity level Report An Error “Associated symptoms incinde lypersomnia or insomnia, muscle or joint pain, without Iymphadenopatty, headaches and cognitive dysfunction, Red flag symptoms which suggest another dagnosis include significant weight loss, inflammatory arthropathy or connective tissue disease and Localizing or focal neurological signs. 2/27/2014 2:30:37 PM ‘Mark this question & => Question Id : 30708 Question 9 of 30 A 38 year old woman has a history of weakness, diplopia and fatigue. Rheumatoid arthritis kad been recently diagnosed in her. On examination there was bilateral ptosis and wealiness of abduction of both eyes and mild proximal weakness of the arms and legs but sensation and reflexes were nonmal. Which of the following ss the most likely diagnosis? a) Guilain-Barre syndrome 1b) Myasthenia gravis ©) Mononeusitis Multiples. 4) Miutipie sclerosis 2) Gout Answer [ Bxrianaton | Other User's Explanation Report An Error Question Explanation: The most likely diagnosis is myasthenia gravis. There is an association between myasthenia gravis and thyroid disease, pemicious anaemia, systemic lupus erthematosis and rheumatoid arthritis. The condition is more common in women with a pede incidence around the age of 30. Ibis characterized by wealmness and fatignability ofthe proximal lim muscles, ocular and bulbar muscles. 75% of patients intially complain of ocular disturbance, mainly ptosis and ciploplia. Reflexes are initiclly preserved but may be fatiguable In Guillein-Barre syndrome there is a post-infective weakness and numbness in the distal limbs which ascends over days and weeks Multiple sclerosis has a predilection for the optics nerve resulting in optic neuritis or atrophy following an attack, however brain-stem. demyelination may produces diploplia. Chronic progressive external opthalmoplegia (CPEO) is a disorder characterized by slowly progressive paralysis of the extraocular muscles. Patients usually experience bilateral, symmetrical, progressive ptosis followed by opthalmoparesis morths to years later. CPEO is the most frequent manifestation of mitochondrial myopathies. 2/27/2014 2:30:37 PM ‘Mark this question & => Question Id : 30708 Question 9 of 30 A 38 year old woman has a history of weakness, diplopia and fatigue. Rheumatoid arthritis kad been recently diagnosed in her. On examination there was bilateral ptosis and wealiness of abduction of both eyes and mild proximal weakness of the arms and legs but sensation and reflexes were nonmal. Which of the following ss the most likely diagnosis? a) Guilain-Barre syndrome © b) Myasthenia gravis ©) Mononeusitis Multiples. 4) Miutipie sclerosis 2) Gout Answer [ Bxrianaton | Other User's Explanation Report An Error Question Explanation: The most likely diagnosis is myasthenia gravis. There is an association between myasthenia gravis and thyroid disease, pemicious anaemia, systemic lupus erthematosis and rheumatoid arthritis. The condition is more common in women with a pede incidence around the age of 30. Ibis characterized by wealmness and fatignability ofthe proximal lim muscles, ocular and bulbar muscles. 75% of patients intially complain of ocular disturbance, mainly ptosis and ciploplia. Reflexes are initiclly preserved but may be fatiguable In Guillein-Barre syndrome there is a post-infective weakness and numbness in the distal limbs which ascends over days and weeks Multiple sclerosis has a predilection for the optics nerve resulting in optic neuritis or atrophy following an attack, however brain-stem. demyelination may produces diploplia. Chronic progressive external opthalmoplegia (CPEO) is a disorder characterized by slowly progressive paralysis of the extraocular muscles. Patients usually experience bilateral, symmetrical, progressive ptosis followed by opthalmoparesis morths to years later. CPEO is the most frequent manifestation of mitochondrial myopathies. 2027/2014 PM ‘Mark this question = => Question 10 of 30 A40-year old smoker with rheumatoid arthvitis has increasing shortness of breath with a progressive fall inthe FEV1. The residual volume is increased by two liters but diffusion is normal, Diagnose a) Caplan’s syndrome ) Chronic obstructive puimonary disease ©) Organiring pneumonia d) Bronchiolitis obliterans ©) Rheumatoid associated lung fiorosis Question Explanation: ‘All of the possible option can occur in rheumatoid arthritis, but a progressive and relentless fallin the FEV1 indicates bronchiolitis obliterans. Inflammation in the small cstal airways leads to obstructive Spirometry, and this is relentlessly progressive. Air trapping occurs 2s a consequence leading to increased ling volumes 2027/2014 PM ‘Mark this question = => Question 10 of 30 A.40-year old smoker with rheumatoid artivitis has increasing shortness of breath with a progressive fallin the FEV1. The residual volume is increased by two liters but diffision is normal. Diagnose: a) Caplan’s syndrome ) Chronic obstructive puimonary disease c) Organizing pneumonia ¥ © & Bronchiolitis obliterans ©) Rheumatoid associated inng frosis Question Explanation: ‘All of the possible option can occur in rheumatoid arthritis, but a progressive and relentless fallin the FEV1 indicates bronchiolitis obliterans. Inflammation in the small cstal airways leads to obstructive Spirometry, and this is relentlessly progressive. Air trapping occurs 2s a consequence leading to increased ling volumes ‘Maré this question = => Question Td : 54919 Question 11 of 30 A 70-year-old wornan presents with Septic arthritis of right ankle with fever of 33.1°C. The tight ankle was swollen and very tender with a reduced range of movement, Which ofthe folowing wil be most helpful? a) Blood cukures, b) ESR ©) Semm urate level ) Aspirate the right ankle €) Radiography of the right ankle Ancwor UEENARY) othertissr< Explanation Repost An Era Question Explanation: Septic arthritis is a medical emergency and this is the most likely diagnosis in tis case. Ibis essential thet the joint is aspiratediin order to establish a microbiological diagnosis that will guide appropriate treatment. |All of the other investigations listed would be of value in managing this patient, but in this setting joint aspiration is critical ‘Maré this question = => Question Td : 54919 Question 11 of 30 A 70-year-old wornan presents with Septic arthritis of right ankle with fever of 33.1°C. The tight ankle was swollen and very tender with a reduced range of movement, Which ofthe folowing wil be most helpful? a) Blood cutures b) ESR ) Serum urate level Y © od) Aspirate the right ankle €) Radiography of the right ankle Ancwor UEENARY) othertissr< Explanation Repost An Era Question Explanation: Septe arthritis is a medical emergency and this is the most likely diagnosis in this case. It is essential that the joint is aspirated in order to establish a microbiological diagnosis that will guide appropriate treatment. |All of the other investigations listed would be of value in managing this patient, but in this setting joint aspiration is critical 2272014 2:31:14 PM. ‘Mark this question Question Td : 55362 Question 12 of 30 A 26-year-old female has intermittent diplopia. She says that she chokes on her food and regurgtates i, sometimes through her nose. Physical examination reveals drooping eyelids and bilateral facial muscle weakness without atrophy, deep tendon rellexes are nonmadl She is most likely suffering from a) Familial periodic paralysis +b) Muscular dystrophy c) Polymyositis 4) Multiple sclerosis c) Myasthenia gravis (Question Explanation: ‘Myasthenia gravis is an autoimune disorder in which communication between nerves and muscles is impaired, resulting in episodes of muscle weakness. Myasthenia gravis is more common among wemen, It usually develops in women between the ages of 20 and 40. However, the disorder may affect men or women at any age. The most common symptoms are weak, drooping eyelids; weak eve muscles, which cause double vision and excessive fatigue of specific muscles after exercise. In 40% of people with myasthenia gravis, the eve muscles are affected first, but 85% eventually have this problem. In 15% of people, only the eve muscles are, affected, tut, in most people, the whole body is affected. Difficulty speaking and swallowing and weakness of the arms and legs are common. Hand rip may akemate between weak and normal; this Buctuating-srip ic called millemaid'e’ grip. The neck muscles may become weak. Sensation is not affected ‘When a person with myasthenia gravis uses a muscle repetitively, the muscle usually becomes weak. For example, a person who oace coulduse ahammer well becomes weak after hammering for several minutes. However, muscle weakness varies in intensity from hour to hour and ftom day to day, and the course of the disease varies widely. 2272014 2:31:14 PM. ‘Mark this question Question Td : 55362 Question 12 of 30 A 26-year-old female has intermittent diplopia. She says that she chokes on her food and regurgtates i, sometimes through her nose. Physical examination reveals drooping eyelids and bilateral facial muscle weakness without atrophy, deep tendon rellexes are nonmadl She is most likely suffering from a) Familial periodic paralysic b) Muscular dystroply c) Polymyositis ) Multiple sclerosis Y © c) Myasthenia gravis (Question Explanation: ‘Myasthenia gravis is an autoimune disorder in which communication between nerves and muscles is impaired, resulting in episodes of muscle weakness. Myasthenia gravis is more common among wemen, It usually develops in women between the ages of 20 and 40. However, the disorder may affect men or women at any age. The most common symptoms are weak, drooping eyelids; weak eve muscles, which cause double vision and excessive fatigue of specific muscles after exercise. In 40% of people with myasthenia gravis, the eve muscles are affected first, but 85% eventually have this problem. In 15% of people, only the eve muscles are, affected, tut, in most people, the whole body is affected. Difficulty speaking and swallowing and weakness of the arms and legs are common. Hand rip may akemate between weak and normal; this Buctuating-srip ic called millemaid'e’ grip. The neck muscles may become weak. Sensation is not affected ‘When a person with myasthenia gravis uses a muscle repetitively, the muscle usually becomes weak. For example, a person who oace coulduse ahammer well becomes weak after hammering for several minutes. However, muscle weakness varies in intensity from hour to hour and ftom day to day, and the course of the disease varies widely. 2/27/2014 2:31:24 PM ‘Mark this question & => Question Id : 59883 Question 13 of 30 A.22 year old boy has acute fever, malaise and pain in and just below the left knee. There has not been any injury. Which physical finding is most consistent with acute osteomyelitis? a) An effision into the joint ») Masked localized tenderness over the tial metaphysis ) Inability to flex the knee because of pain 4) Swelling of the prepatellar bursa ©) Relief of pain by rest and elevation of the leg, Answer | Bxplanaton Other User's Explanation Report An Error Question Explanation: Osteomyelitis is inflammation and destruction of bone cansed by bacteria, mycobacteria, or fang. Common symptoms are localized bone pain and tendemess with constitutional symptoms (n acute osteomyeliis) or without constitutional symptoms (in chronic osteomyelitis) Patients with acute osteomyelitis of peripheral bones usually experience weight loss, fetigue, fever, and localized warmth, swelling, erythema, and tendemess. Diagnosis is by radiography and cultures. Treatment is with antibiotics and sometimes surgery. 2/27/2014 2:31:24 PM ‘Mark this question & => Question Id : 59883 Question 13 of 30 A.22 year old boy has acute fever, malaise and pain in and just below the left knee. There has not been any injury. Which physical finding is most consistent with acute osteomyelitis? a) An effision into the joint V © ») Marked localized tendemess over the tial metaphysis ) Inability to flex the knee because of pain 4) Swelling of the prepatellar bursa ©) Relief of pain by rest and elevation of the leg, Answer | Bxplanaton Other User's Explanation Report An Error Question Explanation: Osteomyelitis is inflammation and destruction of bone cansed by bacteria, mycobacteria, or fang. Common symptoms are localized bone pain and tendemess with constitutional symptoms (n acute osteomyeliis) or without constitutional symptoms (in chronic osteomyelitis) Patients with acute osteomyelitis of peripheral bones usually experience weight loss, fetigue, fever, and localized warmth, swelling, erythema, and tendemess. Diagnosis is by radiography and cultures. Treatment is with antibiotics and sometimes surgery. ‘Mark this question => Question Id: 61728 Question 14 of 30 ‘The radiographic feanare thet is most consistent with osteoarthritis of the knee is a) Marginal erosions +) Justa-articular osteopenia (demineralization) ©) Loss of articular cartilage with narrowing of the radiologic joint space @) Ostoonectosis (avascular necrosis) of the medical femoral condyle 2) Syndesmophyte formation Question Explanation: Osteoarthritis is a chronic arthropathy of an entire joint characterized by disruption and potential loss of joint cartilage along with other joint changes, including bone hypertrophy (osteophyte formation’). Symptoms include gradually developing pain aggravated or triggered by activity, stiftiess relieved < 30 min after activity, and occasional joint swelling. Diagnosis is confirmed by X-rays. X-rays generally reveal margnal osteophytes, netrowing of the joint space, increased density of the subchondral bone, subchondral cyst formation, bony remodeling, and joint effusions ‘Mark this question => Question Id: 61728 Question 14 of 30 ‘The radiographic feature that is most consistent with osteoarthritis of the knee is a) Marginal erosions ») Justa-erticular osteopenia (demineralization) Y © 0) Loss of articular cartilage with narrowing of the radiologic joint space d) Ostconecrosis (avascular necrosis) of the medical femoral condyle 2) Syndesmophyte formation Question Explanation: Osteoarthritis is a chronic arthropathy of an entire joint characterized by disruption and potential loss of joint cartilage along with other joint changes, including bone hypertrophy (osteophyte formation’). Symptoms include gradually developing pain aggravated or triggered by activity, stiftiess relieved < 30 min after activity, and occasional joint swelling. Diagnosis is confirmed by X-rays. X-rays generally reveal margnal osteophytes, netrowing of the joint space, increased density of the subchondral bone, subchondral cyst formation, bony remodeling, and joint effusions 2/27/2014 2:31:49 PM ‘Mark this question & => Question Td : 76085 Question 15 of 30 A.37-year old woman with glucose-6-phosphate dehycrogenase deficiency has rheumatoid arthritis. Which one of the following medications should NOT be prescribed for her? a) Hydroxychloroguine ») Azathioprine ¢) Ibuprofen 4) Prednisone 2) Penicillamine Question Explanation: Ingestion of the antimalarial hydrcxychloroquine can cause hemolysis in patients with G6PD deficiency, None of the other agents cen induce hemolysis in this condition. Evror 2/27/2014 2:31:49 PM ‘Mark this question & => Question Td : 76085 Question 15 of 30 A.37-year old woman with glucose-6-phosphate dehycrogenase deficiency has rheumatoid arthritis. Which one of the following medications should NOT be prescribed for her? Y © a) Hydrozychloroquine ») Azathioprine ¢) Ibuprofen 4) Prednisone 2) Penicillamine Question Explanation: Ingestion of the antimalarial hydrcxychloroquine can cause hemolysis in patients with G6PD deficiency, None of the other agents cen induce hemolysis in this condition. Evror 2/27/2014 2:32:03 PM ‘Mark this question & => Question Td: 76175 Question 16 of 30 The test most appropriate for the diagnosis of dermatomyositis is 8) Grara stain and cultare of eynovial uid ) Examination of synovial uid for presence of urate crystals c) Radiographic evaluation with clinical examination 4) Serum muscle enzyme measurement. ¢) Punch biopsy. Aver ENGI) ote: Users Explanation Report An Enos Question Explanation: Dermatomyositis is an autoimmune disorder of T-cels, T-cells atack skin and muscle tissue, Consequently, high levels of muscle- specific enzymes such at creatine kinase or lactate dehydrogenase are released. Clinical evaluation with serum muscle enzyme measurement is sufficient to malce the diagnosis of dermatomyositis 2/27/2014 2:32:03 PM ‘Mark this question & => Question Td: 76175 Question 16 of 30 ‘The test most appropriate for the diagnosis of dermatomyositis is a) Gram stain and cultare of synovial fluid. ) Examination of synovial id for presence of urate crystals, c) Radiographic evaluation with clinical examination Y © 4) Serum muscle enzyme measurement. 2) Punch biopsy. Aver ENGI) ote: Users Explanation Report An Enos Question Explanation: Dermatomyositis is an autoimmune disorder of T-cels, T-cells atack skin and muscle tissue, Consequently, high levels of muscle- specific enzymes such at creatine kinase or lactate dehydrogenase are released. Clinical evaluation with serum muscle enzyme measurement is sufficient to malce the diagnosis of dermatomyositis ‘Mark this question & => Question Td : 81482 Question 17 of 30 A.38 year old female presents with complaint of bone pain, X-rays are taken anda cagnosis of osteosarcoma is made. Tae most likely appearance of the aflecied bone on X- ray is a) 'Punched-out" regions ofthe skull ) "Onion peel! bones ©) 'Stnburst" bones 4) "Soap bubble" bones €) "Target" appearance of bones Anewor (UEIANIER) ote: sors Explanation Ropar An Error Question Explanatio Osteosarcoma presents as a ‘sunburst’ lesion on X-ray. "Punched-ott" regions of the skull (A) indicate a muhiple myelotra, ‘Onion peel" bones (B) are seen in Ewing's sarcoma, while "soap bubble" bones (D) present in giant cell tumors of the bones. Finally, the “target” appearance of bones (E) is found in osteoid osteoma ‘Mark this question ‘Question Id: 83522 Question 18 of 30 A. 39 year old cab driver presents with excruciating pain n his hands, He has a 30-pack-year history of cigarette smoking, has hyperchelesterclemia, has not been compliant with medicaticns. He admits to drinking in excess "Barly often." Physical examination is remarkable for severe ulcerations on the fingertips and atrophic nail changes. The 4th dist on the left hand is gangrenous at the tp The most ikely dieanosis is a) Bechet syndrome »b) Buerger disease ) Kawasaki disease 4) Polyarteritis nodosa €) Venous stasis newer [ERISRENY other usore Expan Question Explanation: This is a typical presentation of Buerger disease, which is a recurrent acute and chronic inflammatory disorder that can cause segmental thrombosis in arteries and veins of the extremities. It ypically occurs in heavy smokers who started smoking at an easly age. Major features may include severe pain (claudication) in the affected extremities, peripheral ischemia (especially of the upper extremities) faint/absent pulses in small an medium sized arteries gengrene/ulceration of the fingers and toes. and Reynaud phenomenon, Behget syndrome (Choice A) is 2 multi-system disorder presentira with uveitis as well as recumrent aphthous ulcers on the genitalia or in the mouth, Kawasaki disease (choice C) caucee a vasculitis affecting the coronary arteries in children. Polyarteritis nodosa (choice D) is a recurrent necrotizing rascultis of meckum and small muscular arteries. It is distinguished by its nultiple-organ involvement and constitutional symptoms inclding fever, weight loss, myalgias, and arthralgias. Venous stasis (choice E) results in venous hypertension and predisposes individuals to ulcerative lesions, Venous ulceration accounts for 70 to 80% of lower-imb ulceration. Diseases of the upper extremity is unusual n Report An Error “Marke this question = => ‘Question Id: 83522 Question 18 of 30 A. 39 year old cab driver presents with excruciating pain n his hands, He has a 30-pack-year history of cigarette smoking, has hyperchelesterclemia, has not been compliant with medicaticns. He admits to drinking in excess "Barly often." Physical examination is remarkable for severe ulcerations on the fingertips and atrophic nail changes. The 4th dist on the left hand is gangrenous at the tp The most ikely dieanosis is a) Bechet syndrome Y © b) Buerger disease ) Kawasaki disease 4) Polyarteritis nodosa e) Venous stasis newer [ERISRENY other usore Expan Question Explanation: This is a typical presentation of Buerger disease, which is a recurrent acute and chronic inflammatory disorder that can cause segmental thrombosis in arteries and veins of the extremities. It ypically occurs in heavy smokers who started smoking at an easly age. Major features may include severe pain (claudication) in the affected extremities, peripheral ischemia (especially of the upper extremities) faint/absent pulses in small an medium sized arteries gengrene/ulceration of the fingers and toes. and Reynaud phenomenon, Behget syndrome (Choice A) is 2 multi-system disorder presentira with uveitis as well as recumrent aphthous ulcers on the genitalia or in the mouth, Kawasaki disease (choice C) caucee a vasculitis affecting the coronary arteries in children. Polyarteritis nodosa (choice D) is a recurrent necrotizing rascultis of meckum and small muscular arteries. It is distinguished by its nultiple-organ involvement and constitutional symptoms inclding fever, weight loss, myalgias, and arthralgias. Venous stasis (choice E) results in venous hypertension and predisposes individuals to ulcerative lesions, Venous ulceration accounts for 70 to 80% of lower-imb ulceration. Diseases of the upper extremity is unusual n Report An Error ‘Mark this question e => (Question Td : 88592 Question 19 of 30 A 39 year old man is brought to the emergency room with multple arm and leg fractures following a minor fall. Examination reveals a shght weakness of facial muscles on the left. Blood investigations show a mild anemia. X-ray studies demonstrate a generalized bony ‘widening with partial obiteration of marrow spaces. The patients condition is most likely due to abnormal fimctioning of which of the following type of cell? a) Granulocytic stem cells b) Megakaryocytes c) Plasma cells 4) Osteoblasts ©) Osteoclasts Answer (Explanation | Other User's Explanation Report An Error Question Explanation: The disease is osteopetrosis (Albers-Schorberg disease), which ic a group of hereditary diseases in which impaired osteoclast finction leads to reduced bone resorption. The abnormal osteoclasts frequently are enlarged, with bizarre shapes. The bones become thick and brittle; other features include anemia secondary to marrow loss and cranial nerve deficits secondlary to narrowing of bony ostea, An autosomal-recessive, severe form of the disease produces death in childhood, A relaiively benign autosomal dominant form, presents in adulthood, Abnormal proliferation of granulocytic stem cells can produce myelocytic leukemias. Megakaryocyte abnormalities can produce platelet disorders but not deficient bone resorption, In muliple myeloma, neoplastic plasma cells can cause lytic bone lesions characterized by excessive resorption of tone. Abnormally low osteoblast, rather than osteoblast, function is apparently the problem in osteopetrosis. ‘Mark this question e => (Question Td : 88592 Question 19 of 30 A 39 year old man is brought to the emergency room with multiple arm and leg fractures following a minor fall, Examination reveals a shght weakness of facial muscles on the left. Blood investigations show a mild anemia. X-ray studies demonstrate a generalized bony ‘widening with partial obiteration of marrow spaces. The patients condition is most likely due to abnormal fimctioning of which of the following type of cell? a) Gramulocytic stem cells b) Megakaryocytes ) Plasma cells 4) Osteoblasts VY © ©) Dsteorlasts Answer (Explanation | Other User's Explanation Report An Error Question Explanation: The disease is osteopetrosis (Albers-Schorberg disease), which ic a group of hereditary diseases in which impaired osteoclast finction leads to reduced bone resorption. The abnormal osteoclasts frequently are enlarged, with bizarre shapes. The bones become thick and brittle; other features include anemia secondary to marrow loss and cranial nerve deficits secondlary to narrowing of bony ostea, An autosomal-recessive, severe form of the disease produces death in childhood, A relaiively benign autosomal dominant form, presents in adulthood, Abnormal proliferation of granulocytic stem cells can produce myelocytic leukemias. Megakaryocyte abnormalities can produce platelet disorders but not deficient bone resorption, In muliple myeloma, neoplastic plasma cells can cause lytic bone lesions characterized by excessive resorption of tone. Abnormally low osteoblast, rather than osteoblast, function is apparently the problem in osteopetrosis. ‘Mark this question eo Question 20 of 30 Carpal tunnel may result from all of the following EXCEPT a) Acromegaly. b) Repetitive motion. c) Arnyloid. od) Ulnar artery aneurysm. ©) Hypothyroidsm, Question Explanation: Carpal tunnel syndrome is produced by entrapment of the median nerve at the wrist. Itmay result ffom conditions. such as acromegaly, which produce thickening of connective tissue at the wrist Deposition of amyloid, either as a primary or secondary disease, also produces the syndrome. Any other condition which produces pressure on the median nerve as it passes through the bony flexor compartment also produces the syndrome. Ancurysms of the median artery and not the ulnar artery cause the syndrome. The median artery is occasionally found in bumaans in 8% of individuals. Tt runs with the median nerve and supplies the same structures as that nerve. It may be undateral or bilateral, ‘Mark this question eo Question 20 of 30 Carpal tunnel may result from all of the following EXCEPT a) Acromegaly. b) Repetitive motion. c) Arnyloid. Y © A) Ular artery aneurysm. ©) Hypothyroidsm, Question Explanation: Carpal tunnel syndrome is produced by entrapment of the median nerve at the wrist. Itmay result ffom conditions. such as acromegaly, which produce thickening of connective tissue at the wrist Deposition of amyloid, either as a primary or secondary disease, also produces the syndrome. Any other condition which produces pressure on the median nerve as it passes through the bony flexor compartment also produces the syndrome. Ancurysms of the median artery and not the ulnar artery cause the syndrome. The median artery is occasionally found in bumaans in 8% of individuals. Tt runs with the median nerve and supplies the same structures as that nerve. It may be undateral or bilateral, 227 2014 2:33:13 PM ‘Mare this question = => Question 21 of 30 The disease that most predisposes an individual to the development of secondary amyloidosis is a) Inflammatory bowel disease b) Rheumatoid arthritis ©) Sclerodenma 4) Tuberculosis ©) Bronchiectasis Answer ( Exoianation) Other User's Explanation Report An Error Question Explanation: Question Id : 95659 ‘While all of the choices are associated with increased risk of development of secondary amyloidosis, rheumatoid arthritis is associated with the greatest nsk. 227 2014 2:33:13 PM ‘Mare this question = => Question 21 of 30 The disease that most predisposes an individual to the development of secondary amyloidosis is a) Inflammatory bowel disease Y © b) Rheumatoid arthritis ©) Sclerodenma d) Tuberculosis, ©) Bronchiestasis Answer ( Exoianation) Other User's Explanation Report An Error Question Explanation: Question Id : 95659 ‘While all of the choices are associated with increased risk of development of secondary amyloidosis, rheumatoid arthritis is associated with the greatest nsk. 2/27 2014 2:33:33 PM. ‘Matte this question e => Question Td : 98210 Question 22 of 30 A.42 year old woman compleins of aumbress and tngling in her thumb and index fingers which gets worse just prior to menstruation Physical finding liely to be positive is her is which of the following? a) Shober's test b) Schirmer’s test ©) Cervical compression test 4) Phalen‘s test ©) Firkelstein’s tect Question Explanation: This patient's symptoms are most consistent with carpal tunnel syndrome, which can be elicited by the Phalen’s test, forced hyperflexion of the wrists. Shober’s test is associated with sacroilitis. Schirmer’s testis positive in keratoconjunctivtis sicca. The cervical compressioa test is associated with psychosomatic complaints. Finkelstein’s test is positive in De Quervain's tenosynovitis, 2/27 2014 2:33:33 PM. ‘Matte this question e => Question Td : 98210 Question 22 of 30 A.42 year old woman compleins of aumbress and tngling in her thumb and index fingers which gets worse just prior to menstruation Physical finding liely to be positive is her is which of the following? a) Shober's test b) Schirmer’s test ©) Cervical compression test Y © & Phalen’s test ©) Firkelstenn's tect Question Explanation: This patient's symptoms are most consistent with carpal tunnel syndrome, which can be elicited by the Phalen’s test, forced hyperflexion of the wrists. Shober’s test is associated with sacroilitis. Schirmer’s testis positive in keratoconjunctivtis sicca. The cervical compressioa test is associated with psychosomatic complaints. Finkelstein’s test is positive in De Quervain's tenosynovitis, 2/27/2014 2:33:45 PM ‘Mark this question ==> Question Id 95252 Question 23 of 30 “What is the mainstay in the treatment of sheumetoid antaritis in adults? a) Systemic bed rest 6) Diet ) Hydration 4) Physical therapy. ¢) High impact aerobics Question Explanation: Physical and occupatonal therapy are very anportant aspecis of treatment in rheumatoid arthritis to maintain muscle mass and tone and prevent contrachires and loss of function. Sysiemic bed res is no longer thought to be beneficial and can lead to cisuse atrophy ofmscles surrounding affected joints. There is no evidence that aay alterations in the diet are beneficial in the management of rheumatoid arthritis. Hydration is not a specific patt of therapy. High impact aerobic exercises should be avoided because of their potential for traumatizing inflamed joints 2027/2014 PM Mark ths question qc Question 23 of 30 “What is the mainstay in the treatment of sheumetoid antaritis in adults? a) Systemic bed rest 6) Diet ) Hydration Y © A) Physical therapy. ¢) High impact aerobics Question Explanation: Physical and occupatonal therapy are very anportant aspecis of treatment in rheumatoid arthritis to maintain muscle mass and tone and prevent contrachires and loss of function. Sysiemic bed res is no longer thought to be beneficial and can lead to cisuse atrophy ofmscles surrounding affected joints. There is no evidence that aay alterations in the diet are beneficial in the management of rheumatoid arthritis. Hydration is not a specific patt of therapy. High impact aerobic exercises should be avoided because of their potential for traumatizing inflamed joints 2/27/2014 2:33:59 PM ‘Maric this question & => Question Id : 104924 Question 24 of 30 A.25 year old woman presents with fever, weightloss, and malaise, Examination reveals a "butterfly" rash and rashes over sunlight exposed. The autoantibodies that would strengthen the diagnosis of systemic lupus erythematosus are a) Anti-neutrophil cytoplasmic antibodies b) Ani-RNA antibodies c) Anti-dsDNA antibodies 4) Anii-Ro antibodies €) Anii-To-1 antibodies Question Explanation: “Although anti-double-stranded DMA antibodies are found in a very small number of rheumatoid arthritis cases, they are seen primarily in the context of systemic lupus erythematosus. 2/27/2014 2:33:59 PM ‘Maric this question & => Question Id : 104924 Question 24 of 30 A 25 year old women presents with fever. weight loss. and malaise, Examination reveals a "butterfly" rash and rashes over sunlight exposed, The autoantibodies that would strenathen the diagnosis of systemic lupus erythematosus are 2) Anti-neutrophil cytoplasmic antibodies b) Ani-RNA antibodies v © c) Anti-dsDNA antibodies 4) Anti-Ro antibodies ¢) Anti-Jo-1 antibodies Question Explanation: “Although anti-double-stranded DMA antibodies are found in a very small number of rheumatoid arthritis cases, they are seen primarily in the context of systemic lupus erythematosus. ‘Mark this question Question 25 of 30 Question Id : 107579 “Which of the following has NOT been shown to be useful in decreasing the severity of osteoporosis in women? a) Calcium. ) Estrogen ) Activity 4) Prednisore. ©) Fluoride Ancwor [NEBR oter Users explanation Report An Evo Question Explanation: Prednisone is associated with the incuction of osteoporosis when given for more then a short course. ‘There is evidence that the other modalities can improve bone density, although none are curative. ‘Mark this question Question 25 of 30 Question Id : 107579 “Which of the following has NOT been shown to be useful in decreasing the severity of osteoporosis in women? a) Calcium. ) Estrogen ) Activity Y © d)Predeisone. ©) Fluoride Ancwor [NEBR oter Users explanation Report An Evo Question Explanation: Prednisone is associated with the incuction of osteoporosis when given for more then a short course. ‘There is evidence that the other modalities can improve bone density, although none are curative. 2/27/2014 2:34:25 PM ‘Mark this question = => Question Id : 107852 Question 26 of 30 ‘What is the most effective treatment for gout ina patient allergic to cochicine? a) Sodium urate ) Indomethacin. c) Aspirin 4) Methowrexate ¢) Prednisone Question Explanation: Colchicine and indomethacin are the mainstays of the medical management of gout. Aspitin is not effective, Methotrexate and predaisione are indicated in rheumatcid arthritis, not gouty arthuitis. Sodium urate is the crystal thats found in the joint fuid of joints affected by gout 2/27/2014 2:34:25 PM ‘Mark this question = => Question Id : 107852 Question 26 of 30 ‘What is the most effective treatment for gout in a patient allergic to cochivine? a) Sodium wrate Y © b) Indomethacin. ) Aspirin 4) Methotrexate ¢) Prednisone Question Explanation: Colchicine and indomethacin are the mainstays of the medical management of gout. Aspitin is not effective, Methotrexate and predaisione are indicated in rheumatcid arthritis, not gouty arthuitis. Sodium urate is the crystal thats found in the joint fuid of joints affected by gout 2/27/2014 PM ‘Mark this question = => Question Id: 112456 Question 27 of 30 A 36 year old female presents with a gritty sensation in her eyes, dry mouth, and early morning stiffness in her hands, mists, knees, and feet Tnvestigetions would probably show which one of the following? a) A posttive rheumatoid factor. b) Low C3 levels c) An elevated uric acid. 4) Anti-Jo-1 antibodies ¢) Eosinophila Answer Other User's Explanation Report An Error Question Explanation: This patient probably has seropostive rheumatoid artis with Sjégren’s syndrome. This syndrome is not associated with low C3 levels, elevated serum uric acid, anti-Jo-1 antibodies, which can be associated with polymyositis, or eosinophilia, 2/27/2014 PM ‘Mark this question = => Question Id: 112456 Question 27 of 30 A 36 year old female presents with a gritty sensation in her eyes, dry mouth, and early morning stiffness in her hands, mists, knees, and feet Tnvestigetions would probably show which one of the following? Y © a) A positive rheumatoid factor. b) Low C3 levels c) An elevated uric acid. 4) Anti-Jo-1 antibodies ¢) Eosinophila Answer Other User's Explanation Report An Error Question Explanation: This patient probably has seropostive rheumatoid artis with Sjégren’s syndrome. This syndrome is not associated with low C3 levels, elevated serum uric acid, anti-Jo-1 antibodies, which can be associated with polymyositis, or eosinophilia, ‘Mark this question e => Question Td: 118636 Question 28 of 30 ‘A. 32 year old female presents to her physician because of weakness, discomfort, and coldness in both arms that are exacerbated vwihen she does overhead work or tries to carry anything, She has hed generalzed fatigue for several months and has lost about 15 pounds. Which of the fellowing finding would be present on the physical examination? a) Orthostatic hypotension, b) Vi ) Telangiectases. 4) Diminished radial pulses «) Stocking glove paresthesias. Answer (Biotanation) Other User's Explanation Report An Error Question Explanation: This patient's syndrome is most consistent with Takayasu's arteritis, or pulseless disease. The arteries of the aortic arch are most commonly involved and distal pulses of the upper extremities are often impossible to detect. Orthostatic hypotension, vitiligo, telangiectases, and stocking glove paresthesias are not associated with this condition, ‘Mark this question e => Question Td: 118636 Question 28 of 30 ‘A. 32 year old female presents to her physician because of weakness, discomfort, and coldness in both arms that are exacerbated vwihen she does overhead work or tries to carry anything, She has hed generalzed fatigue for several months and has lost about 15 pounds. Which of the fellowing finding would be present on the physical examination? a) Orthostatic kypotension. b) Vit ) Telangiectases. Y © @ Diminished radial pulses. 2) Stocking glove paresthesias. Answer (Biotanation) Other User's Explanation Report An Error Question Explanation: This patient's syndrome is most consistent with Takayasu's arteritis, or pulseless disease. The arteries of the aortic arch are most commonly involved and distal pulses of the upper extremities are often impossible to detect. Orthostatic hypotension, vitiligo, telangiectases, and stocking glove paresthesias are not associated with this condition, 2:27 2014 2:35:03 PM ‘Mare this question = => Question Td : 126073 Question 29 of 30 Pseudogoutis associated with which one of the following? a) Bamboo spine +b) Chondrosaleinosie c) DIP sclerosis 4) Justa-articular demineralization ©) Pencil-in-cup erosions £) Overhanging erosions (Question Explanation: Pseudogout-chondrocalcinosis-this is often seen in the fibrocartilaginous tissue of the pubic symphysis, the triangular ligament of the ‘wrist, and the menisci 2:27 2014 2:35:03 PM ‘Mare this question = => Question Td : 126073 Question 29 of 30 Pseudogoutis associated with which one of the following? a) Bamboo spine WY © b) Chondrocaleinesis c) DIP sclerosis 4) Justa-articular demineralization @) Pencil-in-cup erosions £) Overhanging erosions (Question Explanation: Pseudogout-chondrocalcinosis-this is often seen in the fibrocartilaginous tissue of the pubic symphysis, the triangular ligament of the ‘wrist, and the menisci 2/27/2014 2:35:20 PM / a C1 unread) - emacomr = ) 9% www.nterface.ecupkin x \__ —— % € 2 @ DB www interfare.edupk /medical-exams/test-analysis phpauti & apps FJ coogle A settings [) signin Cl imported Fromie 2 ge dle © oe yo Gaull Saye » © other bookmar ‘Mack this question <= Question Id: 216856] Total Questions Question 30 of 30 A 66-year-old man with chronic thigh pain slips and hits his le thigh to the ground, followed by a "snap." He also has decreased hearing in both ears and a 2/6 mideystolic crescendo-decrescendo murmur. X-ray of the left femur shows a midshaft fracture as well a9 an expanded, deformed cortical contour. X-ray of the sight shows an expanded, bowed femur. A bone scan shows focal areas of intense uptake in both thighs. Laboratory data are: [Sodium 149 mEq/L Potassium 4.8 mEgiL [Chioride 105 mEq/L [Bicarbonate 24 mEq [Grea nitrogen 13 mefal. [Creatinine 1.0 mg/dL 12 [Calcium [92mg fa Li Phosphate 29 mafal. 4 [asr. 2r0L [axT. 2570L ea [Alkaline phosphatase: [400 TUL ‘ [sor 1S 1UL (normal: 9-50 IU/L} ‘i [ESR 18 mavfhour Other than referral to orthopedic surgery, what therapy should be offered to this patient? a) Calcium supplementaion ) Bisphosphonates c) Doxombicin 4d) High-dose corticosteroids €) Hydrochlorothiazide Answer | Baoianation | Other User's Explanation Report An Error Question Explanatio: ‘This patient has Paget disease (osteitis deformars). ‘The disease is characterized by bone pain, skeletal deformity, pathologic fractures, cranial nerve deficits and diminished hearing secondary to expansion of the calvarium, hish-output cardiac Faure, and increased incidence of valve calcification. The underlying pathophysiology is increased osteoclastic activity accompanied by a compensator increase in osteoblastic activity, resuling in chaotic bone formation. Tae bisphosphonates act by irhibting osteoclastic activity, They also inhibit bone formation at a slower rate and are the main treatmert modality for Paget dsease Calcium supplementation is incorrect because this man's serum calcium is normal, and calcium supplemertation does not after the course of Paget disease Doxonbicin is incorrect because the bone scan is typical for osteitis defoumans and no abno:mal focal site concerning for osteosarcoma is visualized. Itis important to note that Paget disease is associated with an increased rate of osteosarcoma. High-dose corticostercid is an incorrect choive because the proximal thigh pain is secondary to Paget, no: to pelymyaigia rheumatica “Additionally, the ESR is not elevated. [Biodvschiecs teianivie ae Si GAS woe besauce the Gahenk ic uonnetenne and hac ue eesdence obuechrolthiace AXA AAR AA AA AA AA a a or ECE CE ERE REE Ee eek EERE tee 2/27/2014 2:35:20 PM / a C1 unread) - emacomr = ) 9% www.nterface.ecupkin x \__ —— % € 2 @ DB www interfare.edupk /medical-exams/test-analysis phpauti & apps FJ coogle A settings [) signin Cl imported Fromie 2 ge dle © oe yo Gaull Saye » © other bookmar ‘Mack this question <= Question Id: 216856] Total Questions Question 30 of 30 A 66-year-old man with chronic thigh pain slips and hits his le thigh to the ground, followed by a "snap." He also has decreased hearing in both ears and a 2/6 mideystolic crescendo-decrescendo murmur. X-ray of the left femur shows a midshaft fracture as well a9 an expanded, deformed cortical contour. X-ray of the sight shows an expanded, bowed femur. A bone scan shows focal areas of intense uptake in both thighs. Laboratory data are: [Sodium 149 mEq/L Potassium 4.8 mEgiL [Chioride 105 mEq/L [Bicarbonate 24 mEq [Grea nitrogen 13 mefal. [Creatinine 1.0 mg/dL 12 [Calcium [92mg fa Li Phosphate 29 mafal. 4 [asr. 2r0L [axT. 2570L ea [Alkaline phosphatase: [400 TUL ‘ [sor 1S 1UL (normal: 9-50 IU/L} ‘i [ESR 18 mavfhour Other than referral to orthopedic surgery, what therapy should be offered to this patient? a) Calcium supplementaion Y © b) Bisphosphonates c) Doxombicin 4d) High-dose corticosteroids €) Hydrochlorothiazide Answer | Baoianation | Other User's Explanation Report An Error Question Explanatio: ‘This patient has Paget disease (osteitis deformars). ‘The disease is characterized by bone pain, skeletal deformity, pathologic fractures, cranial nerve deficits and diminished hearing secondary to expansion of the calvarium, hish-output cardiac Faure, and increased incidence of valve calcification. The underlying pathophysiology is increased osteoclastic activity accompanied by a compensator increase in osteoblastic activity, resuling in chaotic bone formation. Tae bisphosphonates act by irhibting osteoclastic activity, They also inhibit bone formation at a slower rate and are the main treatmert modality for Paget dsease Calcium supplementation is incorrect because this man's serum calcium is normal, and calcium supplemertation does not after the course of Paget disease Doxonbicin is incorrect because the bone scan is typical for osteitis defoumans and no abno:mal focal site concerning for osteosarcoma is visualized. Itis important to note that Paget disease is associated with an increased rate of osteosarcoma. High-dose corticostercid is an incorrect choive because the proximal thigh pain is secondary to Paget, no: to pelymyaigia rheumatica “Additionally, the ESR is not elevated. [Biodvschiecs teianivie ae Si GAS woe besauce the Gahenk ic uonnetenne and hac ue eesdence obuechrolthiace AXA AAR AA AA AA AA a a or ECE CE ERE REE Ee eek EERE tee 2/27/2014 4:47:07 PM ‘Mark this question => Question Id : 28182 Question 1 of 4 ‘A 28-year-old man presents with fever, urethritis and arthralgia. Hie is found to have a swollen ankle with a pustular rash on the dorsal aspect of his foot ‘What is the most likely diaanosis? a) Lyme disease tb) Reiter's syndrome ©) Staphylococcal arthritis d) Gonococeal sepsis €) Arhrtis due to Pasteurella Muocida Answer | Branatin | Other User's Explanation Report An Error Question Explanation: ‘The most likely cause for this acute presentation is gonococcal septicemia-with a pustular rash on the dorsum ofhis foot, fever, wethritis and oligoarthnts. Reiter’s is associated with an acute infection- urcthitis/diarrhea and later the development of arthritis. Arthritis due to Pasteurella ‘Mubocida will follow an animal bite along inflammation at bite sits. 2/27/2014 4:47:07 PM ‘Mark this question => Question Id : 28182 Question 1 of 4 A 28-year-old man presents with fever, urethritis and arthralgia. He is found to have a swollen ankle with a pustular rash on the dorsal aspect of his foot ‘What is the most likely diagnosis? a) Lyme disease b) Reiter's syndrome c) Staphylococcal arthritis Y © d)Gonococcal sepsis ©) Arthtiis due to Pasteurella Mulocida Answer | Branatin | Other User's Explanation Report An Error Question Explanation: ‘The most likely cause for this acute presentation is gonococcal septicemia-with a pustular rash on the dorsum ofhis foot, fever, wethritis and oligoarthnts. Reiter’s is associated with an acute infection- urcthitis/diarrhea and later the development of arthritis. Arthritis due to Pasteurella ‘Mubocida will follow an animal bite along inflammation at bite sits. 22772018 4:47:22 PM ‘Mark this question = => (Question Td : 55053 Question 2 of 4 “Which of the following statements is tue regarding neck of fernur fracture? a) Extracapsular fractures have a higher incidence of avascular necrosis compared to intracapsular fractares, +) Internal rotation of the affected limb is a common clinical finding ©) In the young displaced fractures should be reduced and fixed urgently ) Mortality for elderly peticnts is >80% et one year €) Non-union is not a complication (Question Explanation: Tntracapsular femoral fractures are at increased risk of avascular necrosis due to disruption of the capsular blood supply at the time of injury. Patients more commonly present with a short, externally rotated limb, In young patients the fracture should be reduced and fixed early to decrease the risks of avascular nectosis and non-union Mortality at one year is approximately 25%. 22772018 4:47:22 PM ‘Mark this question = => (Question Td : 55053 Question 2 of 4 “Which of the following statements is tue regarding neck of fernur fracture? a) Extracapsular fractures have a higher incidence of avascular necrosis compared to intracapsular fractares, +) Internal rotation of the affected limb is a common clinical finding Y © c) In the young displaced fractures should be reduced and fixed urgently ) Mortality for elderly peticnts is >80% et one year €) Non-union is not a complication (Question Explanation: Tntracapsular femoral fractures are at increased risk of avascular necrosis due to disruption of the capsular blood supply at the time of injury. Patients more commonly present with a short, externally rotated limb, In young patients the fracture should be reduced and fixed early to decrease the risks of avascular nectosis and non-union Mortality at one year is approximately 25%. 3530 ‘Mark this question ez Question Id : Question3 of 4 ‘A 55-year-old alcoholic male presents to the emergency room complaining of severe pain and swelling of his right toe. He denies aay recent trauma, urogetital infections, diarrhea, rashes, or sexmal activity. There is no family history of arthritis, This pain usvally occurs after binge dritking and subsides with Nonsteroidal anti-nflammatory drugs. Whats the most licely diagnosis in this patient? a) Reiter's syndrome b) Lyme disease monoarthritis ©) Septic arthritis 4) Acute gout €) Calcium pyrophosphate dinydrate deposition Answer [UEQIBBIRN) other User's Explanation Report An Evan Question Explanation: Gout usually is associated with alcohol intake and presents with podagra, acute swelling and pain in the first metatarsal phalangeal joint. Gout is more common in diabetics and in men. Aspiration of uid ftom the involved joint would reveal urate crystals under the ppolatizing microscope. The pain is usually relieved with non-steroidals or prednisone. Reiter's syncrome is associated with HLAB2? and is more commonin young to middle-aged men. Itis associated with a sexually transmitted disease, urethritis, and arthritis. Lyme disease is caused by a deer tick bite and presents with arash and! or myalgias and fever. Arthritis is common in the knee joints. This man did not have a rash. Septic arthritis could be life threatening and is associated with a fever and elevated white count. Aspiration of the joint Quid would reveal an extremely elevated white count greater than 50,000 cells/ml, This pain would not be relieved with nonstcroidals alone antibiotics would be needed. CPDD is more commoa in the elderly, and aspiration of the joint. Flud would reveal rhomboid-shaped crystals which are positively bireffingent under the polarizing microscope. The knee joints are more commonly affected, 3530 ‘Mark this question ez Question Id : Question3 of 4 ‘A 55-year-old alcoholic male presents to the emergency room complaining of severe pain and swelling of his right toe. He denies aay recent trauma, urogetital infections, diarrhea, rashes, or sexmal activity. There is no family history of arthritis, This pain usvally occurs after binge dritking and subsides with Nonsteroidal anti-nflammatory drugs. Whats the most licely diagnosis in this patient? a) Reiter's syndrome b) Lyme disease monoarthritis ©) Septic arthritis Y © 4) Aeute gout €) Calcium pyrophosphate dinydrate deposition Answer [UEQIBBIRN) other User's Explanation Report An Evan Question Explanation: Gout usually is associated with alcohol intake and presents with podagra, acute swelling and pain in the first metatarsal phalangeal joint. Gout is more common in diabetics and in men. Aspiration of uid ftom the involved joint would reveal urate crystals under the ppolatizing microscope. The pain is usually relieved with non-steroidals or prednisone. Reiter's syncrome is associated with HLAB2? and is more commonin young to middle-aged men. Itis associated with a sexually transmitted disease, urethritis, and arthritis. Lyme disease is caused by a deer tick bite and presents with arash and! or myalgias and fever. Arthritis is common in the knee joints. This man did not have a rash. Septic arthritis could be life threatening and is associated with a fever and elevated white count. Aspiration of the joint Quid would reveal an extremely elevated white count greater than 50,000 cells/ml, This pain would not be relieved with nonstcroidals alone antibiotics would be needed. CPDD is more commoa in the elderly, and aspiration of the joint. Flud would reveal rhomboid-shaped crystals which are positively bireffingent under the polarizing microscope. The knee joints are more commonly affected, ‘Mark this question & Question Id: 109973 Question 4 of 4 A.21 year old woman presents to her physician afier noticing blurred vision attacks. On physical examination absert pulses in the regions of the aortic arch and the arms are revealed. The most lkely clagnosis is a) Takayasu's arteritis b) Kawasaki's disease c) Buerger’s disease 4) Wegener’s granulomatosis +) von Hippel-Lindau disease Answer | Boanation | Other User's Explanation Report An Error Question Explanation: Takayasu's arteritis, or the ‘pulseless disease," is a disease mostly of young Asian women, Typically, pulses are absent in the arms. The ascending aorta is the most commonly affected site. Kawasaki's disease is a lymph node syndrome, which often presents with antertis of the coronary vessels. Buerger’s disease is an arteritis of smokers, while Wegener's granulomatvsis is a disease of small and medium sized vessels, but usually affects middle aged persons. Von Hippel Lindau disease is a disease of cavemous hemangiomas in the CNS. ‘Mark this question & Question Id: 109973 Question 4 of 4 A.21 year old woman presents to her physician afler noticing blurred vision attacks, On physical examination absent pulses in the regions of the aortic arch and the arms are revealed, The most iely ciagnosis is Y © a) Takayasu's arteritis b) Kawasaki's disease c) Buerger’s disease 4) Wegener's granulomatosis *) von Hippel-Lindau disease Answer | Boanation | Other User's Explanation Report An Error Question Explanation: Takayasu's arteritis, or the ‘pulseless disease," is a disease mostly of young Asian women, Typically, pulses are absent in the arms. The ascending aorta is the most commonly affected site. Kawasaki's disease is a lymph node syndrome, which often presents with antertis of the coronary vessels. Buerger’s disease is an arteritis of smokers, while Wegener's granulomatvsis is a disease of small and medium sized vessels, but usually affects middle aged persons. Von Hippel Lindau disease is a disease of cavemous hemangiomas in the CNS. ‘Mark this question => Question Id : 28342 Question 1 of 10 A 60-year-old has increasing back and leg pain for several years. X-Rays reveal bony sclerosis of the sacroiliac, lower vertebral and upper tial regions with cortical thickening and radiolucent areas. He has greater dificuty hearing on the left. He has orthopnea and pedal edema Blood tests reveal an elevated serum alkaline shosphatase. What is the most likely pathologic process that explains these findings? a) Paget's disease of bone b) Decreased bone mass c) Metastatic adenocarcinoma 4) Renal failure with renal osteodystrophy ¢) Vitamin D deficiency Question Explanation: This man has Paget's disease, with high output cardiac failure and sensorineural deafness Renal osteodystrophy leads to lesions of osteitis fbrosa cystica admixed with osteomalacias which are focal in nature. Metastatic disease to bons produces focal lesions, not more diffuse enlargement ‘Mark this question => Question Id : 28342 Question 1 of 10 A 60-year-old has increasing back and leg pain for several years. X-Rays reveal bony sclerosis of the sacroiliac, lower vertebral and upper tial regions with cortical thickening and radiolucent areas. He has greater dificuty hearing on the left. He has orthopnea and pedal edema Blood tests reveal an elevated serum alkaline phosphatase. What is the most lely pathologic process that explains these findings? Y © a) Paget's disease of bone b) Decreased bone mass ©) Metastatic adenocarcinoma 4) Renal failure with renal osteodystrophy ¢) Vitamin D deficiency Question Explanation: This man has Paget's disease, with high output cardiac failure and sensorineural deafness Renal osteodystrophy leads to lesions of osteitis fbrosa cystica admixed with osteomalacias which are focal in nature. Metastatic disease to bons produces focal lesions, not more diffuse enlargement 22772014 4 PM ‘Mark this question ez (Question Td : 54909 Question 2 of 10 “Which of the following is a pro-inflammatory cytokine? a) C-Reactive protein 'b) Tumowr necrosis factor 0. o) Id a L-10 ©) Serum Amyloid precursor protein (Question Explanation: (C- reactive protein and serum amyloid precursor protein ate acute phase reactants IL-4 and I-10 are enti-inflammatory cytokines TINF-alphais a pro-inflammatory cytokine In inflammatory disorder such as theumatoid arthritis, the levels of TNE-alpha are markedly elevated in inflamed joints. Treatments directed at the inhibiton of TNE-alpha such as inflaimab (a monoclonal antibody egainst TNE-alpha) have been shown to be very effective in the treatment of rheumatoid arthritis and also effective in fistalating Crohn's disease. 22772014 4 PM ‘Mark this question ez (Question Td : 54909 Question 2 of 10 ‘Which of the following is a pro-inflammatory cytokine? a) C-Reactive protein Y¥ © b) Tumow necrosis factor a c) a d) L-10 e) Serum Amylcid precursor protein (Question Explanation: (C- reactive protein and serum amyloid precursor protein ate acute phase reactants IL-4 and I-10 are enti-inflammatory cytokines TINF-alphais a pro-inflammatory cytokine In inflammatory disorder such as theumatoid arthritis, the levels of TNE-alpha are markedly elevated in inflamed joints. Treatments directed at the inhibiton of TNE-alpha such as inflaimab (a monoclonal antibody egainst TNE-alpha) have been shown to be very effective in the treatment of rheumatoid arthritis and also effective in fistalating Crohn's disease. ‘Mark this question ez Question Td Question 3 of 10 A 73-year-old afebrile man has acutely painful right knee that is hot and swollen, His white cell count is 12.8 x109/L (4-11 2109) and aknee X-ray shows reduced joint space and calcifiction of the articular cautilage. Culture of aspirated fuid revealed no growth, Diagnose a) Pseado-gout b) Gout ©) Psotiatic monoarthropathy 4) Rheumatoid arthritis €) Septic arthritis Question Explanation: This is a typical presentation of pseudo-gour / calcium pyrophosphate (CPP) arthropathy with evidence of osteoarthrits, calification ofthe articular canilage and no growth on culture, The differential does inchide gout but there is nothing else within the history to suggest this as the diagnosis. Distinguishing between the two depends on analysis of the crystals with CPP crystal demonstrating a postive birefringence and urate crystals demonstrating a negative birefringence. ‘Mark this question ez Question Td Question 3 of 10 A 73-year-old afebrile man has acutely painful right knee that is hot and swollen, His white cell count is 12.8 x109/L (4-11 2109) and aknee X-ray shows reduced joint space and calcifiction of the articular cautilage. Culture of aspirated fuid revealed no growth, Diagnose Y © a) Pseudo-gout b) Gout ©) Psotiatic monoarthropathy 4) Rheumatoid arthritis €) Septic arthritis Question Explanation: This is a typical presentation of pseudo-gour / calcium pyrophosphate (CPP) arthropathy with evidence of osteoarthrits, calification ofthe articular canilage and no growth on culture, The differential does inchide gout but there is nothing else within the history to suggest this as the diagnosis. Distinguishing between the two depends on analysis of the crystals with CPP crystal demonstrating a postive birefringence and urate crystals demonstrating a negative birefringence. ‘Marc this question ¢<-~ Question 4 of 10 4.30 year old individual with a haman leukocyte antigen type of HLA-B27 will be at risk for the development of which one of the following diseases? a) Reiter's syndrome 'b) Psoriatic arthritis ©) Arthritis associated with gastrointestinal defects d) Rhcurnatoid arthritis ©) Osteoarthritis (Question Explanation: Reiter’s syndrome, a spondyloarthropathy and ankylosing spondylitis have a strong association with HLA-B27. Psoriatic arthritis, arthntis associated with gastrointestinal defects, rheumatoid arthritis, end osteoarthritis do not have a strong association wth HLA- B2v. ‘Marc this question ¢<-~ Question 4 of 10 4.30 year old individual with a human leukocyte antigen type of HLA-B27 will be at risk for the development of which one of the following diseases? Y © a) Retter's syndrome 'b) Peorietic artis ©) Arthritis associated with gastrointestinal defects d) Rheumatoid arthritis €) Osteoarthritis (Question Explanation: Reiter’s syndrome, a spondyloarthropathy and ankylosing spondylitis have a strong association with HLA-B27. Psoriatic arthritis, arthntis associated with gastrointestinal defects, rheumatoid arthritis, end osteoarthritis do not have a strong association wth HLA- B2v. 22772014 11:09:10 PM. € CB www interface edu.ok/medical-exams/test-analysis.phputid=1 4. Apps EJ coogle A settings [signin Co imparted Fromie go. wlohe ulus co cea Bape oe Gi other bookinar ‘Mark this question << => Question Td : 87964 Question 5 of 10 A Si year old female has a long standing history of morning stiffness of her jeints, anemia, and bilateral symmetric arthritis of her hands and feet. Physician examination shows ulnar deviation of her Singers and subcutaneous nodules. She is being treated with, corticosteroids and nonsteroidal ant-inflammatory drugs (NSAIDs). Tae cardiac complication that may arise in this clinical setting is a) Constrictive Pericarditis, b) Dilated cardiomyopathy c) Hypersensitivity myocarditis 4) Hypertrophic cardiomyopathy e) Restrictive cardiomyopathy Question Explanation: Rheumatoid arthritis is a chronic inflammatory autoimmune disorder affecting multiple systems. It causes symmetrical joint deformities, arthritis, subcutaneous nodules, joint stiffness, pulmonary fibrosis, and multiple other extra articular afflictions. Amyloid links theumetoid arthritis (KA) to restrictive cardiomyopathy. Long stancing inflammatory conditions such as RA are associated with deposition of a form of amyloid known as AA (amyloid associated protein), which may involve kidneys, heart, liver, skeletal muscle, and skin, for example. Amyloid deposition in the myocardium results in decreased compliance and impaired diastolic filling, ie., restrictive cardiomyopathy. The myocardium has a rigid and waxy texture This form of amyloid, as well as any other biochemical form can be visualized on tissue section by staining with Congo red, which acquires a characteristic apole green birefringence under polarized light. Constrictive pericarditis is due to any pathologic process that results in fibrous thickening of the pericardium with resultant impaired compliance. Clinically, therefore, this condition manilests with a picture similar to restrictive cardiomyopathy because of impaired diastolic filling, Constrictive pericardiis is usually caused by previous episodes of acute pericarditis, especially hemorthagic, suppurative, and caseous pericanditis. Dilated cardiomyopathy is characterized by massive ventricular diatetion and maybe caused by genetic alterations, myocarditis toxic insults (alcohol) metabolic disorders (hemochromatosis), ett. Most cases are idiopathic. The main pathophysiologic alteration is impaired contractility. You may think that this patient is prone to developing hypersenstivity myocardits, but this form of myocardial disease has been reported after treatment with scme antihypertensive agexts, antibiotics, and diuretics not with corticosteroids or NSAIDs Furthermore, myocarditis manifests acutely with errhythmics and heart failure, and chronically with diated cardiomyopathy and congestive heart failure. Most cases of hynertorhic cardiomyopathy are familial and due to mutations in one of the genes encoding proteins of the sarcomeres. most frequently myosin heavy chain. This form of cardiomyopathy leads to asymmetric hypertrophy of the left ventricle with predominant thickening of the interventricular septum, In oes are ee ae Total Questions fe PO KD FS te xxxxKXxXKXKXKX 22772014 11:09:10 PM. € CB www interface edu.ok/medical-exams/test-analysis.phputid=1 4. Apps EJ coogle A settings [signin Co imparted Fromie go. wlohe ulus co cea Bape oe Gi other bookinar ‘Mark this question << => Question Td : 87964 Question 5 of 10 A. 54 year old female has a long standing history of maming stiffhess of her joints, anemia, and bilateral symametric arthritis ofher bands and feet. Physician examination shows ulnar deviation of her Sngers and subcutaneous nodules. She is being treated with corticosteroide and nonsteroidal ant-inflammatory druge (NSAIDs). Tae cardiac complication that may arise in this clinical cetting ie a) Constrictive Pericarditis ) Dilated cardiomyopathy c) Hypersensitivity myocarditis 4) Hypertrophic cardiomyopathy Y © e) Restrictive cardiomyopathy Question Explanation: Rheumatoid arthritis is a chronic inflammatory autoimmune disorder affecting multiple systems. It causes symmetrical joint deformities, arthritis, subcutaneous nodules, joint stiffness, pulmonary fibrosis, and multiple other extra articular afflictions. Amyloid links theumetoid arthritis (KA) to restrictive cardiomyopathy. Long stancing inflammatory conditions such as RA are associated with deposition of a form of amyloid known as AA (amyloid associated protein), which may involve kidneys, heart, liver, skeletal muscle, and skin, for example. Amyloid deposition in the myocardium results in decreased compliance and impaired diastolic filling, ie., restrictive cardiomyopathy. The myocardium has a rigid and waxy texture This form of amyloid, as well as any other biochemical form can be visualized on tissue section by staining with Congo red, which acquires a characteristic apole green birefringence under polarized light. Constrictive pericarditis is due to any pathologic process that results in fibrous thickening of the pericardium with resultant impaired compliance. Clinically, therefore, this condition manilests with a picture similar to restrictive cardiomyopathy because of impaired diastolic filling, Constrictive pericardiis is usually caused by previous episodes of acute pericarditis, especially hemorthagic, suppurative, and caseous pericanditis. Dilated cardiomyopathy is characterized by massive ventricular diatetion and maybe caused by genetic alterations, myocarditis toxic insults (alcohol) metabolic disorders (hemochromatosis), ett. Most cases are idiopathic. The main pathophysiologic alteration is impaired contractility. You may think that this patient is prone to developing hypersenstivity myocardits, but this form of myocardial disease has been reported after treatment with scme antihypertensive agexts, antibiotics, and diuretics not with corticosteroids or NSAIDs Furthermore, myocarditis manifests acutely with errhythmics and heart failure, and chronically with diated cardiomyopathy and congestive heart failure. Most cases of hynertorhic cardiomyopathy are familial and due to mutations in one of the genes encoding proteins of the sarcomeres. most frequently myosin heavy chain. This form of cardiomyopathy leads to asymmetric hypertrophy of the left ventricle with predominant thickening of the interventricular septum, In oes are ee ae Total Questions fe PO KD FS te xxxxKXxXKXKXKX ‘Mark this question & => (Question Id: 93905 Question 6 of 10 45 year old woman has had severe joint pain for several years. Initially she was bothered primarily by stifftess, pain, and swelling in her wrists, hands, ankles, and feet. Recently her knees, elbow, and shoulders have become affected. Although she feels better at times, she consistently has an extended period of siffhess during the early moming hours. A biopsy ofthe synovium oflher knee would mostlikely reveal which of the following? a) A nearly normal synovium with scattered inflammatory cells b) A non-proliferative synovitis with abscess formation c) A non-proliferative synovitis with many neutrophils 4) A proliferative smovitis with many eosinophils, neutrophils, and plasma ells 2) A proliferative synovitis with many Iymphocytes, macrophages, and plasma cells Question Explanation: Rheumatoid arthritis is a chronic systemic inflammatory disease primarily involving the synovial membranes and articuler structures of ‘multiple joints. Severe rheumatoid arthritis causes a proliferative synovitis with extensive damage to the synovium of the joint. The synovial membrane becomes markedly thickened (pannus formation), with edematous villous projections that extend into the joint space. The intense inflammatory, inflate that is present is typically composed of plasma cells, lymphocytes, and macrophages. ‘Mark this question & => (Question Id: 93905 Question 6 of 10 45 year old woman has had severe joint pain for several years. Initially she was bothered primarily by stifftess, pain, and swelling in her wrists, hands, ankles, and feet. Recently her knees, elbow, and shoulders have become affected. Although she feels better at times, she consistently has an extended period of siffhess during the early moming hours. A biopsy ofthe synovium oflher knee would mostlikely reveal which of the following? 4) A nearly normal synowinm with scattered inflammatary cells ) A non-proliftrative synovilis with abscess formation c) A non-proliferative synovitis with many neutrophils 4d) A proliferative smovitis with many eosinophils, neutrophils, and plasma ells JY © 2) A proliferative synovitic with mary lymphocytes, macrophages, and plasma cells Question Explanation: Rheumatoid arthritis is a chronic systemic inflammatory disease primarily involving the synovial membranes and articuler structures of ‘multiple joints. Severe rheumatoid arthritis causes a proliferative synovitis with extensive damage to the synovium of the joint. The synovial membrane becomes markedly thickened (pannus formation), with edematous villous projections that extend into the joint space. The intense inflammatory, inflate that is present is typically composed of plasma cells, lymphocytes, and macrophages. ‘Mari tis question ez Question Td 113520 Question 7 of 10 A 34 year old female factory worker uses a stapling gun every day in her right hand. After several months she notices pain and sore numbness in her right thunb and index finger. Whet is the most icely cause of her discomfor:? a) Repetitive use syndrome. ) Carpal tunnel syndrome ©) Cubital tonne! syndrome 9) Reflex sympathetic dystrophy. €) Tarsal tunnel syndrome. Answer [NBQIINSHBR) other Users Explanation Report An Error (Question Explanation: An injury resulting from continuous use of the same movements of any part of the musculoskeletal system is called a "repetitive use syndrome” The distributon of this patient's discomfort is not consistent with compression of the medien nerve in the carpal tunnel or with cubital tunnel compression. ‘The symptoms are not typical of reflex sympathetic dystrophy. The tarsal tunnel is located in the leg ‘Mari tis question ez Question Td 113520 Question 7 of 10 A 34 year old female factory worker uses a stapling gun every day in her right hand. After several months she notices pain and sore sumnbness in her right thunb and index finger. Whet is the most icely cause of her discomfor:? Y © a) Repetitive use syndrome. ) Carpal tunnel syndrome ©) Cubital tonne! syndrome 9) Reflex sympathetic dystrophy. €) Tarsal tunnel syndrome. Answer [NBQIINSHBR) other Users Explanation Report An Error (Question Explanation: An injury resulting from continuous use of the same movements of any part of the musculoskeletal system is called a "repetitive use syndrome” The distributon of this patient's discomfort is not consistent with compression of the medien nerve in the carpal tunnel or with cubital tunnel compression. ‘The symptoms are not typical of reflex sympathetic dystrophy. The tarsal tunnel is located in the leg 2/27/2014 11:09:47 PM ‘Mark this question & => Question Id : 120766 Question of 10 A.74 year old male presents with high output cardiac failure. Further examination demonstrates hypercalcemia, elevated alkaline phosphatase, and a disabling wealness secondary to left petvic bone pain. The most lkely diagnosis is a) Metastatic prostate cancer b) Ricket’s c) Paget's disease 4) Hypoparathyroidism €) Berbesi Answer | Exvianaton | Other User's Explanation Report An Error Question Explanation: Paget's disease is commonin the elderly and is caused by increased bone tumnover. There is elevation of urinary hydroxyproline and increased bone resorption secondary to osteoclasts. In severe cases, high output cardiac failure can occur, Metastatic prostate cancer can cause ostzosclerosis, which is a diffuse increase in cancellous bone mass. Rickets is commonin children, Osteomalacia, with decreases in calcium, phosphorous, and vitamin D, is present. Hypoparathyroidism would cause hypocalcemia, not hypercalcemia secondary to decrease in secretion of parathyroid hormone. Berber is a disease which can cause high output cardice falure secondary to thiamine deficiency. 2/27/2014 11:09:47 PM ‘Mark this question & => Question Id : 120766 Question of 10 A.74 year old male presents with high output cardiac failure. Further examination demonstrates hypercalcemia, elevated alkaline phosphatase, and a disabling wealness secondary to left petvic bone pain. The most lkely diagnosis is a) Metastatic prostate cancer b) Ricket's Y © c) Paget's disease 9) Hypoparathyroidism ¢) Benberi Answer | Exvianaton | Other User's Explanation Report An Error Question Explanation: Paget's disease is commonin the elderly and is caused by increased bone tumnover. There is elevation of urinary hydroxyproline and increased bone resorption secondary to osteoclasts. In severe cases, high output cardiac failure can occur, Metastatic prostate cancer can cause ostzosclerosis, which is a diffuse increase in cancellous bone mass. Rickets is commonin children, Osteomalacia, with decreases in calcium, phosphorous, and vitamin D, is present. Hypoparathyroidism would cause hypocalcemia, not hypercalcemia secondary to decrease in secretion of parathyroid hormone. Berber is a disease which can cause high output cardice falure secondary to thiamine deficiency. ‘Mark this question & => (Question Id : 139226 Question 9 of 10 A.woman with deformed wrist joints presents for routine evaluation, Hand examination shows decreased range of moton also marked ulnar deviation of the fingers. She has recently had seven dental caries filled. What marker would be specific to identifying the present ines? a) Anticentromere b) p-ANCA ©) SSB La) 4) Anti-ds DNA ¢) Antimicrosomal antibodies Question Explanation: ‘There are two markers (antibodies) associated with Sjoaren syadrome: SS-A (Ro) and $S-B (La), SS-B is more specific (70%) to confirm the diagnosis ofthe disease than SS-A. due to the fact that it is associated with nmerous other autoimmune conditions such fas neonatal lupus. Sjégren syndrome classically presents with keratocanjanctivitis (dry eyes) and xerostomia (dry month often resubing in dertal caries and fissures in the oral mucosa). These symptoms are due to autoimmme involvement with subsequent scarving of the salivary and lacrimal glands. Parotid gland enlargement is commen as are vasculiis, Raynand phenomenon hyperviscotity syndrome, and perpheral neuropathy. Rheumatoid arthritis (RA) can coexist with a wide variety of autoimmune disorders, butit is mostly associated with Sjégren syndrome. Anticentromere antibodies are present in CREST syndrome which is manifested by calcinosis, Raynaud phenomena esophageal dysmotility, sclerodectyly, and telangicctasias. CREST is a milder form of scleroderma, with limited skin involvement to the face and hand and itis not associated with dental caries. p-ANCA is associated with microscopic polyangiits, 2 systemic necrotizing vasculits (Rbrinoid necrosis) of small arteries that classically involves the kidneys and the lungs. Iris not related to the development of dental caries. Anti-ds DMTA is associated with systemic lupus erythernatosus (SLE), which is an autoimmune disease characrerized by vasculitis, rash, renal disease, hemolytic anemia, and neurologic disturbances. Antimicrosomal antibodies are associated with Hashimoto thyroicitis, an autoimmune disorder that results in hypothyroidism. Itis not related to the development of dental caries, ‘Mark this question & => (Question Id : 139226 Question 9 of 10 A.woman with deformed wrist joints presents for routine evaluation, Hand examination shows decreased range of moton also marked ulnar deviation of the fingers. She has recently had seven dental caries filled. What marker would be specific to identifying the present ines? a) Anticentromere b) p-ANCA Y Oc) SSB La) 4) Anti-ds DNA €) Antimicrosomal antibodies Question Explanation: ‘There are two markers (antibodies) associated with Sjoaren syadrome: SS-A (Ro) and $S-B (La), SS-B is more specific (70%) to confirm the diagnosis ofthe disease than SS-A. due to the fact that it is associated with nmerous other autoimmune conditions such fas neonatal lupus. Sjégren syndrome classically presents with keratocanjanctivitis (dry eyes) and xerostomia (dry month often resubing in dertal caries and fissures in the oral mucosa). These symptoms are due to autoimmme involvement with subsequent scarving of the salivary and lacrimal glands. Parotid gland enlargement is commen as are vasculiis, Raynand phenomenon hyperviscotity syndrome, and perpheral neuropathy. Rheumatoid arthritis (RA) can coexist with a wide variety of autoimmune disorders, butit is mostly associated with Sjégren syndrome. Anticentromere antibodies are present in CREST syndrome which is manifested by calcinosis, Raynaud phenomena esophageal dysmotility, sclerodectyly, and telangicctasias. CREST is a milder form of scleroderma, with limited skin involvement to the face and hand and itis not associated with dental caries. p-ANCA is associated with microscopic polyangiits, 2 systemic necrotizing vasculits (Rbrinoid necrosis) of small arteries that classically involves the kidneys and the lungs. Iris not related to the development of dental caries. Anti-ds DMTA is associated with systemic lupus erythernatosus (SLE), which is an autoimmune disease characrerized by vasculitis, rash, renal disease, hemolytic anemia, and neurologic disturbances. Antimicrosomal antibodies are associated with Hashimoto thyroicitis, an autoimmune disorder that results in hypothyroidism. Itis not related to the development of dental caries, 2/27/2014 11:10:20 PM. € (unread) - emackomr- x \ @? missona’s profile on gr x) 9% www nterface.eduipk/n x e www. interface edu,pk/medical analysis php2utid=14759 Apps EJ coogle A settings [signin Ci imparted Fomie go. wlohe ulus Jose ecco rear oo ceuall Bye Pe [is tab Ww Gi other bookinar Mask this question = Question Id: 217185 Question 10 of 10 A 60-year-old femele has shoulder pain for the last 3 months after treatment of a fractured radius that involved casting of the wrist end elbow and use of en arm sling for 8 weeks, It is severe at night. Stiffness and défculty moving the affected shoulder exists. Exam reveals limited active and passive range of motion of the shoulder, and arthrography demonstrates decreased capacity of the joint space. Which of the following is the most ilcely diagnosis? a) Adhesive capsulitis b) Pseudogout ©) Rheumatoid arthritis ) Septic arthritis €) Subacromial bursitis Question Explanatio: ‘This patient is suéfering from adhesive capsulitis of the shoulder (‘frozen shoulder’). This condition is characterized by pain and immobility of the shoulder at the gleaokumeral joint it typically affects older females and may occur in a variety of settings. The most typical history is one of prolonged shoulder immobiity (as in this case) or of recent trauma or inflammation of the shoulder structures, Diabetics are at a significantly higher risk of developing this condition when compared to the general population, The pain from this condition typically occurs during the night and can also be proveked by attempted active or passive range of motion at the shoulder, particularly with motions invoWving intemal rotation, Arthregrephy, a procedure where an z-ray is taken following injection of radiocontrast material into the jomt classically shows decreased volume of the joint space. This is a seE limited condition thet typically resolves within approximately 1 year. Pseudogout typically causes amonoartiritis (typically ofthe knee) resulting from deposition of calcium pyrophosphate dehydrate crystals in the synovium. Rheumatoid arthritis typically manifests as a symmetric inflammatory arthritis. Patients complain of moming stiffness lasting over 1 hour and constitutional symptoms in addition to joint pain. The carpal, MCP, and PIP joints are preferentally affecied Septic artivits is usually an acute monoarthits that results from hematogenous seeding of organisms irto the joint. Staphylococcus ewe is the most common cause in the general population. Symptoms would include acute onset pan, tendemess, edema, and erythema of the affected joint. A fever may be present as wel Sub-acromial bursitis typically causes pain when the arm is actively raised laterally above the level of the shoulder. Pain is usually Le Report An Error Total Questions HNO OMI xxxXxxXXXKXXKX 2/27/2014 11:10:20 PM. € (unread) - emackomr- x \ @? missona’s profile on gr x) 9% www nterface.eduipk/n x e www. interface edu,pk/medical analysis php2utid=14759 Apps EJ coogle A settings [signin Ci imparted Fomie go. wlohe ulus Jose ecco rear oo ceuall Bye Pe [is tab Ww Gi other bookinar Mask this question = Question Id: 217185 Question 10 of 10 ‘A. 60-year-old female has shoulder pain for the last 3 months after treatment of a fractured radive that involved casting of the wrist cad elbow and we of en arm sling for 8 weeks. It ie severe at night. Stifhess and difficulty moving the affected shoulder exists. Ester oveals limited active and passive range of motion of the shoulder, and arthrography demonstrates decreased capacity of the joint space. Which of the following is the most likely diagnosis? Y © a) Adhesive capsulitis )Pseudogout ©) Rheumatoid arthritis 4) Septic axtheitis ©) Subacromial bursitis Question Explanatio: ‘This patient is suéfering from adhesive capsulitis of the shoulder (‘frozen shoulder’). This condition is characterized by pain and immobility of the shoulder at the gleaokumeral joint it typically affects older females and may occur in a variety of settings. The most typical history is one of prolonged shoulder immobiity (as in this case) or of recent trauma or inflammation of the shoulder structures, Diabetics are at a significantly higher risk of developing this condition when compared to the general population, The pain from this condition typically occurs during the night and can also be proveked by attempted active or passive range of motion at the shoulder, particularly with motions invoWving intemal rotation, Arthregrephy, a procedure where an z-ray is taken following injection of radiocontrast material into the jomt classically shows decreased volume of the joint space. This is a seE limited condition thet typically resolves within approximately 1 year. Pseudogout typically causes amonoartiritis (typically ofthe knee) resulting from deposition of calcium pyrophosphate dehydrate crystals in the synovium. Rheumatoid arthritis typically manifests as a symmetric inflammatory arthritis. Patients complain of moming stiffness lasting over 1 hour and constitutional symptoms in addition to joint pain. The carpal, MCP, and PIP joints are preferentally affecied Septic artivits is usually an acute monoarthits that results from hematogenous seeding of organisms irto the joint. Staphylococcus ewe is the most common cause in the general population. Symptoms would include acute onset pan, tendemess, edema, and erythema of the affected joint. A fever may be present as wel Sub-acromial bursitis typically causes pain when the arm is actively raised laterally above the level of the shoulder. Pain is usually Le Report An Error Total Questions HNO OMI xxxXxxXXXKXXKX 2/27/2014 11:11:29 PM. ‘Mark this question => Question Td: 22715 Question 1 of 20 A.48-year-old male went for an insurance medical examination. He was entirely asymptomatic, but his serum urate conceatration was noted to be 0.5 mmol (0.23 - 0.46). What is the most appropriate management for this patient? ) Allepurinal ) Lifestyle intervention c) Colehicine 4) Ibuprofen ©) Probenecid Avower (ERNE) otter Users Explanation Report An Enos Question Explanation: This asymptomatic patients requires only lifestyle advice ie. weight loss if appropriate, an appropriate diet and reduction of alcohol. 2/27/2014 11:11:29 PM. ‘Mark this question => Question Td: 22715 Question 1 of 20 A.48-year-old male went for an insurance medical examination. He was entirely asymptomatic, but his serum urate conceatration was noted to be 0.5 mmol (0.23 - 0.46). What is the most appropriate management for this patient? ) Allepurinal Y © ») Lifestyle intervention c) Colehicine 4) Ibuprofen ©) Probenecid Avower (ERNE) otter Users Explanation Report An Enos Question Explanation: This asymptomatic patients requires only lifestyle advice ie. weight loss if appropriate, an appropriate diet and reduction of alcohol. 2/27/2014 11:11:48 PM. ‘Mark this question & => Question Id: 28172 Question 2 of 20 A 73-year-old female has tenderness and bony swelling over the base of first metacarpal and wasting of right thenar eminence. Labs show: ESR 30mm/Ist hour, CRP normal, Rheumetoid factor 60TU/L, X-ray shows loss of joint space with articular sclerosis and osteophytes of the first carpo-metacarpal joint. Diagnosis is: a) Osteoporosis, b) Gouty arthritis c) Paget's disease of bone 4) Osteoarthritis ©) Rheumatoid arthritis £) Seronegative spondyloarthitis (Question Explanation: This woman has clinical and radiological features consistent with osteoarthritis (O.A) of the Ist night carpometacarpal (CMC) jon (OAis characterized by joint pain, crepitus, and stiffness after mobility, and Lmitation of motion. The CIMIC jointis involved in arpping and twisting, The clinical joint symptoms are associated with defects in the articular cartilage and underlying bone, outlined in this wornan’s x-ray findings. Joint swelling is bony in nature, unlice the boggy swelling which occurs in inflammatory arthritis. This woman “ESR ss not signtficantly raised and her CRP is within normal range making an inflammatory arthntis unlikely. A positive cheumatoid factor does not make the diagnosis of rheumatoid arthritis, The frequency of positive rheumatoid factor in normal indinduals of age 701s upto 10-20%. Thenar wasting occurs in OA of the Ist CMC joint due to disuse. 2/27/2014 11:11:48 PM. ‘Mark this question & => Question Id: 28172 Question 2 of 20 ‘A.73-year-old female has tendemess and bony swelling over the base offiret metazarpal and wasting of right thenar eminence, Labs show: ESR 30mm/Ist hour, CRP notmal, Rheumatoid factor 6OTU/L, X-ray shows loss of joint space we articular sclerosis and odteophytes of the first carpo-metacarpal joint. Diagnosis is a) Osteoporosis +b) Gonty arthritis ) Paget's disease of tone Y © 4) Osteoarthritis c) Rheumatoid artis £) Seronegative spendsloarthritis (Question Explanation: This woman has clinical and radiological features consistent with osteoarthritis (O.A) of the Ist night carpometacarpal (CMC) jon (OAis characterized by joint pain, crepitus, and stiffness after mobility, and Lmitation of motion. The CIMIC jointis involved in arpping and twisting, The clinical joint symptoms are associated with defects in the articular cartilage and underlying bone, outlined in this wornan’s x-ray findings. Joint swelling is bony in nature, unlice the boggy swelling which occurs in inflammatory arthritis. This woman “ESR ss not signtficantly raised and her CRP is within normal range making an inflammatory arthntis unlikely. A positive cheumatoid factor does not make the diagnosis of rheumatoid arthritis, The frequency of positive rheumatoid factor in normal indinduals of age 701s upto 10-20%. Thenar wasting occurs in OA of the Ist CMC joint due to disuse. 22772014 11:11:58 PM ‘Mark this question & => ion Td: Question 3 of 20 A general practice covers a population of 20,000 patients, How many patients with Rheumatoid Arthitis would be expected in this population? a) 100 ») 450 ©) 200 470 2) 50 Answer | Boanation Other User's Explanation Report An Error Question Explanation: The prevalence of theumateid Arthritis is approximately 12% Thus in a practice of 20,000 the number of patients with RA would be 200, a not insignificant number. Approximately 2-3 females are affected for each male 22772014 11:11:58 PM ‘Mark this question & => ion Td: Question 3 of 20 A general practice covers a population of 20,000 patients, How many patients with Rheumatoid Arthitis would be expected in this population? a) 100 ») 450 ¥ ©») 200 470 2) 50 Answer | Boanation Other User's Explanation Report An Error Question Explanation: The prevalence of theumateid Arthritis is approximately 12% Thus in a practice of 20,000 the number of patients with RA would be 200, a not insignificant number. Approximately 2-3 females are affected for each male ‘Mark this question = => Question Id : 46780 Question 4 of 20 A.72 year old woman refums to the office because of aching and weakness her arms to the point where she canaotlft her arm to ‘brush her hair. No muscle tenderness or other evidence of joint disease is present in both arms. The aching improves whea she takes the prescribed nonsteroidal ani-inflammatery drug (TSAID). She also describes tenderness over the right parietal area of her scalp, Physical examination of the scalp shows no lesions, The most appropriate nest step is a) Increases the dose of the NSAID b) Order determination of serum rheumatoid factor c) Order x-ray films of the cermical spine 4) Order determination of erythrocyte sedimentation rate +) Refer her for psychiatric counseling Answer (Birisnanon) Other User's Explanation Report An Error Question Explanation: The tenderness over the right temporal area of the patient's scalp is worrisome for temporal arteritis, which is offen ascociated with an elevated erythrocyte sedimentation rate, Prompt treatment with steroids is often indicated to prevent blindness. The diagnosis can bbe established with a temporal artery biopsy. Increasing her dose of NSAIDs is incorrect. The most common treatments for pain are the non-steroidal anti-inflammetory deugs, or NSAIDs. I this is ineffective, oral steroids such as prednisone should be started. Since this patient does not have signs or symptoms consistent with any type of arthritis (no joint disease: arthritis is a disease of joints), a serum RF is not useful A radiograph of the cervical spine is not indicated here as this patient does not describe radicular symptoms that could be attributed to compression of single nerve root, Rather, her symptoms are of generalized weakness. This is nota symptom that a cervical disc could be responsible for. ‘Mark this question = => Question Id : 46780 Question 4 of 20 A.72 year old woman refums to the office because of aching and weakness her arms to the point where she canaotlft her arm to ‘brush her hair. No muscle tenderness or other evidence of joint disease is present in both arms. The aching improves whea she takes the prescribed nonsteroidal ani-inflammatery drug (TSAID). She also describes tenderness over the right parietal area of her scalp, Physical examination of the scalp shows no lesions, The most appropriate nest step is a) Increases the dose of the NSAID b) Order determination of serum rheumatoid factor c) Order x-ray films of the cermcal spine Y © 4) Order determination of erythrocyte sedimentation rate 2) Refer her for psychiatric counseling Answer (Birisnanon) Other User's Explanation Report An Error Question Explanation: The tenderness over the right temporal area of the patient's scalp is worrisome for temporal arteritis, which is offen ascociated with an elevated erythrocyte sedimentation rate, Prompt treatment with steroids is often indicated to prevent blindness. The diagnosis can bbe established with a temporal artery biopsy. Increasing her dose of NSAIDs is incorrect. The most common treatments for pain are the non-steroidal anti-inflammetory deugs, or NSAIDs. I this is ineffective, oral steroids such as prednisone should be started. Since this patient does not have signs or symptoms consistent with any type of arthritis (no joint disease: arthritis is a disease of joints), a serum RF is not useful A radiograph of the cervical spine is not indicated here as this patient does not describe radicular symptoms that could be attributed to compression of single nerve root, Rather, her symptoms are of generalized weakness. This is nota symptom that a cervical disc could be responsible for. ‘Maré this question => Question Id : 54889 Question 5 of 20 A 39-year-old lady presents to clinic complaining of an 18 month history of dorsoradial wrist pain, She is a keen tennis player. On exattination she has tendemess localized to the dorsoradial aspect of the wrist and passive motion of the thumb causes crepitus in the same region. Finkelstein’s test is positive, What is the litely diagnosis? a) Caspal manel syndrome b) Golfer's elbow ©) Tennis elbow 4d) De Quervain’s tenosynovitis €) Compartinent synerome Question Explanation: De Quervain’s tenosynovitis is thought to be related to overuce and is commen in golfers and racquet «port players. Most affected ace females 30-50 years old, Finkelstein’s test (Dexion of the thumb into the palm, making a first over the thurc and ulnar deviation of the wrist causes pain in the first dorsal extensor compartment) is diagnostic ‘Maré this question => Question Id : 54889 Question 5 of 20 A 39-year-old lady presents to clinic complaining of an 18 month history of dorsoradial wrist pain, She is a keen tennis player. On exattination she has tendemess localized to the dorsoradial aspect of the wrist and passive motion of the thumb causes crepitus in the came region, Finkelstein’s test ie positive. What is the likely diagnosie? a) Carpal muanel syndrome 'b) Golfer's elbow ©) Tennis elbow Y © d)De Quervain’s tenosynovitis ¢) Compartment syncrome Question Explanation: De Quervain’s tenosynovitis is thought to be related to overuce and is commen in golfers and racquet «port players. Most affected ace females 30-50 years old, Finkelstein’s test (Dexion of the thumb into the palm, making a first over the thurc and ulnar deviation of the wrist causes pain in the first dorsal extensor compartment) is diagnostic ‘Matte this question <= => Question Td : 59551 Question 6 of 20 A78 year old man with a previous history of prostate cancer has a lumbar spine films suggesting osteopenia, Subsequent bone density studies show aT score of 2-2.7. The appropriate therapy would be a) Testosterone ) Calcitonin nasal spray (Micalcin) c) Ralonifene Evista) 4) Alenctonate Fosamax) Answer [JERI] oter Users Explanation Report An Eso Question Explanation: ‘The only approved treatments for male osteoporosis are alendronate and recombinant parathyroid hormone. Several drugs have been tested in clinical tals, and more pharmacologi treatments should become available in the future as male osteoporosis is increasingly recognized, Testosterone should not be used in this patient because of his history of prostate cancer ‘Matte this question <= => Question Td : 59551 Question 6 of 20 A.78 year old man with a previous history of prostate cancer has a inmbar spine films suggesting osteopenia. Subsequent bone density studies show a T score of 2-2.7. The appropriate therapy would be a) Testosterone ) Calcitonin nasal spray (Micalcin) ©) Ralossfine Evista) Y © 4) Alendronate (Fosamax) Answer [JERI] oter Users Explanation Report An Eso Question Explanation: ‘The only approved treatments for male osteoporosis are alendronate and recombinant parathyroid hormone. Several drugs have been tested in clinical tals, and more pharmacologi treatments should become available in the future as male osteoporosis is increasingly recognized, Testosterone should not be used in this patient because of his history of prostate cancer ‘Mark this question << => (Question Id: 61778 Question 7 of 20 A positive flexion abduction exsernal rotation (ABER) test that elicits posterior pain indicates involvement of which of the following joints? a) Shoulder ») Sacroilac *) Ankle a) Wrist 2) Knee Anower EEENGEERY cine: sorebxplanaion Report An Ever Question Explanation: “When the flexion ab dacton external rotation (FABER) test elicts pain pesteriody, it indicates sacroiliac involvemert, Anterior pain indicates hip involvement ‘Mark this question << => (Question Id: 61778 Question 7 of 20 A positive flexion abduction extemal rotation (EABER) test that elicits posterior pain indicates involvement of which of the following jonts? 2) Shoulder Y © 6) Sacroiliac 2) Ande d) Wrist 2) Knee Anowor ERTGEERY cn: sore bxptanas Question Explanation: “When the flexion ab dacton external rotation (FABER) test elicts pain pesteriody, it indicates sacroiliac involvemert, Anterior pain indicates hip involvement Report An Error ‘Mark this question =z Question Td 63590 Question 8 of 20, “Which of the following is NOT true? a) Psoriatic artiritis is always associated with skin lesions . overlying the involved joints ») Psoriasis can be associated with spondylitis and sacroilts ©) Psoriatic arthritis can appear similar to rheumatoid arthritis 4) Psoriatic arthnits offen demon strates radiographic manifestations «) Psoriatic artiritis is not usually associated with a positive sheumatoid factor. newer (REINER other Users Expton Question Explanation: Report An Eror ‘Psoriatic lesions sometimes, but not usually can overiie affected joints, In fact, patients with psoriatic arthntis often have very minimal skin findngs. ‘The other four statements are all true. ‘Mark this question =z Question Td 63590 Question 8 of 20 ‘Which of the following is MOT true? Y © a) Psoriatic arthritis is always associated with skin lesions overlying the involved joints. ») Psoriasis can be associated with spondyliis and sacroilts, ©) Psoriatic arthritis can appear similar to rheumatoid arthritis, d) Psoriatic arthritis often demon strates radiographic manifestations. 2) Psoriatic arthritis is not usually associated with a positive sheumatoid factor. newer (REINER other Users Expton Question Explanation: Report An Eror ‘Psoriatic lesions sometimes, but not usually can overiie affected joints, In fact, patients with psoriatic arthntis often have very minimal skin findngs. ‘The other four statements are all true. 2/27/2014 11:13:09 PM ‘Mark this question = => Question Td : 64572 Question 9 of 20 A.27 year old woman complains of fatigue, stiffiess, and a diffise aching in her neck, shoulders, and back for several months. She is not sleeping well. A physical examination is unremarkable except for tendemess over the upper trapezius, second costochondral Junctions, lateral epicondyles, end medial knees. A CBC, erythrocyte sedimentation rate, and rheumatoid factor are unremarkable. ‘The most appropriate management at this time is a) Prescribe Amitriptyline (Elavi), 10-25 mg every aight at bedtime, along with exercise aimed at improving her overall level of fines b) Presonbe a 2-week course ofa short-acting, sedative-hypaotic such as Triazolam (Halcior). 0.25 mg every night at bedtitne c) Schedule electromyography and nerve conduction studies to male out nearopathies 4d) Refer the patient to a rheumatologist for suspected seronegative theumetoid arthritis Question Explanation: This patient has a classic presentation of fibrositis fibromyalgia syndrome. In addition to small bedtime doses of amtriptyline or cyclobenzaprine, symptoms are improved by an increase in physical ness, stress reduction, regulation of sleep schedules, and reassurance. Opiate analgesics and sedative hypnotics are not recommended for long term treatment of this disorder. 2/27/2014 11:13:09 PM ‘Mark this question = => Question Td : 64572 Question 9 of 20 A.27 year old woman complains of fatigue, stiffiess, and a diffise aching in her neck, shoulders, and back for several months. She is not sleeping well. A physical examination is unremarkable except for tendemess over the upper trapezius, second costochondral Junctions, lateral epicondyles, end medial knees. A CBC, erythrocyte sedimentation rate, and rheumatoid factor are unremarkable. ‘The most appropriate management at this time is Y © a) Prescribe Amitriptyline Elvi), 10-25 mg every aight at bedtime, along with exercise aimed at improving her overall level of fines b) Presonbe a 2-week course ofa short-acting, sedative-hypaotic such as Triazolam (Halcior). 0.25 mg every night at bedtitne c) Schedule electromyography and nerve conduction studies to male out nearopathies 4d) Refer the patient to a rheumatologist for suspected seronegative theumetoid arthritis Question Explanation: This patient has a classic presentation of fibrositis fibromyalgia syndrome. In addition to small bedtime doses of amtriptyline or cyclobenzaprine, symptoms are improved by an increase in physical ness, stress reduction, regulation of sleep schedules, and reassurance. Opiate analgesics and sedative hypnotics are not recommended for long term treatment of this disorder. 22772014 11:13:22 PM ‘Marc this question = => Question Td : 70187 Question 10 of 20 ‘Which one of the following is NOT characteristic of limited systemic scleroderma? a) Ramnand’s phenomenon ) Esophageal disease, ©) Caleinosis of subcutaneous tissue ) Scleredactyly ©) Synovitis Question Explanation: ‘Synovitis is not considered part of the symptoms associated with limited systemic scleroderma, or the CREST syndrome, CREST is an acronym that accounts for the following syinptoms: calcinosis of subcutaneous tissue; Raynaud's phenomenon, esophageal disease; and sclerodactyly, tightening of the hand, face, and feet, Telangiectasia also occurs in the limited scleroderma type (CREST). 22772014 11:13:22 PM ‘Marc this question = => Question Td : 70187 Question 10 of 20 ‘Which one of the following is NOT characteristic of limited systemic scleroderma? a) Ramnand’s phenomenon ) Esophageal disease, ©) Caleinosis of subcutaneous tissue A) Sclerodastyiy Y © ¢) Synovitis Question Explanation: ‘Synovitis is not considered part of the symptoms associated with limited systemic scleroderma, or the CREST syndrome, CREST is an acronym that accounts for the following syinptoms: calcinosis of subcutaneous tissue; Raynaud's phenomenon, esophageal disease; and sclerodactyly, tightening of the hand, face, and feet, Telangiectasia also occurs in the limited scleroderma type (CREST). 2er2014 4 M ‘Mark this question << => Question Td: 74184 Question 11 of 20 A. 16-year-old has five day old swollen knee and a low-grade fever. She had pain while urinating and vaginal discharge two weeks ago, which she thought to be an yeast infection, Her knee is aspirated and the cell count shows 40,000 WBCs and 10 RBCs. Cultures are pending, The most appropriate treatment would be a) Ceftriaxone. 6) Cefazoin, ©) To consult orthopaedics for joint ivigation and debridement 4) Miconezole. ¢) To give the patient ibuprofen and a knee immobilizer for treatment of her traumatic injury Question Explanation: This patient has a clinical history and lab results most consistent with gonoccocal infection for which ceftriaxone is the treatment of choice. Gonoccocol arthritis usually responds to antibiotics and rarely requires jomt irigation and debridement. The 40,000 WBC is not consistent with septic arthrits (greater than 100,000 WBC). Ifthis wes an isolated yeest infection, itis not licely that there would bea knee effusion and fever, thus miconazole, which is used to treat yeast infections, is an incorrect answer. The absence of greater than 500 RBCs makes trauma an unlikely source for her elfission, rukng out answer choice. 2er2014 4 M ‘Mark this question << => Question Td: 74184 Question 11 of 20 A. 16-year-old has five day old swollen knee and a low-grade fever. She had pain while urinating and vaginal discharge two weeks ago, which she thought to be an yeast infection, Her knee is aspirated and the cell count shows 40,000 WBCs and 10 RBCs. Cultures are pending, The most appropriate treatment would be Y © a) Ceftriaxone. 6) Cefazoin, ©) To consult orthopaedics for joint ivigation and debridement 4) Miconezole. ¢) To give the patient ibuprofen and a knee immobilizer for treatment of her traumatic injury Question Explanation: This patient has a clinical history and lab results most consistent with gonoccocal infection for which ceftriaxone is the treatment of choice. Gonoccocol arthritis usually responds to antibiotics and rarely requires jomt irigation and debridement. The 40,000 WBC is not consistent with septic arthrits (greater than 100,000 WBC). Ifthis wes an isolated yeest infection, itis not licely that there would bea knee effusion and fever, thus miconazole, which is used to treat yeast infections, is an incorrect answer. The absence of greater than 500 RBCs makes trauma an unlikely source for her elfission, rukng out answer choice. 2272014 11:13:49 PM ‘Matte this question & => Question Ta : 78628 Question 12 of 20 A.Si-year-old woman with long history of theumatcid arthritis is taking nonsteroidal anti-inflammatory drugs. ‘Which one of the following is associated with her ilness? a) Elevated Tz. 1b) Tnereased risk of vertebral spine fractures ©) Pleural effusion 4) Chibbing of the fingers e) Lymphoma 8) Analgesic Nephropathy Answer (Explanation) Other User's Explanation Report An Error Question Explanation: Pleural effisions with alow glucose level in the effusion are associated with long-standing rheumatoid arthritis. An elevation of sheumatoid factor TgM is associated with rheumatoid arthritis-not IgA. An increased risk of femoral neck and vertebral spine fiactures is known with osteoporosis, not rheumatoid arthritis. Clubbing of the fingers occurs with long-standing hypoxemia, which is not a feanure of rheumatoid arthitis Lymphoma is a malignancy which can occur with Sjogren's syndrome-not rheumatoid arthritis, Analgesic Nephropathy occurs as a result of drugs containing mixtures of Acetaminophen and aspirin, 2272014 11:13:49 PM ‘Matte this question & => Question Ta : 78628 Question 12 of 20 A Sl-year-old woman with ¢ long history of rheumatoid arthritis is taking nonsteroidal ant-inflammatory drugs. Which one of the following is associated with her ilness? a) Elevated Ig 1b) Increased risk of vertebral spine fractures Y © ©) Pleural fusion 4) Clubbing of the fingers e) Lymphoma 8) Analgesic Nephropathy Answer (Explanation) Other User's Explanation Report An Error Question Explanation: Pleural effisions with alow glucose level in the effusion are associated with long-standing rheumatoid arthritis. An elevation of sheumatoid factor TgM is associated with rheumatoid arthritis-not IgA. An increased risk of femoral neck and vertebral spine fiactures is known with osteoporosis, not rheumatoid arthritis. Clubbing of the fingers occurs with long-standing hypoxemia, which is not a feanure of rheumatoid arthitis Lymphoma is a malignancy which can occur with Sjogren's syndrome-not rheumatoid arthritis, Analgesic Nephropathy occurs as a result of drugs containing mixtures of Acetaminophen and aspirin, 2/27/2014 4 PM ‘Mari this question <= => Question Td: 93201 Question 13 of 20 “Which is NOT a predisposing factor for hemiated disc disease? a) Loss of fd in the mcleus pulposus b) Tears in the anulas fibrosus ©) Loss of resiliency of the miclens pulposas ) Degeneration of the postesior longitudinal ligaments. €) Ostroporosis ofthe lumbar vertebrae. (Question Explanation: ‘A fibrocartlaginous intervertebral disc rests berween each two vertebral bodies. The disc consists of a soft inner nucleus pulposus and a thick surrounding anulus fibrosis. The nucleus pulposus loses fluid, volume, and resliency with age, increasing its susceptibility to compression [(A) and (C)]. Tears of the anulus fibrosus and degeneration of the posterior longitudinal igaments predispose to herniation of the nucleus pulposus [(B) and (D)]. Osteoporosis results in fractures of the vertebrae, a separate entity from disc disease (E) 2/27/2014 4 PM ‘Mari this question <= => Question Td: 93201 Question 13 of 20 “Which is NOT a predisposing factor for hemiaied disc disease? a) Loss of tid in the micleus pulposus. ) Tears in the emulas fibrosus ©) Loss of resiliency of the miclens pulposas ) Degeneration of the posterior longidinal ligaments Y © €) Osteoporosis of the lumbar vertebrae. (Question Explanation: ‘A fibrocartlaginous intervertebral disc rests berween each two vertebral bodies. The disc consists of a soft inner nucleus pulposus and a thick surrounding anulus fibrosis. The nucleus pulposus loses fluid, volume, and resliency with age, increasing its susceptibility to compression [(A) and (C)]. Tears of the anulus fibrosus and degeneration of the posterior longitudinal igaments predispose to herniation of the nucleus pulposus [(B) and (D)]. Osteoporosis results in fractures of the vertebrae, a separate entity from disc disease (E) ‘Mark this question = Question 14 of 20 Question Id : 96165 A.42 year old woman presents with a history of white cyanotic appearing digits when she is exposed to cold whichis promptly relieved by heat with a resultant hyperemia, Ske has normal pulses and has chronic nail and skin changes. “Which of the following is NOT a treatment option for her? a) Heparin ») Limitation of cold exposure ¢) Nicotine restriction, ® Vasodlators 2) Regional sympathectomy. Question Explanation: Report An Error Heparin has no role in the management of this condition because itis due t> vasospastn, not embolism, Nicotine and cold are avoided because of their vascconstrictive effects. Vasodilators have been shown to be of some help in counteracting the vasoconstriction. Regional sympathectomy is also effective in controlling the vasospasm, ‘Mark this question e& => Question Td : 96165 Question 14 of 20 A.42 year old woman presents with a history of white cyanotic appearing digits when she is exposed to cold whichis promptly relieved by heat with a resultant hyperemia, Ske has normal pulses and has chronic nail and skin changes. “Which of the following is NOT a treatment option for her? JY © a) Heparin. ») Linitation of cold exposue. ¢) Nicotine restriction, ® Vasodlators 2) Regional sympathectomy. Question Explanation: Heparin has no role in the management of this condition because itis due t> vasospastn, not embolism, Nicotine and cold are avoided because of their vascconstrictive effects. Vasodilators have been shown to be of some help in counteracting the vasoconstriction. Regional sympathectomy is also effective in controlling the vasospasm, 20772014 11:14:32 PM ‘Maric this question eq => Antibody study of a patient Question 15 of 20 shown as folows [Ant-Centromere [High titer [Ant-Sol-70 [Not detected [antiPol-T [Not detected] [ant-REP [Not detected [ant-dsDNA [Not detected The disease most strongly suggested by the above antibody study is a) CREST ) Diffuse Scleroderma c) Mixed connective tissue disease 4) Sjogren syndrome €) Systemic lupus erythematosus Question Explanation: Localized scleroderma/CREST syndrome is an autoimmune disease characterized by Sbroblast stimulation and deposition of collagen in the skin and internal organs. Cinical feanires include involvement of the skin of the fece and hands late involvement of visceral organs, and a relatively benign course, Anti-centromere antibodies are a marker for the CREST (calcinosis, Raynaud phenomenon, esophageal dysfunction, sclerodactyly telangiectasias) variant of scleroderma, The markers for diffuse scleroderma ie., atti-Sol-70 and anti-Pol-T, are usually negative in CREST syndrome, as are most other autoantibodies ‘The marker for mixed connective tissue disease is anti-RINP. ‘The markers for Sjégren syndrome are anti-SSA, anti-SSB and anb-RAP. Systemic lupus erythematosus typically produces a large number of autoantibodies, of which anti-dsDNA and anti-Sm are the most distinctive. Question Td : 101683 20772014 11:14:32 PM ‘Maric this question eq => Antibody study of a patient Question 15 of 20 shown as folows [Ant-Centromere [High titer [Ant-Sol-70 [Not detected [antiPol-T [Not detected] [ant-REP [Not detected [ant-dsDNA [Not detected Tae disease most strongly suggested by the above antibody study is v © a) CREST ) Difise Scleroderma c) Mixed connective tissue disease 4) Sjogren syndrome €) Systemic lupus erythematosus Question Explanation: Localized scleroderma/CREST syndrome is an autoimmune disease characterized by Sbroblast stimulation and deposition of collagen in the skin and internal organs. Cinical feanires include involvement of the skin of the fece and hands late involvement of visceral organs, and a relatively benign course, Anti-centromere antibodies are a marker for the CREST (calcinosis, Raynaud phenomenon, esophageal dysfunction, sclerodactyly telangiectasias) variant of scleroderma, The markers for diffuse scleroderma ie., atti-Sol-70 and anti-Pol-T, are usually negative in CREST syndrome, as are most other autoantibodies ‘The marker for mixed connective tissue disease is anti-RINP. ‘The markers for Sjégren syndrome are anti-SSA, anti-SSB and anb-RAP. Systemic lupus erythematosus typically produces a large number of autoantibodies, of which anti-dsDNA and anti-Sm are the most distinctive. Question Td : 101683 2/27 2014 11:14:46 PM. ‘Mark this question & => Question 16 of 20 ‘Which testis most appropriate for the diagnosis of esteoauthrtis? a) Gram stan and culture of synovial uid. 'b) Examination of synovial fnid for presence of urate crystals. ©) Radiographic evaluation with clinical examination ) Semm mascle enzyme measurement. ©) Arteriogram. Anewor (UERNMIREN) Othercors Explanation Repost An Eros Question Explanation: Question Ta : 109478 ‘Because osteoarthritis is a degenerative disease, Gram-stain and examination of synovial fuid are useless. Diagnosis of osteoarthritis is based on clnical and radiologic findirgs. 2/27 2014 11:14:46 PM. ‘Mark this question & => Question 16 of 20 ‘Which testis most appropriate for the diagnosis of esteoauthrtis? a) Gram stan and culture of synovial uid. 'b) Examination of synovial fnid for presence of urate crystals. JY © ©) Radiographic evaluation with clinical examination. ) Semm mascle enzyme measurement. ©) Arteriogram. Anewor (UERNMIREN) Othercors Explanation Repost An Eros Question Explanation: Question Ta : 109478 ‘Because osteoarthritis is a degenerative disease, Gram-stain and examination of synovial fuid are useless. Diagnosis of osteoarthritis is based on clnical and radiologic findirgs. ‘Mark this question & => Question Td: 118512 Question 17 of 20 “Whhat is the most important aspect of the management of septic arthritis of the hip? a) Elevation. ) Immobilization c) IV antibiotics 4) Scrial aspiration ) Iimgation and debridement. Question Explanation ‘All are important parts of the management of septic arthntis, however, thorough irigation and debridement is the most unportant aspect of management, with the remainder of the choices being important adjuvants. ‘Mark this question & => Question Td: 118512 Question 17 of 20 ‘What is the most important aspect of the management of septic arthritis of the hip? a) Elevation b) Immobilization c) IV antibiotics. ) Serial aspiration. © @) Imgation and debndement. Question Explanation ‘All are important parts of the management of septic arthntis, however, thorough irigation and debridement is the most unportant aspect of management, with the remainder of the choices being important adjuvants. ‘Mark. tis question ==> Question Id: 126207 Question 18 of 20 A.35 year old male presents with 'sausage" digts involving the distal interphalangeal joints. Skin lesions are also present and his nails are pitted, The patient is most likely suffering from which one of the following? a) Reiter's syndrome ) Psoriatic arthritis ©) Arthritis associated with gastraintestinal defects ) Racumatoid arthritis 2) Osteoarthritis Answer [UEQSINSHRN) other User's Ee Question Explanation: The commen presentation of psoriatic arthritis is the presence of skin lesions prior to the development of arthritis, Psoriatic arthritis is considered a spondyloarthropathy that does not have a strong association with HLA-B27. Itis the only spondyloarthropathy that has a significant presentation of skia related problems ation Report An Error ‘Mark. tis question ==> Question Id: 126207 Question 18 of 20 A.35 year old male presents with 'sausage" digts involving the distal interphalangeal joints. Skin lesions are also present and his nails are pitted, The patient is most likely suffering from which one of the following? a) Reiter's syndrome ¥ © ’) Psoriatic arthritis ©) Arthritis associated with gastrointestinal defects d) Rheumatoid arthritis 2) Osteoarthritis Answer [UEQSINSHRN) other User's Ee Question Explanation: The commen presentation of psoriatic arthritis is the presence of skin lesions prior to the development of arthritis, Psoriatic arthritis is considered a spondyloarthropathy that does not have a strong association with HLA-B27. Itis the only spondyloarthropathy that has a significant presentation of skia related problems ation Report An Error ‘L unread) - emackomr= x \ @? misso64’s profile on Qh. CE www interface.edu.pk, ams /test-analysis ppt Apps [Ej Google settings [5 Signin Cl Imported FromiE 2. lobe dul © on go well Sys Paw WNIERTACE Pew wun EDUCATION FARINIER Pe eis Gi Other bookmar Mack this question = => Question Id : 150720 Question 19 of 20 A 34 year old single mother of two young children presenis with an oral ulcer. Systemic review is signiicant for fatigue, myalgia, and joint pain, Investigations reveal leucopenia and a high titer of antinuclear antibody. A speckled staining pattern due to anti-Sm is seen with immunofuorescence; urinary protein is elevated, Adcitional condition that is most likely to be found in this patient is a) Antibodies to Centromeres ) Antibodies to the glomenalar basement membrane ©) Antibodies to $S-A nbomucleoproteia 4) Complement-medated Iysis of blood vessels in the face €) Epstein-Barr virus infection Question Explanation: Systemic lupus erythematosus (SLE) is a prototype connective-tissue disease. The diagnosis requires that four of eleven possible criteria be met: malar rash, discoid rash, photosensitivity, oral ulcers, arthritis, serostis, renal disorder, neurologic disorder. hematologic disorder, immunologic disorder, and antinuclear antbody. This patient also has anti-Sm, which is pathognomonic for SLE but is found in only 20% of the affected patients. Antinuclear antibodies (ANIA) are present in 95 to 100% of SLE cases; anti- double stranded DNA is found in 70% of cates. The vasculiic and rash of SLE are associated with immmme complex: deposition in the small-diameter bloed vessels followed by complement mediated lysis Antbodies to Centromeres are found in 90% of patients with limited scleroderma er CREST (Calcinosis, Raynaud syadrome, esophageal dysmotilty, cleroderma, and telanglectasias). They would be found in less than 5% of patients with SLE. Antibodies to the glomenular basement membrane are found in Goodpasture syndrome. This causes the linear disposition of immunoglobulin, and often C3, along the glomemter basement metabrane. It results in glomenulonepbaitis, pulmonary hemorshage, and occasionally idiopathic pulmonary hemosiderosis, “Antibodies to SS-A ribonucleoproteins are found in Sjogren syndrome, which is characterized by keratoconjunctivitis sicca and xerostomia. The secondary form of this disease may be associated with rheumatoid arthritis, SLE, polymyostis, sclerodesma, vasculitis, mixed connective tissue disease, or thyroiditis. Antibodies to core proteins of smal! nuclear ribonucleoprotein particles (Gauith antigen) are found in 20 t> 30% of SLE patients, and antibodies against SS-A sibomucleoprotems are found in 30 to 50% of patients with SLE, Epstein-Barr virus infection has been suggested to be Inked to the development of chronic fatigue syndrome (CES). This contition, whichis of unknown etiology, involves severe mental and physical exhaustion thatis not selieved by rest and may be worsened by even trivial exertion. The symptoms must be present for atleast 6 months and must be idiopathic. Other signs that are nct included In all diganostic criteria may be found, including muscle weakness, cognitive dysfunction, hypersensitivity, orthostatic intolerance. digestive disturbances depression, poor immune response. and cardiac end respiratory problems, Since chronic faticue is not emphasized in thie vionette. and antimuclear antibodies were found here. CFS is not lileelw to he the djacnosis. Total Questions fs KO Keo FY nS fee BeeEEEEE xxKXKXKXXKXKXKXKXKXKKXKKXKKXKKXK ‘L unread) - emackomr= x \ @? misso64’s profile on Qh. CE www interface.edu.pk, ams /test-analysis ppt Apps [Ej Google settings [5 Signin Cl Imported FromiE 2. lobe dul © on go well Sys Paw WNIERTACE Pew wun EDUCATION FARINIER Pe eis Gi Other bookmar Mack this question = => Question Id : 150720 Question 19 of 20 A 34 year old single mother of two young children presenis with an oral ulcer. Systemic review is signiicant for fatigue, myalgia, and joint pain, Investigations reveal leucopenia and a high titer of antinuclear antibody. A speckled staining pattern due to anti-Sm is seen with immunofuorescence; urinary protein is elevated, Adcitional condition that is most likely to be found in this patient is a) Antibodies to Centromeres ) Antibodies to the glomenalar basement membrane ©) Antibodies to $S-A nbomucleoproteia Y © 4) Cotplemert-medated lysis of blood vessels in the face €) Epstein-Barr virus infection Question Explanation: Systemic lupus erythematosus (SLE) is a prototype connective-tissue disease. The diagnosis requires that four of eleven possible criteria be met: malar rash, discoid rash, photosensitivity, oral ulcers, arthritis, serostis, renal disorder, neurologic disorder. hematologic disorder, immunologic disorder, and antinuclear antbody. This patient also has anti-Sm, which is pathognomonic for SLE but is found in only 20% of the affected patients. Antinuclear antibodies (ANIA) are present in 95 to 100% of SLE cases; anti- double stranded DNA is found in 70% of cates. The vasculiic and rash of SLE are associated with immmme complex: deposition in the small-diameter bloed vessels followed by complement mediated lysis Antbodies to Centromeres are found in 90% of patients with limited scleroderma er CREST (Calcinosis, Raynaud syadrome, esophageal dysmotilty, cleroderma, and telanglectasias). They would be found in less than 5% of patients with SLE. Antibodies to the glomenular basement membrane are found in Goodpasture syndrome. This causes the linear disposition of immunoglobulin, and often C3, along the glomemter basement metabrane. It results in glomenulonepbaitis, pulmonary hemorshage, and occasionally idiopathic pulmonary hemosiderosis, “Antibodies to SS-A ribonucleoproteins are found in Sjogren syndrome, which is characterized by keratoconjunctivitis sicca and xerostomia. The secondary form of this disease may be associated with rheumatoid arthritis, SLE, polymyostis, sclerodesma, vasculitis, mixed connective tissue disease, or thyroiditis. Antibodies to core proteins of smal! nuclear ribonucleoprotein particles (Gauith antigen) are found in 20 t> 30% of SLE patients, and antibodies against SS-A sibomucleoprotems are found in 30 to 50% of patients with SLE, Epstein-Barr virus infection has been suggested to be Inked to the development of chronic fatigue syndrome (CES). This contition, whichis of unknown etiology, involves severe mental and physical exhaustion thatis not selieved by rest and may be worsened by even trivial exertion. The symptoms must be present for atleast 6 months and must be idiopathic. Other signs that are nct included In all diganostic criteria may be found, including muscle weakness, cognitive dysfunction, hypersensitivity, orthostatic intolerance. digestive disturbances depression, poor immune response. and cardiac end respiratory problems, Since chronic faticue is not emphasized in thie vionette. and antimuclear antibodies were found here. CFS is not lileelw to he the djacnosis. Total Questions fs KO Keo FY nS fee BeeEEEEE xxKXKXKXXKXKXKXKXKXKKXKKXKKXKKXK ‘ww interface ectupk/ analysis pnpautid=14759 e ops J coogle \ Settings [5 Signin Cl Imported FromiE 2. wba Qala © ou go cell aye Gi Other bookmar Mavk this question = Question Td : 164305 Question 20 of 20 A 52 year old man has generalized ‘stiffiess" lasting a few hours each moming, He has a recent 10 Ib weight loss and a past medical history of duodenal ulcers. On exam, his temperature is 38. 1°C (100.5°F), lymphadenopathy and mild splenomegaly. Subcutaneous “nodules” are palpable over selected bony prommences. Which would be recommended for the treatment his acute pain? a) Acetaminophen 1b) Celecoxib c) Indomethacin d) Methotrexate €) Sulfasalazine Question Explanatior Rheumatord arthritis (RA) is a chronic, multisystem, inflammatory disorder of unknown etology. The primary feature of RA is persistent inflammatory synovitis, symmetrically involving the peripheral joints. The synovial inflammation leads to cartilage destraction and bone erosions, which can cause substantial joint deformities In around 10 to 20% of patients the onset of LA is acute and accomparied by constitutional syraptoms such as fever, lymphadenopathy, splenomegaly, and weight lose, Subcutencous nodules, plewitis, pulmonary fibrosis, pericarditis, nerve entrapment syndromes, ocular changes, and vasculiis may also be seen, Moming stiffiess > 1 hour is one ofthe distinguishing factors ofinflammatory arthritis as compared to non-inflammatory arthritis. ‘The question is essentially asking for the most appropriate agent for the alleviation of the patient's acute signs and syrnptoms Celecoxib is a selective cyclooxygenase-2 (COK-2) inhibitor with antiinflammatory, analgesic, and antipyretic effects COX-1 is involved in the production of prostaglandins that protect the GI ining, whereas COX-2 is important for the synthesis of prostaglandins involved in inflammation and pain Therefore, a selective COX-2 inhibitor can be used for management of KA ina patient with a past history of ulcers. Please note that there has been controversy regarding COX-2 inhibitors because of associated adverse cardiac events. Currently, Celebrex (celecoxib) remains on the market, whereas Bextra (valdecoxib) and Vioscx (rofecoxib) have been withdcawn from the market. Acetaminophen has analgesic and antipyretic effects but not ant-inflammatory effects. ‘Therefore; this agent would not be indicated for treatment of a patient with RA. Indomethacin is a nonsteroidal anti-inflammetory ceug indicated for the treatment of acute pain caused by inflammation. Aithough indomethacin can be used in the treatment of RA, it would not be recommended in a patient with a past medical history of ulcers. Both methotrexate and sulfasalazine are used in the chronic management of RA as part of disease modifying antirheumatic drug (DMARD) therapy: however. these agents take several weeks to imonthe to elicit their beneficial effects. Therefore. these acente chould not be used in the acute management of rheumatoid arthritis. Total Questions BEER Mm Rm mB we xKXXKKKAKAAKKKKKRKKKKK ‘ww interface ectupk/ analysis pnpautid=14759 e ops J coogle \ Settings [5 Signin Cl Imported FromiE 2. wba Qala © ou go cell aye Gi Other bookmar Mavk this question = Question Td : 164305 Question 20 of 20 A 52 year old man has generalized ‘stffness" lasting a few hours each moming. He has a recent 10 lb weightloss and a past medical tistory of duodenal ulcers. On exam, his temperanure is 38. 1°C (100.5°E), lymphadenopathy and mild splenomegaly. Subcutaneous nodules" are palpable over selected bony promences. Which would be recommended for the treatment his acute pain? a) Acetaminophen Jo Ob) Celecoxib ) Indomethacin 4) Methotrexate €) Sulfasalazine Question Explanatior Rheumatord arthritis (RA) is a chronic, multisystem, inflammatory disorder of unknown etology. The primary feature of RA is persistent inflammatory synovitis, symmetrically involving the peripheral joints. The synovial inflammation leads to cartilage destraction and bone erosions, which can cause substantial joint deformities In around 10 to 20% of patients the onset of LA is acute and accomparied by constitutional syraptoms such as fever, lymphadenopathy, splenomegaly, and weight lose, Subcutencous nodules, plewitis, pulmonary fibrosis, pericarditis, nerve entrapment syndromes, ocular changes, and vasculiis may also be seen, Moming stiffiess > 1 hour is one ofthe distinguishing factors ofinflammatory arthritis as compared to non-inflammatory arthritis. ‘The question is essentially asking for the most appropriate agent for the alleviation of the patient's acute signs and syrnptoms Celecoxib is a selective cyclooxygenase-2 (COK-2) inhibitor with antiinflammatory, analgesic, and antipyretic effects COX-1 is involved in the production of prostaglandins that protect the GI ining, whereas COX-2 is important for the synthesis of prostaglandins involved in inflammation and pain Therefore, a selective COX-2 inhibitor can be used for management of KA ina patient with a past history of ulcers. Please note that there has been controversy regarding COX-2 inhibitors because of associated adverse cardiac events. Currently, Celebrex (celecoxib) remains on the market, whereas Bextra (valdecoxib) and Vioscx (rofecoxib) have been withdcawn from the market. Acetaminophen has analgesic and antipyretic effects but not ant-inflammatory effects. ‘Therefore; this agent would not be indicated for treatment of a patient with RA. Indomethacin is a nonsteroidal anti-inflammetory ceug indicated for the treatment of acute pain caused by inflammation. Aithough indomethacin can be used in the treatment of RA, it would not be recommended in a patient with a past medical history of ulcers. Both methotrexate and sulfasalazine are used in the chronic management of RA as part of disease modifying antirheumatic drug (DMARD) therapy: however. these agents take several weeks to imonthe to elicit their beneficial effects. Therefore. these acente chould not be used in the acute management of rheumatoid arthritis. Total Questions BEER Mm Rm mB we xKXXKKKAKAAKKKKKRKKKKK ‘Mark this question => (Question Id: 22651 Question 1 of 10 A 73-year-old female is diagnosed with giant call arteritis and i treated with Predrisolone 60 mg per day. What is the most appropriate treatment for the prevention of steroid induced osteoporosis? a) Alfacalcidol ) Caleiumn c) Raloxifene 4d) Tibolone e) Vitarnin D Question Explanation: The National Osteoporosis Sociery/RCP Guidelines were updated in 2002. Patients older than 65 years are considered at high risk of osteoporotic fractures secondary steroid induced osteoporosis. The algotithm for treatment can be found on the National Osteoporosis Society website, Daily intake 1,500mg of caloium and 800 of Vit D3 is recommended, Bone mass measurements at baseline and follow up measurement will guide future therapeutic decisions in patients on long term steroids. There is also evidence to support the use of Bisphosponates and Calcitonin in these patents. ‘Mark this question => (Question Id: 22651 Question 1 of 10 A73. year-old female ic diagnosed with giant cell arteritc and ic treated with Predricolone 60 mg per day. What is the most appropriate treatment for the prevention of steroid induced osteoporosis? ¥ © a) Alfacalcidol ) Calcium c) Raloxifene 4) Tiboloae ©) Vitamin D Question Explanation: The National Osteoporosis Sociery/RCP Guidelines were updated in 2002. Patients older than 65 years are considered at high risk of osteoporotic fractures secondary steroid induced osteoporosis. The algotithm for treatment can be found on the National Osteoporosis Society website, Daily intake 1,500mg of caloium and 800 of Vit D3 is recommended, Bone mass measurements at baseline and follow up measurement will guide future therapeutic decisions in patients on long term steroids. There is also evidence to support the use of Bisphosponates and Calcitonin in these patents. 2/28/2014 3:01:16 AM ‘Mark this question & => Question Td: 28152 Question 2 of 10 An 80-year-old female suffers a fracture neck of femur following a fall athome. Investigations are normal buther X-ray shows the bones to be rather ‘thin’. Its assumed that she is osteoporotic and she is started on alendronate therapy. Which of the following is correct concerning this drug? a) Facilitates vitamin D action on bone ) Inhibits osteoclast activity ©) Stinmulates absorption of calefum 4) Inhibits calcitonin ©) Increases the action of estrogen on bone Question Explanation: The bisphosphonetes of which alendronate is one. Increase Bone mineralization by intibiting osteoclastic activity. They have been demonstrated in numerous studies to reduce subsequent risk of fracture 2/28/2014 3:01:16 AM ‘Mark this question & => Question Td: 28152 Question 2 of 10 An 80-year-old female suffers a fracture neck of femur following a fall athome. Investigations are normal buther X-ray shows the bones to be rather ‘thin’. Its assumed that she is osteoporotic and she is started on alendronate therapy. Which of the following is correct concerning this drug? a) Facilitates vitamin D action on bone Y © ») Inhibits osteoclast activity 2) Stimulates absorption of calcium 4) Inhibits calcitonin ©) Increases the action of estrogen on bone Question Explanation: The bisphosphonetes of which alendronate is one. Increase Bone mineralization by intibiting osteoclastic activity. They have been demonstrated in numerous studies to reduce subsequent risk of fracture 228/201; AM ‘Mark this question ez Question Td : 28232 Question 3 of 10 “Which of the following regarding Infliximab is most true? a) Itis antibody to TNF-a +b) Is authorized for the treatment of severe ulberative colitis ©) Is apolyclonal antibody 4) It prevents relapse of Crohn's disease in patients who are in remission ¢) Must not be used in combination with methotrexate due to increased toxicity Answer | Beanation Other User's Explanation Report An Error Question Explanation: Inflizamab is a monoclonal entibody to tumour necrosis factor (TNE) alpha. It is licensed for the treatment of- 1. Severe active Crohn's discase refiactory to corticosteroid or immunosuppressant therapy and for refectory fistulas 2. Rheumatoid arthritis in adult whose response to disease modifying anti-rheumatic drugs is inadequate Before stariing therapy and through out treatment, patients should be evaluaied carefilly for tuberculosis as there have besn reports ofthe onset or reactivation of TB including miliary TB and some unusual extrapulmonary TB, Inflixmeb must be given concomitantly with methotrexate and requires iniravenous infusion in a hospital setting. Some other monoclonal antibodies in clinical use include 1. Digibind-digoxin-bincing antibody for treatment of overdoses (creases clearance) 2. Abcisimab: glycoprotein Ilb-Ia receptor (for unstable angina) 3, Pexelizunab: anti-C5 (complement) - antiinflammatory: reduces myocardial infarction and death following CABG. 228/201; AM ‘Mark this question ez Question Td : 28232 Question 3 of 10 “Which of the following regarding Infiximab is most trve? Y¥ © a) ltis antibody to TNE-c. 'b) Is authorized for the treatment of severe ulcerative colitis ) Is apolyclonal antibody 4) It prevents relapse of Crohn's disease in patients who are in remission ¢) Must not be used in combination with methotrexate due to increased toxicity Answer | Beanation Other User's Explanation Report An Error Question Explanation: Inflizamab is a monoclonal entibody to tumour necrosis factor (TNE) alpha. It is licensed for the treatment of- 1. Severe active Crohn's discase refiactory to corticosteroid or immunosuppressant therapy and for refectory fistulas 2. Rheumatoid arthritis in adult whose response to disease modifying anti-rheumatic drugs is inadequate Before stariing therapy and through out treatment, patients should be evaluaied carefilly for tuberculosis as there have besn reports ofthe onset or reactivation of TB including miliary TB and some unusual extrapulmonary TB, Inflixmeb must be given concomitantly with methotrexate and requires iniravenous infusion in a hospital setting. Some other monoclonal antibodies in clinical use include 1. Digibind-digoxin-bincing antibody for treatment of overdoses (creases clearance) 2. Abcisimab: glycoprotein Ilb-Ia receptor (for unstable angina) 3, Pexelizunab: anti-C5 (complement) - antiinflammatory: reduces myocardial infarction and death following CABG. 2/28/2014 3:01:42 AM ‘Mare this question & => Question Ta : 28352 Question 4 of 10 A 43-year-old had pain and stifiness in her shoulders and wrists for a week worse in the momings. There was synovitis of both wrists but no proximal muscle tendemess and weakness. ESR was 52 mame. Diagnose a) Rheumatoid arthritis +b) Polyrayalgia cheumatica @MR) ©) Polymyositis 4) Reactive arthritis ) Sjogren’s syndrome. Answer | Bxpianaton | Other User's Explanation Report An Error Question Explanation: Tn this middle aged female the acute bilateral arthritis of shoulders and wrists together with synovitis and raised ESR are highly suggestive of acute rheumatoid arthritis. Sjogren's syndrome will have more fincings along some specific antbodies not mentioned in the scenario. Weakness and myalgia would be expected with polymyositis. There is no prior precipitant to suggest a reactive arthritis and synovitis would be again unusual, PME would be less likely in this age group (PME usually occurs over 50 years of age) Proximal weakness in the moming with the gel phenomenon would be expected and synovitis in the wrists would be less likely in PMR, 2/28/2014 3:01:42 AM ‘Mare this question & => Question Ta : 28352 Question 4 of 10 4.43-year-old had pain and stiffness in her shoulders and wrists for a week worse in the momings. There was synovitis of both wrists ‘but no proximal muscle tendemess and weakness, ESR was 52 mm‘br. Diagnose Y © a) Rheumatcid arthritis 'b) Polymyalgia rheumatica PMR) c) Polymyositis 4) Reactive arthritis ©) Sjogren's syadrome Answer | Bxpianaton | Other User's Explanation Report An Error Question Explanation: Tn this middle aged female the acute bilateral arthritis of shoulders and wrists together with synovitis and raised ESR are highly suggestive of acute rheumatoid arthritis. Sjogren's syndrome will have more fincings along some specific antbodies not mentioned in the scenario. Weakness and myalgia would be expected with polymyositis. There is no prior precipitant to suggest a reactive arthritis and synovitis would be again unusual, PME would be less likely in this age group (PME usually occurs over 50 years of age) Proximal weakness in the moming with the gel phenomenon would be expected and synovitis in the wrists would be less likely in PMR, ‘Mare this question e& => Question Id : 54879 Question 5 of 10 Congenital Heart block in a Neonate given birth by a mother maybe due to which antibody in the mother’s serum? a) Anti-Smooth Muscle antibody b) Anti-endomysiel antibodiss c) Ani-SCL70 antibodies 4) Anti-Ro/SSA antibodies ©) Rheumatoid factor Answer | Explanation Other User's Explanation Report An Error Question Explanation: The majority of cases of congenital heart block: are due to the presence of anti-Ro/SSA antibodies in the maternal serum. The mother may have no evidence of a connective tissue disorder. The risks of congenital heart block in mothers with ani-Ro/SSA antibodies and congenital heart block is very strong The heart block is generally permanent (unlike other features of neonatal lupus) and insertion of a permanent pacemaker is frequentiy requred. ‘Mare this question e& => Question Id : 54879 Question 5 of 10 Congenital Heart block in a Neonate given birth by a mother maybe due to which antibody in the mother's serum? a) Anti-Smooth Muscle antibody b) Anti-endomysial antibodies c) Ani-SCL70 antibodies V © 4) Anti-Ro/SSA antibodies e) Rheumatoid factor Answer | Explanation Other User's Explanation Report An Error Question Explanation: The majority of cases of congenital heart block: are due to the presence of anti-Ro/SSA antibodies in the maternal serum. The mother may have no evidence of a connective tissue disorder. The risks of congenital heart block in mothers with ani-Ro/SSA antibodies and congenital heart block is very strong The heart block is generally permanent (unlike other features of neonatal lupus) and insertion of a permanent pacemaker is frequentiy requred. ‘Mari this question ez Question Id: $1238 Question 6 of 10 A.25 year old Turkish rman has had recurrent genital and oral ulcers for several years as well as waxing and waning joint pains. On examnination, he was found to have cutaneous vasculitis and inflammation in the posterior chamber of the eyes on slit-lamp exam, X- rays reveal erosive changes of the hip and knee joints. The most likely diagnosis is a) Sarcoidosis b) Reiter’s syndrome ©) Polymyositis 9) Polymyalgia sheumatica €) Behcet's disease Question Explanation: Behcet's disease is of unknown etiology and is common in Micdle Eastem and Turkish men. Ibis associated with HLA B51. Ibis a vasculitis of unknown etiology Meningoencephalitis, phlebitis, and pulmonary aneurysms are also seen in this disorder. Sarcoidosis (A) also presents with uveits, but genital and oral isolated ulcers are not common, Reiter’s syndrome (B) is associated with sexually transmitted diseases, It presents as atthnitis and urethnitis. Itis associated with HLA B27. Polymyositis (C) is iflammetion of the skeletal muscles. Elevation of creetinine kinase is common, Polymyalgia theumetica (D) is more common in the elderly population and genital rashes are not seen. ‘Mari this question ez Question Id: $1238 Question 6 of 10 A 25 year old Turkish man has had recurrent genital and oral uicers for several years as well as waxing and waning joint pains. On examination, he was found to Lave culaneous vasculitis and inflammation in the posterior chamber of the eyes on slit-lamp exam, X- rays reveal erosive changes of the hip and knee joints The most likely diagnosis is a) Sarcoidosis b) Reiter's syndrome €) Polymyostis <) Polymyalgia theumetica Y © ©) Behcet's disease Question Explanation: Behcet's disease is of unknown etiology and is common in Micdle Eastem and Turkish men. Ibis associated with HLA B51. Ibis a vasculitis of unknown etiology Meningoencephalitis, phlebitis, and pulmonary aneurysms are also seen in this disorder. Sarcoidosis (A) also presents with uveits, but genital and oral isolated ulcers are not common, Reiter’s syndrome (B) is associated with sexually transmitted diseases, It presents as atthnitis and urethnitis. Itis associated with HLA B27. Polymyositis (C) is iflammetion of the skeletal muscles. Elevation of creetinine kinase is common, Polymyalgia theumetica (D) is more common in the elderly population and genital rashes are not seen. 2/28/2014 3:02:21 AM ‘Marte this question & => Question 7 of 10 A 32 year old woman presents to a physician with nonspecific complaints of fatigue, malaise, low grade fever, and arthralgias Screening serum chemistries demonstrate elevated blood urea nitrogen (BUN). The urine is positive for protemuria. The most likely diagnosis would be established by testing of which one of the following? a) Anticentromeric antibody 'b) Anti-double stranded DIA antibody c) Antimitochondtial antbody 4) Anti-thyroid stimulating hormone (TSH) receptor antibody €) Rheumatoid tactor Answer | Exvianaton) Other Users Explanation Report An Error Question Explanation: Systemic lupus erythematosus should be considered in any 15 to 45 year old woman with chronic, poorly defined symptoms. Cues that help are malar rach, anti-double stranded DMA antibodies, and renal involvement: Antinuclear antibody is a good screening test and should be used first in this patient's workup. However, antinuclear antibody is a nonspecific serum marker. Renal involvement is not as common in most other autoimmane diseases, Anticentromeric antibody (choice A) is a marker for the CREST, form of scleroderma, Antimnitochondriel antibody (choice C) is a marker for prima bilary cicrhosis. “Anti TSH receptor antibody (choice D) is a marker for Graves’s disease. Rheumatoid factor (choice E) is a marker for rheumatoid arthritis, 2/28/2014 3:02:21 AM ‘Marte this question & => Question 7 of 10 A 32 year old woman presents to a physician with nonspecific complaints of fatigue, malaise, low grade fever, and arthralgias Screening serum chemistries demonstrate elevated blood urea nitrogen (BUN). The urine is positive for protemuria. The most likely diagnosis would be established by testing of which one of the following? a) Anticentromeric antibody Y © b) Anti-double stranded DA astibody c) Antimitochondtial antbody 4) Anti-thyroid stimulating hormone (TSH) receptor antibody €) Rheumatoid tactor Answer | Exvianaton) Other Users Explanation Report An Error Question Explanation: Systemic lupus erythematosus should be considered in any 15 to 45 year old woman with chronic, poorly defined symptoms. Cues that help are malar rach, anti-double stranded DMA antibodies, and renal involvement: Antinuclear antibody is a good screening test and should be used first in this patient's workup. However, antinuclear antibody is a nonspecific serum marker. Renal involvement is not as common in most other autoimmane diseases, Anticentromeric antibody (choice A) is a marker for the CREST, form of scleroderma, Antimnitochondriel antibody (choice C) is a marker for prima bilary cicrhosis. “Anti TSH receptor antibody (choice D) is a marker for Graves’s disease. Rheumatoid factor (choice E) is a marker for rheumatoid arthritis, 2/28/2014 3:02:37 AM ‘Mark this question & => Question 8 of 10 “Which one of the following s the most common site of gout attacks? a) Previously traumatized joints b) The hip. c) The lumbosacral spine. 4) Middle-aged women. +) The knee. Answer | Bavianaton | Other User's Explanation Report An Error Question Explanation: Question Id: 89609 Previously traumatized joints arc fequently the site of gout and other inflammatory conditions, such as septic arthritis. Gout is not particularly common in hips, lumbosacral spine, or knees, unless previously injured, or in middle aged women. 2/28/2014 3:02:37 AM ‘Mark this question & => Question 8 of 10 “Which one of the followings the most common site of gout attacks? Y © a) Previously traumatized joints b) The hip. c) The lumbosacral spine. 4) Middle-aged women. +) The knee. Answer | Bavianaton | Other User's Explanation Report An Error Question Explanation: Question Id: 89609 Previously traumatized joints arc fequently the site of gout and other inflammatory conditions, such as septic arthritis. Gout is not particularly common in hips, lumbosacral spine, or knees, unless previously injured, or in middle aged women. 2/28/2014 3:02:54 AM ‘Mark this question = => Question Td: 104892 Question 9 of 10 ‘Which of the following is NOT usef in the management of Paget's disease? a) Salmon calcitonin. ) Human calcitonin. ©) Fitidrinate disodium 4) Pamidronate disodiam. 8) Naproxen sodium. Question Explanation: ‘Naproxen sodium is an NSATD that normally does not play a role in the treatment of Paget's disease. The bisphosphonates work by inhibiting bone tamover, while calcitonin has the same effect by inhibiting osteoclastic bone resorption. Pamidronate is the newest of the bisphosphonates, has a decreased side effect profile, and can be taken orally. Calcitonin must be given subcutaneously. fn Report An Error 2/28/2014 3:02:54 AM ‘Mark this question = => Question Td: 104892 Question 9 of 10 ‘Which of the following is NOT usef in the management of Paget's disease? a) Salmon calcitonin. ) Human calcitonin. ©) Fitidrinate disodium 4) Pamidronate disodiam. Y © ¢) Naproxen sodium. Question Explanation: ‘Naproxen sodium is an NSATD that normally does not play a role in the treatment of Paget's disease. The bisphosphonates work by inhibiting bone tamover, while calcitonin has the same effect by inhibiting osteoclastic bone resorption. Pamidronate is the newest of the bisphosphonates, has a decreased side effect profile, and can be taken orally. Calcitonin must be given subcutaneously. fn Report An Error ‘Mark this question <= Question Id : 124775 Question 10 of 10 ‘Which of the following is a manifestation of rheumatoid arthritis? a) Bamboo spine 6) Chondrocaleinosis ¢) DIP sclerosis 4) Juxta-articular demineralization ¢) Pencil-in-cup erosions Answer | Bxplanation Other User's Explanation Report An Error Question Explanation: Rheumatoid arthritis-juxta-aticular demineralization-this is often the earliest radiographic manifestation of rheumatoid arthritis, presumably due to peri-articular effects of prostaglandins secreted by the inflamed synovial tissue ‘Mark this question <= Question Id : 124775 Question 10 of 10 “Which of the following is a manifestation of rheumatoid artaritis? a) Bamboo spine b) Chondrocalcinosis ) DIP sclerosis Y © 4) Justa-articular demineralization ¢) Pencil-in-cup erosions Answer | Bxplanation Other User's Explanation Report An Error Question Explanation: Rheumatoid arthritis-juxta-aticular demineralization-this is often the earliest radiographic manifestation of rheumatoid arthritis, presumably due to peri-articular effects of prostaglandins secreted by the inflamed synovial tissue ‘Mark this question => Question Td: 19514 Question 1 of 20 An 86-year-old woman presented with bileteral esteoauthrts of the knees. She had no history of previous gastrointestinal diseace. ‘Which of the following is the most appropriate initial treatment for let? a) Paracetamol ») Celecoxib «) Ibuprofen @) Ditydrocodeine 2) Topical Diclofenac 1) Oral Steroids Question Explanation: The recommendations of the American College of Rheumatology published in Arthritis and Rheumatism 2000, recommend. acetaminophen (paracetamol) together with non-pharmacolagical interventions (exercise, diet) as fist ne therapy of mildimoderate OA of hips or knees. For mild to moderate symptoms, its effectiveness is simlar to NSAIDs, though for more severe symptoms NSAIDs may be more effective, Non-steroidal antiinflammatory dmge (NSAID) such as ibuprofen while more effective in severe cases are associated with greater side effects such as gestrointestinal bleeding Another class of NSAIDs, COX-2 selective inhibitors (such as celecoxib) are equally effective to NSAIDs but no sefer in terms of side effects. They are however much more expensive There are several NSAIDs available for topical use including diclofenac. They have fewer systemic side-effects and at least some therapeutic effect, While opioid analgesic such as morphine and fentanyl improve pain this benef is outweighed by Gequent adverse events and shus they should not routinely be used. Oral steroids are not recommended in the treatment of OA because of their modest benefit and high rate of adverse effects ‘Mark this question => Question Td: 19514 Question 1 of 20 An 86-year-old woman presented with bilateral osteoarthritis of the knees. She had no history of previous gastrointestinal disease. ‘Which of the following is the most appropriate initial treatment for her? Y © a) Paracetamol ») Celecoxib 2) Ibuprofen d) Dihydrocedeine 2) Topical Diclofenac 1) Oral Steroids Question Explanation: The recommendations of the American College of Rheumatology published in Arthritis and Rheumatism 2000, recommend. acetaminophen (paracetamol) together with non-pharmacolagical interventions (exercise, diet) as fist ne therapy of mildimoderate OA of hips or knees. For mild to moderate symptoms, its effectiveness is simlar to NSAIDs, though for more severe symptoms NSAIDs may be more effective, Non-steroidal antiinflammatory dmge (NSAID) such as ibuprofen while more effective in severe cases are associated with greater side effects such as gestrointestinal bleeding Another class of NSAIDs, COX-2 selective inhibitors (such as celecoxib) are equally effective to NSAIDs but no sefer in terms of side effects. They are however much more expensive There are several NSAIDs available for topical use including diclofenac. They have fewer systemic side-effects and at least some therapeutic effect, While opioid analgesic such as morphine and fentanyl improve pain this benef is outweighed by Gequent adverse events and shus they should not routinely be used. Oral steroids are not recommended in the treatment of OA because of their modest benefit and high rate of adverse effects ‘Mark this question => Question Id : 28162 Question 2 of 20 AAl-year-old hes Raynaud's phenomenon, dyspepsia and arthralgias. along sclerodactyly and synovitis ofthe small joints of the hhands. ESR =41 mmfhr (10) but Rf and ANA are both negative. Which of the following is most likely to develop? a) Malabsorption ) Anterior uveitis c) Butterfly rash ) Erosive joint disease ©) Erythema nodosum Question Explanation: This woman has features of a mized connective tisme disorder lie CRESTisystemic sclerosis with scleradactyly, Raynaud’, dyspepria and arthralgia. The absence of ANA found in 90% of systemic sclerosis malces this diagnosis less lcely and these antibodies plus Anti-centromere antibodies arc alse associated with CREST. The mest likely development would be a malabsorption which is associated with hypomotiliy of the small intestine. Erosive anhropathy is sare as is uveitis, with Keratoconjunctivitis sicca ‘being more common, ‘Mark this question => Question Id : 28162 Question 2 of 20 AAl-year-old hes Raynaud's phenomenon, dyspepsia and arthralgias. along sclerodactyly and synovitis ofthe small joints of the hhands. ESR =41 mmfhr (<10) but Rf and ANA are both negative. Which of the following is most likely to develop? Y © a) Malabsorption 'b) Anterior uveitis c) Butterfly rash ) Erosive joint disease ¢) Erythema nodosum Question Explanation: This woman has features of a mized connective tisme disorder lie CRESTisystemic sclerosis with scleradactyly, Raynaud’, dyspepria and arthralgia. The absence of ANA found in 90% of systemic sclerosis malces this diagnosis less lcely and these antibodies plus Anti-centromere antibodies arc alse associated with CREST. The mest likely development would be a malabsorption which is associated with hypomotiliy of the small intestine. Erosive anhropathy is sare as is uveitis, with Keratoconjunctivitis sicca ‘being more common, ‘Mark this question = cp» Question3 of 20 A previously well, 63-year-old hypertensive builder presents with pain, redness and swelling in the right knee, which started 12 hours ago. There is 2 family history of hypertension and joint problems. ‘What investigation is most important? a) C-Reactive protein b) ILA statas c) Radiology 4) Joint aspiration for microscopy and culture €) Serology Question Explanation: ‘This patent hes an acute monoarthropathy with pain sweling and erythema of single joint, this maybe arthritis and he aeeds joint aspiration for microscopy and culture to identify any infective organism so appropriate therapy caa be guided. X-ray is of a0 value in septic arthntis it only becomes abnormal following joint destruction n Report An Error ‘Mark this question = cp» Question3 of 20 A previously well, 63-year-old hypertensive builder presents with pain, redness and swelling in the right knee, which started 12 hours ago. There is 2 family history of hypertension and joint problems. ‘What investigation is most important? a) C-Reactive protein b) ILA statas ) Radiology Y © @) Joint aspiration for microscopy and culture 2) Serology Question Explanation: ‘This patent hes an acute monoarthropathy with pain sweling and erythema of single joint, this maybe arthritis and he aeeds joint aspiration for microscopy and culture to identify any infective organism so appropriate therapy caa be guided. X-ray is of a0 value in septic arthntis it only becomes abnormal following joint destruction ‘Mark this question = => Question Id: 28252 Question 4 of 20 ‘Which of the following statements is true immunology of theumatoid arthritis? a) Ibis an example of an organ-specific disease 'b) It is likely that joint specific antigens have been sequestered during the time when immunological tolerance was being established, c) Joint damage is the consequence of mast cell degranulation 4) RF is an IgM autoantibody that has specificity for the Fe portion of IgG ¢) Rheumatoid factar is detected by a test utilizing the patient's B lymphocytes Answer | Explanation) Other User's Explanation Report An Error Question Explanation: Rheumatoid arthvits is associated with several antibodies such as theumatoid factor, collagen antivody, capable of reaction at sites other than the joints Additionally, the disease is not confined to the joints Damage is mediated by several means, including macrophages activated by CD4+T cells, and by complement fixing immune complexes There is no evidence for the creation of joint-specific antibodies in development, All the components of the joint are present during feral life ‘The rheumatoid factor test utlizes the patient’s serum, to agglutinate cells coated with antibody. Rheumatoid factor (RF) is an antibody whose specificity is directed to a domain situated within the Fc portion of IgG. The rheumatoid factor may be of IgM, IgA class. ‘The conventional (agglutination) test, detects only IgM RE. ‘Mark this question = => Question Id: 28252 Question 4 of 20 ‘Which of the following statements is true immunology of theumatoid arthritis? a) Ibis an example of an organ-specific disease 'b) It is likely that joint specific antigens have been sequestered during the time when immunological tolerance was being established, c) Joint damage is the consequence of mast cell degranulation Y © @)RF is an IgM autoantibody that has specificity for the Fe portion of IgG ¢) Rheumatoid factar is detected by a test utilizing the patient's B lymphocytes Answer | Explanation) Other User's Explanation Report An Error Question Explanation: Rheumatoid arthvits is associated with several antibodies such as theumatoid factor, collagen antivody, capable of reaction at sites other than the joints Additionally, the disease is not confined to the joints Damage is mediated by several means, including macrophages activated by CD4+T cells, and by complement fixing immune complexes There is no evidence for the creation of joint-specific antibodies in development, All the components of the joint are present during feral life ‘The rheumatoid factor test utlizes the patient’s serum, to agglutinate cells coated with antibody. Rheumatoid factor (RF) is an antibody whose specificity is directed to a domain situated within the Fc portion of IgG. The rheumatoid factor may be of IgM, IgA class. ‘The conventional (agglutination) test, detects only IgM RE. 2/28/2014 3:07:41 AM ‘Mark this question = => Question Td : 30451 Question 5 of 20 A. women of 52 years of age has a history of arthitis and eweling for 4 months. On exemination, she has a symmettical inflammetion, ‘Movements ofhands and feet are painfil and swelling is preseat on both knees, suggesting a diagnosis of rheumetoid arthrts, Regarding her joint disease, which of the following suggest an adverse prognosis? a) Acuteness of presentation b) Elevated C-reactive protein c) Enthesttis 4) Astioular erosions on X-ray ¢) Sero-negative for Rheumatoid Factor Answer | Beplanation Other User's Explanation Report An Error Question Explanation: Articalar erosins in rheumatoid arthritis occurring early on in the course of the disease, especially within the first € months of presentation, indicate a poor prognosis. Over tims joint damage will relate to disabilty. A positive rheumatoid factor is associated with more severe erosive disease, extra-articular manifestations including subcutaneous nodules and increased mortality. An acute onset of presentation is not a poor prognostic factor. Raised inflammatory markers (CRP, ESR) and the daration of the early moming stiffness both correlate with disease activity, 2/28/2014 3:07:41 AM ‘Mark this question = => Question Td : 30451 Question 5 of 20 A. women of 52 years of age has a history of arthitis and eweling for 4 months. On exemination, she has a symmettical inflammetion, ‘Movements ofhands and feet are painfil and swelling is preseat on both knees, suggesting a diagnosis of rheumetoid arthrts, Regarding her joint disease, which of the following suggest an adverse prognosis? a) Acuteness of presentation b) Elevated C-reactive protein c) Enthesttis Y © 4) Articular erosions on X-ray ¢) Sero-negative for Rheumatoid Factor Answer | Beplanation Other User's Explanation Report An Error Question Explanation: Articalar erosins in rheumatoid arthritis occurring early on in the course of the disease, especially within the first € months of presentation, indicate a poor prognosis. Over tims joint damage will relate to disabilty. A positive rheumatoid factor is associated with more severe erosive disease, extra-articular manifestations including subcutaneous nodules and increased mortality. An acute onset of presentation is not a poor prognostic factor. Raised inflammatory markers (CRP, ESR) and the daration of the early moming stiffness both correlate with disease activity, ‘Mari this question <= >> Question 1d: 30471 Question 6 of 20 A.37 year old women has a history of joint pain and stiffness of hands and feet for six months, Synovitis of distal interphalangeal joints fle index finger and right ring finger along with right wrist and ankle joints is present on examination, Her ESR wes 35 mmuihr (0- 10). This pattern of joint involvement is most lely to be present in which condition? a) Osteoarthritis b) Reactive arthritis c) Rheumatoid arthritis 4) Psoriatic arthritis €) Systemic lupus erythematosus Question Explanation: This woman is most likely to have psoriatic arthriis Psoriatic arthrtis has been subclassiied according to different pattems of arthritis © Asymmetrical oligoertheitis © Symmetric polyarthritis Spondgloarthropathy Artluitis mutlans Tn about 20% of patients there is a chronic, progressive and deforming arthropathy with an asyrrmetrical pattern, including distal interphalangeal joint involvement. Osteoarthritis in this age group is unlikely. Rheumatoid arthritis is a symmetrical erthntis typically affecting the metacarpophalangeal joints bilaterally. Arthrits does occur in systemic lupus erthematosus, however there are several other clinical feebures that form part ofthe diagnostic criteria. Viral arthritis s self-limting ‘Mari this question <= >> Question 1d: 30471 Question 6 of 20 A.37 year old woman has a history of joint pain and stiffness of hands and feet for six months, Synovitis of distal interphalangeal joints oflleft index finger and right ring finger along with right vist and ankle joints is present on examination, Her ESR was 35 mmnir (0- 10). This pattern of join: involvement is most ikely to be present in which condition? a) Osteoarthritis b) Reactive arthritis ©) Rheumateid arthritis Y © 4) Psoriatic arthritis ¢) Systemic lupus erythematosus Question Explanation: This woman is most likely to have psoriatic arthriis Psoriatic arthrtis has been subclassiied according to different pattems of arthritis © Asymmetrical oligoertheitis © Symmetric polyarthritis Spondgloarthropathy Artluitis mutlans Tn about 20% of patients there is a chronic, progressive and deforming arthropathy with an asyrrmetrical pattern, including distal interphalangeal joint involvement. Osteoarthritis in this age group is unlikely. Rheumatoid arthritis is a symmetrical erthntis typically affecting the metacarpophalangeal joints bilaterally. Arthrits does occur in systemic lupus erthematosus, however there are several other clinical feebures that form part ofthe diagnostic criteria. Viral arthritis s self-limting ‘Mare this question & => (Question Id: 30481 Question 7 of 20 Arman aged 25 years has a three months history of arthralgia, mouth ulceration and eye irritation On examination he did not have fever but had some ulceration of the mouth, bilaterally swollen wnists and effusions, with reduced range of movements of both knees. Scrotal uloer was preseat on external genitlia, His investigations showed ‘White cell count 12x 1091 G-11 x 109) C-reactive protein 120mg (<10) Rheumatoid factor Negative “What is the most likely diagnosis? 2) Inflammatory bewel disease 'b) Psoriatic arthitis c) Behcet's syndrome 4) Reiter's syndrome ©) Sjogren's syndrome Answer (Biotanation)) Other User's Explanation Report An Error Question Explanation: This man has Behcet's on the basis of his crogenital ulcsration and oligoarihritis Behcet's syncvome is a multisystem disorder characterized by, + Recurrent oral and genital ulceration + Bye lesions (anterior or posterior uveitis or retinal vasculitis) + Skin lesions Erythema nodosum, papulopustular lesions or folliculitis) and © A positive pathergy test. Other features include musuloskeletal involvement with a mono- or oligoarthropathy, venous thromboembolism, neurologival and gastrointestinal features, Reiter's syndrome is a clinical iad of urethritis, conjanctivitis and arthritis after infective dysentery. Genital ulceration is not a feature of systemic lupus erthematosus, sheumatoid artiuitis or Sjogren’s syndrome, ‘Mare this question & => (Question Id: 30481 Question 7 of 20 Arman aged 25 years has a three months history of arthralgia, mouth ulceration and eye irritation On examination he did not have fever but had some ulceration of the mouth, bilaterally swollen wnists and effusions, with reduced range of movements of both knees. Scrotal uloer was preseat on external genitlia, His investigations showed ‘White cell count 12x 1091-11 x 109) C-reactive protein 120mg (<10) Eheumatoid factor Negative “What is the most lcely diagnosis? a) Inflammatory bowel disease b) Psoriatic arthritis Y © c) Behcet's syndrome 4) Reiter's syndrome 2) Sjogren's syndrome Answer (Biotanation)) Other User's Explanation Report An Error Question Explanation: This man has Behcet's on the basis of his crogenital ulcsration and oligoarihritis Behcet's syncvome is a multisystem disorder characterized by, + Recurrent oral and genital ulceration + Bye lesions (anterior or posterior uveitis or retinal vasculitis) + Skin lesions Erythema nodosum, papulopustular lesions or folliculitis) and © A positive pathergy test. Other features include musuloskeletal involvement with a mono- or oligoarthropathy, venous thromboembolism, neurologival and gastrointestinal features, Reiter's syndrome is a clinical iad of urethritis, conjanctivitis and arthritis after infective dysentery. Genital ulceration is not a feature of systemic lupus erthematosus, sheumatoid artiuitis or Sjogren’s syndrome, € CD www interface edu pk /medical-oxams/test-analysis phputid=14759 Apps [J coogle A settings [signin Cl imported rromie go. wlohe Cutan oo ceuill Bye 2/28/2014 3:08:32 AM mmad.omnran= Yahoo Ni |) muslim matimenial |) 9S wwwanterfare.eckupkr Ww G other bookmar ‘Mark this question & => (Question Td : 30525 Question 8 of 20 A male aged 52 years has a six-week history of general malaise and a2 day history of a tight foot drop, left ulnar nerve palsy and a widespread purpuric resh. He also complained of arthralgia but no clinical evidence offinflammatory joint disease was presert. Investigations showed ESR 100 memstr (0-20) ANCA Negative ANA Megative ‘Rheumatoid factor strongly positive 3 08 gL (075-16) 4 0.02 afl (0.14-0.9 Urine dipstick Blood+ +, No protein An echocardiogram was normal and two sets ofblood cultures were negative, What is the most likely diagnosis? a) ANA negative SLE ) Polyarteritis nodosa ) Infective endocarditis 4) Cryoglobulinaesnia ¢) Rheumatoid arthritis Anower (UBQIENEAR) othe: Users Expon Question Explanation: The history is strongly suggestive of systemic vascultis with mononeuritis mubiplex, purpusic rash and haematuria, lis important to exclude condtioas which can mimic vasculitis such as infective endocarditis. The normal echocardiogram and negative blood cultures make this unlikely. Whist polyartertis nodosa can present with exactly this clinical picture, the marked consumption ofC4 together with a strongly positive rheumatcid factor strongly suggest cryoglobulirs as the underlying cause, Cryoglobulins are immunoglobulins which precipitate in the cold, They can be type I (monoclonal, type II (mized monoclonal and polyclonal) or type III (polycional) ‘Type I eryoglobulinaemia is associated wih haematological ciseases such as myeloma and Waldenstrom’s Type II and tissue disorders, chronic infections and most important, hepatitis C infection which should always be exchided Treatment of lheekniolwinastia aoa scinde= fiaemanhorecit tah Aace etercide and tor isehacibamile: Report An Error Total Questions REREEM eR eee BRBE EE xxXxxXKXKXKXKXKXKKKKXKKKKKKK € CD www interface edu pk /medical-oxams/test-analysis phputid=14759 Apps [J coogle A settings [signin Cl imported rromie go. wlohe Cutan oo ceuill Bye 2/28/2014 3:08:32 AM mmad.omnran= Yahoo Ni |) muslim matimenial |) 9S wwwanterfare.eckupkr Ww G other bookmar ‘Mark this question & => (Question Td : 30525 Question 8 of 20 A male aged 52 years has a six-week history of general malaise and a2 day history of a tight foot drop, left ulnar nerve palsy and a widespread purpuric resh. He also complained of arthralgia but no clinical evidence offinflammatory joint disease was presert. Investigations showed ESR 100 memstr (0-20) ANCA Negative ANA Megative ‘Rheumatoid factor strongly positive 3 08 gL (075-16) 4 0.02 afl (0.14-0.9 Urine dipstick Blood+ +, No protein An echocardiogram was normal and two sets ofblood cultures were negative, What is the most likely diagnosis? a) ANA negative SLE ) Polyarteritis nodosa ) Infective endocarditis Y © d) Cryoglobulinaemia ¢) Rheumatoid arthritis Anower (UBQIENEAR) othe: Users Expon Question Explanation: The history is strongly suggestive of systemic vascultis with mononeuritis mubiplex, purpusic rash and haematuria, lis important to exclude condtioas which can mimic vasculitis such as infective endocarditis. The normal echocardiogram and negative blood cultures make this unlikely. Whist polyartertis nodosa can present with exactly this clinical picture, the marked consumption ofC4 together with a strongly positive rheumatcid factor strongly suggest cryoglobulirs as the underlying cause, Cryoglobulins are immunoglobulins which precipitate in the cold, They can be type I (monoclonal, type II (mized monoclonal and polyclonal) or type III (polycional) ‘Type I eryoglobulinaemia is associated wih haematological ciseases such as myeloma and Waldenstrom’s Type II and tissue disorders, chronic infections and most important, hepatitis C infection which should always be exchided Treatment of lheekniolwinastia aoa scinde= fiaemanhorecit tah Aace etercide and tor isehacibamile: Report An Error Total Questions REREEM eR eee BRBE EE xxXxxXKXKXKXKXKXKKKKXKKKKKKK ‘Mark this question e& => Question Ta : $5033 Question 9 of 20 Aa 86-year-old patient with recurrent falls has a fracture of the distal ulna Which of the following statements is/are correct? a) Bone fiactures attributable to vitamin D deficiency are due to bone density b) Low Vitamin D levels are not associated with muscle weakness ©) The toxic levels of vitamin D occur at approximately twice the therapeutic dose 4) Vitamin D deficiency in the elderly is rare 2) Vitemin D replacement reduces the incidlence of fractures in the elderly Question Explanation: ‘The che is in the question! Vitamin D deficiency in the elderly is common, especially in the housebound and those in residential and narsing homes, wath an overall frequency of 17% in women aged 85 and over, Vitamin D deficiency is associated with muscle wealeness, as well as Vitamin D deficiency are often not attributable to reduced bone density. Vitamins D replacement (800 TU daily) with calcium has been shown to reduce falls and fractures by 47% compared with controls who received calcium only. The comect dose of vitamin D replacement should be 800 TU. Lowest doses at which adverse events have been observed with ‘Vitamin D replacement are 200nmol which equates to a daily dose of 40,000 TU. ‘Mark this question e& => Question Ta : $5033 Question 9 of 20 Aa 86-year-old patient with recurrent falls has a fracture of the distal ulna Which of the following statements is/are correct? a) Bone fiactures attributable to vitamin D deciency are due to bone density. b) Low Vitamin D levels are not associated with muscle weakness ©) The toxic levels of vitamin D occur at approximately twice the therapeutic dose 4) Vitamin D deficiency in the elderiy is rare Y © ®) Vitamin D replacement reduces the incidence of fractures in the elderly Question Explanation: ‘The che is in the question! Vitamin D deficiency in the elderly is common, especially in the housebound and those in residential and narsing homes, wath an overall frequency of 17% in women aged 85 and over, Vitamin D deficiency is associated with muscle wealeness, as well as Vitamin D deficiency are often not attributable to reduced bone density. Vitamins D replacement (800 TU daily) with calcium has been shown to reduce falls and fractures by 47% compared with controls who received calcium only. The comect dose of vitamin D replacement should be 800 TU. Lowest doses at which adverse events have been observed with ‘Vitamin D replacement are 200nmol which equates to a daily dose of 40,000 TU. ‘Mark this question ez (Question Id : 59451 Question 10 of 20 4.66 years old hypertensive female has bea treated with hydrochlorothiazide, 25 mg/day, etenolol (Tenormin), 100 mg/day, and Hydralazine (Apresoline), 50 mg 4 timesiday for several years. Her blood pressure has been well controled on this regimen, She has experienced malaise, along with difuse joint pains that involve symmetric sites in the fingers, hands, elbows, and knees over the past 3 months. A pleural fiction rub is noted on examination Laboratory testing shows that the patient has mild anemia and leucopenia, with a negative rheumatoid factor and 2 positive antinuclear antibody (ANA) titer of 1:640, What is the most appropriate initial treatment? a) Discontinue the Hy¢ralazine +) Discontinue the thiazide diuretic and switch to a loop diaretic such as furosemide (Lasix) c) Begn treatment with prednisone, 40 mg/day orally @) Treat with hydroxychloroquine Plaquenil), 400 mg/day *) Obtain renal finction studies and anticipate that a renal biopsy will be needed Answer (Botaraion Other User's Explanation Report An Error Question Explanation: There are many drugs that can indace a syndrome resembling systemic lupus erythematosus, but the most common offender is procainamide, followed by hydralazine. There is a genetic predisposition for this dag induced lupus, determined by drug acetylation rates, Polyerthrtis and pleuropsricarditis occur in half of patents, but fortunately, CNS and renal Involvement are rare. While all patients with this condition have positive ANAs and mos have antibodies to histones, antibodies to double stranded DNA and decreased complement levels are rare, which distinguishes crug induced lupus froma idiopathic lupus. The best initial management is to withdraw the drug, and most patients improve in a few weeks. For those with severe symptoms, a short course of corticosteroids is indicated. Once the offending drug is discontinued, symptoms seldorn last beyond 5 months ‘Mark this question ez (Question Id : 59451 Question 10 of 20 A. 66 years old hypertensive female has been treated with hydrochlorothiazide, 25 mg/day; atenolol (Tenormin), 100 mg/day, and Hydralasine (Apresolne), 5) mg 4 timesiday for several years, Her blood pressure hes been well controlled on this regimen, She has experienced malaise, along with diffice joint pans that involve symmetric sites in the fingers, hards, elbows, and knees over the past 3 months. A pleural friction mu is noted on examnation Laboratory testing shows that the patent has mild anemia and leucopenia, with a negative rheumatoid factor and a positive antinuclear antibody (ANA) titer of 1:640. What is the most appropriate intial treatment? Y © a) Discontinue the Hycralazine >) Discontinue the thiazide diuretic and switch to a loop diaretic such as furosemide (Lasix) c) Begn treatment with prednisone, 40 mg/day orally 4) Treat with hydroxychloroqnine (Plaquenil), 400 mg/day 2) Obtain reral finction studies and anticipate that a renal biopsy will be needed Answer (Botaraion Other User's Explanation Report An Error Question Explanation: There are many drugs that can indace a syndrome resembling systemic lupus erythematosus, but the most common offender is procainamide, followed by hydralazine. There is a genetic predisposition for this dag induced lupus, determined by drug acetylation rates, Polyerthrtis and pleuropsricarditis occur in half of patents, but fortunately, CNS and renal Involvement are rare. While all patients with this condition have positive ANAs and mos have antibodies to histones, antibodies to double stranded DNA and decreased complement levels are rare, which distinguishes crug induced lupus froma idiopathic lupus. The best initial management is to withdraw the drug, and most patients improve in a few weeks. For those with severe symptoms, a short course of corticosteroids is indicated. Once the offending drug is discontinued, symptoms seldorn last beyond 5 months ‘Mari this question => Question Id: 64602 Question 11 of 20 AT? year old Asian woman comes to your office with a history of the gradual onset of localized low back pain for 6 weeks. The pain is increased with walking or standing and relives by siting ard lying, Pusting a shopping cart dramatically relieves the pain. Which one ofthe following is the most ikkely diagnosis? a) Spinal cord tumor b) Epidural abscess ©) Osteoporotic vertebral fracture 9) Sciatica €) Spinal stenosis Avewor [UEQNIIRGAN) other ucors Explanation Report An Ever (Question Explanation: Spinal stenosis pain is often referred to as pseudoclaudication because the pain is worsened by walking Any movement that flexes the spine, such as pushing a shopping cart, relieves the pain, Pain from tumor of infection is persistent. Osteoporotic vertebral fractures have a sudden onset, Sciatica does not have the typical findings of spinal stenosss. ‘Mari this question => Question Id: 64602 Question 11 of 20 AT? year old Asian woman comes to your office with a history of the gradual onset of localized low back pain for 6 weeks. The pain is increased with walking or standing and relives by siting ard lying, Pusting a shopping cart dramatically relieves the pain. Which one ofthe following is the most ikkely diagnosis? a) Spinal cord tumor b) Epidural abscess ©) Osteoporotic vertebral fracture 9) Sciatica Y © €) Spinal stenosis Avewor [UEQNIIRGAN) other ucors Explanation Report An Ever (Question Explanation: Spinal stenosis pain is often referred to as pseudoclaudication because the pain is worsened by walking Any movement that flexes the spine, such as pushing a shopping cart, relieves the pain, Pain from tumor of infection is persistent. Osteoporotic vertebral fractures have a sudden onset, Sciatica does not have the typical findings of spinal stenosss. 2/28/2014 3:09:30 AM ‘Mark this question <= => (Question Td : 81452 Question 12 of 20 ‘The triad of urethritis, uveitis, and arthritis is most commonly found in which of the following disorder? a) Tietze's syndrome. ) Reiter's syndrome. ©) Ward-Romano syndrome 4) Cryoglobulinemia €) Sarcoidosis Ancwor [UEISINRIANY) other User's Explanation Report An Exar (Question Explanation: Reiter’s syndrome is an inflammatory condition that can present with urethritis, uveitis, and arthritis. Tietze’s syndrome (A) is an inflammatory costochondrits, demonstrated by chest wall pain. Ward-Romano (C) presents as lorg QT syndrome Cryoglobulinemia (D) can present with a vasculitis and glomerulonephritis. Sarcoidosis (E) presents with hilar adenopathy and eye findings, 2/28/2014 3:09:30 AM ‘Mark this question <= => (Question Td : 81452 Question 12 of 20 ‘The triad of urethritis, uveitis, and arthritis is most commonly found in which of the following disorder? a) Tietze's syndrome Y¥ © b) Reiter's syndrome. c) Ward-Romano syndrome d) Cryoglobulinemia, €) Sarcoidosis. Ancwor [UEISINRIANY) other User's Explanation Report An Exar (Question Explanation: Reiter’s syndrome is an inflammatory condition that can present with urethritis, uveitis, and arthritis. Tietze’s syndrome (A) is an inflammatory costochondrits, demonstrated by chest wall pain. Ward-Romano (C) presents as lorg QT syndrome Cryoglobulinemia (D) can present with a vasculitis and glomerulonephritis. Sarcoidosis (E) presents with hilar adenopathy and eye findings, 22872014 3:09:45 AM ‘Mark this question e => Question Td : 83551 Question 13 of 20 A.women with swelling of the oral mucosa and dry mouth is found have intense destructive inflammation of the salivary glands and antibodies agaist the nbonacleoprotein (SS-B). What is also associated with this syndrome? a) Conjunctivitis ») Goiter ©) Hemolytic anemia 4) Proximal muscle weakness @) Uretheitis Question Explanation: ‘The patienthas Sjogrens syndrome, an autoimmune disease characterized by dry eves (keratoconjunctivitis) and a dry mouth (eerostomia) due to destruction of the lacrimal and salivary glands. Sjégrens syndrome is also characterized by autoantibody production. The most diagnostic autoantibodies are those aganst ribomucleoproteins Ro (SS~A) and La (SSV-B), although co- existing chenmatoid factor and lupus antibodies are not uncommon. The occurrence of aon-Hodkin lymphoma is the most serious complication of Sjégren syndrome, and approximately 5% of patients with Sj5gren syndrome will develop a marginal zone lymphoma, Geiters (choice B) are not typical of Sjégren syndrome. Although autoimmune thyroiditis is associated with Sjégren syndrome, ocular involvementis muuch more characteristic than thyroid involvement. Hemolytic anemia (choice C) is not characieristic of Sjogren syndrome. Primary autoantibodies, drugs, and systemic lupus erythematosus may be associated with hemolytic anemia, but the findings of anti-La and inflammation of the salivary glands indicate that this patient has Sjogren syndrome. Proximal mascle weakness (choice D) in association with autoantibodies is expected in polymyositis or dermatomyositis. Although polymyositis may occur in association with Sjégren syndrome, keratoconjunctivitis would be much more commen than muscle weakness. Urethntis (choice B) in autounmune disease 1s typical of Reiter's syndrome, not Sjogren syndrome. 22872014 3:09:45 AM ‘Mark this question e => Question Td : 83551 Question 13 of 20 A. women with welling of the oral macosa and dry mouth is found have intense destructive inflammation of the salivary glands and ankibodies against the nbonacleoprotein (S$-B). What is also associated with this syndrome? Y © a) Conjunctivitis b) Goiter c) Hemolytic anemia 4) Proximal muscle weakness @) Uretheitis Question Explanation: ‘The patienthas Sjogrens syndrome, an autoimmune disease characterized by dry eves (keratoconjunctivitis) and a dry mouth (eerostomia) due to destruction of the lacrimal and salivary glands. Sjégrens syndrome is also characterized by autoantibody production. The most diagnostic autoantibodies are those aganst ribomucleoproteins Ro (SS~A) and La (SSV-B), although co- existing chenmatoid factor and lupus antibodies are not uncommon. The occurrence of aon-Hodkin lymphoma is the most serious complication of Sjégren syndrome, and approximately 5% of patients with Sj5gren syndrome will develop a marginal zone lymphoma, Geiters (choice B) are not typical of Sjégren syndrome. Although autoimmune thyroiditis is associated with Sjégren syndrome, ocular involvementis muuch more characteristic than thyroid involvement. Hemolytic anemia (choice C) is not characieristic of Sjogren syndrome. Primary autoantibodies, drugs, and systemic lupus erythematosus may be associated with hemolytic anemia, but the findings of anti-La and inflammation of the salivary glands indicate that this patient has Sjogren syndrome. Proximal mascle weakness (choice D) in association with autoantibodies is expected in polymyositis or dermatomyositis. Although polymyositis may occur in association with Sjégren syndrome, keratoconjunctivitis would be much more commen than muscle weakness. Urethntis (choice B) in autounmune disease 1s typical of Reiter's syndrome, not Sjogren syndrome. ‘Marie this question q => (Question Td : 86283 Question 14 of 20 A 33 year old men presents to a physician because of @ painfil, swollen knee for several weeks. The patient reports having had several episodes of painful urination prior to the onset of his kaee pain. Physical examination reveals an edematous kne2, and bilateral conjunctival injection with mucopurulent discharge. The synovial duid ifaspirated from his knee would most closely resemble synovial fluid aspirated from a joint in a patient with which disease? a) Gout +b) Grara-negative bacillary arthritic c) Osteochendiits dissecans 4) Pigmented vilonodular tenosynovitis ¢) Rheumatoid arthritis Question Explanation: This patient has Reiter syndrome (urethritis, cervicitis in females, conjunctivitis, arthritis, and rmacocutaneous lesions). Patients may also demonstrate findings consistent with keratoderma blennorrhagicum, lesions of the palins and soles thet resemble pustular pronasis, Tongue lesions may also be present and are due to erosions histologically characterized by prominent spongiform pnstiles Lesions oftthe glans penis (balanitis circinata) may also be present. Reiter syndrome is associated with chlamydial infection in 70% (or more) of cases, in addition to several enteric infections. The arthritis that can be seen inthe disease is a reactive inflammatory artarits similar to that of theumatoid arthritis. Reactive arthritis is associated with human leukocyte antigen HLA-B27. Goutis a crystal acthritis; other examples of crystal arthritis inchade apatite associated arthritis and pscudogout, due to calciam pyrophosphate dhydraic crystals. Ifyou thought thet the patient has gonococcal arthritis (a septic avthritis), then gram-negative bacilary arthuitis would be 2 good choice. However, Neisseria gonorhorae is not noted for causing conjunctivitis, which should have steered you toward Reiter syndrome. Osteochondritis dissecans, more commonly known as "joint mice,” are litle pieces of broken-off cartilage in the joint they produce a non-inflammatory arthritis with synovial fuid similar to that seen in osteoarthritis. Pigmented vilonodalar tenosynovitis is a proliferative process with giant cells, macrophages, collagen production, and hemosiderin depostion that can involve synovium. This condition can, produce either a non inflammatory or a hemorrhagic synovial uid. ‘Marie this question q => (Question Td : 86283 Question 14 of 20 A 33 year oldman presents to a physician because of a painful, swollen knee for several weeks. The patient reports having had several episodes of painful urination prior to the onset of his knee pain. Physical examination reveals an edematous knee, and bilateral conjunctival injection with mucopurulent discharge. The synovial uid if aspirated from his knee would most closely resemble synovial uid aspirated from a joint in @ patient with which disease? a) Gout b) Gram-negative bacillary arthritis c) Osteochondritis dissecans 4) Pigmented vilonodular tenosynovitis V © 2) Rheumatoid arthritis Question Explanation: This patient has Reiter syndrome (urethritis, cervicitis in females, conjunctivitis, arthritis, and rmacocutaneous lesions). Patients may also demonstrate findings consistent with keratoderma blennorrhagicum, lesions of the palins and soles thet resemble pustular pronasis, Tongue lesions may also be present and are due to erosions histologically characterized by prominent spongiform pnstiles Lesions oftthe glans penis (balanitis circinata) may also be present. Reiter syndrome is associated with chlamydial infection in 70% (or more) of cases, in addition to several enteric infections. The arthritis that can be seen inthe disease is a reactive inflammatory artarits similar to that of theumatoid arthritis. Reactive arthritis is associated with human leukocyte antigen HLA-B27. Goutis a crystal acthritis; other examples of crystal arthritis inchade apatite associated arthritis and pscudogout, due to calciam pyrophosphate dhydraic crystals. Ifyou thought thet the patient has gonococcal arthritis (a septic avthritis), then gram-negative bacilary arthuitis would be 2 good choice. However, Neisseria gonorhorae is not noted for causing conjunctivitis, which should have steered you toward Reiter syndrome. Osteochondritis dissecans, more commonly known as "joint mice,” are litle pieces of broken-off cartilage in the joint they produce a non-inflammatory arthritis with synovial fuid similar to that seen in osteoarthritis. Pigmented vilonodalar tenosynovitis is a proliferative process with giant cells, macrophages, collagen production, and hemosiderin depostion that can involve synovium. This condition can, produce either a non inflammatory or a hemorrhagic synovial uid. ‘Mark this question e& => (Question Id: 87542 Question 15 of 20 A.14 year cld boy suifers fiom sporadic episodes of severe muscle weakness. The episodes occur after severe exercise and after large meals rich in carbohycrates and have also occurred in his father and brother. The boy has also had aight time attacks, ftom which he awakes aearly paralyzed Serum electrolyte studies when ordered duriag one ofthe patient's attacks would most likely show a) Decreased bicarbonate ») Decreaced calcium c) Decreased potassium 4) Increased glucose 2) Increased potassium Question Explanation: This is a classic presentation of the rare condition known as hypokalemic periodic paralysis. There is often a family history suggesting, autosomal dominant inheritence. Symptoms tend to appear late in the first decade or in the second decade of li (when physical education instructors or other kids may criticize the child for being weak), In addition to the triggers listed in the question stem, other precipitating factors include tension, ansisty, and a habitual high salt diet. The condition may be difficult to demonstrate, since serum chemistries are normal between attacks. Muscle biopsy may demonstrate vecuolation or damage to myofibrils. The pathophysiology of this condition was previously poorly understood, but recent advances suggest that the primary defect is in calcium channels. Treatment is with potassium supplementation daring acuts attacks, and prophylactic acetazolamide, triamterene, amiloride, or spironolactone, Decreased serum bicarbonate is a characteristic of metabolic acidosis Hypocalcemia can cause muscle weakness (and more seriously, cardiac arrhythmias), but this patients history suggests a specific alternative etiology. Although calcium channels may be involved in this disorder, the serum calcium level is generally normal. Increased serum ghicose is seen in diabetes melitus, not hypokalemic periodic paralysis. Decreased, not increased, potassimmis the hallmark of this disease, ‘Mark this question e& => (Question Id: 87542 Question 15 of 20 A.14 year cld boy suifers fiom sporadic episodes of severe muscle weakness. The episodes occur after severe exercise and after large meals rich in carbohycrates and have also occurred in his father and brother. The boy has also had aight time attacks, ftom which he awakes aearly paralyzed Serum electrolyte studies when ordered duriag one ofthe patient's attacks would most likely show a) Decreased bicarbonate ») Decreaced calcium Y © c) Decreased potassium 4) Increased glucose ) Increased potassium Question Explanation: This is a classic presentation of the rare condition known as hypokalemic periodic paralysis. There is often a family history suggesting, autosomal dominant inheritence. Symptoms tend to appear late in the first decade or in the second decade of li (when physical education instructors or other kids may criticize the child for being weak), In addition to the triggers listed in the question stem, other precipitating factors include tension, ansisty, and a habitual high salt diet. The condition may be difficult to demonstrate, since serum chemistries are normal between attacks. Muscle biopsy may demonstrate vecuolation or damage to myofibrils. The pathophysiology of this condition was previously poorly understood, but recent advances suggest that the primary defect is in calcium channels. Treatment is with potassium supplementation daring acuts attacks, and prophylactic acetazolamide, triamterene, amiloride, or spironolactone, Decreased serum bicarbonate is a characteristic of metabolic acidosis Hypocalcemia can cause muscle weakness (and more seriously, cardiac arrhythmias), but this patients history suggests a specific alternative etiology. Although calcium channels may be involved in this disorder, the serum calcium level is generally normal. Increased serum ghicose is seen in diabetes melitus, not hypokalemic periodic paralysis. Decreased, not increased, potassimmis the hallmark of this disease, 2/28/2014 3:10:28 AM ‘Marie this question eq => Question Ta Question 16 of 20 A.44 year old women presents with severe bilateral wrist pain, low grade fever, and swelling of the metacarpopkalangeal joints. On ezaminztion she has decreased range of motion of the wrist joints and swan-neck deformities are seen on both hands along with ulnar deviation. Her theumatoid factor was positive. Which of the following is NOT a complication of cheumatoid artheitis? a) Arthits of the cricoarytenoid joints +b) Splenomegaly and leukopenia c) Pancreatitis 4) Cervical spine disease ©) Cardiovascular disease Answer | Explanation Other User's Explanation Report An Error Question Explanation: Rheumatoid arthritis has multisystem organ involvement, but the pancreas is not usually affected. Hoarseness, pain radiating to the ears, speech discomfort, and a sensation of illaess in the throat are all manifestations of laryngeal involvement in sheumetoid arthritis, Arthritis ofthe cricoarytenoid joints (A) leading to narrowing of the glottis and fixed adduction of the vocal cords can occur and lead to problems daring endotracheal intubation during anesthesia. Felty’s syndrome is a form of rheumatoid arthriis associated with both splenomegaly and leukopenia (B). Tue upper cervical spine (D) may be affected by rheumatoid arthritis, Cardiovascular disease) isin the form of pericarditis or coronary arertis, is seen in some patiemss with rheumatoid arthritis, but rarely results in serious conditions such as cardiac tamponade, angina pectotis, or myocardial infarction. 2/28/2014 3:10:28 AM ‘Marie this question eq => Question Ta Question 16 of 20 A.44 year old women presents with severe bilateral wrist pain, low grade fever, and swelling of the metacarpopkalangeal joints. On ezaminztion she has decreased range of motion of the wrist joints and swan-neck deformities are seen on both hands along with ulnar deviation. Her theumatoid factor was positive. Which of the following is NOT a complication of cheumatoid artheitis? a) Arthits of the cricoarytenoid joints +b) Splenomegaly and leukopenia Y © c) Pancreatitis, 4) Cervical spine disease ®) Cardiovascular disease Answer | Explanation Other User's Explanation Report An Error Question Explanation: Rheumatoid arthritis has multisystem organ involvement, but the pancreas is not usually affected. Hoarseness, pain radiating to the ears, speech discomfort, and a sensation of illaess in the throat are all manifestations of laryngeal involvement in sheumetoid arthritis, Arthritis ofthe cricoarytenoid joints (A) leading to narrowing of the glottis and fixed adduction of the vocal cords can occur and lead to problems daring endotracheal intubation during anesthesia. Felty’s syndrome is a form of rheumatoid arthriis associated with both splenomegaly and leukopenia (B). Tue upper cervical spine (D) may be affected by rheumatoid arthritis, Cardiovascular disease) isin the form of pericarditis or coronary arertis, is seen in some patiemss with rheumatoid arthritis, but rarely results in serious conditions such as cardiac tamponade, angina pectotis, or myocardial infarction. ‘Mark this question «= => (Question Td : 95452 Question 17 of 20 A patient mentions that after many years without teeth problems, she has recently developed severe caries. According to her she has had pain and swelling in her hands and knees for many years. More specifically, she finds that her MOP joints and wrists are warm and tender, and are most painfil in the morning ‘The test that would most likely reveal the definitive diagnosis for this patient is a) Antithyroglobulin antibody assay 1b) de-DINA antibody assay ©) Extensive examination of the oral cavity 4) p-ANCA assay ©) SS-A and S8-B antibody assay Question Explanation: Rheumatoid arthritis can coexist with a variety of autoimmune diseases. It is most frequently associated with Sjogren syndrome. diagnosed with a positive SS-A or SS-B titer. is cue to autoimmune involvement with subsequent scarring of the salivary anc lacrintal glands leading to dry eyes and dry mouth. Secondary effects incade parotid gland enlargement, recurrent tracheobronchitis, and dental caries, this due to the inability to produce saliva Remember that saliva is rich in calcinm and phosphates, facilitating remineralization of early carious lesions. There is evidence that remineralization is associated with an increase in the size of enamel crystals and a consequent increase in resistance to caries. Antithyroglobulin antibodies associated with Graves’ disease, result in thyrctonicosis and produce insomnia weight loss, tremors heat intelerance excessive sweating and frequent bowel movements or diarrhea Anti-dsDNA antibodies are specific for systemic Inpus erythematosus, an autoimmune disease characterized by vascultis (which may produce a variety of symptoms depending on the ste of the lesion) rash renal disease hemolytic anemia, and acurologic disturbances. Extensive examination of the oral cavity would reveal the caries and may demonstrate dry mucous membranes. However, nothing specific enough for Sjgren would be found. p-ANCA antibocies are associated with polyartetitis nodosa, a systemic necrotizing vasculitis. Patients present with low grade fever weakness and weight loss. They may also have abdominal pain hematuria renal failure hypertension, and leukocytosis ‘Mark this question «= => (Question Td : 95452 Question 17 of 20 A patient mentons that after many years without teeth problems, she has recently developed severe caries. According to her she has had pain and swelling in her hands and knees for many years. More specifically, she finds that her MOP joints and wrists are warm and tender, and are most painfil in the morning ‘The test that would most likely reveal the defintive diagnosis for this patient is a) Antthyroglobulin antibody assay 'b) ds-DINA antibody assay c) Extensive examination of the oral cavity 4) p-ANCA assay ¥ © €)SS-A and SS-B antibody assay Question Explanation: Rheumatoid arthritis can coexist with a variety of autoimmune diseases. It is most frequently associated with Sjogren syndrome. diagnosed with a positive SS-A or SS-B titer. is cue to autoimmune involvement with subsequent scarring of the salivary anc lacrintal glands leading to dry eyes and dry mouth. Secondary effects incade parotid gland enlargement, recurrent tracheobronchitis, and dental caries, this due to the inability to produce saliva Remember that saliva is rich in calcinm and phosphates, facilitating remineralization of early carious lesions. There is evidence that remineralization is associated with an increase in the size of enamel crystals and a consequent increase in resistance to caries. Antithyroglobulin antibodies associated with Graves’ disease, result in thyrctonicosis and produce insomnia weight loss, tremors heat intelerance excessive sweating and frequent bowel movements or diarrhea Anti-dsDNA antibodies are specific for systemic Inpus erythematosus, an autoimmune disease characterized by vascultis (which may produce a variety of symptoms depending on the ste of the lesion) rash renal disease hemolytic anemia, and acurologic disturbances. Extensive examination of the oral cavity would reveal the caries and may demonstrate dry mucous membranes. However, nothing specific enough for Sjgren would be found. p-ANCA antibocies are associated with polyartetitis nodosa, a systemic necrotizing vasculitis. Patients present with low grade fever weakness and weight loss. They may also have abdominal pain hematuria renal failure hypertension, and leukocytosis ‘Marie this question e& => (Question Id: 120756 Question 18 of 20 A.56 year oldman has arthritic pain in his frst metatarsophalangeal joint. Serum analysis demonstrates a uric acid level of 8.3 mg/dl Arthrocentesis reveals the presence of tophi, The statement LEAST likely appropriate to this man's disorder is a) The best treatment for this man is nonstercidal anti-inflammatory drugs. b) Colchicine can also be used for the treatment of this man, but has significantly more noticeable side effects. c) Involvernent of the first metatarsophalangeal joint is a common finding, 4) This disease typically presents with an absence of urate crystals in the arthrocentesis fuid ©) Men over 30 years of age are typically the majority of patients with this disease Answer | Bolanation Other User's Explanation Report An Error Question Explanation: The presence of tophi in synovial fuid, especially with elevated uric acid levels, is indicative of gout Goutis a metabolic disorder, typically in men over 30, which results in hyperuricemia and uric acid crystal deposition in tissues, including joints. Tophi, which arc farge masses of anosphous uric acid crystals surrounded by inflammatory cells (macrophages, lymphocytes, and fibroblasts), in the synovial Muid, are diagnostic of gout, The joint most often involved in gout is the jont of the greet toe, or the metatarsophialangeal joint The firstline drug therapy for the treatment of goutis NSAID administration, folowed by colchicine administration. However, colchicine is sometimes avoided because it causes gastrointestinal side effects, such as vomiting, nausea, and cramping ‘Marie this question e& => (Question Id: 120756 Question 18 of 20 A.56 year oldman has arthritic pain in his frst metatarsophalangeal joint. Serum analysis demonstrates a uric acid level of 8.3 mg/dl Arthrocentesis reveals the presence of tophi, The statement LEAST likely appropriate to this man's disorder is a) The best treatment for this man is nonstercidal anti-inflammatory drugs. b) Colchicine can also be used for the treatment of this man, but has significantly more noticeable side effects. c) Involvernent of the first metatarsophalangeal joint is a common finding, Y © 4) This disease typically presents with an absence of urate crystals in the arthrocentesis fuid. ©) Men over 30 years of age are typically the majority of patients with this disease Answer | Bolanation Other User's Explanation Report An Error Question Explanation: The presence of tophi in synovial fuid, especially with elevated uric acid levels, is indicative of gout Goutis a metabolic disorder, typically in men over 30, which results in hyperuricemia and uric acid crystal deposition in tissues, including joints. Tophi, which arc farge masses of anosphous uric acid crystals surrounded by inflammatory cells (macrophages, lymphocytes, and fibroblasts), in the synovial Muid, are diagnostic of gout, The joint most often involved in gout is the jont of the greet toe, or the metatarsophialangeal joint The firstline drug therapy for the treatment of goutis NSAID administration, folowed by colchicine administration. However, colchicine is sometimes avoided because it causes gastrointestinal side effects, such as vomiting, nausea, and cramping ‘Mark this question e& => (Question Id : 139539 Question 19 of 20 47 year old men with hairy cell leukemia presents because of worsening lower extremity pain, especially around the first toe. On examnination his right first toe is tender and warm to the touch, The patient is afebrile with splenomegaly and small nodules are noted on the patient’s ear. The metatarsel-phalangeal joint is aspirated. Fincing that would be presentis which one of the following? ) Amorphous clear crystals when viewed parallel to the polarizer b) Needle shaped, blue crystals when viewed parallel to the polarizer c) Needle shaped, yellow crystal when viewed parallel to the polarizer ) Rhomboid blue crystals when viewed parellel to the polarizer ¢) Rhomboid yellow crystals when viewed parallel to the polarizer Anewar (ERENGERNIY) ote sere Explanation Report An Ever Question Explanation: ‘The patient has gout, which is due to precipitation of monosodium urate crystals in joint spaces (notably the great toe) and soft tssues (causing tophi, which are often found on the external ears). Precipitation of the urate crystal occurs with high circulating levels of uric acid, These increased levels can be secondary to enzyme deficiencies, lifestyle choice or drug treatment (especially chemotherapy as inthis case), Gout crystals are described needle shaped and negatwely breffingent These refers to the yellow appearance to the crystals when viewed parallel to the polarizer and blue when viewed perpendicular to the polarizer. Gout crystals are not amorphous. ‘The monosodium urate crystals appear blue when viewed perpendicular to the polarizer. Calcium pyrophosphate crystals are deposited in pseudogout, which classically affects the knee or other large joints. Some of these crystals are positively bireftingent exhibiting a blue color when viewed in parallel and yellow when perpendicular. ‘Mark this question e& => (Question Id : 139539 Question 19 of 20 47 year old men with hairy cell leukemia presents because of worsening lower extremity pain, especially around the first toe. On examination his right frst toe is tender and warm to the touch. The patient is afebrile with splenomegaly and small nodules are noted on the patient's ear. The metatarsal-phalangeal join is aspirated. Finding that would be present is which one of he following? a) Amorphous clear crystals when viewed parallel to the polarizer ) Needle shaped, blue crystals when viewed parallel to the polerizer JY © 2) Needle chaped, yellow crystal when viewed parallel to the polarizer 4) Rhomboid blue crystals when viewed parallel to the polarizer ¢) Rhomboid yellow crystals when viewed parallel to the polarizer Anewar (ERENGERNIY) ote sere Explanation Report An Ever Question Explanation: ‘The patient has gout, which is due to precipitation of monosodium urate crystals in joint spaces (notably the great toe) and soft tssues (causing tophi, which are often found on the external ears). Precipitation of the urate crystal occurs with high circulating levels of uric acid, These increased levels can be secondary to enzyme deficiencies, lifestyle choice or drug treatment (especially chemotherapy as inthis case), Gout crystals are described needle shaped and negatwely breffingent These refers to the yellow appearance to the crystals when viewed parallel to the polarizer and blue when viewed perpendicular to the polarizer. Gout crystals are not amorphous. ‘The monosodium urate crystals appear blue when viewed perpendicular to the polarizer. Calcium pyrophosphate crystals are deposited in pseudogout, which classically affects the knee or other large joints. Some of these crystals are positively bireftingent exhibiting a blue color when viewed in parallel and yellow when perpendicular. 75569, ‘Marre this question e Question Id Question 20 of 20 During auscultatcn cf the heart of 30 year old male a faint murmur consistent with aortic regurgitation is heard, Masculoskceletal system examination shows decreased range of motion of the hips bilaterally and of the spine in rotation and forward bending. No scoliosis is seen, Eyes and mouth exams unremarkable, His ckin is smoota and dry The doctor thiaks of a soondyloathropathy. ‘Which test would best differentiate this patient's disease from the other spondyloartiropathies? a) HLA-Ba? b) MRI of femoral head c) Rheumatoid factor @) Spinal x-ray. e) Stool culture Anewor [UEQGIREN) Other cers Explanation Repost An Ero: Question Explanation: ‘This young man has an inflammatory arthropathy involving his hips and spine. The disease is ankylosing spondyltis which typically occurs in young men. Ankylosing spondylitis begins at the sacroiliac crests then moves upward in the spine, causing inflammation and destruction of the posterior elements oftthe vertebral bodies. The posterior aspects of the vertebrae fice, reducing the range of motion land partially taking the weight off the vertebrae, leading to an atrophy osteoporosis. The spinal X-ray will show fission of the disks ard possible intervertebral dist ossification Patients may also have associated inflammation and fibrosis of the proximal aorta, leading ta aortic regurgitation HLA B27ie a clacs [histecompatibility antigen that has a strong astociation with arleylosing spondyltic, Reiter eyndeome, peotiatic arthritis, and certain entoropathic artaritides, Tris foundin all spondyloaethropathies and therefore can not help te distinguish between them, MRI of the femoral head would reveal inflammatory arthritis ia, that location, but that would not be enough to differentiate ankylosing, spondyliis fom the other entities. The spinal findings are more characteristic of this disease. Rheumatoid factor is typically negative for all ofthe spondyloarthropathies, as are other serologe markers of rheumatoid artaritis. Stool culture is not of agnostic mportance in distinguishing these entines. Anklosing spondylitis is not associated with diarrhea or stool pathogens. 75569, ‘Marre this question e Question Id Question 20 of 20 During auscultation of the heart of a 30 year old male a faint murmur consistent with aortic regurgitation is heard. Musculoskeletal system examination shows decreased range of motion of the hips bilaterally and of the spine in rotation and forward bending, No scoliosis is seen. Eyes and mouth examis unremarkable, His skin is smooth and dry. The doctor thinks ofa sondyloarthropathy. ‘Which test would best differentiate this patient's disease from the other spondyloarthropathies? a) HLA-B27 'b) MRI of femoral head c) Rheumatoid factor ¥ © & Spinal x-ray €) Stool culture Anewor [UEQGIREN) Other cers Explanation Repost An Ero: Question Explanation: ‘This young man has an inflammatory arthropathy involving his hips and spine. The disease is ankylosing spondyltis which typically occurs in young men. Ankylosing spondylitis begins at the sacroiliac crests then moves upward in the spine, causing inflammation and destruction of the posterior elements oftthe vertebral bodies. The posterior aspects of the vertebrae fice, reducing the range of motion land partially taking the weight off the vertebrae, leading to an atrophy osteoporosis. The spinal X-ray will show fission of the disks ard possible intervertebral dist ossification Patients may also have associated inflammation and fibrosis of the proximal aorta, leading ta aortic regurgitation HLA B27ie a clacs [histecompatibility antigen that has a strong astociation with arleylosing spondyltic, Reiter eyndeome, peotiatic arthritis, and certain entoropathic artaritides, Tris foundin all spondyloaethropathies and therefore can not help te distinguish between them, MRI of the femoral head would reveal inflammatory arthritis ia, that location, but that would not be enough to differentiate ankylosing, spondyliis fom the other entities. The spinal findings are more characteristic of this disease. Rheumatoid factor is typically negative for all ofthe spondyloarthropathies, as are other serologe markers of rheumatoid artaritis. Stool culture is not of agnostic mportance in distinguishing these entines. Anklosing spondylitis is not associated with diarrhea or stool pathogens.

You might also like